You are on page 1of 486

Hens a

NI Tr
Heat and Mass Transfer
Heat and Mass Transfer
Second Edition

G. S. Sawhney
Dept. of Mechanical Engineering
Greater Noida Institute of Technology
Greater Noida, Ghaziabad, U.P.
India

I.K. International Publishing House Pvt. Ltd.


NEW DELHI • BANGALORE
Published by
I.K. International Publishing House Pvt. Ltd.
S-25, Green Park Extension, Uphaar Cinema Market
New Delhi - 110 016 (India)
E-mail: infogildnternational.com

ISBN 978-93-80578-39-2

10 9 8 7 6 5 4 3 2 1

© 2010 I.K. International Publishing House Pvt. Ltd.

All rights reserved. No part of this publication may be reproduced, stored in a retrieval system, or
transmitted in any form or any means: electronic, mechanical, photocopying, recording, or otherwise,
without the prior written permission from the publisher.

Published by Krishan Makhijani for I.K. International Publishing House Pvt. Ltd.,
S-25, Green Park Extension, Uphaar Cinema Market, New Delhi - 110 016 and Printed by Rekha Printers
Pvt. Ltd., Okhla Industrial Area, Phase II, New Delhi - 110 020.
PREFACE TO THE SECOND EDITION

In the second edition, the book has been thoroughly revised and enlarged. The chapter on steady
state one-dimensional heat conduction has been modified to include problems on two-dimensional
heat conduction. Finite heat difference method of solving such problems has been covered.
Modification has also been included in the text as per the suggestions obtained from various
sources. Additional typical problems based on the examination papers of various technical
universities have been included with solutions for easy understanding by the students.

I thank all the faculty members of Mechanical Department of GNIT, Greater Noida for their
assistance in the completion of this second edition of the book.

I once again request students and teachers to send constructive suggestions and criticism by
e-mailing chamii-sawhney@hotmail.com

G. S. Sawhney
PREFACE TO THE FIRST EDITION

Heat and Mass Transfer is considered a basic and essential subject for engineering students of
Mechanical and Chemical disciplines. The concepts of heat and mass transfer have a variety of
applications in industry. Realizing the importance of the subject, all technical universities have
introduced this subject in their syllabi. This subject is taught to undergraduate students only for
one semester which is generally fifth semester. Efforts have been made to cover the syllabi of
all technical universities.
Based on my teaching experience, I have tried to explain principles and concepts of heat and
mass transfer in simple and clear terms. The endeavour is to present the subject matter in the
most comprehensive and usable form. The derivations of fundamental relations have been kept
as simple as possible. Diagrams have been used abundantly to elucidate the difficult concepts
which are difficult to explain otherwise. The theory part is further supported with illustrations
and suitable examples. This book has easy to read style as it is written in question-answer form
and it is going to benefit the readers immensely.
Above all, I wish to record my sincere thanks to my wife, Jasbeer Kaur for patience shown
throughout the preparation of this book. I am grateful to Dr. S. Prasad for everlasting support
always extended to me.
I would appreciate constructive suggestions and objective criticism from students and
teachers alike with a view to enhance further usefulness of this book. They may mail me their
views at channi_sawhney@hotmail.com

G. S. Sawhney
CONTENTS

Preface to the Second Edition v

Preface to the First Edition vi

1. Introduction to Heat Transfer 1


2. General Heat Conduction Equation 30
3. Steady State Heat Conduction 50
4. Heat Transfer from Extended Surfaces 135
5. Transient Heat Conduction 172
6. Forced Convection Heat Transfer 214
7. Free Convection 266
8. Radiation Heat Transfer 301
9. Heat Exchangers 370
10. Condensation and Boiling 425
11. Mass Transfer 443
Index 477
Chapter 1

INTRODUCTION TO HEAT
TRANSFER

KEYWORDS AND TOPICS


A HEAT ENERGY A RADIATION
A TRANSIT ENERGY A NATURAL CONVECTION
A STORED ENERGY A FORCED CONVECTION
A INTERNAL ENERGY A FOURIER'S LAW
A THERMAL EQUILIBRIUM A STEADY STATE
A HEAT FLUX A THERMAL CONDUCTIVITY
A MASS TRANSFER A LOCAL COEFFICIENT OF CONVECTION
A TEMPERATURE GRADIENT A AVERAGE COEFFICIENT OF CONVECTION
A CONCENTRATION GRADIENT A STEFAN-BOLTZMAN'S LAW
A CONDUCTION A EMISSIVITY
A CONVECTION A BLACK BODY

INTRODUCTION

Heat transfer is a science which helps in determining the rate of heat energy transfer taking place
between bodies having temperature differences. Heat transfer helps in determining not only how
heat may be transfered but also the rate at which this heat exchange will take place under the
specified conditions. The heat transfer rate forms the basis for difference between the science
of heat transfer and thermodynamics. There are three distinct modes of heat transfer which are:
(i) conduction, (ii) convection, and (iii) radiation. These modes have a common feature that the
heat energy transfer always takes place in the direction of the decreasing temperature. However,
these modes of heat transfer differ entirely in the physical mechanism and laws by which they
are governed. The general mechanism of each of these modes is as shown in the figure.
2 Heat and Mass Transfer

T2

4
q = ocT
Q

To
/////////// /////////, T

To >To.
T = absolute temperature

> T2
(b) Convection (c) Radiation

(a) Conduction

Modes of Heat Transfer

1. What is heat energy?

When a hot body is placed in contact with a cold body, the hot body cools down while the
cold body warms up. The energy transferred from the hot body to the cold body as a result
of temperature difference is called heat energy.

2. What is transit energy?

Energy possessed by a system which can cross its boundary is called transit energy. Heat
and work are transit energy.

3. What is stored energy?

Energy possessed by a system within its boundary is called stored energy. Total stored
energy is the sum of potential energy, kinetic energy and internal energy.

4. Is heat energy a property of a system?

Heat energy is not a property of a system. It appears as a boundary phenomenon, but it is


a path function. The magnitude of heat transfer depends upon the path followed by a system
during the process. Hence, heat energy is inexact differential i.e. rdQ # Q2 — Qi
1

5. How is the magnitude of heat energy measured?

The area under the process or area enclosed in the cyclic process on a TS diagram is the
magnitude of heat energy.
Introduction to Heat Transfer 3

t
t
T T
r%
S —.- S --
Heat transfer in process: Heat utilised in cyclic
shaded portion process: shaded portion

6. What is internal energy?

Internal energy is stored energy and it depends upon temperature of the system.

7. How is the internal energy related to heat energy?

When heat energy enters the system, internal energy increases. Similarly, internal energy
decreases when heat energy flows out of the system.

8. Does internal energy of a system change, if temperature remains constant?

The internal energy depends upon temperature only. Hence, internal energy of a system does
not change when temperature remains constant.

9. Why does heat transfer take place?

Heat transfer takes place by virtue of temperature difference. Heat transfer takes place from
a hot body to a cold body. If temperature of two bodies is same, then no heat transfer takes
place. Heat transfer takes place also in a system, if there is a temperature distribution within
the system.

10. What is thermal equilibrium?

Two systems are said to be in thermal equilibrium with each other if no transfer of heat takes
place.

11. What is the major difference between sciences of thermodynamics and heat
transfer?

Thermodynamics deals with the conversion of heat into work or other forms of energy and
vice versa. However, heat transfer is concerned with the analysis of the rate of heat transfer.
Heat flow takes place only when there is a temperature gradient in a system.

12. What is heat flux?

The heat flux is the amount of heat transfer per unit area and per unit time.
4 Heat and Mass Transfer

13. What does temperature indicate?

Temperature indicates the relative hotness or coldness of a body with respect to its
surroundings.

14. Using an example, explain the difference between sciences of thermodynamics and
heat transfer?

Consider the cooling of a hot body placed in water. Thermodynamical analysis concerns
with the final equilibrium temperature of the body and water. It cannot find out the
temperature of the body after a certain length of time or the period of time would be lapsed
before the body and water can reach the final equilibrium temperature. However, heat
transfer study can predict the temperature of both the body and water as a function of time.

15. What is the reason for the lack of information obtainable from thermodynamics
analysis?

The main reason is the absence of time as variable.

16. What is the difference between the steady and unsteady state of heat transfer?

In steady state heat transfer, the temperature at any location within the system does not vary
with time. The temperature in steady state is the function of space coordinates while it is
independent of time i.e. temperature is given as—

T = f(x, y, z) and
In unsteady state heat transfer, the temperature varies with time as well as location within
aT
the system i.e. temperature T = f(x, y, z, t) and — # 0.
at

17. What are the purposes for studying heat transfer?

The purposes for studying heat transfer are—


(a) to estimate the amount of heat flow through a system under steady and transient
conditions
(b) to determine the temperature distribution within the system under steady and transient
conditions.

18. What are different types of heat transfer based on the configuration of the system?

The heat transfer can be (1) one dimensional, (2) two dimensional, or (3) three dimensional.

19. What are the applications of heat transfer study?

The applications of heat transfer study are—


Introduction to Heat Transfer 5

1. Refrigeration and air-conditioning system: The design of compressors, condensers


and evaporators which involves the study of heat transfer.

T2 < Ti Ti> T2
Surrounding
Surrounding (T2)
(Ti)
02

Refrigeration
plant
Cooling Heating

2. Heat production and conversion system: The thermal design of boilers, steam
turbines, condensers, heat exchangers and cooling towers are carried out on the basis
of heat transfer analysis.

Steam
Vapour (low pressure)

t ir
Steam Cold
water:
I - I

.-----r" r"V—
Vater:
Heat
(0) Boiler 1 Fluid
Condenser u Steam
(high pressure)
Turbine

3. Electric Machines: When current flows through electric motors, generators or


transformers, heat is generated which has to be dissipated effectively to surroundings
to safegaurd these equipment. The heat transfer analysis is carried out for each of these
equipment before fabricating.

Stator

Pipes to
Fins to cool oil
dissipate
heat

AC Motor Transformer
6 Heat and Mass Transfer

4. IC Engine: The cylinder of the engine has to be kept at sufficiently lower temperature
to avoid melting of piston, cylinder and other accessories. Water jacket and fins are
provided to efficiently remove the heat to surroundings. This is possible by heat
transfer analysis.
Head
Piston

AI Cylinder
Water
110
jacket

Connecting
Fins -------14 rod
4
IC Engine

20. What is mass transfer?

Mass transfer is the movement of mass from higher concentration region to lower
concentration region.

21. What is the importance of mass transfer in heat transfer study?

In many applications, heat transfer processes also involve mass transfer. Mass transfer
process is analogous to heat transfer process in many respects.

22. What are the conditions for the stoppage of (1) heat transfer process, and (2) mass
transfer process?

Heat transfer process stops when temperature gradient reduces to zero. The mass transfer
process stops when concentration gradient reduces to zero.

23. Why the concentration of water molecules is higher above the water surface of a
pond as compared to the main portion of air stream?

Due to concentration difference, the water molecules are transported from the water
surface to the air in contact with the water. Later water molecules are transported to main
air stream from this region.

24. Is mass transfer by diffusion affected by molecular spacing of the substance?

The mass transfer depends upon the molecular spacing. More is spacing, more is the mass
Introduction to Heat Transfer 7

transfer. Mass transfer by diffusion occurs more readily in gases as compared to liquid and
least readily in solids.

25. What are different modes of mass transfer?

There are basically two modes of mass transfer which are-


1. Diffusion mass transfer
2. Convective mass transfer

26. What are different modes of heat transfer?

Heat transfer has three modes of heat transfer which are-


1. Conduction
2. Convection
3. Radiation
Conduction usually takes place in solids, convection in liquids and gases, and no medium
is required for radiation.

27. How do these three modes of heat transfer differ? What is essential for all those
modes?

It is essential for all these modes of heat transfer to take place only when the temperature
difference must exist as heat transfer has to take place in the direction of decreasing
temperature. However, the methodology of heat transfer in these modes differs. This can
be explained with the help of an example. Suppose a teacher wishes to give a book to a
student who is sitting at the end of the row. He can adopt any of the following methods-
1. He can deliver the book to the student nearest to him who delivers the same to the next
and so on till the book reaches the last student. Here book corresponds to heat energy
while students correspond to the molecules. Heat transfer takes place from one
molecule to next molecule without the displacement of any molecule.

Book
— --,

Teacher
44-
Student Student Student Student

Conduction

2. The teacher could go to the last student and deliver the book to him. Similarly, in liquid
and gases, the molecule from the hot region moves to cold region and transfers heat
energy and vice versa.
8 Heat and Mass Transfer

Teacher Teacher Teacher Teacher

----"r 4———i,

Book

Teacher Student Student Student Student Student


..-------
....------

Convection

3. The teacher can throw the book so that it reaches the last student. Heat can move from
hot region to cold region without any movement of molecules. No medium is required.

Book
-.I

Teacher
CO
Student Student Student Student Student Student
Radiation

28. Explain heat transfer by conduction.

Conduction is a mode of heat transfer from one part of a substance to another part of the
same substance or from one substance to another in case in physical contact without any
appreciable displacement of molecule forming the substance.
In solids, heat is conducted by following mechanisms-
1. Lattice vibration: Fast moving molecules in the hottest region transfer heat to
adjacent molecules by impact.
2. Transport of free electrons: Free electrons provide heat energy flux in the direction
of decreasing temperature. In case of gases, molecules are in a continuous random
motion exchanging energy and momentum. The kinetic energy of a molecule is a
function of its temperature. When molecule from hot region collides with a molecule
of cold region, it loses heat energy by collision. In liquids also, the mechanism of heat
transfer is similar to that of gases.

29. Explain heat transfer by convection.

Convection is a mode of heat transfer within a fluid by mixing of one portion of the fluid
with another. The physical movement of fluid molecules is made possible by the existing
temperature gradient.
Introduction to Heat Transfer 9

The convection heat transfer comprises two mechanisms. First is transfer of heat due to
random molecular motion which is called diffusion. Second is transfer of heat due to bulk
motion of the fluid which is called advection. The bulk motion is due to difference of density
of fluid particles on account of difference of temperature. For example, when water is
heated, the water at the bottom gets heated and its density decreases and it consequently
rises. Thus, the heat is carried from the bottom to the top by the actual movement of the
hot water.

30. How can convection heat transfer be classified?

Convection heat transfer can be classified according to the nature of fluid flow. If fluid
motion is setup by buoyancy effects, the heat transfer is said to be the free or natural
convection. If fluid motion is artificially induced by a pump, fan or blower, then the heat
transfer is called forced convection.
Plate Plate

t
Heat 4 i

8=
Blower

Natural or free convection Forced convection

31. Natural convection in water in lake plays important role in saving the lives of fishes
when the atmospheric temperature falls below 0°C. Comment.

As water at the surface of lake is cooled, it becomes denser and it goes down. The lesser
cold water from the bottom of the lake rises up to the surface and gets cooled. The entire
water attains the temperature of 4°C. Water has maximum density at 4°C and its density falls
when temperature falls below 4°C. Surface water on cooling below 4°C remains at surface
only and it keeps on getting cooled there only till it freezes. Heat is now lost to the
atmosphere by the water in lake only due to conduction through the ice at the surface. As
ice is a poor conductor of heat, the further freezing of water below the frozen upper surface
is very slow. The temperature of water below the frozen upper surface remains at 4°C and
fishes can survive.

32. The heat transfer inside a human body is carried out by a forced convection. Explain.

Blood inside a human body is circulated to all parts of the body by heart acting as a pump.
Heat is lost to the surroundings by the body through conduction, convection and radiation.
The rate of heat loss depends upon: (1) clothing, (2) perspiration, (3) outside temperature,
(4) air speed, (5) humidity and other factors. However, our body has a forced convection
I 0 Heat and Mass Transfer

system with blood flow by heart. The blood flow transports the needed amount of heat to
maintain a remarkable constant temperatures within our body.
Jugular vein Carotid artery
Subclavian vein
Arch of the arota

Superior Pulmonary
vena cave vein
t< 0°C
0°C Ice
-- 4°C water Pulmonary
artery
//////////////////////////
Inferior Thoracic arota
Survival of fishes in lake below vena cave
0°C by natural convection
Abdominal arota

Forced Convection: Blood flow

33. What is radiation?


Radiation is a process of the transfer of heat through space or matter other than conduction
or convection. The process of radiation does not need any material medium for heat transfer.
Energy is emitted by a body and thus energy travels in space just like light as electromagnetic
waves. All bodies radiate heat. Transfer of heat by radiation occurs because hot body emits
more heat than it receives. A cold body receives more heat than it emits. The heat from the
sun reaches the earth by radiation, travelling millions of kilometres of empty space.

34. What are the properties of radiant heat?


The properties of radiant heat are-
1. Radiation does not require any material medium for travelling.
2. Radiant heat can be reflected like light from the surface. It also obeys the laws of
reflection like light.
3. Radiant heat travels with same velocity of light.
4. Radiant heat also shows interference, diffraction and polarisation like light.
5. Radiant heat also follows the law of inverse square.
6. Radiant heat is electromagnetic wave having longer wavelength. Hence, radiant heat is
invisible to our eyes.
Introduction to Heat Transfer I I

35. What is the basic law of heat conduction?


Or
What is Fourier's law of heat conduction? Why is a negative sign inserted in its
expression?
(UPTU — 2003)
The basic law of heat conduction or Fourier's law of heat conduction states that the rate
of heat flow by conduction in material is equal to the product of three quantities as given
below-
1. Thermal conductivity (k) which is a property of the material.
2. Cross-sectional area (A) which is normal to the direction of flow.
all
3. Temperature gradient along the direction of flow of heat (
ax
dQ
Rate of heat flow = = q
dt
aT
or q= —m ax
The minus sign is inserted to make the heat flow rate (q) positive as temperature
gradient is negative.

36. Find the expression for heat conducted (q) through a slab thickness (L), cross-
sectional area (A) having temperature T1 and T2 at both ends for steady state.
As per heat conduction law-
q = —m aT
ax

1
Temp
T1

T2

x1 x2
-,.- x
H L H Thickness

q • dx = —kAaT
In steady state, temperature distribution does not change and q is constant. In case
thermal conductivity (k) is constant, then
5 q • ax = —f kAaT
I2 Heat and Mass Transfer

X2 T2

or q fdx = —kA f aT
xi T1

q(x2 — x1) = —kA(T2 — T1)


(T
or qq _ Mai — T2)
X1

Mai — T2)
= L

37. The inner surface of a plane brick wall is at 60°C and outer surface is at 35°C.
Calculate the rate of heat transfer per sq metre surface area of the wall which is 220
mm thick. Thermal conductivity of the brick is 0.51 W/mk.
(AMIE — 1998)

Heat conduction equation is—


q = kA(T — T2)
L
= 0.51 x 1(60 — 35)
0.22
= 57.95 W/m2

38. What are assumptions made for Fourier's law of heat conduction?

The assumptions are-


1. Conduction of heat takes place under steady state conditions.
2. Temperature gradient is constant i.e. linear temperature distribution.
3. Heat flow is unidirectional.
4. Material is homogeneous and isotropic i.e. thermal conductivity is constant in all
directions.
5. No internal heat generation.

39. Explain thermal conductivity.

The ability of a material to conduct heat is measured by thermal conductivity (k). It is a


physical property of the material. It is defined as the amount of heat conducted through a
body of unit cross-sectional area, unit thickness and in unit time when temperature
difference is unit.
Materials with high thermal conductivity are good conductors of heat. Materials with low
thermal conductivity are good thermal insulators of heat.
Solids have higher thermal conductivity than liquids and liquids have better thermal
conductivity than gases. Metals have higher thermal conductivity than non-metals. Higher
conductivity is possible as metals have a large number of free electrons which help in
carrying thermal energy from one place to another.
Introduction to Heat Transfer I3

40. Why are cooking utensils made of metals whereas their handles are made of wood
or plastic?

Metals have high thermal conductivity. Therefore, cooking utensils are made of metals so
that heat can easily pass from burner to food being cooked in the metallic utensil. However,
wood or plastic has low thermal conductivity so that heat current going to handle is smaller
and it is not heated up.

Utensil

Handle
Q1 >> Q2
Q1

I— —I

Utensils and Thermal Conductivity

41. Why does a rug placed in bright sun on a tiled floor help to facilitate bare foot
standing?

The rug and tiled floor when placed outside in sun aquire same hot temperature. But it is
much easier to stay bare foot on the rug as compared to tiled floor. This is possible as the
thermal conductivity of the rug is lower than the tiled floor. Hence, the heat current going
into our feet is smaller in case of rug.

02 << 01

Thermal Conductivity of Rug and Tiled Floor

42. Give the value of thermal conductivity (k) of metals, liquids and non-metals.

The values of thermal conductivity decrease from metals to non-metals, metals to liquids
and then liquids to gases as tabulated below.

Material Thermal conductivity (k) (W/mk)


1. Metals
(a) Silver 410
(b) Copper 385
Contd.
I4 Heat and Mass Transfer

(c) Aluminium 225


(d) Cast Iron 55-65
(e) Steel 20-45
2. Liquids
(a) Water 0.585
(b) Transformer oil 0.176
3. Gases
(a) Air 0.0238
(b) Hydrogen 0.167
(c) Oxygen 0.0242
4. Non-metals
(a) Concrete 1.3
(b) Glass wool 0.042
(c) Wood 0.146
(d) Asbestos 0.17

43. Give the range of thermal conductivities of various states of matter.

The thermal conductivity is highest for solids and it is lowest for gases. The thermal
conductivity of liquids is lower than solids but higher than gases.

CO2 H2
Gases
Oil Mercury
Liquids
1
Molecular spacing

Insulator
Plastics Oxides
Non-metals

Alloys

Zinc Silver
Pure metal

0.01 0.1 1 10 100 1000


Thermal conductivity

44. Write a short note on the effect of temperature on thermal conductivity of good
conductors.
(UPTU — 2003 — 04)
or
How do thermal conductivities of gases and liquids vary with temperature?
(UPTU — 2006 — 07)
Introduction to Heat Transfer I5

A solid material comprises atoms bound in a lattice and free electrons. The conduction of
heat is due to: (1) lattice vibration and (2) migration of free electrons. The total thermal
conduction (k) is sum of the electronic conductance (ke) and the lattice conductance (k1)
k = ke + k1
Good conductors are made from pure metals. In pure metals there is always a presence
of a large number of free electrons which would increase the electronic conductance (ke)
as electronic conductance (ke) increases with temperature i.e. ke ... 7'. However, thermal
vibration of lattice also increases with rising temperature which retards the motion of free
electrons. As a result, thermal conductivity of pure metals or good conductors decreases
with rising temperature with aluminium and uranium as exception.

500

I 400
>,

=
C
u
o
To
E 200 Aluminium
43
_c
1—
-------------------------"'----....,,J!on
o I 1
100 500 1000
Temperature (k) —,-

In alloys or non-metals, there is an absence of free electrons. Hence, heat conducted in


alloys is only due to lattice vibration. As temperature increases, the lattice vibration
increases, resulting into rise in heat flow. Thus, the thermal conductivity of alloys increases
with increase in the temperature.
Thermal conductivity

Stainless
steel

Temp —..-

For gases, the molecules are in continuous random motion. As temperature increases,
velocity of molecules becomes higher as compared to velocities of molecules in lower
I6 Heat and Mass Transfer

temperature region. The molecules move from high temperature region to lower temperature
region and give up their energy through collision to lower energy molecules. Therefore, the
thermal conductivity of gases increases with rise in temperature and the increase of thermal
conductivity is proportional to square root of their absolute temperatures.

I 0.3
1
5
0.2
////,
8
0.1 Air

CO2

200 400 600 800


Temperature (K)

Note: The thermal conductivity of most liquids remains constant except water and thermal
conductivity of insulating material generally increases with increasing temperature.
Insulators have generally voids in which air is entrapped, resulting high thermal resistance.

45. A plane wall 20 cm thick has a thermal conductivity given by the following relation:
k = 2 + 0.0005 T W/mK
Where T is temperature in Kelvin. If two surfaces of the wall are at 150°C and 50°C,
determine the rate of heat transfer for a wall of 3 m x 5 m.
(UPTU — 2003 — 04)

aT
Q = —A • K •
ax

or Q ax = —kaT

Now integrating both sides


T2

f Qax = —5 kaT
0
T2

•L = (2 + o.our) a T
A

= 2(T1 — T2) + 0.005 [T12 — Ti]


= 2(150 — 50) + 2.5 x 10-3[1502 — 502]
= 200 + 2.5 x 10-3 (22500 — 2500)
= 200 + 50
Introduction to Heat Transfer I 7
250 x A
or Q- L
250 x 3x 5
=
0.2
= 18.75 kW

46. Why do two thin blankets appear warmer than a single blanket of double thickness?

There is air entrapped in between two thin blankets. Air has very low thermal conductivity
(k = 0.024 W/m-k). Hence, two blankets offer more resistance to the heat flow from human
body to the surroundings as compared to a single blanket with double thickness.

Blankets

Air Blanket

Qi Q2
Q2 > Q1
Single vs two blankets

6. What is Wiedermann and Frantz law?

The ratio of the thermal and electrical conductivities is same for all metals at the same
temperature which is directly proportional to the absolute temperature of the metal.
k
... T
C
where k = thermal conductivity
C = electrical conductivity
T = absolute temperature
It is apparent that good conductors of electricity are also good conductors of heat.

47. What is the basic law of convection?


Or
What is the Newton's law of cooling?

The rate of heat transfer by convection is—


Q = hA(T, — Toc )
or Q = q = h(T, — Toc)
A
I8 Heat and Mass Transfer

Fluid flow at temperature


T„ Ts > To
Q

/////////////////1//// /

Hot surface at Ts
Convective heat transfer

In above equation—
Q = q = rate of heat transfer per unit area by convection
h = average convective heat transfer coefficient
T, = temperature of solid surface
T., = temperature of fluid
49. What are the parameters on which the value of convective heat transfer coefficient
depends?

As it can be seen from the Newton's law of cooling, the value of h depends upon—
(a) The geometry of the solid surface
(b) The properties of the fluid in touch with the solid surface
(c) The velocity of the fluid
(d) The temperature difference between solid surface and the fluid i.e. (T, — To.).

50. What do you understand from the local and average convective heat transfer
coefficients?

Since the value of h depends on (i) the geometry of the solid surface, (ii) the velocity and
properties of the fluid and (iii) the temperature difference (7's — T) and these quantities do
not remain constant over the solid surface, hence the value of h vary from point to point
on the surface. Therefore, h, denotes the value of h at a point on the surface at distance
x from the leading edge of the surface and it is called local convective heat transfer
coefficient. The average convective heat transfer coefficient (h) is defined as—
h = f h., dA
AA
or h= Q
A(Ts —L)
The average convective heat transfer h (also called film heat transfer coefficient) is defined
as the amount of heat transferred for: (i) a unit temperature difference between the fluid and
surface, (ii) a unit area of solid surface, and (iii) in unit time.

51. A hot plate lm x 2m is maintained at 320°C. Air at 20°C moves over the plate. If
h = 40 watts/m2°C, find the rate of heat transfer.
Introduction to Heat Transfer I9

Q = hA(T, — To j
= 40 x (1 x 2) (320 — 20)
= 40 x 2 x 300
= 24 x 103 watts
= 24 kW

52. An electric wire carrying current is submerged in water. The wire is 1.5 mm in
diameter and 300 mm length. If h = 4500 watts/m2°C, and wire surface remains at
120°C when water boils (100°C), find electric power supplied to the wire.

A = surface area of wire


= MI = ir x 1.5 x 10-3 x 300x 10-3
= 14.13 x le m2
Now heat transfer rates
Q = h • A • (T, — Tw)
= 4500 x 14.13 x le x (120 — 100)
= 127.2 W
Since 127.2 W heat is transferred from the wire, we have to supply 127.2 W power to the
wire.

53. What is the basic law of radiation?


Or
What is Stefan-Boltzman's law?

Stefan-Boltzman's law states that the total emissive power of a black body (Eb) is
proportional to the fourth power of its absolute temperature.
Eb oc T 4
or Eb = a 74
where a = Stefan-Boltzman constant
= 5.67 x 10-8 W/m2k4
The rate of heat emission from a black body to the surroundings is given by
Q = a A(T 14 — Ti')
where
Q = Maximum rate of heat emission from a black body
A = Surface area
T1 = Absolute temperature of the body in Kelvin
T2 = Absolute temperature of surrounding
a = Stefan-Boltzman constant
= 5.67 x 10-8 W/m2K4

54. How much heat is radiated from one real surface to another?

Real surface emits heat at lower rate than black body. Also, the emission of heat depends
20 Heat and Mass Transfer

upon the geometry of two surfaces. Hence, the net heat exchange between two real bodies
is given by—
Q1-2 = Cr Ai El Fi2(V — T24)

where
E1= emissivity of surface 1
F1_2 = shape factor depending upon the geometry of two surfaces.

55. What do you understand from black body and total emissive power? Out of snow and
surface coated with lamp black, which is a black body?

A black body is an ideal hypothetical body which absorbs all thermal radiation falling on it
and it does not transmit or reflect any radiation falling on it. Total emissive power is the rate
of thermal radiation emitted by the body in all directions.
A surface coated with lamp black appears black but it is not a black body. Although snow
appears white but it absorbs all radiation falling on it and hence it is a black body.

56. What do you understand from combined heat transfer mechanism?


(UPTU — 2006 — 07)

There are three mechanisms of heat transfer viz. (1) conduction (2) convection and
(3) radiation. In actual practice and in many situations, heat is transferred in successive
steps by different mechanisms. Heat transfer can occur—
(a) By one mechanism
(b) By more than one mechanism in steps.

57. Explain the combined mechanism of cooling of hot coffee kept in a thermos flask.

Flask
Coffee
Air layer

Cover

q2

q4 Room
Air T.,
q6

q8
Cooling of coffee
Introduction to Heat Transfer 2I
The mechanisms for heat transfer from hot coffee to air include:
(a) Free convection from hot coffee to the walls of the flask (q1)
(b) Conduction through wall of the flask (q2)
(c) Free convection from flask to air layer (q3)
(d) Free convection from air layer to the plastic cover (q4)
(e) Radiation from flask wall to the plastic cover (q5)
(f) Conduction of heat in the cover (q6)
(g) Free convection from cover to room air (q7)
(h) Radiation from cover to room air (q8)

58. Explain the combined mechanism of heat transfer from outside to inside of cooled
room (neglect radiation).

=> qcond

T., Room temperature


Tr

The pathways for heat transfer from outside to the cooled room are-
(a) Heat transfer by convection from surrounding (11,) to the wall at T1
qconvi = hiA(T- - T1)
(b) Heat transfer by conduction from outer wall (T1) to inner wall (T2)
-kA(T2 - li)
qcond = L
where L = thickness of wall
(c) Heat transfer by convection from inner wall (T2) to room air (Tr)
qconv2 = h2A(T2 — T-)
The quantity of heat flow through each step is same, hence

qconvl = qcond = qconv2 = q


Also we can write-

T - T1 =
qconv, q (1)
-
hi A hi A
T1 - T2 = qwnd — q (2)
kAIL kA
L
22 Heat and Mass Transfer

qC0//1,2 _ q
T2 — T = _ (3)
r h2 A h2A
If we add equation (1) to (3), we get

(T — Tr) = q( 1 1
hi A + kNL + h2A
or q = UA(L — Tr)
u=( 1 1+ 1j
where
h1 +k h2
Here U is known as the overall heat transfer coefficient.

59. A black body of surface area 2 x 10-3 m2 is heated to 127°C and is suspended in a
room having temperature 27°C. Find the initial rate of loss of heat from the black
body.
From Stefan-Boltzman law we have
Q = a A(Tb4 — V)
= (5.67 x 10-8) (2 x 10-3) (4004 — 3004)
= 1.98 W

60. The inside temperature of a furnace wall is 1000°C. It has k = 2.0 W/mK and
thickness = 200 mm. The convective heat transfer coefficient (h) at outside is—
h = 8 + 0.1 AT
If surrounding temperature is 20°C, find the outer wall temperature and heat
transfer per unit area.

To, = 20°C

k(Ti — T2 ) = h(7, — Too


q= 2
L
2.0(1000 — T2)
or = [8 + 0.1(T2 — 20)] [T2 — 20]
L
or 10(1000 — T2) = 8 T2 — 160 + 0.1 (T2 — 20)2
or 10000 — 10 T2 = 8 T2 — 160 + 0.1(T22 —400-40 T2)
or T1 — 40 T2 + 400 = 10(10160 — 18 T2)
or T22 + 140 T2 — 101200 =0
Introduction to Heat Transfer 23

- 140 ± )11402 + 4 x1012 x102


T2 —
2
- 140 ± 236.48
=
2
= 96.48
Now heat transfer per unit area is
k
q= — (1000 - 96.48)

= 2 (903.52)
0.2
= 9035.2 W/m2

61. The walls of a house are 0.3 m thick and total surface area of the walls is 100 m2.
Thermal conductivity (k) of the walls = 1 W/m2K. Temperature of outside air is 37°C
and inside air is 27°C. Heat transfer coefficients inside and outside are 20 W/m2K
and 10W/m2K. Calculate the inside and outside wall temperature, heat flux and heat
transfer rate through the walls.

-Q
T.,2 = 37°C

2
= 20 w/m k

0.3 m

Overall heat transfer coefficient-


1 L 1
U= ++
h1 k h2
= 1 + 0.3 + 1
10 1 20
= 0.1 + 0.3 + 0.05
= 0.45
Q= U.A. (T-2 - T-1)
= 0.45 x 100 (37 - 27)
= 450 W
Now for steady state and for interface of inside air and inside wall, we have-
Q = hiA(T 1 - To,1)
450 = 10 x 100(T1 - 27)
T1 - 27 = 0.45
24 Heat and Mass Transfer

T1 = 27.45°C
Similarly for outside air and outside wall, we have—
Q = h2A(T.2 — T2)
450 = 20 x 100 (37 — T2)
37 — T2 = 40 = 0.225
20 x 100
or T2 = 36.775°C.

62. An electric heater emits 1000 watts of thermal radiation. The filament has surface
area 0.06 m2 and may be presumed as black body. Find its temperature if a = 6 x
10-8 W/m2K4.
4
Q=A• a• T
1000 = 0.06 x 6 x 10-8 x T4
T4 _ 1000
or
6 x 10-8 x 0.06
= 2.77 x 1011
or T = 725.5°K

63. Wet clothes are being on a clothesline outdoor in subzero weather. After a day, the
clothes are brought into the house and observed to be dry. The process of drying is
best explained as—
(a) vaporisation (b) sublimation
(c) melting (d) condensation
(GRE)

The clothes are drying due to change of frozen water in the clothes into the vapour. Hence,
it is the process of sublimation.
Option (b) is correct.

64. Aluminium foil used for cooking food and storage sometimes has one shiny surface
and one dull surface. Should the shiny side or the dull side be on the outside when
the food is wrapped for baking and freezing respectively?
(a) shiny side, shiny side (b) dull side, dull side
(c) shiny side, dull side (d) dull side and shiny side
(10 L)

Shiny side can reflect the heat while dull side can absorb the heat most efficiently. We require
most heat to be absorbed during baking which necessitates dull side is to kept outside.
However, while freezing, we want heat to leave the food. We want heat should not be
reflected back. Hence, shiny side should be outside.
The option (d) is correct.
Introduction to Heat Transfer 25

65. An outer wall of a furnace is at '7'„,' Kelvin and it faces an environment at '1'e' . The
wall's average convective heat transfer coefficient 'h' is 4 W/m2K. For radiation heat
transfer, the wall's emissivity 'e' equals its absorbity 'a' and equals 0.8. The ratio of
net heat loss by radiation to heat loss by convection from the wall is closed to—
Tw4 — T4 T.: — Te4
(a) 5 a e
TH, — Te
(b) 2.5a
TH, — Te

(d) 0.2a T .
T4
w — T4 — Te4
(c) 0.5 a e
TH, — Te TH, — Te
(IES 89)

QRad = ea (T,',,I — Te4) x A


= 0.8 a (Tw4 — Te4) x A
Qconv = hA(T, — Te)
= 4 A(T, — Te )

QRad _ 0 .8 a (1;4,, — Te4 )


Qconv — 4 (Tw — Te)

(T 4 —7' 4)
= 0.2 a w e
(Tw — Te)
Option (d) is correct.

66. A furnace is made of a red brick wall of thickness 0.5 m and conductivity 0.7 W/mK.
For the same heat loss and temperature drop, this can be replaced by a layer of
diatomic earth of conductivity 0.14 W/mK and thickness:
(a) 0.5 m (b) 0.1 m
(c) 0.2 m (d) 0.5 m
(IES — 93)

Qi = —kill dT
dx1
dT
= —0.7 x A x
0.5
dT
Q2 = —0.14 x A x
dx2
Now Qi = Q2
dT dT
0.7 • A • = 0.14 x A x
0.5 dx2
0.14 x 0.5
or dx2 =
0.7
26 Heat and Mass Transfer

= 0.2 x 0.5
= 0.1 m
Option (b) is correct.

67. A thin flat plate 2 m x 2 m is hanging freely in air. The temperature of the
surroundings is 25°C. Solar radiation is falling on one side of the plate at the rate
of 500 W/m2. The temperature of the plate will remain constant at 30°C, if the
convective heat transfer coefficient (in W/m2°C) is:
(a) 25 (b) 50
(c) 120 (d) 200
(IES — 93)
QRad = 500 watt
Qconv = hA(Tw — To0
= h x (2 x 2) (30 — 25)
QRad = Qconv
500 = h x 4 x 5
500
or h= = 25 W/m2°C
20
Option (a) is correct.

68. A wire is plastically deformed (bent) by supplying a force of 40 N over a distance of


0.8 m (the force moves in directions in which the distance is measured). If the wire
has a mass of 0.2 kg and a sp. heat of 0.5 kJ/kg°C, estimate the maximum increase
in the average temperature of the wire
(a) 0.03°C (b) 0.3°C
(c) 3°C (d) 30°C
(GRE)

Work = Force x distance


= 40 x 0.8 = 32 J
Now 32 J = mCp x dT
= 0.2 x 0.5 x 103 x dT
or 32
dT = = .032°C
103
Option (a) is correct.

69. In a long cylinder rod of radius R and a surface heat flux of 90, the uniform internal
heat generation rate is
Introduction to Heat Transfer 27

(a) 290/R (b) 290


90 (d) 90
(c)
2R R2
(GATE)
Heat flux x Area = Heat generation per volume x volume
qo x 27 rR x L = qg x 4ir R2 x L
_ qo x2nRxL
or
qg — 47rR2 x L

_ go
2R
Option (c) is correct.

70. For a given heat flow and for same thickness, the temperature drop across the
material will be maximum for
(a) Copper (b) Steel
(c) Glass wool (d) Refractory brick
(GATE 1996)
dT
q = kA = constant
dx

. dT . I-
k
Hence, temperature fall is maximum when 'V is low. Since copper has lowest I', hence
temperature fall for copper is maximum.
Option (a) is correct.

71. A 2 kW, 40 litres water heater is switched on for 20 minutes. The heat capacity C,
for water is 4.2 kJ/kg K. Assuming all the electrical energy has gone into heating the
water, increase of the water temperature in degree centigrade is—
(a) 2.7 (b) 4.0
(c) 14.3 (d) 25.25
(GATE 2003)
Heat supplied = Heat gained
Heat supplied = power x time
= 2000 x 20 x 60
= 24 x 105W
Heat gained = m Cp x AT

25 x 10",= 40000 x 4.2 x 1000 x AT


1000
28 Heat and Mass Transfer

25 x 105
or AT = = 14.3°C.
4x104 x4.2

72. A solar collector receiving solar radiation at the rate of 0.6 kW/m2 transforms it to
the internal energy of a fluid at an overall efficiency of 50%. The fluid heated to
350 K is used to run a heat engine which rejects heat at 315 K. If the heat engine
is to deliver 2.5 kW power, the minimum area of the solar collector required would
be—
(a) 83.33 m2 (b) 16.66 m2
(c) 39.68 m2 (d) 79.36 m2
(GATE 2004)
Solar radiation = 600 W/m2
With 50% T = 0.5 x 600 = 300 W/m2
1 T2
=1 315
11engine =
350
W 35
or •= =0.1
Q 350

25 x 103
or Q= = 25000 W
0.1
25000
Now A=
Solar radiation
25000
300
= 83.33 m2

73. An electrically heated plate dissipates heat by convection at a rate of 8000 W/m2 into
ambient air at 25°C. If the surface of the hot plate is at 125°C. Calculate, the heat
transfer coefficient for convection between the plate and the air.
(Annamalai University 2004-5, 2005-8)
Q = hA(T, — To,)

or Q = h(T, — To,)

8000 = h(125 — 25)


8000
or h=
100
= 80 W/m2°C
Introduction to Heat Transfer 29

74. For the three dimensional object shown in the figure below, five faces are insulated.
The sixth face (PQRS), which is not insulated and it interacts thermally with the
ambient, with a convective heat transfer coefficient of 10 W/m2K. The ambient
temperature is 30°C. Heat is uniformly generated inside the object at the rate of
100 W/m3. Assuming the face PQRS to be at uniform temperature, its steady state
temperature is—
(a) 10°C (b) 20°C
(c) 30°C (d) 40°C
(GATE 2008)

h x A x (Ts. — To ) = heat convected out


Now heat convected out = heat generated
10 x 2 x 2 x (Ts — 30) = 100 x 2 x 2 x 1
or Ts = 30 + 10
= 40°C
Option (d) is correct.
Chapter 2
GENERAL HEAT CONDUCTION
EQUATION

A CARTESIAN COORDINATES A ONE-DIMENSIONAL HEAT CONDUCTION


A CYLINDRICAL COORDINATES A LINEAR HEAT FLOW
A SPHERICAL COORDINATES A RADIAL HEAT FLOW
A THERMAL DIFFUSIVITY A LINEAR TEMPERATURE DISTRIBUTION
A HEAT ACCUMULATION A RADIAL TEMPERATURE DISTRIBUTION
A GENERAL HEAT CONDUCTION
EQUATION

INTRODUCTION

Conduction is primarily a molecular phenomenon requiring temperature gradient as driving force


for heat transfer to take place. Heat conducting systems can have different shapes and
geometries. Hence, heat conduction equations have to be derived using cartesian, cylindrical and
spherical coordinates. The Fourier's law for unidirectional heat flow helps in derivation of three-
dimensional heat conduction equations.

1. Why are different coordinate systems being used?

Heat conducting systems can have different shapes and geometries. Heat conducting
systems can basically have rectangular, cylindrical or spherical geometries. It is therefore
easier to derive the heat conduction equations by using cartesian, cylindrical or spherical
coordinates system depending on the shape of the heat conducting system.

2. How are point, surface area and volume defined in (1) cartesian coordinates (2)
cylindrical coordinates, and (3) spherical coordinates?
General Heat Conduction Equation 3I

Cartesian coordinates system

In cartesian coordinate system, we have—


(a) Volume AV = dx • dy • dz
(b) Surface area S1 = dy • dz
(c) Surface area S2 = dx • dz
(d) Surface area S3 = dx • dy

Cylindrical coordinates system

In cylindrical coordinates, we have—


(a) Volume AV = dr • rd0 • dz
(b) Surface area S1 = rd0 • dz
(c) Surface area S2 = dr • dz
(d) Surface area S3 = dr • rd0

Spherical coordinates system

In spherical system, we have-


1. Volume dV = rd0 • dr • r sin Od0
2. Surface area S1 = r sin 0 • dO • rde
3. Surface area S2 = dr • rde
4. Surface area S3 = dr • r sin MO
32 Heat and Mass Transfer

3. Derive one-dimensional heat conduction equation in cartesian coordinates.


(UPTU 2003)

Consider a small volume of material in cartesian coordinates of sides dx, dy and dz as shown
in the figure.

Qx+dx

Heat entering the control volume along the x-direction in time dt is—
Qx = — kx (dy • dz) • g • dt

Heat leaving the control volume along the x-direction in time dt is—

Qx±dx = Qx ,ax (eodx


Net heat accumulation in the control volume for heat flowing in x-direction is—

dQx = Qx — [Qx + (Qx)dx]


ax

= x (Qx)dx
a
- a [—k (dy • dz) aT • chi • dx
ax x

— (k aT )dx • dy • dz • dt
ax x ax
Similarly, the heat accumulation in the control volume due to heat flow along y and z
directions are—

dQy = [k 1clx • dy • dz • dt
ay Y .

dQz = a [k dyd d
z az x. z. t
Now total heat accumulation in the control volume from all directions

a (k. ), a (k aT)1d x • dy•dz•dt


(dOx+y+z = [ax(-
kx ax) ay -Y az z az
ay
General Heat Conduction Equation 33

If qg is the heat generated per unit volume and unit time, then total heat generated in the
control volume is—
dQg = qg • dx • dy • dz • dt
Total heat accumulation in control volume is—
dQtotal = dQx+y+z ÷ dQg
Due to heat accumulation, the temperature of control volume will increase and thermal
energy increase in time dt is—
dQtotal = Mass x sp heat x rate of change of temperature x time
= (p • dx • dy • dz) x C, x f x dt
t
where p = density and C = sp. heat
From thermal energy balance, we get—
( aT) + a ( al + a (kzaT)].
[ kx
ax ax) ayky ay az az
dx • dy • dz • dt + qg • dx • dy • dz • dt

= p • dx • dy • dz • p • aaT • dt
t

or a [ a T 1 + a [ k ay
a T1 ÷ [ Ice a T1 + = aT p . c .

ax kx ax ay ay :z az qg p
at

In case of homogeneous and isotropic material, k = kx = ky = kz i.e. uniform thermal


conductivity. Therefore we have—
a2T a2T a2T qg P • cp aT
ax2+aye + az2 + k — k at
In case of steady state condition, the temperature at any point does not change with time
(aT = 0) then we have—
at '
a 2T + a2T + a 2T + =
qg 0
axe aye az2 k
In case there is no heat generated in the controlled volume (qg = 0), then we have—
a2T + a2 T + a2 T =
0 (Laplace equation)
ax2 aye az2
If temperature varies in one direction (say along x-axis), then we have—
a2 T
=0
ax2
4. What is thermal diffusivity? What is its significance?

Three-dimensional heat conduction equation is—


a2T a2T a2T qg aT
P • cp
ax2+aye + az2 + k — k at
34 Heat and Mass Transfer

1 aT
__ — —
a at

where a = k = thermal diffusivity. The significance of diffusivity is—


pCp

1. Larger is the diffusivity (a), the shorter is time required for the supplied heat to
penetrate deeper into solid i.e. aT is low.
at
2. Larger value of diffusivity is possible either due to high value of thermal conductivity
(k) or low value of heat capacity (p • cr).
3. Lower is the heat capacity, less amount of heat is absorbed to raise the temperature and
bulk heat is available for further transmission.
4. Metals and gases have high value of diffusivity and they respond quickly to any
temperature change. The thermal diffusivity of non-metals is low and they do not
respond quickly to any temperature change.

5. Where can heat conduction equation in cylindrical coordinates be used?

In many situations, we may have heat conducting systems having cylindrical


geometries such as cylinders, rods and pipes. For these systems, we can derive the heat
conduction equations by using cylindrical coordinates system.

6. Derive general heat conduction equation in cylindrical coordinates for homogeneous


and isotropic materials.
(UPTU 2002-03, 2004-5)

Consider an infinitesimal cylindrical volume of sides dr, rdo and dz as shown in the figure.

--"'",..
00

Q0+4

The element volume dV = dr • rd0 • dz


Heat flow in through (z, 0) plane in time dt is—

Qr = —k(dz • rdo) a T • dt
r
Heat flow out through (z, 0) plane in time dt is—

Qr+dr = Qr + ar(Q0CIT
General Heat Conduction Equation 35
Heat accumulated in the element volume due to heat flowing in and out in (z, (0) plane in time
dt is—
(Qr)acc = Qr — Qr+dr

= Qr — (Qr + a l. Q,. • dr)

= — a Q • dr
ar r

= a (lc • dz • rdeP • dt) dr


ar ar
= k • dr • d0 • dz • t (r aT ) • dt
r

= k(dr • d0 • dz)( aT + r a2Tjdt


ar ar2

1 aT)
= k(dr • rd0 • dz)( a2T
r2 + a r dt
a r
Heat flowing in through (r, z) plane in time dt is—
aT
Q0 .—k(dr • dz) rao dt

Heat flowing out through (r, z) plane in time dt is—

Q0+0 = Qo + (Q0) • rd0


raa(1)
Heat accumulated in the element volume due to heat flowing in and out in (r, z) plane in time
dt is—

(Q0)acc = QO — Q0+0
a (Q0)
= Qo — Qo r d0
rag)

= (Q0) • r • d0
ra0

= rah
a [—k(dr • dz) aT dti• r • d0
r •d0

= k(dr • dz • d0) • as (1 aT •dt


a)

= k(dr rd0 • dz) [ 1 a2T ) dt


r 2 a2 0
Heat flowing in the element volume through (r, 0) plane is—
36 Heat and Mass Transfer

aT
Q, = —k(dr • rdep) —dt
az
Heat flowing out from the element where through (r, 0) plane in time dt is—
a dz
Qz+dz = Qz (Qz) •
az
Heat accumulated in the element volume due to flow in and out of the heat through (r, 0)
plane in time dt is—
(Qz)acc = Qz Qz+dz

= Qz [Qz +
az
(Qz) • dz]
a
az (Qz) • dz
= --

a[ k(nhp • dr) • dT • dt ] dz
z dz
a2T
= k(dr • rd0 • dz) 2 dt
az
Total heat accumulated in the element volume is—
Qacc = (Qr)acc + (Q(p)acc + (Qz)acc
If heat generation in the element is qz per unit volume, then heat generated in time dt is—
Qg = (dr • rd0 • dz)qg • dt
Increase of thermal energy of the element volume in time dt is—
dE = p • (dr • rd0 • dz)cp • daT • dt
t
where p = density and cp = sp heat
The energy balance is
Qacc + Qg = dE

k[
2T
+
aT + 1 a2T + a 2T1 dt • [dr • rd0. dz] + qg(dr • rd0. dz) • dt
ar2 r ar r2 a02 a z2

dE = p • (dr • rd0 • dz) • cp • a


aT dt
t

a2T 1 aT
or k[ + + 1 a2T _ L a2T aT
ar2 r ar r2 a02 az2 1 qg = P cP . at

or a2T 1 aT + 1 a2T a2T qg aT


ar2 r ar r 2 atp2 az2 k k at
1 aT
a at
General Heat Conduction Equation 37

k
a= = thermal diffusivity
pcp
Note: For steady state and one-dimensional heat conduction in radial direction with no heat
generation (qg = 0), the conduction equation reduces to—
a2T + 1 aT = o
art r ar

7. Where can heat conduction equation in spherical coordinates be used?

In many situations, we may have heat conducting systems having spherical geometries such
as hollow spherical vessels. For these systems, we can derive the heat conduction equations
by using spherical coordinates system.

8. Derive one-dimensional heat conduction equation in spherical coordinates.


(UPTU 2003 — 04)

Consider an infinitesimal spherical volume element of size dr, rd0 and r sin 0 thip as shown
in the figure. Assume uniform conductivity as k,

density = p, sp. heat = c and uniform heat generation rate = qg per unit volume per unit time.
Now dV = dr • rd0 • r sin Od0
Now heat flow through (r, 0) plane in 0-direction is—
Q0 = —k(dr • rd0) aTedo dt
r sin
Heat flowing out through (r, 0) plane in 0-direction is-
1 a p)r sin Odo
Q0+d° = Q° + r sin e ao (Q,
Heat accumulated in the elemental volume due to heat flow in and out in the 0-direction is—
dap = Q0 — 00+0
—1
= (Q0)r sin Odo
r sin 0 a0
38 Heat and Mass Transfer

—1 a [ — k (dr •• rde) •• 1 aT • dti• r sin 0 • chip


= .
r sine ao
rsin e ao

1
= k (dr • rde • r sin 0 • d0) • r 2 sin 2
a
2 0 ao2Tdt
Now heat flow through (r, 0) plane in 0-direction is—
Q9 = —k (dr • r sin e • d0) aaTe dt
r
Heat flowing out in 0-direction is—
a
Qe+de = Qe + ra 0 (Q0 • rde
Heat accumulated in the elemental volume due to heat flow in and out in 0-direction is—
da, = Qe — Qe+de
a (Q9) • r • dO
= rail
aT
=— [ k(dr • rsin 0 d0) rae dt r • de
rag ]

. (dr • rde • r sin 0 d0) •


=‘
4 1 a (sin OL
T )dt
sine ae
Now heat flow in (0, 0) plane in r-direction is—
aT
Qr = —k(r • de • r sin 0 • dtP) • dt
ar
Heat flow out in (0, 0) plane in r-direction—
,
Qr+dr = Qr -1-
ar (Qr) • dr
Heat accumulated in the elemental volume due to heat flow in and out from the r-direction
is—
dQr = Qr — Qr+dr

= --
ar (Qr)dr

= a [ k(r de • rsin e • d0) a T dt]• dr


ar r

= k • de • sin 0 • dO • dr • 1(1-2 aT
y)dt
r

= k(dr • rde • r sin 0 d0) -2, 1(1-2 aaT )dt


7 r
Now (dQ),,,, = dQ0 + dQ 19 + dQr
General Heat Conduction Equation 39
Now heat generated in thin control volume—
Qg = qg(dr • rd0 • r sin 0 • d0) x dt
Rate of change of thermal energy within the elemental volume—
dT
dE = p (dr • rd0 • r sin 0 • d0) • cp at dt
Applying now energy balance—
(dQ)acc + dQg = dE
1 a2T + 1
a02
a (sine ail, 1 a (r 2 al
ad r2 ar ar
+ qgk = p •cp
k
aT = 1 aT
at a at
r2 sin2 0 r2 sin 0 ad
Now for steady state unidirectional heat flow in the radial direction for a sphere without
internal heat generation is-
1a ail _
r2 al. r al. — a

9. What is the aim in analysing a heat conduction problem in a medium?


The aim in analysing a heat conduction equation is to determine the temperature field in a
medium resulting from the conditions that are imposed on the boundaries of the medium.
The determination of temperature distribution helps in—
(a) Finding heat transfer rate
(b) Finding the structural integrity of the medium through determination of thermal
stresses, expansion and deflections
(c) Optimising the thickness of an insulating material for a conductor.

10. What are the conditions on which the solution of heat conduction equation depends?
The solution of heat conduction equation depends upon following conditions—
(a) Boundary conditions
(b) Initial conditions

11. Why do we require initial conditions?


If the solution of heat conduction equation is time dependent, initial conditions are required.
The initial conditions describe the temperature distribution in a medium at the initial moment
of time and these are needed for the time dependent problems (transient or unsteady state).
The initial conditions can be expressed as—
(i) t = 0, T = T(x, y, z)
(ii) t = 0, T= To = Constant

12. What are the boundary conditions? How many are needed?
40 Heat and Mass Transfer

Boundary conditions are the physical conditions existing at the boundaries of the medium.
If heat conduction equation is of first order, only one initial condition is required. If heat
equation is of second order, two boundary conditions are needed for each coordinate in
determining temperature distribution.

13. What are the different kinds of boundary conditions?


There are three kinds of boundary conditions which are-
(a) Constant surface temperature-
T(0, t) = T, = Temperature at surface
(b) Constant surface heat flux-
(i) Finite heat flux-
(-k aT
, ) = q, = fixed heat
ax x=o
(ii) Zero heat flux due to insulated surface-
( dT -0
ax)x=o -
(c) Equal conductive and convective heat flux at surface condition-
(_ k dT
= h[T, - T(0, t)]
dx )x=o

14. Using one-dimension heat conduction equation with uniform thermal conductivity,
steady state and no heat generation, find heat `Q' to be supplied to a wall of width
`I,' to maintain temperature difference of (T1 - T2) across it.

H L -.-I
One-dimensional heat conduction equation is-
a 2T
0 as qg = 0, aT= 0
axe =
Now integrating two times, we get-
T = Cix + C2
Using boundary conditions
x = 0, T = T1 —> C2 = T1
X = L, T = T2 —> T2 = CiL + T1
General Heat Conduction Equation 4I

T2 — Ti .
or C1 =
L
Now putting the values of C1 and C2 , we get—
—(T2 Ti )
T x + Ti
L
The above is temperature distribution inside the wall along x-direction. Now to find heat, we
have to find the change of temperature w.r.t. x coordinate.

aT _ T2 — T1
ax — L
As per Fourier's law, heat flow is—
Q = —kA
aT
ax
= —kA ( T2 — 11 )
L )
— kA(Ti—T2)
L

15. The temperature distribution across a wall of 2 m thick is given by—


T(x) = 820 — 300x — 50x2
where T is in degree and x is meter. If the wall has surface area = 5 m2, internal heat
generation qg = 10 kw/m3, k = 40W/mk, find—
(a) Rate of heat entering and leaving the wall
(b) Rate of change of internal energy

x= 0 x=2

Given T = 820 — 300x — 50x2


aT = —300 — 100x
ax
aT
Heat entering Qx=0 = —kA ()
ax x=o
or Qx=0 =—40 x 5 x (-300 — 1004=o
= 200 x 300
= 60 kW
42 Heat and Mass Transfer

Heat leaving Qx=2 = —kA (


ax 1=2
or Qx=2 = —40 x 5 x (-300 — 100x)x =2
= —200(-300 — 200)
= +200 x 500
= 100 kW
Rate of change of internal energy (Au) is—
Au = Rate of heat entering + Rate of heat generation — Rate of heat leaving
= 60 + qg xA xL— 100
= 60 + 10 x 5 x 2 — 100
= 60 + 100 — 100
= 60 kW

16. The temperature distribution at a certain instant of time in a concrete slab during
curing is given by—
T = 3x2 + 3x + 16
Where x is in cm and T is in Kelvin. The rate of change of temperature with time
is given by (assume diffusivity to be 0.0003 cm2/s)
(a) 9 x 10-4 k/s (b) 48 x 10-4 k/s
(c) 12 x 10-4 k/s (d) 18 x 10-4 k/s
(IES — 94)
T = 3x2 + 3x + 16
= 6x + 3
ax
ax
a2 7
xx2 = 6

Now a2T = 1 aT
ax2 a at
aT a2T
or =a
at ax2
= 0.0003 x 6
= 0.0018 k/s
Option (d) is correct.

17. Consider the following statements:


dT
The Fourier heat conduction equation Q = —kA , presume
dx
1. Steady-state conditions
General Heat Conduction Equation 43

2. Constant value of thermal conductivity


3. Uniform temperatures at the wall surfaces
4. One-dimensional heat flow of these statements
(a) 1, 2 and 3 are correct
(b) 1, 2 and 4 are correct
(c) 2, 3 and 4 are correct
(d) 1, 3 and 4 are correct
(IES - 98)
The statement (b) is correct.

18. A steam pipe is passed through a room in which air and wall temperature are at 30°C
while surface temperature of the pipe is 400°C. If the diameter of the pipe is 40 mm
and average heat transfer coefficient is 20 W/m2°C, what is the rate of heat loss from
the pipe for one metre length of pipe.

Ta = 30°C
i
i T ., = 400°C
steam i
i
1) pipe
\ ,

''-------'

Q =hxAx(Ts - Too )
= 20 x (,rdL) x (400 - 30)
= 20 x ir x 40 x 10-3 xlx 370
= 0.93 kW
19. A copper pipe of diameter 5 cm is kept at a temperature of 50°C in a large room
where air and wall are at 20°C. If the average heat transfer coefficient is 6.5 W/m2°C
and emissivity of the body is 0.8, calculate the total heat lost by the pipe per unit
length.

Qconv = hA(T1 - T2)


= h(ndL)(Ti - T2)
= 6.5 x 7r x 5 x 10-2 x 1 x (50 - 30)
= 30.6 W/m
QRad = aA E (T14 — T24)
= 5.67 x 10-8 x (7r x 5 x 10-2 x 1) x 0.8 x (3234 - 2934)
= 25 W/m
Total heat lost Qtotal = QConv + QRad
= 30.6 + 25
= 55.6 W/m
44 Heat and Mass Transfer

20. The radial temperature distribution at a certain instant of time in a cylindrical


column during curing is given by
T = 5r + 250
where r is in cm and T is in Kelvin. Find the rate of change of temperature with time
if diffusivity is 0.002 cm2/s.

One-dimensional steady-state heat conduction in radial direction is given by—


a ( aT) _ 1 aT
ar C. ar ) — a at
Now T = 5r + 250
aT 5
ar -
Hence,

aar (r
• 5) =
1 at
a at
or 5= 1 aT
a al-
dT
or =ax5
at
= 0.002 x 5
= 0.01 k/s

21. The radial temperature distribution in a sphere at a certain time is given by


T = 6r2 + 200
where r is in cm and T is in Kelvin. Find the rate of change of temperature with time
if diffusivity is 0.0004 cm2/s.

One-dimensional steady-state radial heat distribution is-

1 a ( r 2 an = 1 aT
r2 ar ar ) a at
Now T = 6r2 -F 200
aT _ 12r
ar -
I- a [r2(12r)] = 1 aT
r2 ar a at
or z 1
— x 12.3•r2 =
r2
1 aT
a at
or aT _ a x 36 = 0.0004 x 36 = 0.0144 k/s
at -
General Heat Conduction Equation 45

22. In steady-state the heat generation rate per unit volume is qg at radius r of solid rod
given by
2
qg = 500[1 —(i)
1 X 106

Find the maximum temperature of the rod having R = 40 cm when temperature at


surface is 20°C. Take k = 100 kW/m°C.
For cylindrical surface and radial heat flow, we have-

a ;) = qk r
t(r al
or aar (r aaT ) + x 500[1- (f )2 1 X 106 = 0
i. z

On integration, we get-
(r aT) soo x io6 r2 r4 = C
a r ) 100 x io3 L 2 4R2
Now aT = 0
at r = 0, —
=0
aT = -5000 (' - ' 3
ar 2 4R2
Now integrating again
[r2 r4
T=- + C2
5000 4 16R2
Now T = Tmwhen r = 0
Tmax = C2

2 4
T = —5000L r ,1 + Tmax
4 16R 2
[r2 4
or Tmax = T + 5000
4
16R 2 ]
Now when r = R, T = Ts = 20°C
or Tmax = 20 + 500OUR2 )

= 20 + 5000 x 3 x 16 x 10-2
6
= 20 + 150 = 170°C
46 Heat and Mass Transfer

23. The temperature distribution in a long cylindrical to be is—

T(r) = 800 + 1200r — 3000r2


where T is in centrigrade and r is in metre. The cylindrical tube has inner radius =
250 mm and outer radius = 400 mm. Find the heat entering and leaving the cylinder.

Given T(r) = 800 + 1200r — 3000r2


... aT = 1200 — 6000r
ar
Hence, heat flow per unit length is—
—kA(— aT
Qr=0.25 =
ar 1=0.25
= —50 X (27r X 0.25)(+1200 — 6000r)r=o.25
= —50(0.570(1200 — 1500)
= 257r(300)
= 23.55 kW
Similarly, heat flowing out per unit length is—

Q r=o.4 = —kA
( a Tr) r = 0.4
Qo.4 = —50 x (27r x 0.4)(1200 — 6000 • r)r=o.4
= —40 is • (1200 — 2400)
= 40 • ir • 1200
= 150.72 kW

24. The temperature field in a body varies according to the equation T(x, y) = x 3 + 4xy.
The direction of fastest variation in temperature at the point (1, 0) is given by
(a) 3i + 8j (b) j
(c) 0.6i + 0.8j (d) 0.5i + 0.866j
(GATE — 1967)

T = x3 + 4y
Now aT = 3x 2 and aT = 4
ax ay
General Heat Conduction Equation 47

Hence, PT)
ax x=i
=3, (al =4
ay y=o
Hence, direction of fastest variation in temperature at (1, 0) is given by 3i + 4j or 0.6i
+ 0.8j.
.. Option (c) is correct.

25. Two surfaces of a 1.1 m thick wall are maintained at temperature of 200°C and 60°C
respectively. Determine the heat flux through the wall, if the thermal conductivity
of the wall material varies with temperature as follows—
k = 0.2(1 + 0.04T)
where k is in W/mk and T is in °C.
(UPTU — 2007-08)

q = Q = —ky
OT
A x
or qax = —kaT
On integration
L 200
q f dx = —f [0.2 (1 + 0.04T)]aT
o 60
2 200
q x L = —0.2 [T + 0.04 xT
2 60

= —0.2 [(200 — 60) + 0.04 (200 2_ 602)]


= —0.2[140 + 0.02[4 — 0.36] x 104]
= —0.2[140 + 728]
= 173.6 W/m
q = 173.6 = 173.6 _ 157.8 W/m2
L 1.1 —

26. A 50 cm thick steel plate (ksteei = 36 W/mk) of large surface area generates heat at
a uniform rate of 20,000 W/m3. Both the surfaces of the plate are exposed to a gas
(hgas = 25 W/m2k) at temperature of 100°C. Find the temperature at: (i) centre, (ii)
surface, and (iii) at 10 cm right

2
h = 25 W/m k

ksteel = 36 W/mk

T., = 100°C

.- X
x=0 x = 25 cm
48 Heat and Mass Transfer

2T qg n
+— =v
x2 k
a 2 r, qg
or
axe — k

or aT _ qg
+ Cl
ax — k x
aT = 0 at x = 0, hence C1 = 0
ax
qg
Now on integration, T = —T x2 + C2 (i)
At the surface, we have-

-1c( axx=o.2,5
T) = h(Ts — T—)

—k(— X 0.25) = 25 (Ts — 100)


k

20 x 103 x 0.35
or — Ts — 100
25
or Ts = 100 + 280
= 380°C
Putting the value of x = 0.25 in equation (i), we have-
— 25 x 103
380 = x (0.25)2 + c2
36
or C2 = 380 + 43.4
= 423.4 °C
. T = —694.4x2 + 423.4°C
The temperature is maximum at x = 0,
Tx=0 = 423.4°C
Temperature at x = 0.1 m
Tx=0.1 = 423.4 — 694.4 x (0.1)2 = 416.5°C

27. A flat plate has thickness 5 cm, thermal conductivity 1 W/mk, convective heat
transfer coefficient on its two flat faces are 10 W/m2k and 20 W/m2. The overall heat
transfer coefficient for the plate is—
(a) 5 W/m2K (b) 6.33 W/m2K
(c) 20 W/m2K (d) 30 W/m2K
(ESE — 2002)
General Heat Conduction Equation 49
k = 1 w/mk

2
hi = 10 w/m2k = 20 w/m k

H— 5 cm -P-

1 5 x 10-2 1
ER = + +
hi k h2
5 x 10-2 1
= 1 + +
10 1 20
= 0.1 + 0.05 + 0.05
= 0.2

U =E 1 = 5 W/m2K
R = 0.2
Option (a) is correct.

28. A 0.5 m thick plane wall has its two surfaces kept at 300°C and 200°C. Thermal
conductivity of the wall varies linearly with temperature and its value at 300°C and
200°C and 25 W/mK and 15 W/mK respectively. Then steady heat flux through the
wall is—
(a) 8 kW/m2 (b) 5 kW/m2
(c) 4 kW/m2 (d) 3 kW/m2
(ESE = 2001)
k1 +k1 25+15
k —— = 20
2 2
Q=k (300 — 200) _ 20 x 100
x = 4 kW/m2
A 0.5 — 0.5
Option (c) is correct.
Chapter 3
STEADY STATE HEAT CONDUCTION

A LOG-MEAN AREA A THERMAL CONTACT RESISTANCE


A CONCEPT OF THERMAL RESISTANCE A COMPOSITE CYLINDER
A THERMAL RESISTANCE OF SLAB A COMPOSITE SPHERE
A THERMAL RESISTANCE OF SPHERICAL A CRITICAL THICKNESS FOR WIRE
WALL A CRITICAL THICKNESS FOR SPHERE
A COMPOSITE WALL A LAPLACE EQUATION 2D HEAT
CONDUCTION
A COMPLEX COMPOSITE WALL A SHAPE FACTOR
A OVERALL HEAT TRANSFER A FINITE DIFFERENCE EQUATION
CO-EFFICIENT A RELAXATION METHOD

INTRODUCTION

One-dimensional heat flow means that there is a single predominant direction in which
temperature gradient is existing. Hence, heat flow takes place in one direction while heat flow
in other directions is neglected. Therefore, only one space coordinate is required to describe the
distribution of temperature within the heat conducting body. The term of steady state means that
the temperature distribution within the body does not change with time. Fairly good estimates
of heat conduction can be made in many cases by making the assumption of one dimensional
heat conduction. Heat flow through: (i) a plane wall, (ii) a very long hollow cylinder or pipe, and
(iii) a very thin wire or rod can be found out easily using this method. Multidimensional heat
conduction problems are generally complex. Laplace and Fourier obtained analytical solution of
partial differential equations for different heat conduction problems. The analytic approach can
be applied to such problems which involve simple geometrical shapes and boundary conditions.
Finite difference forms of Laplace equations are numerical methods which give good
approximation to complex heat conduction problems. Finite difference equations can be easily
solved with the help of high speed computers.
Steady State Heat Conduction 5I

1. Derive an expression under one-dimensional steady state heat conduction for


temperature distribution for plane wall or slab.

T2

L —d
x x=L
x=0
Heat conduction through slab

Consider a plane wall with thickness `L'. Heat is flowing under steady state condition in
such a way that temperatures of two faces be maintained at T1 and T2. If thermal
conductivity 'lc' is uniform, then general heat conduction equation is
a 2 r, a2 r, a2 r, 1 al,
+qg
ax2+ a y2+ z2 K a at
For one-dimensional steady state heat conduction without internal heat generation
a2 , a2 , a 2 7,
we have y2 = 0, z2 = 0, t2 = 0 and qg 0. =

The equation becomes


a2T
=0
axe
On integration, we get
aT
= C1 = constant
ax
On integration again, we get
T = C1 x + C2
Applying boundary conditions, we get
x = 0, T = Ti or Ti = C2
and x = L, T = T2 or T2 = CiL + T2
T2 — T1
or
Cl L
The temperature distribution becomes
T2 — Ti
T= x + Ti
L
T2 - T1 _ x
or
T2 — T1 — L
52 Heat and Mass Transfer

Now heat flowing from the plane wall is—

q= aT
—m a x

=-kA4[71-1 (T1 T2)1


kA(T — T2 )
L

2. Derive an expression for temperature distribution under one dimensional steady


state heat conduction and without internal heat generation for a hollow cylinder or
tube.

Consider a hollow cylinder of length 'I,' with outer and inner radius as ro and r1. For steady
state one-dimensional heat conduction in radial direction with no heat generation, the
equation is
a2T 1 aT
are r ar =u

Or
aT ) _°
ar r
On integration, we get
raT
= C1 = constant
ar
Or
aT Cl
ar = r

Radial heat flow in a hollow cylinder

Integrating again, we get


T = C1 log r+ C2
Applying boundary conditions, we get
At r =ri,T=Ti or Ti = Ci log ri+ C2
and r = ro, T = To or To = Ci log ro -F C2
Steady State Heat Conduction 53

From there two equations, we get


To —
Ci =
log

ro
Ti log + (Ti — To) log ri
ri
C2 -
log r
0

The temperature distribution equation becomes

Ti log r° + (Ti — To) log ri


To — Ti ri
T— tog r +
log 11) log i
i)
rt

log —
r
T —Ti ri
or
To — Ti log /11

The rate of heat flow is


OT
Q = —M
ar

log r
= —kA a [ +(To Ti ) ri
r Tt log —

ri
1
= —kA[(To Ti ) r
log i
il
ri
kA (To — Ti )
=—
r log °
r

But area A = 27r r • L


2 irkL(Ti— To)
Q—
log( 713

3. Find log-mean area for a hollow cylinder. What is its purpose?

Consider a hollow cylinder and plane wall made of same material with same wall thickness
(ro — r) and temperatures at two sides as Ti and To. In order to have the same heat
conduction from the hollow cylinder and the plane wall, the problem is to find log-mean area
54 Heat and Mass Transfer

(Am) for the plane wall which will give the same heat transfer from the plane wall as it is
from the cylinder.

o — r1-

Cylinder and plain wall with same thickness and temperatures

Heat flow through hollow cylinder is


27r Lk(Ti — To )
Q= (i)
loge 115

Heat flow through the plane wall is


Q - k Am (Ti —To)
(r - ri)
Now on equating Equation (i) and (ii), we get
2irLk(T —To ) kAm(Ti — To)
log 0r r0 — r,

2 7rL(ro — L(27rro — 2 7rri)


or Am =
2 7rr0 • L
log —
r0 log
27rri •L

Ao — Ai
log A—°
Ai
Here Ao and Ai are outer and inner surface areas of the cylinder.
The purpose of log-mean area of the cylinder is that the problem of heat transfer from
the cylinder can be transformed into that of from the plane wall so that the solution of the
problem becomes simpler.

4. Derive an expression for temperature distribution under one-dimensional steady


state heat conduction for a hollow sphere.

Consider a hollow sphere with radius r0 and ri with steady temperatures at To and Ti. The
radial heat flow in the hollow sphere is
Steady State Heat Conduction 55

a2T 2 aT
+ =0
are r ar

or (r2 aT) = o
ar ar
Now on integration, we get—
aT
r r2
Integrating again, we get—

l + C2
T=—C (i)
r
Applying boundary conditions, we get—
at r = ri, T = = + C2
ri

at r = ro, T = To To = + C2
ro
On solving, we get—
Ti — To
= 1
Cl 1
(to rij

and 1 (Ti — To)



C2 = T
ri
ro
Putting the values of C1 and C2, in equation (i), we get-
1 1
T— Cr — rij
To —

Now rate of heat flow is—


aT
Q = kA
ar
= —kA 1(T—To)
r2 1)
ro )

= —k • 4ir r2 • —
1 — To)
r2 (1 1)
To ri
4,rk(T —To)
ri ro
(ro —
56 Heat and Mass Transfer

5. Find log-mean area for a hollow sphere.

zink(Ti — T0 )
Qsphere = Pi • To • • • (i)
fro — n)
k Am (Ti — To)
Qslab • • •00
(ro — ri)
where ro — ri = thickness of slab and Am is log-mean area of the slab. Now equating both
equations, we get—
Qsphere = Qslab
4 k (Ti — T0 )ri • r0 kAm(Ti — T0)
(ro — ri) (ro — ri)
or Am = 47cri ro
or Amt = (47r ri r0)2
= (47r ri2 • 4ir ro2)
Am = Ai • Ao
where Ai = inner surface area of sphere
Ao = outer surface area of sphere.

6. Explain electrical analogy for heat transfer. Find thermal resistance for conduction
and convection heat transfer.
Consider heat flow through a plain wall or slab, then heat flow is—
(T1 — T2)Ak
Q— L
— T2 — AT _ Thermal Potential
L L LI Ak
Ak Ak
Comparing this equation with equation for electric circuit—
AV Electric Potential
Current = 1 =
Hence, we can have following electrical analogy—
(a) The rate of heat transfer (Q) is analogus to current (I)
(b) Thermal potential (T1 — T2) is analogus to electric potential (V1 — V2 = AV)
(c) Thermal resistance (Rth) is analogus to electric resistance (R)

0--MAF-0 V2 T1 0--Wr-0 T2
R Rth L
th k
Electric circuit Thermal circuit

Concept of resistance
Steady State Heat Conduction 57

Similarly, for convective heat transfer, we have

Q = hA(Ti — T2)
T1 —T2 AT
or Q= -
1 Rth
hA
1
where Rth = which can be shown as—
hA
Q
T1 0-I \ AAJ-0 T2

Rth = hA
I

Thermal circuit

7. Derive an expression for thermal resistance of cylindrical wall.

Guidance: First derive the heat transfer through the cylindrical wall as done in question 2
2 • K • L(Ti — To)
Q-
log /11
ri

T —To - To
or Q=
log r
0 Rth

27rK • L

log r0
r
Here Rol = which can be shown as—
27r KiL

Q
0—"AA/-0 T,
ro
log
Rth
2irkL
Thermal circuit

8. Derive an expression for thermal resistance of spherical wall.


(UPTU 2003 — 4, 2008 — 9)

Guidance: First derive the equation of heat transfer of spherical wall as done in question 4
and then find thermal resistance as explained below-
4irk(Ti — To)
Q= ri • ro
(ro—
58 Heat and Mass Transfer

T — To
ro —
47rk• ri •ro

T — To
Rth
1-0 —
where Rth — which can be shown as—
k • ri • ro

Q
0—AAA,--0 To
ro —
Rth — 4.1*- .
ro
Thermal circuit

9. What is a composite wall?

When a wall is consisting of a number of slabs of different materials and thicknesses as


shown in the figure, it is called composite wall.

T1
T2
T3
T4
Q
Q ® ® ®

1 2 3 4
IT
LA —+- LB 4._ Lc H
Composite wall

LA LB . Lc
Here RthA = , RthB = , AthC =
kAA kB A kc A

In case there is a perfect contact between the different layers and there is no temperature
drop occurring at interface (as shown in the figure), we can draw the analogous electric
circuit as shown below—

Q
RthA RthB RthC

Since all resistances are in series, the heat transfer is—


Steady State Heat Conduction 59

T1—T4
Q =
LA LB LC
+— +
kAA kB A kc il

10. What is an overall heat transfer coefficient (u)?

The overall heat transfer coefficient is defined as the heat transmitted per unit area, per unit
time and per degree of temperature difference between hot and cold fluids on either side of
a slab. Therefore thermal resistance to heat flowing through any body consists of sum of
convective and conduction resistance. Hence heat transfer is—
Q = U • A • (Th1 — Th 2)
1
where U=
1 L 1
—+—+—
h1 k h2

Th1

Th2
F.- L --]

Overall heat transfer coefficient

11. Derive an expression for overall heat transfer coefficient for a plane wall.
(UPTU — 2008 — 9)

Consider a plain wall with thermal conductivity l' and it has hot fluid with heat transfer
coefficient `1/1' at one side and cold fluid with heat transfer coefficient '122' as shown in the
figure

Hot fluid Cold fluid


(h1) (h2)

h2

Heat transfer coefficient for plane wall


60 Heat and Mass Transfer

The equivalent electrical circuit is as given below—

T
hl

1 L 1
h1 A kA h2 A

The total thermal resistance is—


1 L 1
R= + +
h1A kA h 2A
Hence heat transfer is—

Th.,— The
Q— 1 L 1
-I-
hiA kA h2
h2 A

Ax (Thi — Th2 )
1 L 1
—+ — + —
h1 k h2
= Ux A x (Thi — Th2)
1
where u=
1 L 1
—+ — + —
h1 k h2

12. Draw the analogus electric circuit of heat flow for a complex composite wall as shown
below:

10 ® ®
Q
® Q

® DO
The analogus electric circuit is—

RthB RthE
-AVVV-- _iv\ p
Q
—.-0—i\N\F-9 0-1NA/-0
RthA -JW\r- RthD

RthC RthF RthG


Steady State Heat Conduction 6I

13. What is thermal contact resistance? How can it be reduced?


(UPTU — 2008 — 9)

At the interface of two slabs, there is a discontinuity in temperature gradient due to surface
roughness and void space in between the interface of two slabs. There is a large resistance
to heat flow at the interface. This resistance is called thermal contact resistance. The drop
of temperature from T2 to T3 is due to the thermal contact resistance.

T1
T2

Q T3

T4

Temperature drop at the interface

The contact resistance is given by—


T2 — T3
Rcontact QIA

The contact resistance can be reduced by—


(a) fine surface finish of the slabs
(b) continuous contact of slabs without any void space
(c) high contact pressure
(d) similar metals.

14. The temperature distribution at certain instant of time in a plane wall of 50 cm thick,
is given by the relation—
T = 450 — 500x + 100x2 + 150x3
where temperature T is in degree Celsius and x in metres measured from the hot
surface at 450°C. The thermal conductivity of the wall material is 10 W/mK.
Calculate the rate of heat energy stored per unit area of the wall at that instant of
time.
(UPTU — 2002 — 3)

Given T = 400 — 500 x + 100 x2 + 150 x3


dT
= —500 + 200 x + 450 x2
dx
Heat entering the wall from the face being heated i.e. x = 0 is—
(dT
= —kA
d xj x=o
= —10 x 1 x (-500 + 200 x + 400 x2),=0
62 Heat and Mass Transfer

= —10(-500) = 5000 W
Heat leaving the wall i.e. at x = 0.5 m is—
(dT
Qout = —kA X
dxj x.0.5
= —10 X 1 X (-500 + 200 x + 450 x2)x=o.5
= —10 x (-500 + 100 + 112.5)
= 2875 W
Qstored = Qin — Qout
= 5000 — 2875
= 2125 W

15. The wall of a building is a composite consisting of 250 mm layer of common brick
(k = 0.72 W/mK) and 10 mm layers of Gypsum plaster (k = 0.12 W/mK) on both the
sides of the bricks. During a hot day (at steady state), the temperature of outside
plaster (exposed to ambient air) is 40°C and the temperature of inside plaster
(exposed to inside air) is 25°C. Find (i) the heat flow rate through the wall per unit
area of the wall (ii) the temperature of the interface of brick and outside plaster.
(UPTU — 2003)
Gypsum
1 12 3
Bricks
Q
40°C Gypsum

Q
25

—H 10 250

The equivalent electric circuit is—

40 25

R3 = L3
R1 = R2 =
ki A k2 A 9 k3 A
Total thermal resistance Rth

= + 4 + 4
ki A k2 A k3A
Steady State Heat Conduction 63

1 31
A ki23
+k +k
= 1 F 10 + 250 + 10 1 x
A L0.12 0.72 0.12]
=1-(83.3 + 347.2 + 83.3) x 10-3
1
= 0.514°C/W
= Ti - T4 _ 40 - 25
Now Q Rth 0.514
= 29.18 W
or Q = Q12 = Q23 = Q34
-k2 A(T2 - 40)
Q12 =
4-2
-0.12(T2 - 40
29.18 =
10 x10-3
40 = -29.18x 10-2 = 2.43
or T2 -
0.12
T2 = 40 - 2.43 = 37.57 °C.
-k3 A(T3 - T2)
Similarly, Q23 =
L23
29.18 - +0.72 x 1(37.57 - T3)
250 x 10-3

or 3757 - T3 = 02.18
9 x 250 x 10-3
.72
= 10.13
or T3 = 37.57 - 10.13
= 27.44 °C

16. One side of a plane wall is maintained at 100°C while other side is exposed to a
convection environment having T = 10°C and h = 10 W/m2K. The wall has the
dimensions of 3 x 5 m, has k = 1.6 W/mk and is 40 cm thick. Calculate heat transfer
rate through the wall.
(UPTU - 2003 - 4)
Area = A = 3 x 5 = 15 m2
aT
Q = -kA = hA(Toc - T2)
ax

or +1.6 x (ii - T 2) = 10(T2 - 10)


0. 4
64 Heat and Mass Transfer

2
h=1.0W/M k

H— 40 cm —0- T. = 10°C
T.1 = 100°C T2

10
or Ti — T2 = (T2 — 10)
4
100 — T2 = 2.5 T2 — 25
125
or T2 = = 35.7°C
3.5
1.6 x (3 x 5)(100 — 35.7)
Q=
0.4
= 3.858 kW

17. The total thickness of a furnace wall which is made of an inner layer of fire brick
covered with a layer of insulation is 32 cm. Thermal conductivities of fire brick and
the insulation are 0.84 and 0.16 W/m°C respectively. The furnace temperature is
1325°C and the temperature of surrounding is 25°C. Calculate the thickness of the
fire brick and that of the insulation for minimum heat loss through the wall. Assume
the maximum heat loss through the wall. Assume that the maximum temperature
in the insulating material should not exceed 1200°C.
(UPTU — 2004 — 5)
k= 0.84

k— 0.16
T.i = 1325°C

T.,= 25°C

T2 = 1200

1 2 3
32 cm —'I
Steady State Heat Conduction 65

The equivalent electric circuit is-


Ta

Rw Rin

Let thickness of insulation = x


Hence, thickness of wall = 0.32 - x
Now, thermal resistance of wall is-
L„, 0.32 - x
R,, = - 1.19(8.32 - x)
k„, A 0.84 x 1
Thermal resistance of insulation is

= Lin = x = 6.25 x
kin • A 0.16 x 1
Therefore, heat transfer is-
T1 - T
Q1.3 -
R„,+ Rin
1325 - 25 1300
1.19(0.32 - x) + 6.25 x 0.38 + 5.06 x
Similarly,
Tl - T2
Q12 -
R„,
1325 -1200 125
1.19(0.32 - x) 0.38 -1.19 x
Now Q1.3 = Q1.2 = Q2.3

or 1300 - 125
0.3 + 5.06 x 0.35 - 1.19 x
or 10.4 (0.35 - 1.19 x) = 0.38 + 5.06 x
or 3.64 - 12.38 x = 0.38 + 5.06 x
or 17.44 x = 3.26
or x = 0.186 m
Thickness of insulating material = 18.6 cm
Thickness of wall = 32 - 18.6 = 13.4 cm

18. A board is composed of three equal layers, middle being of packed saw dust. (k = 0.02
W/me) with the side layers made of plywood each of 2 cm thickness (k = 0.12 W/
m°C). The three layers joined together by bolting using four steel bolts of 1 cm
diameter at the corners (k = 40W/m°c). Calculate the heat flow per m2 area if one
surface is at 35°C and the other at 20°C.
(UPTU - 2004 - 5)
66 Heat and Mass Transfer

k= 0.12 k= 0.02

Bolts (4 Nos)
having k = 40
T.i = 35°C

T2 = 20°C

Equivalent electrical circuit is—

L
Rp1 = Rp2 = = 0.02 = 0.167
kp A 1.12 x 1
L 0.02 = 1
RSD =
kspx A 0.02 x 1

Area of bolt = LT x (0.01)2


4
= 0.785 x 10-4
Rb = L =
0.06
kb • Ab 40 x 0.785 x 10-4
= 19.11
Resistance of four bolts is-
1
1+
= - 1+ --E —
-E --E
Rbt Rb Rb Rb Rb

or 19.11
Rb
Rbt =
4 = 4
= 4.77
Now 2 Rp + RsD = 0.167 x 2 + 1
or R2p+SD = 1.33
Steady State Heat Conduction 67
Above is based on area = 1 m2, but actual area is in fact without bolt area.
Actual R2p±sp = 1.33 x 1
1 — 4 x 0.785 x 10 -4
1.33
1— 0.0003
1.33 — 1.33
0.9997
Now equivalent circuit is—
R 2P+SD

T1 o T2
Rbt

Rbt R2P + SD
R =
e- R2P + SD X Rbt
4.77 + 1.33
Reg' — 4.77 x 1.33
6.10
4.77 x 1.33
= 0.96
— T2 35.20
Q=
Rego — 0.96
15 = 15.625 W/m2
0.96

19. A mild steel tank of wall thickness 10 mm contains water at 90°C where atmospheric
temperature is 15°C. The thermal conductivity of mild steel is 50 W/mk and heat
transfer coefficients for the inside and outside of the tank are 2800 and 11W/m2K
respectively. Calculate:
(i) rate of heat loss per unit area of tank surface
(ii) temperature of the outside surface of the tank.
(UPTU — 2006 — 7)
Heat transfer from each material is equal,
Q12 = Q23 = Q34 = Q14 = heat flow per unit area
k (T2 — T3)
hi (T — T2) = = h2(T3 — T4)

Tl — T4
Q1-4 = 1
1
— + 10 X 10-3 +
hl k 17,2,
68 Heat and Mass Transfer

~ 10 mm-H

90 —15
1 10-2 +1
2800 + 50 11
75
3.57 x10-4 + 0.2 x10-3 + 0.09
75
(3.57 + 2 + 900) x10-4
75 x 104
905.57
= 0.08 x 104
= 800 W/m2
Now Q14 = Q34
800 = h2(T3 — 15)
= 11(1'3 — 15)
or T3 — 15 = 72.72
or T3 = 87.72°C

20. Two aluminium plates 5 mm thick with ground roughness of 2.54 mm are bolted
together with a contact pressure of 20 bar. The overall temperature difference across
the plate is 80°C. Calculate the temperature drop across the contact joint. The
contact resistance of the joint is estimated to be 0.88 x le m2 K/m and the thermal
conductivity of aluminium is 202 W/mK.
(UPTU — 2002 — 3)
Total thermal resistance per unit area is—

R= x 2+ kontact
k AL x A
5x10 3
x 2 + 0.88 x 10-4
202
Steady State Heat Conduction 69
T1 T2

AT= Ti — T2 = 80°C

AL AL

-I- 5
H 5 —I-

= 0.49 x + 0.88 x 10-4


= 1.37 x 10-4
AT 80
Q
R 1.37 x10-4
= 58.2 x 104 W/m2
Same heat (Q) will be flowing through the contact. Hence, we have—
Temperature drop across contanct (ATc)
Q-
Rcontact
••• ATc = Q X Rcontact
= 58.2 x 104 x 0.88 x 10-4
= 51.2°C

21. A conical cylinder of length with radius `r1' and `r2' (where r2 > r1) is insulated
along its outer surface. `T1' & `T2' are the temperatures maintained at two ends. If
heat flow is along its axis only, calculate heat flow through the cylinder.

r2

Consider heat flow through distance 'x' from the end of the cylinder where radius is rx
r2 —
Now rx = r +
)2
r2 —
Hence, Ax = 7r ( ri +
70 Heat and Mass Transfer

Therefore, heat flow is—


aT
Q = — kAx a x

r2 — ri
= —kir (n + x x )2 Xa T
L ax

Qa x
or = —k x iv x )T
r2— ri.
ri. -I- L X)2
C
L T2
Q • ax
or )2 = kxxf aT
0 r2 — 11 x Ti
ri. +
( L
iL
1 1
or Q = i rk (T 1 — T2)
r2 — ri x r2 — 11
ri+
L L o

1
or -Q [ r2 — n
- 1 = irk (Ti — T2)
r2 — ri
7'2 ri
L L

1 1 =
or —Q x L irk (Ti — T2)
r2 — ri r2 11
Q•L=
or 7rk (Ti — T2)
ri r2
irk (Ti — T2) ri • r2
or Q= L

22. A composite wall is made of different slabs as shown in the figure. If the heat flow
is one dimensional, find the heat transfer per unit area.

A B D

Ab = 1 m2
kb = 100 W/mk
A, = 2 m2 Ad = 2 m2
600°C 100°C
ka = 50 W/mk kr) . 10 W/mk
C

A, = 1 m2
k, = 25 W/mk

H 10 cm—'H— 10 cm —.-H— 10 cm—.1


Steady State Heat Conduction 7I
The equivalent electric circuit of the composite wall is—
Rb
--AAAr-
600 100
0—WV-0 0—AAN-0
Ra Rd
—JV\Ar—
Rc

Xa
Now Ra = = 0 .1 = 0.1 x 10-2
Aa X ka 2 x 50
4
Rb = = 0.1 = 0.1 x 10-2
Ab X kb 1X100

xc
Rc = = 0.1 = 0.4 x 10-2
cc
A xk lx 25
Xd = 0.1 = 0.5 x 10-2
Rd =
Ad xkd 2x10
Equivalent resistance (Rbc) of Rb and Rc is-
1 = 1 + 1
Rbc Rb Rc
1 1
= +
01)(10-2 0.4)(10-2
= 1000 + 250
or Rbc = = 8 x 10-4
1250
Now the electric circuit reduces to-
600 500

Ra Rbc Rd

Equivalent resistance R = Ra + Rbc + Rd


= 0.1 x 10-2 + 0.08 x 10-2 + 0.5 x 10-2
= 0.68 x 10-2
Now heat flow Q is—
T1 —T2
Q= R
= 600 —100
0.68 x 10-12

= 500 x 102
0.68
72 Heat and Mass Transfer

= 73.5 x 103 W.
= 73.5 kW

23. A plate made of material of thermal conductivity 10 W/m°C is heated from bottom
surface at constant rate such that the upper surface, which is exposed to the
surroundings, is maintained at constant temperature of 250°C. The upper surface
convects and radiates heat to the surroundings. The surroundings' temperature is
110°C. The convection coefficient and radiation factor are respectively 75 W/m2°C
and unity. Calculate the temperature gradient between upper and lower surface of
the state.
(UPTU — 2004 — 5)

QR + Qconv 7:c =110°0

F12 =1 'Ok

h = 75
_---1._--- k = 10
1' T,,,, = 250°C

t I
Heat

Heat flow through conduction is the sum of convection and radiation heat transfer
Q con = Qconv + QR
Qconv = h • A • (Tw — To j
= 75 x 1 x (250 — 110)
= 75 x 140
= 10.5 kW
QR = A • Fi2 • a(T: — TM
= 1 x 1 x 5.67 x 10-8(5224 — 3834)
= 5.67 (742.5 — 215.2)
QR = 5.67 x 527.3
= 2.99 kW
Qconv + QR = 10.5 + 2.99
= 13.49 kW
aT
Now Qcond = — kA
ax

= —10 x 1 x dT = 13.49 x 103


dx
dT
or = —1349°C/m
dx
= —1.35°C/mm
Steady State Heat Conduction 73

24. A square plate heater of surface area 200 cm2 is inserted between two slabs of same
surface area. Slab-I is 1.0 cm thick with 'lc' = 50 W/m°C and slab-II is 2.0 cm thick
with 'lc' = 100 W/m°C. If outside heat transfer coefficients on side I and II are 50
and 100 W/m2°C respectively and surroundings' temperature of air is 30°C, find
maximum temperature of the system. The rating of heater = 1 kW.

Suppose Tna, is temperature at heater.


Q = heat through slab-I + heat through slab-II
T — T_ T — T_
= +
L1 + 1 L2 1
ki k Iii Al k2A2 h2 A2

= (TLi.
x—T1 1
1X10 -2 1
+
50 x 200 x10-4 50 x200 x10-4

+ 1
2 x 10-2 1
+
100 x 200 x 10-4 100 x 200 x 10-4

T — 30) [ 1
—., + 1
—(m 1 x 10 +1 0.5 x 10-2 + 0.5

1000 = (Tmax - 30) [ 1 + 1 ]


1.01 0.505
= (Tmax - 30) (0.99 + 1.98)
Tmax - 30 = 1000- 336.7
2.97
or Tmax = 366.7°C.
25. A refrigerator having inside dimension of 0.5 m x 0.5 m and 1.0 m height is
maintained at 6°C. The walls of the refrigerator are constructed from two mild steel
74 Heat and Mass Transfer

sheets 3 mm thick (k = 46.5 W/m°C) with 50 mm glass wool insulation (k = 0.460 W/


m°C) between them. The average heat transfer coefficient at the inner and outer
surface are 11.6 and 14.5 W/m2°C respectively. The surroundings' temperature is
25°C. Find the rate at which heat must be removed from the interior, and the
temperature at outer surface of the refrigerator.
(UPTU — 2004 — 5)
kw = 0.046 W/m °C

ks = 46.5 W/m °C
T,= 25 °C
ks = 6 °C

ho = 14.5
hi = 11.6

3 m-+— 50 m —+- 3 m-d

T, = 25° TS T,= 6°C

1 Ls LW Ls 1
hoA ksA kA keA h jA

The refrigerator has four walls through which heat is coming in. Hence, area is
Area = 2(LxW+Lxh+hxW)
= 2(0.5 x 0.5 + 0.5 x 1.0 + 1 x 0.5)
= 2(0.25 + 1)
= 2.5 m2
The thermal resistance is—
1 + Ls L„, + Ls + 1
ho A ks A k„,A lc, A h1 A

1[1 2 x3x10-3 50 x10-3


+ 1
A 14.5 46.5 0.046 11.6

= [6.8 x 10-2 + 0.013 + 1.087 + 8.6 x 10-2]


2.5
= 2.1 x 1.254 = 0.5 °C/W
5
The heat transfer is-
To —
Q= ER
Steady State Heat Conduction 75

25-6 19
0.5 0.5
= 38 W
Now if Ts is surface temperature, then heat transfer is—
Q = hoA(To — Ts)
38 = 14.5 x 2.5(25 — Ts)
or 25 — Ts = 38 = 1.05
2.5 x 14.5
or Ts = 25 — 1.05 = 23.95°C

26. A wall is 30 cm thick and area of 15 sq. m is made of bricks (k = 0.35 W/m°C) having
plasters of both sides of thickness 2 cm (k = 0.6 W/m°C). The rate of heat flow is
1.8 kW and temperature at inside and outside of wall are 140°C and 40°C. A glass
window is provided for observation from outside to inside, find the area of glass
window (kg = 1 W m° C).
Brick

Plaster
Plaster

To = 20°C = 100°C

Glass window

0.02 0.02

The equivalent electric circuit is—


To = 20 = 100

= 0.02 = 2.22 x 10-3


LP
R„ =
' k p A 0.6 x 15

Rb =
4 =
0.3 = 57 x 10-3
kb • A 0.35 x 15
76 Heat and Mass Transfer

Lg 0.34 0.34
R -
g kg x Ag 1 x Ag Ag

1 1 +
1
Rtotai 2Rp + Rb Rg

1 Ag
+
2 x 2.22 x 10-3 + 57 x 10— 0.34
= 16.27 + 2.94 Ag
- To
As -Q
Krotai
100 -10
Now = 1800
Ribtai
or 90(16.27 + 2.94 x = 1800
or 1464.3 + 264.6 x Ag = 1800
or 264.6 x Ag = 335.7
or Ag = 1.27 m2

27. Find heat transfer through a composite cylinder.


Or
Derive an expression for the steady state overall heat transfer coefficient for a
composite hollow cylinder whose inner surface is exposed to hot fluid and outside
surface is exposed to a cold fluid.
(UPTU - 2009 - 10)
A composite cylinder consists of one inner hollow cylinder and one or more outer hollow
cylinders. Inner hollow cylinder has radius ri and r1, and outer hollow cylinder has radius
r1 and ro. In case, the material of the inner and 'outer cylinder has conductivity of k1 and
k2, then total thermal resistance can be found out.

T,

To

1 1
hp4, log log h0A,
ri r.
2irk1L 2jrk2L
Steady State Heat Conduction 77

Q Temperature difference
Thermal resistance

— To
Q= r r0
+log ri
1 + log rt
+ 1
hi Ai 27rk1 . 2irk2L ho Ao

28. Find heat transfer through a composite sphere.


Consider a composite sphere consisting of inner and outer hollow spheres having
conductivities of materials as k1 and k2 as shown in the figure.

To

Equivalent electrical network of the thermal system is—


To Ti

1 r0 — r1r1— 1
ho • 4 ird k2 ro n 4 g k1 ri • ri hi 4 nri2

Q Temperature difference
Thermal resistance
To — T
1 — — 1
+ + +
ho • 4nro2 4nk2ro • ri hrr r2

29. A 50 cm diameter pipeline in the Arctic carries hot oil at 30°C and it is exposed to
surroundings temperature of —20°C. A special powder insolation 50 mm thick having
a thermal conductivity of 7 x 10-3 W/mK surrounds pipe. The film heat transfer
coefficient on the outside of the pipe is 12 W/m2K. Calculate the rate of heat loss
from the pipe per meter of length, neglecting thermal resistance of the pipeline.
(UPTU — 2002 — 03)
78 Heat and Mass Transfer

ri = 25 cm

To = —20°C f ro =25 +5 =30 cm

k— 7 x le W/mk
2
h = 12 W/m k

The equivalent thermal resistance


To Ti

hAo log r°

2irkL

—T°
Q= r
log 0
I + 1
27aL hx2nro • L

30 — (-20)
log 0.3
0.25 + 1
27rx7x10— x1 12x 27rx0.3x1

27rx 50
= 26.05 + 0.28
= 11.92 W/m

30. A standard iron pipe having 5 cm inner diameter and 2.5 mm of thickness is insulated
with magnesium insulation (k = 0.02 W/m°C). Temperatures at the interface
between the pipe and the insulation is 300°C. The permissible heat loss through the
pipe is 600 W/m and the temperature of the outer surface of the insulation is not
allowed to exceed 100°C. If the thermal conductivity of the pipe material is
20 W/m°C. Calculate the minimum thickness of insulation required and the
temperature of inside surface of the pipe.
(UPTU — 2004 — 5)
= 20 W/m°C

2.5 cm
2.75 cm

k2 — 0.02 W/m°C
Steady State Heat Conduction 79
The equivalent thermal resistance circuit is—

T2 = 300°C T3 = 100°C

r2 r3
log — log —
ri r2
2jrk1 L 27dc2L

Q _ Temperature Difference
Thermal Resistance
Ti — T2 = T2 — T3
600 =
r P3
log —2 log —
r1 r2
27rk1 L 27rk2 L

T1 — 300 300 —100


or 600 =
log 2.75 =
2.5 /.3
log 2.75
27rx10 xl 27rx 0.02 xL
200 x 2 x x 0.02 x 1
or 600 =
r
log 2.75
8x
or log 2r335 600 = 0.04186

or r3 = e 0.04186 = 1.0427
2.75
or r3 = 23 .867r2

.. Thickness of the insulation = r
= 2.867 — 2.75
= 0.117 cm
T1 — 300
Now 600 =
log2.75
2.5
2 irki L

600 x log 2.75


T1 — 300 — 2.5
2n x 20 x 1
or T1 — 300 = 0454
or T1 = 300.454°C

31. For the situation below, what would happen to the average temperature at face 'C'
if thermal conductivity of the solid-II was increased?
80 Heat and Mass Transfer

Insulated Face 'B'

Solid
Face 'A II
Solid
q0 T = 20°C
Solid
Ill
Face 'C'
///////////////////
Insulated

(a) No change (b) becomes 20°C


(c) increase (d) decrease
(GRE)
The equivalent circuit is—
Rb

TB x2 Tc Ta = 20°C
To
0--/VVV-0 k
..2A2
Xi 1
ki Ai
V— hA1
x2
k3A3

1 1
ER= KL x + + 1
i iii 1 1 hili
A +
x2/ k2 ri2 X2 /k3 A3

When k2 increases, it is apparent that ER will decrease. Now


AT
Q= ER
Hence, Q will increase
Now Q is also

— T — 10
Q e1
hA1
Since Q has increased, Te will also increase.
Hence, option (c) is correct.

32. The temperature drop through a two-layer furnace wall is shown in the figure.
Assume that the external temperatures T1 and T3 are maintained constant and that
T1 > T3. If the thickness of the layers x1 and x2 are the same, which one of the
following statement is correct?
(A) k1 > k2 (b) k1 < k2 (c) k1 = k2
(GRE and GATE — 1997)
Steady State Heat Conduction 8I
Material Material
I II
k1 k2

H xi

Equivalent thermal circuit is—

x x (as x1 = x2 = x)
k1A k2A

Q. Ti — T2 . T3 — T2
x x
ki A k2 A

Q . Ti - T2 .c T3 — T2 ( T1— T T —T
but 2 = slope and 3 3 = slope
x x l x x
ki A k2 A

Slope of T1 — T2 line = Slope of T3 — T2 line


1 1
k1 k2

Slope of (T1 — T2 ) line = k2


or
Slope of (T3 — T2 ) line k1
It is apparent that the slope of (T1 — T2) line > the slope of (T3 — T2) line. Hence, k2 > ki
Hence, option (b) is correct.

33. A 20 cm dia, 1.2 m long cylinder loses heat from its periphery surface by convection.
Surface temperature is constant at 100°C and the field temperature is constant at
20°C. The average convection heat transfer coefficient over the surface of the
cylinder is 20W/m2K. The heat transfer.
(a) 120 ir W (b) 240K W
(c) 320 ir W (d) 480K W
(IES — 1989)
82 Heat and Mass Transfer

10 cm
T. = 20°C
h = 25 W/m2k
= 100°C

Heat transfer through convection at surface


_ T1 — 1: 100 — 20
Q— 1 — 1
hxA 25 x 2 x x 0.1 x 1.2
= 80 x 25 x 2ir x 0.1 x 1.2
= 480x W
Option (d) is correct.

34. A composite slab has two layers of different materials with thermal conductivity `k1'
and `k2'. If each layer has same thickness, the equivalent thermal conductivity of the
slab would be—
(a) kJ. k2 (b) k1 + k2
2 ki k2
(c) (k1 + k2) (k1 k2) (d)
ki + k2
(IES)
The slabs can be shown as below:

k1 k2 k

H x -4-- x ---I F'— 2x —d

x x 2x
ki k2 k

x x 2x
ER1 = + ER2 =
k1 k 2 k
ki + k2
= x
ki .k2
ERi =ER2
ki + k2 = 2
ki •k2 k
Steady State Heat Conduction 83

2 k1• k2
or k—
ki+ k2
Option (d) is correct.

35. A hollow stainless steel spherical container having 20 cm inside diameter and 30 cm
outer diameter (with k = 50 W/mK) holds certain liquid. Its inner and outer surfaces
are at 250 and 100°C. Find radial heat flow and temperature at r = 12.5 cm from the
centre.
Heat transfer is—
T — To
Q- ro — ri
4gro ri x
250 —100
0.15 — 0.10
47r x 0.15x 0.10 x 50
= 28.26 kW

r, - 10 cm
r,,,= 15 cm
r - 12.5 cm

To = 100°C

Now
T (r — ri )
— fro — ri)
250
T— 0.15 (0.125 — 0.10)
100 — 250 0.125 (0.15 — 0.10)

T — 250 = —150 x 0.15 x 0.025


0.125 x 0.05
= 250 — 90
= 160°C

36. A hollow cylinder has inside radius 2.5 cm and outside radius 5 cm. Inside
temperature is 300°C and outside temperature is 110°C. Find the temperature at
84 Heat and Mass Transfer

3.75 cm from centre if k = 70 W/mk. Also find heat flow through the cylinder per
length.
Heat transfer is—
T —To
Q=
ivb —

2irkL
300 —100

27r x 70 x 1
= 126.8 kW/m
Temperature distribution is—
r
T—T
To — 1,15

3.75
T —300
100 — 300
"'5 2.5
3.75
or T — 300 = —200 x
"'5 2.5
= —117
or T = 300 — 117
= 183°C

37. A hollow sphere is made up of two materials. First one with k1 = 70 W/mk with ID
= 10 cm and OD = 30 cm. The second sphere has k2 = 15 W/mk and with OD = 40
cm. Inside temperature is 300°C and outside temperature is 30°C. Find the rate of
heat flow through the sphere.
Steady State Heat Conduction 85

The equivalent thermal circuit is—

= 300°C T3 = 30°C
r2 — r3 — r2
47rk1rir2 4,rk2r2r3

Tl — T3
Q= r2 — r3 — r2
47rkirir2 41rk2r2 r3

300 — 30
0.15 — 0.05 0.2 — 0.15
+
47r x 70 x 0.15 x 0.05 47r x 15 x 0.2 x 0.15
= 11.24 kW

38. What do you understand from critical thickness of insulation?


Or
Derive an expression for critical radius of insulation for cylindrical body. Give
practical example to explain the concept of critical radius.
(UPTU — 2008 — 9)

In case of plane walls, the heat transfer rate decreases as thickness of the wall increases
since thermal resistance R is equal to the thickness of the wall divided by kA i.e. thermal
resistance keeps on increasing. This is not true always for pipes and spheres with the
addition of insulation as in their cases, thermal resistance firstly decreases and after certain
extent of addition only thermal resistance increases. The thermal resistance of pipes and
spheres therefore firstly decreases with the addition of insulation and heat transfer reaches
to a maximum value at certain value of insulation thickness. The outer radius with insulation
when heat transfer is maximum is called critical radius of insulation. If further insulation is
added after the critical radius the thermal resistance increases resulting in the decrease of
heat transfer. The adding of insulation after critical radius is undesirable as heat transfer
from electric wire to surroundings will decrease, resulting in heating and damaging of wire.
86 Heat and Mass Transfer

Consider a cylinder of length with inner radius `r;' and outer radius with `le as
thermal conductivity. The thermal resistance consists of conductive and convective
resistance.

log L.
ER — +
1
2 xkL h • (27r r • L)
Now as insulation is increased, the radius increases which increases conductive
resistance but it decreases convective resistance. Initially the net result is the decrease of
total thermal resistance with the addition of insulation as the rate of increase of conductive
resistance is less than the decrease of convective resistance due to larger surface area. After
reaching certain radius rc, the increase of conductive resistance is higher than the decrease
of convective resistance. This radius is called critical radius and net result is the decrease of
total thermal resistance and the increase of heat transfer when insulation is added beyond rc.

n rc Radius of insulation

39. Where is this principle of the critical thickness of insulation used?

The principle of the critical thickness of insulation is used in electrical industry as we want
to provide such a thickness of insulation to wires and cables carrying current which permits
an increase in the heat transfer from these currents carrying conductors but at the same time
insures adequate insulation. The temperature of the conductors does not increase with
increased heat transfer when provided with critical thickness of insulation and they remain
safe otherwise the insulation of the conductors will burst whenever heat transfer is less and
temperature rises in the conductors.

40. Derive an expression for the critical radius of insulation for a cylinder.
(UPTU —2003)
Steady State Heat Conduction 87

Consider a cylinder with radius 'ri' and it has insulation upto radius 'r' as shown in the
figure. The thermal conductivity is `le and heat transfer coefficient is 'h'.
The equivalent electric circuit of the heat transfer is—

log f: 1
h(2nrL)
2irkL

Total thermal resistance is

log r
ER — + 1
DrkL h(27rrL)

Differentiate ER and put


asR = 0 in order to find minimum total resistance.
a
ri 1
a ER r 1 o
ar 2irkL h(2irrL)r2
k
or r=

This radius is called critical radius of insulation re.


k
— h

41. Derive an expression for critical radius of insulation for a sphere.


(UPTU — 2007 — 8)

Consider a sphere with radius `ri, and outside radius 'r' with insulation. The thermal
conductivity is `le and heat transfer coefficient is 'h' with the surroundings. The equivalent
electrical circuit for the heat transfer is—
88 Heat and Mass Transfer

r — ri 1
4irkr x h(47u)

The total thermal resistance is-


r— 1
ER =
47rk rxri h(47rr2 )
azR
= 0 for minimum thermal resistance
r
azR _ +ri 2 0
ar 4gkr2 xri 4ghr3
2k
or rc =
h

42. It is desired to increase the heat dissipation rate over the surface of an electronic
device of spherical shape of 5 mm radius exposed to convection surroundings with
h = 10 W/m2K by encasing it in a spherical sheath of conductivity 0.04 W/mK. For
maximum heat flow, the diameter of the sheath should be
(a) 18 mm (b) 16 mm
(c) 12 mm (d) 8 mm
(IES — 1996)
For a sphere, critical insulation is—
2k
rc =
h
2 x 0.04
10
= 0.008 m
= 8 mm
Option (d) is correct.

43. A cable of 10 mm diameter is insulated with a material for which k = 0.2 W/mk and
it is exposed to surroundings at 20°C. Find the critical insulation if heat transfer
coefficient is 9.3 W/m2K.

r
_k
_
c h
0.2
9.3
= 0.021 m
= 21 mm
Steady State Heat Conduction 89

44. A pipe covered with insulating material has inner diameter 0.1 m and outer diameter
0.11 m. Thermal conductivity of insulating material is 1.0W/mK. The temperature of
the fluid inside the pipe is 100°C and surroundings temperature is 20°C. Find the
critical radius of insulation and heat loss per metre length of pipe for critical radius
of insulation. Take 'h' = 8W Im2K.

The critical radius is—


1
r =—
k = — = 0.125 m
h 8
Equivalent thermal circuit is—

log 1
h(2nr,L)
2rckL

Q= rc
log
ri 1
+
27rkL 27rrc hL

100 — 20
log 0.125
0.05 + 1
27rx1x1 27rx 0.125x8x1
= 620 W/m

45. Two insulating materials of thermal conductivity 'Ic' and '2 k' are available for
lagging a pipe carrying a hot fluid. If the radial thickness of each material is the
same, then—
(a) Material with higher thermal conductivity should be used for the inner layer and
one with lower thermal conductivity for the outer.
(b) Materials with lower thermal conductivity should be used for the inner layer and
one with high thermal conductivity for the outer.
(c) It is immaterial in which sequence the insulating materials are used.
(d) It is not possible to judge unless numerical values of materials are given.
(GATE — 1994)
The critical radius—

(a) first material (rci) =


2k
(b) second material (rc2) = h
90 Heat and Mass Transfer

It is apparent that rc2 = 2rci


Hence, the first material is to be used inside and second material is to be used for
outside as it has bigger critical radius.
Option (b) is correct.

46. An electric wire of 2 mm diameter is given a 25 mm thick insulation (k = 0.5 W/m°C).


The surroundings has temperature of 25°C and heat transfer coefficient h = 10 W/
m2°C. The wire has surface temperature of 120°C. Compare the heat dissipation with
and without insulation. Find the thickness of insulation when heat dissipation is
maximum and find its value.

The equivalent electric circuit is—


Ti

log 1
J_ h(2772L)
27d<L

Ti —
Qin — r2
log
ri 1
Drki, h(Drr2 L)
27r xl x (120 — 25)
log 3.5
1 + 1
0.5 10 x 3.5 x 10-3
= 19.2 W/m
Heat dissipation from bare wire is—
Qb = h x A(Ti — Toj
= 10 x (2,r x 1 x 10-3 x 1) (120 — 25)
= x 95 x 10°2
= 5.96 W/m
Qin
= 19.2 = 3.22
as 5.96
Steady State Heat Conduction 9I

Now critical radius is—

r =
= = 0.05 = 50 mm
k 10
Hence, critical thickness of insulation = r, — ri
= 50 — 1 = 49 mm
Now maximum heat flow with critical radius of insulation is-
27r L(Ti —
Qmax 1
log k+
h x r,

2 x (120 — 25)
50
log
1 + 1
0.5 10 x 50 x10-3

27r x 95
7.82 + 2
= 60.75 W/m

47. A hollow sphere of inner radius of 30 mm and outer radius of 50 mm is electrically


heated at inner surface with constant rate of 100 kW/m2. The outer surface is
exposed to surroundings at 30°C. If h = 170 W/m2K and k = 20 W/mK, find
temperature at inner and outer surface of the sphere.

The equivalent electric circuit is—

Ti Ta = 30°

Q ra —
4nkra r, h(4 02 )

Q = 100 kW/m2

Ti —
Q=q x ri2 =
ro — + 1
4irk ro ri h(4 ro2)
92 Heat and Mass Transfer

T1 -30
100 x 103 x 4ir x (30 x 10-3) =
50 -30 1
+
47r • 20.50.30.10—3 170[4x x(50 x10-3)2 ]
1; -30
(0.053 + 0.137) x 103
- 30 = 100 x 103 x 4ir x 9 x x 0.24 x 103
= 271.2
= 301.2°C
Also we can write for convective part of heat transfer as-
Q = hA(T2 - To0
= 170 x 4ir x (50 x 10-3)2 (T2 - 30)
30 = 100 x 103 x 47rri2
or T2 —
170x 47rx 25x10-4
T2 = 211.7 + 30 = 241.7°C

48. An electric wire of diameter 10 mm and a resistance per unit length of 10 W/m is
coated with the insulation of k = 0.2 W/mK. The surrounding air temperature is 27°C
and h = 10 W/m2K. If the insulation has a maximum allowable temperature of 177°C,
what is maximum possible current that may be passed in the wire?
(UPTU - 2005 - 6)

Equivalent electric circuit is-


T;

log 1
ri h(2nr2L)
2,rkL

Now for maximum heat transfer has to be critical radius i.e. r2 = r,


r°= k = 0.20 = 0.02 m
h 10
= 20 mm
Steady State Heat Conduction 93

Now
Ti — 7:
Q— r
log ;
+ 1
27rkL h(27rr,L)
177 — 27
QIL =
log 20
+ 1
27r x 0.2 10(27r x 20 x10-3)
12R = 150 = 150
1.104 + 3.18 4.284
= 35
Now /2 x 10 = 35
or / 2 = 35
or I = 1.87 ampere

49. For a plane wall subjected to uniform heat generation and exposed to a fluid at the
same temperature on both sides, derive an expression for temperature distribution
within the wall and prove that the heat generated is equal to the heat loss from the
sides. Assume one-dimensional case.
(UPTU — 2005 — 6, 2008 — 9)

x
x = —L/2 x = 0 x = L/2

One-dimensional heat conduction equation is—


a2 q

+ g = 0 where qg = heat generation


a X2 k
Integrating, we get—
aT qg
+ x =Ci
ax k

Now x=0, a x = 0
ax
94 Heat and Mass Transfer

CI = 0
Integrating again, we get—
,„2
g A,
T+ • =C2
k 2
Now x = LI2, T = Too
qgxL2
C2 = To. +
k •8

g qg L2
T+ • =T +
k 2 8k

qg L2 2k 2
T — Too =
8k L

In order to find maximum temperature, put x = 0 and equation reduces to—


L2
T — Too — ga
8k
Heat transfer is—
OT
= —kA ax

= —kAi qgx1
k
or Q = Aqgx
Heat conducted at x = L/2 is given by
Q = Aqg • L/2
This heat is convected to the surroundings is same as heat conducted. Hence—
Q = A qg • LI2 = hA(T, — Too)
where TS = surface temperatures
qg x L
or
= + 2h
or T =Ts —gg xL/2h
Now putting the value of Too in general equation of temperature, we get—

qg L2 [ (2121
T=Ts + 1— —
8k L
Steady State Heat Conduction 95

L2
q
, = 9
'max 8k
Ts s

L/2 x=0 L/2

50. Derive an expression of temperature for plane wall having uniform heat generation
and exposed to fluid at one surface and other side surface is insulated. Find
maximum temperature and temperature of surface.

Ta

x=0 x=L

One-dimensional heat conduction equation is-


2 T qg —
0
axe + k —
On integration, we get—
aT qg
+ x =c1
ax k
aT
Now at x =o' =o ... cl =o
ax
aT qg
•x
ax = — k

raT qg
=— •L
0 xj x=L k
Now
aT
Q = —kA j = hA(T, — Toc)
fl ax x=L

qg
or x L x kA = hA (7; — To.)
k
96 Heat and Mass Transfer

= ÷q L
or Ts T
g:
To find temperature distribution, we integrate the equation (i) and get—
2
—qg x
T= +C
k 2
Now x = L, T = T,

Ts=
- qg x e +C
k 2

qg x L2
or
C = Ts ÷ 2 x k

g x x2 qg x L2
... T — T, = —q +
2k 2k
If we get Ts value from equation (ii), the equation becomes—
qg • L qg
T — Tc., = + 11,2 _ x2)
h 2k '
Maximum temperature will be x = 0 and above equation reduces to—
qg L qg 2
Tm= Tc., + + xL
h 2k

51. If qg = 1.5 x 105 W/m3, L = 0.1 m 7',. = 100°C, h= 500 W/m2K, k = 15 W/mK and one
side of the slab is insulated, find, (1) maximum temperature and its location: (2)
temperature at the exposed surface of the slab.

Ta = 100

Maximum temperature will be at x = 0

Tmax = T_ + qg qg • L2
—x L+ —
h 2k

10x10-2
= 100 + 1.5 x 105 x 10 x 10-2 1 +
550 2 x 15
Steady State Heat Conduction 97
= 100 + 1.5 x 104
= 100 + 1.5 x 104(1.818 x le + 3.333 x 10-3)
= 100 + 77.23
= 177.23
Now at x=L,T=T,
T, — Tc: = qg •• L
h
x 105
or T, = 100 + x 0.1
1.5500
= 100 + 30
= 130°C

52. A plane wall of length '2 L' has sides maintained at 'Ts'. The left half of the wall has
uniform heat generation = qg W/m3 while the right half has no generation. Assuming
one-dimensional heat conduction, find the expression for temperature distribution in
the left and right region. Find also the location of maximum temperature.

F.- L -.I.- L

One-dimensional heat conduction equation with heat generation qg is-


2 qg
,+ =o
a x` k
On integration, we get—
aT qg
+—x = cl
ax k
On integrating again, we Bet—
x 2z
T+ = Cix + C2
2k
Now at x = 0, T = Ts ... C2 = Ts
qg x 2
T= + Ci x + Ts (i)
2k
98 Heat and Mass Transfer

Heat conduction equation for right portion with qg = 0 iS—


a2 T
=o
a .X 2
On double integration, we get—
T = C3 x + C4
Now x= 2L, T = Ts
... Ts = C3 • 2L + Cet
... Cet = Ts — C3. 2L

Putting the value of C4 in equation (ii)


T = C3x + Ts — C3 • 2L
= C3(x — 2L) + Ts
The temperature at x = L will be same for equations (i) and (iii)

( — qo x2
+ CIX-F Ts )
2k = (C3(x — 2L) + Ts)x=L
x= L

—qo L2
+ CIL + Ts = —C3L + Ts
2k

qo L
or CI + C3 =
2k
Also at x = L1 heat flow from left will be equal heat flow out from right
( kA aT) _ ( kA aT )
ax)x=L — ax) x=L

qg • L
+ CI = C3
k

qg • L
or C1 — C3 =
k
From equations (iv) and (v), we get-
3 qg L —qg L
Cl = and C3 =
4k 4k
2 3 qg L
—qg X
Tleft = + + Ts
2k 4k

—qg L
Tight = 4k (x — 2L) + Ts
Steady State Heat Conduction 99

Maximum temperature will exist in left portion, where-


lieft — qgx + 3qg' L
ax k 4k

or x = 4
3

53. Three rods of material 'A' and three rods of material '13' are connected as shown in
the figure. All rods have same length and cross-sectional areas. If end 1 and 5 are
maintained at 100 and 20°C, find the temperature at junction 2. Given k A = 800 W/
m°C and kB = 410 W/m°C.
3

T1 = 100 = 20°C

The equivalent electric circuit is

T1 = 100

RA = =C = 0.125 x 10-2 x C where C =


- KA • A 800 A
RR = C = 0.25 x 10-2 x C
KB A 400
RA+ RB 3RA RB
ER = RA +
2 2
3 x 0.25 x 10-2 x C + 0.25 x 10-2 C
2
= 0.5 x 10-2 x C
i 00 Heat and Mass Transfer

Ti — T5
Q1-5 — ER

_ 100 — 20
0.5 x 10-2 x C

_ 160 x 102 W
C
Now heat flow from 1 to 2

Q1-5 = Q12 =
Ti - T2
..ii
1.6x104 = 100— T2
C 0.125 X10-2 X C
or 100 — T2 = 1.6 x 104 x 0.125 x 10-2.
= 20
or T2 = 80°C

54. A rod 'AB' of thermal resistance 10 k/W is connected to another 'CD' rod of same
material where 'IV is the mid-point of rod 'CD' as shown in the figure. Temperatures
are as shown at the ends. Find the heat flow in rod AB.

C D

Tc = 80°C B TD = 0°C

TA = 20°C A

The equivalent electric circuit is—

Q BC
X..

D TD = 0°C
RCB
TD = 80 B
C —,-- RBA
QCB A TA = 20°C

QBA

RCD
RCD = 2 = = 2.5
2 W
Steady State Heat Conduction I 0 i

c k
RBA = RCD = 0
w
RCD
RBD = 2 — 2.5
Let temperature at junction B is TB
Tc — TB
. Heat flow QCB
— Rcy
=
80 — TB
2.5
TB — TD
QBD
RBD
_ TB - 0 _ TB
2.5 2.5
TB —TA
QBA -
RBA

= TB- 20
5
Now QCB = QBD + QBA
80 — TB = TB + TB — 20
2.5 2.5 5
TB
or 80 — TB = TB + —10
2
or —5 TB = 90
2
90 x
or TB = 2 = 36°C
5
Now TB — TA
QBA -
RBA
= 36 — 20
5
= 16 = 3.2 W
5

55. A steel pipe 100 mm bore and 7 mm wall thickness carrying steam at 260°C, is
insulated with 40 mm of glass wool covering. This covering is in turn insulated with
60 mm of asbestos felt. The atmospheric temperature is 30°C. The heat transfer
coefficient for the inside and outside surfaces are 550 and 15 W/m2K respectively and
I 02 Heat and Mass Transfer

the thermal conductivities of steel, glass, wool and asbestos felt are 50, 0.09, and 0.07
W/mk respectively. Calculate:
(i) Rate of heat loss per unit length of pipe
(ii) Temperature at each x section of the pipe
(UPTU — 2006 — 7)

= 50 mm
r2 = 57 mm
r3 = 97 mm
r4 = 157 mm

Equivalent electric circuit is


T5 = 260°C T1 T2 T3 T4 Ta = 30

1 r2 r3 r4 1
log r1 log log 6
hi(27711) r2 h2(2274•L)
2irksL 2/dcgwl 2rrkab•L

Q 2
log r log ' log
1 r, r, 1
+ + +
h1(27rr1 L) Drks L Drkg„,• L 2ir kab L h2(27r r4 • L)

(260 — 20)2 ir • 1
3 log 57 log 97 log 157
10
+ 50 57 97 + 103
550 x 50 50 0.09 0.07 15 x 157
240x2xiv
0.4 + 0.003 + 5.9 + 23.12 + 0.425
1507.2
29.85
= 50.5 W/m
— T1
Q- '1
hi (2 ir ri • L)

50.5 = 260 —
0.064
Steady State Heat Conduction I 03

or Tl= 260 — 50.5 x 0.064


= 260 — 3.23
= 256.77 °C
Now we have—
Tl —T2 256.77 — T2
Q=
log 27rksL 0.47 x10-4
rl
or 256.77 — T2 = 50.5 x 0.47 x 10-4
= 0.0035
or T2 = 256.76°C
Now we have for next junction—
T, - T3
Q=
log
r2
27V kgr,XL
50.5 — 256.76 — T3
0.94
or T3 = 256.76 — 50.5 x 0.94
= 256.76 — 47.47
= 209.29°C
Now we have for next junction—
T3 -T4
Q=
log r4
r3
27Ckab •L

50.5 — 209.29 — T4
3.68
or T4 = 209.29 — 50.5 x 3.68
= 209.29 — 185.91
= 233.8°C

56. Given the following data : Inside heat transfer coefficient = 25 W/m2°C, outside heat
transfer coefficient = 2.5 W/m2°C, thermal conductivity of bricks (15 cm thick) =
0.15 W/mK. The overall heat transfer coefficient (in W/m2K) will be closer to the:
(a) inverse of heat transfer coefficient
(b) heat transfer coefficient
I 04 Heat and Mass Transfer

(c) thermal conductivity of bricks


(d) heat transfer coefficient based on the thermal conductivity of the bricks alone.
(IES — 1993)

h1 = 25 k= 0.15 h2 = 25

—15 cm—d

Equivalent electric circuit is-

1 L
hiA kA h2A

1 L 1 2=0.15
ER = + + + for unit area
hi A kA h2 A h k
1
U= 1 —
ER 2 0.15
+
h k
__ 1 __1
2 + 0.15 0.08 + 1
25 0.15
= 1 = 0.925
1.08
kA 0.15 x 1
Ubrick = L = 0.15 — 1
Hence, option (d) is correct.

57. A stainless steel tube (k, = 19 W/mK) of 2 cm ID and 5 cm OD is insulated with 3 cm


thick asbestos (ka = 0.2 W/mK). If the temperature difference between the
innermost and outermost surfaces is 600°C, the heat transfer rate per unit length is—
(a) 0.94 W/m (b) 9.44 W/m
(c) 944.72 W/m (d) 944.7210 W/m
(GATE — 2004)

Total thermal resistance is

log 1.2 r3
log
r1 r2
XR = +
2irk1 L 2ir k2 L
Steady State Heat Conduction I 05

log 2.5 l0 5.5


_ 1 +
27rx19x1 2irx0.2x1
= 0.0077 + 0.627
= 0.635
AT
Q. __ 600
ER 0.635
= 944.88 W/m
Option (d) is correct.

58. Heat is being transferred from water at 48°C to a glass plate whose surface that is
exposed to the water is at 40°C. The thermal conductivity of water is 0.6 W/m and
the thermal conductivity of glass is 1.2 W/mK. The spatial gradient of temperature
in the water at water-glass interface is dTIdy = 1 x 104 k/m. Select option in question
(i) and (ii) as given below:

48°C

Water
40°C

Glass

(i) The value of the temperature gradient in the glass at the water-glass interface
in k/m is-
(a) —2 x 104 (b) 0.0
(c) 0.5 x 104 (d) 2 x 104
(ii) The heat transfer coefficient h in WIm2K is—
(c) 0.0 (f) 4.8
(g) 6 (h) 750
Question (i): At water-glass interface, we have—
(c/ T _ k dT)
q = -kw
d y) w— g(dY g
(dT) kw (dT
dy g kg dyj w

= x 1 x 104
1.2
= 0.5 x 104 k/m
Option (c) is correct—
I 06 Heat and Mass Transfer

Question (ii):
(dT
q = lcw
dyj w
= 0.6 x 1 x 104 = 6000 W/m
Now q =h(T,,—Tg)
6 x 103 = h(48 — 40)
6 x 103
h—
8
= 750 W/m2K
Option (h) is correct.

59. A composite wall, having unit length normal to the plane of paper, is insulated at the
top and bottom as shown in the figure. It comprises four different materials A, B,
C and D
Insulated

—4—

HA = H

hi T2, h2
Li-a1-4— L2 —+- L3-41

The dimensions are HA = HD = 3 cm, HB = He = 1.5 m


L1 = L3 = 0.05 m, L2 = 0.1 m
KA = KD = 50 W/mK, KB = 10 W/mK, Kc = 1 W/mK
The temperature T1 = 200°C, T2 = 25°C and heat-transfer coefficients h1 = 50 W/
m2K, h2 = 10 W/m2K. Assuming one-dimensional conduction:
(a) Sketch the thermal circuit and
(b) Determine the heat transfer through the wall.
(GATE — 2001)
The equivalent thermal circuit is—

R1 -- R 2 -- kAL —MA,— R R
hlAA AA 5 = kDAD 6 h2AD
2
R4 = kDAD
Steady State Heat Conduction I 07
AA = AD = 3 m2 and AB = Ac = 1.5 m2
R1 =
1 1 = 6.67 x 10-3 deg/W
hi i1A 50 x 3
L1
R2 = = 0.05 = 3.34 x le deg/W
k A AA 50 x 3
L2 0.1 = 6.67 x Hp deg/W
R3 = =
k B AB 10x1.5

L2 0.1 = 66.67 x 10-3 deg/W


R4 = =
kc Ac lx1.5

R5 = 4 = 0.05
= 3.34 x le deg/W
kD AD 50 x 3
1 = 1 = 33.34 x le deg/W
R6 =
h2D
A 10 x 3

ER = Ri + R2 ÷ R +X RR: + it5
” + itn6
3 :
667 x 66.67
= 6.67 x 10-3 + 3.34 x 10-4 + ' x 10-3
6.67 x 66.67
+ 3.34 x le + 33.34 x 10-3
= 46.73 x 10-3 deg/W
AT = 200 - 25
Q=
ER 46.73 x 10-3
= 3744.91 W

60. A steel steam pipe 10 cm inner diameter and 11 cm outer diameter is covered with
an insulation having a thermal conductivity of 1 W/mk. If the convective heat
transfer coefficient between the surface of insulation and the surrounding air is 8 W/
m2K, the critical radius of insulation is
(a) 10 cm (b) 11 cm
(c) 12.5 cm (d) 15 cm
(GATE - 2000)
k
Critical radius =
h
=1
8
= 0.125 m
= 12.5 m
Option (d) is correct.
I 08 Heat and Mass Transfer

61. For a current wire of 20 mm diameter exposed to air (h = 20 W/m2K), maximum heat
dissipation occurs when thickness of insulation (k = 0.5 W/mK) is
(a) 20 mm (b) 25 mm
(c) 30 mm (d) 10 mm
(GATE)
Maximum heat transfer takes place at critical radius

r — k — 0.5
c — h — 20
= 0.025 m
= 25 mm
Option (b) is correct.

62. For a current carrying wire of 20 mm diameter exposed to air (h = 25 W/m2K)


maximum heat distribution occurs when the thickness of insulation (k = 0.5 W/mK)
(a) 20 mm (b) 10 mm
(c) 2.5 mm (d) 0 mm
(GATE — 1993)
The critical radius is
k 0.5
R = = 25 = 20 mm
7,
Thickness of insulation = 20 — 10 = 10 mm
Option (b) is correct.

63. The wall of a building is a composite consisting of 250 mm layer of common brick
(k = 0.72 W/mK) and 10 mm layers of gypsum plaster (k = 0.12 W/mK) on both the
sides of the brick. During a hot day (at steady state) the temperature of outside
plaster (exposed to ambient air) is 40°C and the temperature of inside plaster
(exposed to inside air) is 25°C. Find (i) the heat flow rate through the wall (ii) the
temperature of the interface of brick and outside plaster.
(UPTU — 2007 — 8)

T.1 = 25°C
Steady State Heat Conduction I 09

The equivalent electrical circuit is—


= 25 T4 = 40°C

L1 L2 L1
k1 k2 k1

ER = + + = 2 + L2
k1 k2 k1 k1 k2
2x x 10-3 250 x 10-3
+
0.12 0.72
= 16.6 x 10-2 + 34.72 x 10-2
= 51.32 x 10-2
q = T4 — Tl = 40 — 25 = 29.23 W/m2
ER 51.32 x 10-2
Now q 12 = q23 = q34. = q
Take q34 = q
T4 — T3 =q
L1
k1
40 — T3
10 X 10-3
— 29.233
0.12

or 40 — T3 = 29.23 x 10-2
0.12
= 2.44
or T3 = 40 — 2.44
= 37.56°C

64. Prove when heat transfer (Q) from bare and insulated wire of radius ri and ro is equal,
then
k ( r)
log = 1
hr ro

Heat transfer Qi from bare wire is—


Qi = (27r ri x L) h(To — To,)
1 1 0 Heat and Mass Transfer

Heat transfer Q2 from insulated wire is—

(T0 — T)
Q2 = (2r x L) x
log r0
1 r
ho ro k
Now Q1 = Q2

27 r L (To — L)
2ir Lh(To — =
it,
log—
1 r
+ '
hro k
2KL(To —L) = 27 r L
1 ro
hr log
1 li
+
hro k
log r0
or 1 . 1 + r
hri hro k
ro k [1 11
or log =
ri h ri ro

k rl
1--
h ri

65. The exterior wall of a house many be approximated by a 0.1 m layer of common brick
(k = 0.7 W/mK) followed by a 0.04 m layer of gypsum plaster (k = 0.48 W/mK). What
is the thickness of insulation to be added to reduce the heat loss/gain through the
wall by 80% (k, = 0.065 W/mK).
(Annamalai University — 1993 — 94)

Common Gypsum Insulation


brick plaster

Q2 Q2

L2 Ln

Rt R2 R3

Heat flow with and without insulation


Steady State Heat Conduction

Qi is heat flow without insulation-


A • dT A•dT
Qi = T = 0.1 0.04
L1 + L 2
—+
ki k2 0.7 0.48
Q2 is heat flow with insulation-

AdT A•dT
Q2= =
Li L2 L3 0.1 0.04 L3
+ +— + +
k1 k2 k3 0.7 0.48 0.065
Q2 = 0.2 Qi
A•dT A•dT
= 0.2 x
0.1 0.04 L3 0.1 0.04
—++ —+
0.7 0.48 0.065 0.7 0.48
L3 = 0.856 m

66. A boiler surface has effective dimensions of 2 x 3 x 3 m (1, b and h). The walls are
constructed with an inner fire brick wall of 25 cm thick (k = 0.4 W/mK), a layer of
ceramic blanket insulation of 8 mm thick (k = 0.2 W/mk) and a steel protective layer
of 2 mm thick (k = 54 W/mK). The temperature of fire brick layer is 600°C and that
of outside insulation is 60°C. Determine: (i) rate of heat loss through the vertical
wall of the furnace, and (ii) estimate the temperature drop across the steel layer.
(Annamalai University - 2003 - 4)

0
Fire brick Ceramic Steel
Ti T2 T3 T4

L2

Boiler walls

Area 'A' = 2(2 x 3 + 2 x 3 + 3 x 3) = 42 m2


L1 0.25
R1 = = = 1.491 x 10-2 1c/W
k1A 0.4 x 42
L2 = 0.08
R2 = = 0.95 x 10-2 1c/W
k2 A 0.2x42
II2 Heat and Mass Transfer

L3 0.002 =
R3 = = 0.000088 x 10-2 kW
k3 A 54 x 42
Ti — T4
Q=
R1+ R2 + R3
600 — 60
(1.49 + 0.95 + 0.000088) x 10-2

540 x 102
=
2.44
= 22.1 kW
The temperature drop across steel layer
T3 — T4
Q=
R3
T3 — Tet = Q X R3
= 22.1 x 103 x 0.000088 x 10-2
= 0.0194 K

67. A furnace wall is made of inside silica brick (k = 1.86 W/mK) of thickness 10 cm and
outside magnesite brick (k = 5.86 W/mK) of thickness 20 cm. The temperature at
inside surface of the silica brick is 900°C and outside magnesite brick surface
temperature is 150°C. Calculate: (i) heat flow through composite wall, (ii) if contact
resistance between wall is 0.00257 m2 K/W. Also calculate temperature drop in the
interface.
(Annamalai University — 1999 — 2000)

R3 T4

Furnace Wall
Steady State Heat Conduction 1 13

0. 1
R 1 = L1 = = 0.0538 K/W
k1A 1.86 x 1

L2 0.2
R2 = A = = 0.0341 K/W
k2 Pi 5.86 x 1
R, = 0.00257 K/W
ER = R1 + R2 + R,
= 0.0538 + 0.0341 + 0.00257
= 0.0.905 K/W
T1 — T4 900 —150
Q= =
ER 0.0905
= 8290 W
Now temperature drop at contact—

T2 — T3 = R, x Q
A
= 0.00257 x 8290
= 21.3°C

68. A furnace wall is made up of three layers of thickness 250 mm, 100 mm and 150 mm
with thermal conductivity of 1.65, k2 and 9.2 W/m°C respectively. The inside is
exposed to gases at 1250°C with a convection coefficient of 25 W/m2°C and inside
surface is at 1100°C, the outside surface is exposed to air at 250°C with convection
coefficient of 12 W/m2°C. Determine: (i) the unknown thermal conductivity (k2),
(ii) the overall heat transfer coefficient (U), and (iii) all surfaces temperatures.
(Annamalai University — 2005 — 6)

Th
1

RR1
R2 R3

Furnace wall
II4 Heat and Mass Transfer

Now Th1 = 1250°C and T1 = 1110°C


Q = hfl x A x (Th i — T1)
= 25 x 1 x (1250 — 1100)
= 3500 W
Now Th2 = 25° and Th1 = 1250°C
Q = U • A • (Thi — Th2)
3500 = U•/. (1250 — 25)
3500
or U= = 2.86 W/m2°C
1225
Now overall heat transfer coefficient (u) is—
1
U—
1 Li L2 L3 1
-F-F— + +-
1111 k 1 k2 k3 1111

1
2.86 =
1 0.25 0.1 0.15 1
—++—++-
25 1.65 k2 9.2 12
or k 2 = 1.7 W/m2°C
Now have
ki
Q= A (Ti — T2)
L1

3500 = 1.65x 1 (1110 — T2)


0.25
or T2 = 580°C

Also Q = k 2 x A(T2 — T3)


L2

or 3500 = 1.7 x 1 (580 — T3)


0.1
or T 3 = 373.5°C

Also Q= (T3 — T4)


L,3
9.3
3500 = (373.5 — T4)
0.15
or T4 = 316.4°C
Steady State Heat Conduction 1 15
69. A cold storage system has walls made of 220 mm of brick on the outside, 90 mm of
plastic foam and finally 60 mm of wood on the inside. The outside and inside
temperatures are 25°C and-3°C. The inside heat transfer coefficient is 11 W/m2K and
the outside heat transfer coefficient is 30 W/m2K. The thermal conduction of brick,
foam and wood are 0.99, 0.022 and 0.17 W/mK respectively. Determine: (i) the rate
of heat-removed if total wall area is 85 m2, and (ii) the temperature at the inside
surface area of brick.
(Annamalai University — 2005 — 6)

25°C —3°C

L1 L2 L3H

Th o—M-0
i L1 L2 L3 Th2
1
h1 k1 k2 k3 h2

1 1 Li L2 L3 1
= + + + +
U hi ki k2 k3 h2

= 1 + 0.22 + 0.09 + 0.06 + 1


11 0.99 0.022 0.17 30
or U = 0.22 W/m2°C
Now Q = UA(Thi — Th2)
= 0.22 x 85 x (25 + 3)
= 525.3 W
1
Now Q= x A (Th 1 —Th2)
Rh1+ R1
II6 Heat and Mass Transfer

525.3 = 1 (25 — T2)


10.22
11 0.99
or T2 = 23.1°C

70. A plane wall of area 5 m2 and thickness 10 cm is subjected to one dimensional heat
conduction. Find the heat transfer rate if the surfaces of wall are at 400°C and 100°C
and the conductivity of wall varies as follows:

k = 0.5 [1 + 1.0065T) where k is in W/m x K and 'T' is in °C.


(UPTU — 2008 — 9)

T2 dT
q=—fk
T1 dx
T,
or q•dx = f kdT = T1
m dT
0 T2 T2

T1 T2
or q= x

As k varies linearly, hence km is—


T1 + T2
= 0.5 (1 + 0065 Tm) where Tm =
2
400 +100)
= 0.5 (1 +1.0065 x
2 )
= 0.5 x 252.6 = 126.3 W/mK
126.3 x 300
Now a = = 378.94 kW/m2
10 x10-2
Q=qxA= 378.94 x 5 kW
= 1895 kW

71. The interior of a refrigerator having inside dimension 60 x 50 cm base area and 1.2
m hight is to be maintained at 7°C. The walls of the refrigerator are constructed of
two mild steel sheets 3 mm thick with 5 cm of glass wool insulation between them.
If the convective heat transfer coefficient at inner and outer surfaces are 11.6 and
14.5 W/m2°C respectively, estimate the rate at which heat must be removed to
maintain the specified temperature in the kitchen at 28°C. What will be the
temperature at the surface of wall?

k (glass wool) = 0.0464 W/m°C, k (mild steel) = 46.44 W/m°C.


(UPTU — 2009 — 10)
Steady State Heat Conduction 1 17

Th1 = 28°c

Th1 T3 T4 T h2

Walls of a refrigerator

60 50 60
Area = 2 ( x x x 1.2 + 50 x 1.2
100 100 100 100
= 2 (0.3 + 0.72 + 0.6)
= 3.24 m2
1 1 L1 L2 L3 1
= + + + +
U hi ki k2 k3 h2

1 2 X 3 X 10-2 5 X 10-2 1
= + + , +
145 46.44 4.64 x 10-` 11.6
= 6.9 x 10-2 + 0.129 x 10-2 + 107.8 x 10-2 + 8.6 x 10-2
= 123.43 x 10-2 m°C/W
or U = 8.1 W/m°C
Q = UA(Thi — Th2)
= 8.1 x 3.24 (28 — 7)
= 551.12 W
Q = hlA(Thl — T1)
551.12 = 14.5 x 3.24 (28 — T1)
or 28 — T1 = 11.73
II8 Heat and Mass Transfer

= 28 — 11.73 = 16.27°C.

72. An electric cable 1.2 cm OD has to be insulated with rubber (k = 0.15 W/mK). The
cable is to be located in air (ho = 8.7 wim2°C) at 18°C. Calculate: (i) the heat
dissipation from bare pipe (cable), (ii) the maximum heat dissipating, and (iii) the
critical thickness of insulation assuming a cable surface tamperature of 70°C.
(UPTU — 2009 — 10)

Tho

Th
o

Rh Rc

Cable with insulation

1
Rho = ho x2rE L
r,
log
R, =
2ir kL

Now re = k = 0.15
ho 8.7
re = 0.0172 m
Critical thickness of insulation = 0.0172 — 0.012
= 0.0052 m
Steady State Heat Conduction 1 19

1
=
Rho 8.7 x 27r x 0.0172 x 1
= 1.064
0.0172
log
Rc = 0.012
27rx0.15x1
0.36
= 0.382
0.942
Ti - Tito
Now Qins =
Rc + Rho
70 —18
0.382 x 1.064
52
1.446
= 35.96 W
Heat transfer without insulation is—
A X (Ti — Tho)
Quninsulated = hp X
= 8.7 x 2ir x 1.2 x 10-2 x 52
= 28.41 W

73. A steam pipe of 10 cm ID and 11 cm OD is covered with an insulating substance


k = 1 W/mK. The steam temperature is 200°C and ambient temperature is 20°C. If
the convective heat transfer coefficient between insulating surface and air is 8 W/
m2K, find critical radius of insulation. For this value of rc, calculate the heat loss per
meter of pipe and the outer surface temperature. Neglect the resistance of the pipe
material.
(Annamalai University — 2003 — 4)

1
Now r° = = — = 0.125 m
ho 8
10
Also = = 5 cm = 0.05 m
2
Q 2rc (Ti —
L r 1
log +
rl hp r°
I 20 Heat and Mass Transfer

27r (200 — 20)


0.125 1
log +
0.05 8 x 0.125
= 620 W/m

Now 9 h x 2nr,(T3 — To.)

620 = 8 x 2 xxx 0.125 x(T3 — 20)


rr 620
or 3= + 20
6.28
= 98.73 + 20
= 118.73°C

14- Cc -1.9

Steam pipe with insulation

74. What is Laplace equation for two dimensional steady heat conduction in the absence
of any heat source and uniform thermal conductivity?
General heat conduction equation is—

a2T a2 T a2 T qg 1 aT
a 2
ax e aye z k = a at

a2 T =o, qg aT
As = 0 and — =0
az2 k at
Steady State Heat Conduction 12I
Hence 2-D heat conduction equation is—
a2 T a2 T
+ ay2 = 0
ax 2

75. What is the graphical method of solving 2-D steady heat conduction equation?
Or

What is the shape factor? Give conduction shape factors for several configurations
to determine steady heat transfer rate.

The graphical method provides an approximate estimate of the temperature distribution and
heat flow in geometically complex two-dimensional systems. The method involves in
constructing a network of isothermal and adiabatic lines. The adiabatic or heat flux lines are
analogus to streamlines present in fluid flow. They are tangent to the direction of heat flow
at a point. Therefore, no heat can flow across the constant heat flux lines. The isotherms
are analogus to constant potential lines. Heat flux lines are normal to isotherms. To obtain
the temperature distribution, a scale model of two dimensional system is made. The
isotherms and heat flux lines are drawn by trial and error method until they form a network
of curvilinear squares. A constant amount of heat has to flow between any two flux lines.

heat flux
Isotherms line

Isotherm
49 IIIIIIIIIIIII1 49 heat flux
49 INIIIIIIIIIIII 49 line
-1-
11111111111111.1 Aq
49 imumiL 49, Aq
T149 is heat flux
49 IMMIlliiill 49 line
H— AX —I'd
49 111011111.11 49
49 IIIIIIIIIIIIII 49

Network of isotherms and heat flux lines

If there are 'n' isotherms and `m' heat flux lines, then heat which can flow between two
flux lines is—

Aq = Q
n
Similarly the temperature drop between two isotherms is—
T2— Ti
AT =
m
AT
But A q = k • Ay •
Ax
and Q = nAq
I 22 Heat and Mass Transfer

AT
= n • k • Ay •
Ax

Ay T2 — T1
= n •k•
Ax m

n
=k • — • ( T2 - T1 ) if Ax = Ay
m
= k • S • (T2 - T1)
n
S= is called the conduction shape factor. The conduction shape factors for various
m
configurations are as per table below.

Shape and shape factor

S.No. Shape Shape factor


1. Isothermal cylinder of length
L buried in a semi-infinite
medium (L » D and Z > 1.5 D)

7T2

1
i S- 2irL

0 0± log (t

L 1

2. Vertical isothermal cylinder of length L


buried in a semi-infinite medium (L » D)

2,rL
S-
49
log (ff

I-t-DH
Contd.
Steady State Heat Conduction 123

3. Large plain wall

S=L

-,-11_--

4. Disk buried paralld to the surface in a


semi-infinite medium (z » D)

rT2

S = 4D or S = 2D if z = 0

5. Isothermal sphere buried in a


semi-infinite medium

Iz
.4.----- 2,W
S=
D
1 1 — 0.25 x
z

6. Isothermal sphere buriad in a semi-infinite


medium (surface insulated and medium at T2)

Conk!.
I 24 Heat and Mass Transfer

2irD
7-.1 S=
1+ 0.25 x D
z

7. The edge of two adjoining


walls of equal thickness

S = 0.54 W

8. Corner of three walls of equal thickness

S = 0.15 L

76. What is finite difference method?

It is a numerical analysis method which is used to obtain approximate solution in multi-


dimensional steady state condition in a body. Numerical methods are preferred over analy-
tical methods as: (i) useful solution is difficult to be obtained in multidirectional steady state
condition with complicated boundary conditions, and (ii) numerical methods are well suited
to be used with high spreed digital computers. In the analytical solution, the determination
of temperature is made at any point of interest in a medium, whereas in a numerical solution
the determination of temperature is possible at discrete point.

77. How is Laplace equation for two dimensional heat conduction approximated to the
finite difference forms or equations?
Steady State Heat Conduction I 25

Or
Explain: (i) node, (ii) nodal network, and (iii) finite difference form for 2-D heat
conduction.
In numerical method, the temperatures are determined at discrete points. To select the
discrete prints, the body is subdivided into a number of small regions and a reference point
to each is assigned at its centre. The reference point is termed as node or nodal point. The
aggregate of nodal points is called as nodal network or mesh. The body with nodes and nodal
network is as shown in the figure. Consider five modes as shown in the figure which are
assigned reference numbers with respect to the centre node which is assigned as (i, j).
Material between nodal points may be replaced with fictitious rods having the same
conductivity (k). Now 2 D Laplace equation is-

j +1
I
AMENEUEllb•
+ 1, j
Ay MENNII=EMMI
gNIENIMMENF i — 1, j j
valIMMENEr
j "1411111MIP"
j-

Two dimensional nodal network and finite difference approximation

a2T a2 T
ax2 =0

CaT
But
ax),+ Ax
2'

a TI _ J — T-1, j
and
C ax i-v2,j Ax

ax)i+1.;
Now — 2'
l.ax2
Ca )0 Ax
Ti ,j + - , j- 2 ,j
(Ax) 2

r a T)

a2 T' .aY)i,j+1/2 ay )i,J _1


Similarly 2
ay2 Ay
I 26 Heat and Mass Transfer

Ti,j+1+ - 2T,
2
(Ay)
If Ax = Ay, then Laplace e uation from equation (i) and (ii) —
a 2 T a2 T _
0
ax2 ay —
or Ti+1, Ti, j+1 Ti , j_i - 4 Ti, = 0.
Hence, each node represents a certain region and its temperature is a measure of the average
temperature of the region i.e, the temperature of node (i, j). The above equation is the
approximate algebraic equation in finite difference form of the exact Laplace differential
equation in two dimensional. This approximate finite difference form of the heat conduction
equation may be applied to any interior node which is equidistant from its four neighbouring
nodes. This rquation therefore reveals that the sum of the temperatures associated with the
neighbouring nodes should be equal to four times the temperature of the node of interest.
In other words, the algebraic sum of all heat flowing to a point must be equal to zero. (Eq
= 0)

78. What are the nodal finite difference equations for: (i) interior node, (ii) node at an
internal corner with convection, (iii) node at plane surface with convection, (iv) node
at an external corner with convection, and (v) node at insulated boundary.

The nodal finite difference equations are as per Table given below.

Nodal finite difference equations

Configuration Finite difference equation (Ax = Ay = Az)

4 Interior Node

Qio + Q20 + Q30 + Q40 = 0
Alxb
.0 [k(T, — To ) + h(T2 — + k(T3 — TO+ k (T4 —To)] —0
•3 Al
or T1 + T2 + T3 + T — 4 To = 0

Node at an internal corner with convection


2 A/xb Mxb Mxb
kx ( T1 — To) + k ( T4 — To) + k x
4 Al Al 2x A/
A/xb
(T3 — + kx ( T2 — To) + hx Aix b(T_ — To ) = 0
2M
0 •3
2.h•A/ h•
2( + T4) +(T2 + T3) + 2(3+ )To = 0
h1,T.


2
Steady State Heat Conduction I 27

4 Node at plane surface with convection



Alxb Aixb A/ b
kx (T1 — To) + k x (T2— To)+ k
Al 2,6,/ 2,6,/
0
1• h1 T, (T4 — To) + h x Al x b(T_ — To) = 0
2.h.A1To. (h•Al
(27-1 + T2 + T4 ) + 2 +2) To = 0
• k k
2
Node at an external corner with convection

EIxb Alxb
1 —0 kx (Ti To) + k x (T2 — To) + h x Al x b
2A/ 201
hi Ta x (T,, — To) = 0
+2hAx h. AI
2 ( Ti + T2) x 7-0, — 2 ( +1) To = 0
k k

Insulated
Node at insulated boundary
Aix b Aix b Aix b
kx (7-1 To) + k x (T2 — To) + k x x
20 / A/ 20/
(T3 — To) = 0
Ti + 2 7-2 + T3 — 4T0 = 0

79. How can finite difference equations be solved?

The finite difference equations can be solved by—


(a) The relaxation method
(b) the Gauss-Seidel iteration method
(c) the matrix inversion method

80. Describe relaxation method of solving a system of finite difference equations.

The steps involved are—


(a) Stepl. Sundivide the system into a number of small regions and assign a reference
number to each node.
(b) Step2. Assume temperatures for all nodes.
(c) Step3. Using the assumed temperatures, calculate the residuals at each node.
(d) Step4. Release the largest residual to zero by changing the corresponding nodal
temperature by an appropriate amount.
(e) Step5. Change the residuals of the surrounding nodes to correspond with temperature
change in step-4.
(f) Continue to relax residuals and obtain temperatures until all residuals are close to zero.
I 28 Heat and Mass Transfer

81. The boundary temperatures of a thin plate are as shown in the Figure. Ditermine the
temperature at the centre of the plate.

y/
4
200 °C
T 2
0 =— = 1
2
400 °C 100 °C •
H= 2 m 1 0 3

100 °C I
I x 2
1-.—w= 2 m—.-I +-Az -1-

Now 4x To = l'i + T2 ÷ T3 ÷ Tet as Ax = Ay =1


To = 400 +100 +100 + 200
or
4
= 200°C

82. The temperature distribution and boundary condition in various parts of a solid is as
shown below. Determine the temperature at nodes marked as A, B and C. Also find
the heat convected over the surface exposed to convection. Assume k = 1.5 W/mK
and h = 500 W/m2°C.

Insulated

T. = 30 °C
Convection
face
h= 500 W/m2°C

For node 'A', we have


T1+ T2+ T3+ T4
TA =
4

= 172.9 + 200 +132.8+137


4
= 160.7°C
For node 'if , we have
T1 + 2 x T2+ T3
TB =
4
Steady State Heat Conduction I 29

129.4 + 2 x 103.5 + 45.8


= 4
= 95.6°C.
For node 'C', we have
1 hxAx
(27'1+1'2 + TO + xl:
7, _ 2
, hx Ax
+2
k

1 500 5 0.1
(2 x 103.5 + 67 + 45.8) + x 30
=2 1
500 x 0.1
+2
1.5
= 37.4°C.
Now the heat convected out by the exposed surface is—
Qconv =hxAxAT
= (Az x Ay) x E(T — To ) but Az = thickeness = 1
= 500 x 1 x 0.1 x [(45.8 — 30) + (37.4 — 30) + (67 — 30) + 2 x (200 — 30)]
= 7258 W

83. The figure as shown below indicates temperatures in a region of a solid with boundary
conditions. Find: (i) thermal conductivity, and (ii) the heat flow from surface 1.
(Annamalai University — Nov 2005, May 2006)

Insulated

1 -,— 0.1 m—).- D-r--------„,


Surface 1
Ta = 300 °C, h = 10 Whn

(Q)surfacel = h • A • I AT
1
= h • (Az x Ax) x [— x (500 — 300) + (356 — 300) + (337 — 300)1
2
= 10 x 0.1 x [100 + 56 + 37) as Az = 1
= 193 W
Also Q= k • A • EAT
I 30 Heat and Mass Transfer

193 = k • (Ax x Az)[(TE — TF) + (TA — TD) + (TB — Tc)]


= k x 0.1 x 1 x [(500 — 500) + (455 — 356) + (454 337)]
= k x 0.1 x (97 + 117)
193
or k=
0.1 x 196
= 9.85 W/mK

84. The boundary temperatures of a thin plate are shown in the Figure. Determine the
temperature at the centre of the plate.
(Annamalai University — April 2007)

YA
200 °C •4
t
Ay

400 °C 100 °C • •1
4 = 1.5 m 1 0 3

100 °C •
X 2

v=2m--1 —Ax

Here Ax # Ay
2
and Ax = — = lm and Ay = 15 = 0.75 m
2 2
T1 + T2 + 3 + T4
Now To = when Ax = Ay
4T

Ay Ax
(T1+ T3) + (T2+ T4)
Ax Ay
and To = when Ax # Ay
4
0.5 1
(400 + 100) + 0. 7 (100 + 200
1 5
4
1
0.75 x 500 + x 300
0.75
4

= 375+400 = 775
4 4
= 193.75°C
Steady State Heat Conduction I3I

85. Consider two dimensional steady state heat conduction in a square region of side 12
subjected to the boundary conditions as shown in the Figure.

A
Y
T= 400 °C
L

P
0
0
T2. •
N T=600 °C
T4
T3.
1

0 T= 300 °C

L
Calculate Ti., T2, T3 and Ti considering Ax = Ay = — . Calculate the heat transfer rate
3
through the boundary surface at x = L per 1 m length perpendicular to the plane of
figure for L = 0.1 m, k = 20 W/mK.
(Annamalai University — May 2004)

For node T1, we have—


T2 + T4 + 600 + 400
T1 = ...(i)
4
For node T2, we have—
200 + T3 + T1 + 400
T2 = •• •00
4
For node 7'3, we have—
200 + 800 + T4 + T2
T3 = •• .(iii)
4
For node T4, we have—
T3 + 800 + 600 + Ti
T4 = . . .(iv)
4
Rearranging the equations (i), (ii), (iii) and (iv), we have-
1000+ T2 + T4 — 4 T1 = 0
600 + T3 + T1 — 4 T2 = 0
1000 + T2 + T4 — 4 T3 = 0
1400 + T1 + T3 — 4 T4 = 0
Now using the relaxation method—
I 32 Heat and Mass Transfer

S.No. Step Ti R1 T2 R2 7'3 R3 T4 R4


1. Initial assumed value 500 0 450 —200 500 0 550 200
2. change AT2 = —50 500 —50 400 0 500 —50 550 +200
3. change AT4 = +50 500 0 400 0 500 0 600 0

As all residuals are zero on assigned temperatures, hence temperatures are—


Ti = 500°C
T2 = 400°C
T3 = 500°C
T4 = 600°C
Now heat transfer rate at x = L is
dT L
Q=k•b•Ax• but Ax = Ay = and b = 1
dy 3
= 20 x dT
= 20 x [(T3 — 800) + (T4 — 800)]
= 20 x [(500 — 800) + (600 — 800)]
= 20 x [-300 — 200]
= —10 kW

86. Steady two-dimensional heat conduction takes place in the body as shown in the
figure below. The normal temperature gradients over surface P and Q can be
considered to be uniform. The temperature gradient at surface Q is equal to 10
ax
ax
K/m. Surfaces P and Q are maintained at constant temperatures as shown in the
figure, while the remaining part of the boundary is insulated. The body has a constant
conductivity of 0.1 W/m • K. The values of and aT
— at surface P are:
ay ax

(a) = 20 K/m,
aT = 0 Kim (b) aT 20 Kim, aT = 10 Kim
ax
ax ay ax ay

(c) = 10 K/m, aT = 10 K/m (d) = 0 K/m, aT = 20 K/m


ax ay ax
ax ay

Y
Surface Q at 0°C

Surface Pat 100°C


X

Figure
Steady State Heat Conduction I 33

The surface P is horizontal (Parallel to x-axis). Surface P has uniforms temperature. Hence
T
( ax p
0 at surfaceP.Now the surface Q has only temperature gradient —
ax),
10 K/m.
Now balance of heat is—
(Q)P = ( Q)Q

aT
or k • Ap k•A x (
x ay p Q a x)Q

or (1 X b) x (aT) (2 x b) x 10
p
( aT = 20 K/m
ay 1
The option (d) is correct.

87. A long pipe of 0.6 m outside diameter is buried in earth with it axis at a depth of
1.8 m. The surface temperature of pipe and earth are 95°C and 25°C respectively.
Determine the loss of heat per unit length of the pipe. Assume conductivity of earth
= 0.51 W/mK.

/ / / / / / / / / / / / / /

Earth
Z = 1.8m

D = 0.6 m

Pipe buried in earth

= k • S (T pipe
Tearth)

where S = shape factor as given in Answer 75.


2ir x L
7
log 41
D
where D = diameter and Z = depth
2.7rx1
4 x 1.8
log
0.6
= 2.53
I 34 Heat and Mass Transfer

2 = 0.51 x 2.53 (95 — 25)


L
= 90.8 W/m

88. A sphere (diameter = 1.6 m) is buried in earth at a depth = 5.5 m. Heat in sphere
is generated at a rate of 580 W. The earth conductivity = 0.51 W/mK and earth
surface is at 6°C. Determine the temperature of the sphere under steady state
condition.

Earth
f
Z = 5.5 m

D = 1.6 m
Sphere buried in earth

2irD
Shape factor S=
D
1— 0 . 25.—
z

= 2x7rx1.6
1.6
1-0.25x —
55
= 10.8 m
Now /
Q = k • S • (Tsphere Tearth)
580 = 0.51 x 10.8 (Tsphere — 6)
_ 580
Tsphere
6 + 051 x 10.8
= 6 + 104.9
= 110.9°C.
Chapter 4
HEAT TRANSFER FROM EXTENDED
SURFACES

KEYWORDS AND TOPICS


A FINS A FIN EI,PECTIVENESS
A LONG FINS A EFFICIENCY OF FIN
A INSULATED TIP FINS A ERROR IN MEASUREMENT
A FINITE LENGTH FINS A HYPERBOLIC FUNCTIONS

INTRODUCTION

The rapid heat dissipation from a surface to surroundings is required in many applications. The
heat dissipation depends upon: (i) convective heat transfer coefficient (h), (ii) the temperature
difference between surface and surroundings (ts — L), and (iii) surface area (A). The surface
area exposed to the surroundings is frequently increased by the attachment of protrusions to a
surface. This method of increasing surface area provides a easier method of increasing heat
transfer rate from a surface. The protrusions are called fins. The fins can be rectangular, circular,
trapezoidal and parabolic. The selection of a fin for a particular application depends upon factors
which will give: (i) maximum efficiency, (ii) minimum material for cost, weight and space
consideration, (iii) minimum resistance to the heat flow, (iv) adequate strength, and (v) ease of
manufacturing. The heat dissipation from extended surface is found out using one-dimensional
heat flow method.

1. What are fins? Why are they used?

Fins are basically extended surfaces. They are used to increase the heat transfer rate from
a surface to the adjoining fluid. The rate of heat transfer increases as the effective surface
area of the surface increases with the attachment of fins.
I 36 Heat and Mass Transfer

2. How does heat transfer take place from a fin?

The heat transfer takes place from the fin to the adjoining fluid by conduction and
convection.

3. Mention different types of fins?

Fins are of two types:


(a) fins of uniform cross-sectional area, they can have rectangular or circular profiles
(b) fins of non-uniform cross-sectional area: they have tapered shapes with cross-section
as rectangular or circular.

Fins of uniform cross-section

Fins of non-uniform cross-section

Annular fin Splines

Fins of uniform cross-section

4. What are various applications of fins in industry?

Fins are generally used in industry in following applications:


1. Cooling of IC engines
Heat Transfer from Extended Surfaces I 37

2. Cooling of electric motors


3. Cooling of transformers
4. Cooling of refrigerators
5. Cooling of electronic components.

5. Derive an expression in one-dimensional differential equation of temperature for a


fin of uniform cross-section.

For the analysis of heat flow through the fin, following assumptions are made:
1. Steady state heat conduction along x-axis
2. No heat generation in the fin
3. Uniform heat transfer coefficient over the entire surface of the fin
4. The material of the fin is homogenous and isotropic.
5. The fin and base material has no contact resistance.
6. Negligible radiation from the fin.
7. The fin temperature at base is uniform (T0).

Fin

Base

Fin of uniform cross-section

Consider an elemental cross-section `dx' thick at distance 'x' from the base of the bin. On
basis of heat balance for this element, we get:
Conduction heat entering at 'x' = Conduction heat leaving at `x + dx' + Heat leaving by
convection
Qx = Qx+dx + Q.,
But heat leaving at 'x' is
aT
Qx = —kA—
x
Heat leaving at x + dx is
aT a
Qx+dx = —kA + ax(Qx)
ax
I 38 Heat and Mass Transfer

= -kA—
-kA—
aT +
aT (-kAA) • •dx
(-kAA) dx
ax ax ax

= -kA aT kA a2T dx
ax ax2
Also convective heat transfer is
Qconv = h (P • •ddx)(T
x)(T -
where P = perimeter, Ta = temperature of surroundings
On substituting these values, we get
T
kA" d xx -- h(P
h(P•• dx)(T - =0
ax2
a2T
m` (T - =0 (i)
ax2
where m = hP
kA
a 29 a2 7,
Now take 0 = T - Tc, and =
ax2 ax2
Hence, equation (i) becomes
a 29 m2 0 0
ax2
Equation (ii) is a differential equation of second order which has solution as
0 = cie-" + c2emx
Two boundary conditions are required for solving c1 and c2.

6. What are different conditions of fins which can yield a set of two boundary cases for
solving temperature distribution equation for the fins?

There can be three conditions of the fins:


(a) Fin is infinitely long resulting temperature at end of the fin is same as that of the
surrounding fluid. The boundary conditions possible with such fins are:
(i) At x = cc, T=T i.e. 0 = 0
(ii) At x = 0, T = To i.e. 0 = 190 where To is temperature at base.
(b) Fin has its end insulated. The boundary conditions possible with such fins are:

(i) a T = o at x = L
a
T = To at x = 0
(c) Fin has finite length. The boundary conditions possible with such fins are:
Heat Transfer from Extended Surfaces I 39

(i) At x= L, -k—aT
ax
. h(T - 71,0
(ii) At x = 0, T = To i.e. 0 = 00

7. Derive expressions for temperature distribution and rate of heat flow for infinitely
long fin. Also explain why fins are made of high conducting metals.

The general equation for temperature distribution for a fin is


0 = c le-" + c2emx (i)
Now boundary conditions are
(i) x = 0, T = To or 0 = Oo
(ii) x = cc, T = 7'. or 0 = 0
Applying second boundary condition, we get
0 = c le — + c2e-
ci
or 0= + c2e-
e
,,,, ci
= c2e as _ -3 0
e
or c2 = 0 and 0 = cie-mx
from equation (i) (ii)
Using first boundary condition in equation, we get
00 = ci
Hence, temperature distribution in the fin is-
0 = Ooe-mx
0
or — = e-mx
0o
T -T
or = e-mx
To - To
The rate of heat dissipation from the fin is

ems, = -kA( aT )
ax x=o
= -kA(-00 x m x e-mx)x=0
= +kAm00

= +kAliii) x 00
kA
= VkAhP (T0 - Tc.c)
I 40 Heat and Mass Transfer

x
Temperature distribution of infinitely long fin

Temperature distribution in an infinite long fin depends upon the value of 'x' as shown in
the figure.
From the above diagram, it is apparent that temperature of the fin falls from base
temperature (T0) as we move away from the base. As temperature falls along the length of
the fin, heat transfer reduces along the length of the fin. If `m' is higher, the temperature
falls much faster along the length of the fin which is undesirable. The ideal case is that base
temperature remains constant throughout the length of the fin which is possible when 'Ic'
is high. It is the reason why fins are made of high conducting metals such as aluminium,
copper and brass, etc.

8. Derive expressions for temperature distribution and heat flow for a uniform cross-
sectional fin which has insulated end.

The general equation for temperature distribution of a fin is given by


0 = c le" + c2emx (i)
On differentiating, we get
ae
= -cime" + c2memx
ax
Now boundary condition that end is insulated gives
aT ae n
ax = ax= -
at x= L
Heat Transfer from Extended Surfaces 1 41

0= + c2memL
or c1 = c2e 2mL
Now when x = 0, 0 = 00 we get,
From equation (i)
Oo = CI + C2

From equation (ii) and we get


00 00
C2 = and ci = -2mL
1+ e2mL 1+ e
Putting the values of c1 and c2 in equation (iii), we get

emx
e-2mL +
0 = 00[ 1+e-mx
1+ e2mL

emL mx
= ryo
[ e' x e
xe
1+ e-2mL emL 1+ e2mL

e m(L-x) e
m(x-L)
= e0 e mL e-mLe-mL mL
+ +e
em(L-x) e-m(L-x)
= Oo
emL + e

— cosh m(L — x)
Oo cosh mL
Now temperature distribution is
0 cosh m(L — x)
0o cosh mL
T — Ta cosh m(L — x)
To — T — cosh mL
The rate of heat flow is given by
0 = —kA
"
C ax )x=o

= —kA[Qo x
rm sin h m(L — x))1
cosh mL
x=0
Q = kAm00 tanh(mL)
= hP kA Ootanh mL

9. Derive expressions for temperature distribution and heat flow for a fin which has
finite length and loss of heat from fin tip is by convection.
I 42 Heat and Mass Transfer

The general equation of temperature distribution for a fin is


0 = cie-mx + c2emx
At the tip of the fin, we have
Heat loss by conduction = heat loss by convection at x = L
-kA a0
— = hAO at x = L
ax
or hx0+kx— a =0
ax
or h x (cie-mL + c2emL) ÷ k(_ cime-m.L., + c2memL) =0

. cie-na, h il +c2eniqh +11 =0


mk ) mk )
Now we know at x = 0, 0 = 00 and applying it in general equation, we get
00 = c1 + c2
or c1 = 00 - c2
From equations (i) and (ii), we have

e-mq h -1)
mk
Cl = 00 + )ci
+ng,(h
e +1
mk

_niLi h _ 1)-
e
mk )
or ci [1 = 00
e-Frnqh + i j
mk

00 [11+ h lemL 1
R mk ) i
or Cl =
(emL + e-niL)± h (emL — e-mL )
mk

041+ h jemL
mk
and C2= 00 -
(e + e )+ h (emL
mL -mL —-mL
e)
mk

00(emL
[ + Cm L)+ h (e mL e-mL ) (1+ h j emL1
mk mk
=
(emL ± e-mL) ± h (em L _ e _mL )
mk
Heat Transfer from Extended Surfaces I 43

00 (1 — h jet'
mk
(emL
nil.
+ e-nil.
) +
h mi.
(e — e
-mi.
)
mk
Substituting the values of c1 and c2, we get

(e m(L-x) + e-m(L-x)) + h (em(L-x) — e-m(L-x))

0= 00 mk
(emL + e-mL) + h (emL — e-mL)
mk

h
cosh m(L — x) + sin m(L — x)
mk
0 = 00 x
h
cosh (mL) + — sinh (mL)
mk

Now to find heat transfer Q from the fin, we use

Q = —kA( 09 )
x_ o

h
—m sinh (mL) — x m cosh (mL)
= —kA x 00 x mk
h
cosh (mL) + sinh (mL)
mk

h
sinh (mL) + cosh (mL)
mk
= kArrlOo cosh (mL) + h
sinh (mL)
mk

tanh (mL) + h
= V hP kA 00 mk
1+ li tanh (mL)
mk

10. What is the significance of fin effectiveness?


(UPTU — 2006-7)

The ratio of heat transfer rate from a surface with fin and heat transfer from the surface
as such without fin is called fin effectiveness (E). The value of fin effectiveness should be
greater than one.
I44 Heat and Mass Transfer

with fin
Fin effectiveness E —
()without fin
In case we take infinitely long fin, then fin effectiveness is given by
VhPkA00 00
E= —
hA00 hA
kP
For E > 1, we have to have M < 1.
kP

11. What do you understand from the efficiency of a fin?


(UPTU — 2006-7)

The efficiency of a fin is defined as the ratio of actual heat transfer from fin surface to the
maximum heat transferred which could be achieved from the fin. It is apparent that
maximum heat transfer would occur when the temperature of entire fin surface becomes
uniform and equal to the base temperature To
Qfin
nfin =
max

12. The temperature distribution in a stainless fin (k = 0.17 W/cm°C) of constant cross-
sectional area of 2 cm2, and length of 1 cm, when exposed to an ambient of 40°C
(h = 0.0025 W/cm2°C) is given by
(T — Ta) = 3x2 — 5x + 6
where T is in °C and x is in cm. If the base temperature is 100°C, then the heat
dissipated by the fin surface will be:
(a) 6.8 W (b) 3.4 W (c) 1.7 W (d) 0.17 W
(IES — 1994)

We know ao
Qfil, = —k11()
x=0
It is given— T — Ta = 0 = 3x2 — 5x + 6
ao = 6x — 5
ax

or

Qfm = —kA(-5)
= 5 x 0.17 x 2
= 1.7 W
Option (c) is correct.
Heat Transfer from Extended Surfaces I 45

13. Addition of fin to the surface increases the heat transfer if 1 hA is:
kP
(a) equal to one
(b) greater than one
(c) less than one
(d) greater than one but less then two.
(IES - 1996)
The fin can transfer additional heat in case effective (E) is move than one.
Qfin hP kA 9 0
E= >1
Qwithout fin hAeo

1 >1
hA
il kP
hA < 1
kP
Option (c) is correct.

14. A long rod 20 mm in diameter has one end maintained at 110°C. The surface of the
rod is exposed to ambient air at 20°C with convection coefficient of 5 W/m2K.
Calculate the temperature distribution equation and heat loss from the rod having
thermal conductivity of 1.5 W/mK.
(UPTU - 2002 - 03)
Guidance: A long rod applies that it is a infinitely long fin therefore
0 = e-mx
00

where m= hP
kA
P = perimeter = 7cd = 7r x 0.02 = 6.3 x 10-2 m
X42x (2 x 10-2)2
A = area = = 3.14 x 10-4 m2
4 4

5 x 6.3 x 10-2
m = , = 8.16
15x 3.14 x10—
T - Ta
- e-8.16x
To —T
Now heat loss from fin is
Q = jhP kA (To - To.)
I 46 Heat and Mass Transfer

= -\ 5 x 6.3 x 10-2 x 15 x 3.14 x 10-4 x (100 - 20)


= 3.07 W.

15. One end of a long, 2 cm diameter rod is inserted inside a furnace while the other end
projects into the ambient air at 28°C. The steady state temperatures of the rod at
two points 10 cm apart are measured as 125°C and 91°C. Find the thermal
conductivity of the material of rod where h = 15 W/m2K.
(UPTU - 2002 - 2003)
Guidance: The fin is infinitely long and temperature distribution is given by:
0 = Ooe-mx
Fin
Base

Now Ox = 00e-"
and = ooe-m(x+Sx)
Ox+Sx
Now dividing equation (i) and equation (ii)
ex = 0oe' = e m•Sx
ex-Fox Ope -m(x-Fox)
Tx - 7:
= em.Sx
Tx-Fox - To.
125 - 28 m. sx
=e
91 - 28
97 =emxo.i as 8x = 10 cm
63
or 0.1 m = log 97
63
= 0.43
or m = 4.3
But m2 = hP where
kA
P = perimeter =

and A = Area = ird2


4
Heat Transfer from Extended Surfaces I 47

m = = 15xxxd
2 hP
kA rd2
kx
4
(4.3)2 = 15 x 4
k•d
15x4 = 30x100
18.63 =
kx 2x10-2
30 x 100
or k= = 161 W/mK
18.63

16. A fin of circular x-section, diameter 2.5 cm is placed in a furnace with large portion
of it is projecting in a room where temperature is 28°C. After steady state conditions
prevail, the temperature at two points 10 cm apart are found to be 110°C and 85°C.
The value of h = 28 W/m2K. Find value of k.
(UPTU - 2006 - 7)
Guidance: It is like last problem where fin is infinitely long. Also
= em•Sx
ex
e x+ox

Tx - Ts° = emxSx
where Sx = 0.1 m (given)
Tx+ox - Tso
110 - 28 - eo.ixm
85 - 28
82 - eo.im
57
or 0.1 m = log —82
57
= 0.36
m = 3.6
m 2 hP
- - 28 x nd
Now
kA k x ird 2
28
k x 2.5 x 10-2
28 x 102
or k= - 2800
m2 x 2.5 (3.6)2 x 2.5
2800 = 86.42 W/mK
12.96 x 2.5
I 48 Heat and Mass Transfer

17. It is required to heat the oil to 300°C for frying purpose. A long laddle is used in
frying pan. The section of the laddle is 5 mm x 18 mm. The surrounding air is at 30°C
and the thermal conductivity is 205 W/mK. If temperature at a distance of 380 mm
from the oil should not exceed 40°C, determine convective heat transfer coefficient.
(NMU — 2002)

= 30°C

Now as laddle is long, we have temperature distribution as


T—T ,x
—e
To —T
Now it is given that T = 40 at x = 0.38 m
40 — 30 — e_0.38m
300 — 30

or e—0• 38m = 10
270
or m = 8.67
h x 2(0.005 + 0.018)
But m2 — hP —
kA 205 x 0.005 x 0.018
(8.67)2 x 205 x 90 x 10-6
h—
0.046
= 30.17 W/m2K

18. Consider a stainless steel spoon (k = 15.1 W/mK) partially immersed in boiling water
at 95°C in a kitchen at 25°C. The handle of the spoon has a cross-section 0.2 cm x
1 cm and it extends 18 cm in air from the free surface of the water. If h = 15 W/m2K,
find temperature difference across the exposed surface of the spoon handle. State
your assumptions.
(Shivaji University — 1997)
Heat Transfer from Extended Surfaces I 49

Assumptions: The handle of spoon is thin and heat loss from its free end is negligible i.e.
the free end of the spoon is insulated.
For insulated tip fin, temperature distribution is
T — To, cosh m(L — x)
To — T cosh mL
Let TL is temperature at the tip of the handle i.e. at x = L = 0.18 m
71— 7' 1
To — Tos cosh mL

m2 —hp— 15 x 2(0.2 + 1) x 10-2


kA 15.1 x 0.2 x 1 x 10-4
= 34.52 m-1
Now mL = 34.52 x 0.18
= 6.21
cosh (mL) = cosh (6.21) = 250
TL —7' 1 1
To — To, cosh 6.21 250
95 — 25
or — 25 = = 0.28
250
TL = 25.28°C
Now the drop of temperature from base temperature i.e. (To = 95) to TL (25.28)
To — = 95 — 25.28
= 69.72°C.

19. Two long rods of same diameter, one made of brass (k = 85 W/mK) and other made
of copper (k = 375 W/mK) have one of their ends inserted into a furnace. Both the
rods are exposed to same environment. At a distance of 105 mm away from the
furnace, the temperature of brass rod is 120°C. At what distance from the furnace,
the same temperature would be reached in the copper rod?

= 120°C
105 mm
Brass

Furnace

Copper
x
= 120°C

Since both rods are long, we can apply temperature distribution as in the infinite long fin,
I 50 Heat and Mass Transfer

-T
TL p-mx
To - T —
For brass rod at x = 0.105, TL = 120°C
120 - T - e-mb xo.los
To -T
For copper rod TL = 120°C at x
120 - T x
= e _mcx
To -T
Equating equations 1 and 2
mb x 0.105 = mc x x

or x= x 0.105
me

hP
til kb `4 x 0.105
hP
\i' kc A

k
x= x 0.105
kb

;i 375 x 0.105
85
= 0.221 m
= 221 mm

20. A very long rod 25 mm in diameter has one end maintained at 100°C. The surface
of the rod is exposed to ambient air at 25°C with convective coefficient of 10 W/m2K:
1. What are the heat losses from the rods, constructed of pure copper (k = 398 W/
mk) and stainless steel (k = 14 W/mK)?
2. Estimate how long the rods must be to be considered infinite.
(PU - 2003)
For infinitely long fin, the heat transfer is-
Q = NihPkA (To - To.)
For copper fin, k = 398
Qc„= x x 25 x 10-3) x 398 x x (25 x 10-3)2 x (100 - 25)
= 29.37 W
Heat Transfer from Extended Surfaces I5I

For stainless steel fin, k = 14 W/mk


k
Q ss =A; " x Q.
Iccu

— ..\ii 398
14
x 29.37
= 5.51 W
Now to find length of infinite long fin, we equate the heat loss of infinite long fin to the fin
of finite length with tip insulated

VhPkA (To — Ta) = VhPkA (To — Ta)tanh (mL)


. tanh mL = 1
or mL = 2.6
For copper rod, we have
10 x 7.854 x 10-2
m = 1 hP
1 kcu A 398 x 4.90 x 10-4
=2
Now mLci, = 2.6
6 = 1.3 m
L. = 2
For stainless steel rod, we have
10 x 7.854 x 10-2
m= — 10.69
14 x 4.9087 x 10-4
Now mL„ = 2.6
2.6
L„ = = 0.247 m
10.69

21. The aluminuim square fins (0.5 m x 0.5 m) of 1.0 cm length provided on the surface
of an electronic semiconductor device to carry 46 x le W of energy generated by
an electronic device and the temperature at the surface of the device should not
exceed 60°C. The temperature of the surrounding medium is 20°C. Take k = 190 W/
mk & h = 12 W/m2K. Find the number of fins required to carry out the heat transfer
neglecting heat loss from the end of the fins.
The fins can be considered insulated tip fins. Hence,
Q = IhP kA (To — Ta) tank mL

hP _ 12.5 x 2(0.5 + 0.5) x 10-3


m
- / kA — 190 x 05 x 05 x 10-6
I 52 Heat and Mass Transfer

= 23 m-1
mL = 23 x 1 x 10-2
= 0.23
Q = V125 x 2(05 + 03) x 10-3 x 190 x 05 x 0.5 x 10-6 x tanh(0.23) x (60 - 20)
= 11.5 x 10-3 W
Number of fins required = Qt"4.al
Qfin
x 10-3
,=4
11.5 x 10—

22. Determine the heat lost per hour by a mild steel rod of 2 cm diameter and 10 cm
length. The temperature at one face of the rod is 220°C and ambient temperature
is 20°C. Take k = 50 W/mK and h = 20 W/m2K. Neglect the heat lost by the free end
of the rod.
The fin is equivalent to the fin with insulated tip. Hence,
Q = VhP kA tanh mL(To - Too )

hP _ 20 xxxd
m
_ Al kA 50x 7xd2
4

180 = v80 = 8.95


- 50 x 2 x 10-2 = \ 1
mL = 8.95 x 0.1 = 0.895
Q = •NlhP kA tanh mL(To - To.)

= V20 x Ir x 0.02 x 50 x Ir X (0.0D2 x tanh (0.895) x (220 - 20)


= 20 W = 20 Joules/s
Heat loss per hour = 20 x 60 x 60
= 72 x 103 Joules
= 72 kJ

23. A circular fin of 100 m length and 5 mm diameter extends horizontally from a casting
at 200°C. The fin is in an environment with T., = 20°C and h = 20 W/m2K. What is
the temperature at the free end?
Take k = 133 W/mK.
(UPTU - 2003)
The fin has finite length and temperature distribution is
Heat Transfer from Extended Surfaces I 53

h .
T—T
cosh m(L — x) + Ruh m(L — x)
_ mk
To — T h
cosh mL + — sinhmL
mL
Temperature is TL at x = L
TL — 7'
=
1
To — T h
cosh mL + sinh mL
mk

20 xxxd
Now m _ lhP _
i kA x x d2
133 x
4
20 x 4 i 80
= 133 x d i 133 x 0.005

= V120.3 = 10.96
Now mL = 10.96 x 0.1
= 1.096
TL — 20 _ 1
200 — 20 — cosh (1.096) + 10.9620x 133 x sinh (1.096)

= 1
20 x 1.32
1.66 +
10.96 x 133
= 0.595
TL — 20 = 0.595(180)
= 107.26
TL = 127.26°C

24. What do you understand by fin effectiveness and fin efficiency? Derive an expression
for these for a fin of insulated tip using standard expression for the heat flow rate
through the fin.
(UPTU — 2003)
The fin effectiveness is defined as the ratio of fin heat transfer rate to the heat transfer rate
that is possible from the surface without the fin.
Qfin
E—
hA(To — L)
Fin efficiency is the ratio of actual to ideal heat transfer from the fin. It is a parameter which
evaluates the thermal performance of a fin.
I 54 Heat and Mass Transfer

Actual heat transfer


elfin = Ideal heat transfer through fin if entire fin surface is at base temperature

= 9fin where
Qideal

Qideal = hP x L(To — To.)


The above means that fin has constant temperature of 70' throughout its length which is
possible in case fin material has very high 'V .

T, T,

Ideal fin

The differential equation for temperature distribution is—


a 29
axe m0
or 0=c + c2emx

For insulated tip fin, we have—


x = L ae = 0 and x = 0, 0 = 00
'
We get—
9 oemL 9 oe — niL
C1 = ng, + e-m L & C2 ra
e e +e
0 = 0 cosh [m(L — x)]
° cosh mL
Q = —kA where 0 = To —

(t)x=o

rm sinh (mL)1
= —kA (To T)
cosh (mL)

= VhP kA (To — Too) tanh (mL)


For the above fin, we have—
Qfin
hA(To —

hP kA x (T — Too) tanh (mL)


hA (To —
Heat Transfer from Extended Surfaces I 55

_ hA
A tanh (mL)

Gm
hPL(To —

11hP kA • (To — T_) tanh (mL)


hPL(T — 1:)

= l_ A x tanh (mL)
A hp

25. A straight fin of rectangular profile is constructed of stainless 18-8 steel and has a
width of 5 cm and thickness of 2.5 cm. The base temperature is maintained at 100°C
and the fin is exposed to a convection environment at 20°C with h = 47 W/m2K.
Calculate the heat lost by the fin of one meter length.
(UPTU — 2003-4)

The fin is of finite length with end insulated. `k' for 18-8 steel = 14 W/mk
Q = 11hP kA (To — T) tanh (mL)

hP 47 x 2(5 + 25) x 10-2


m_
kA 14 x5x 25x10-4

47 x 15
x 102
14 x 5 x 25

= V402.8 = 20.07
Now mL = 20.07

Q = X147 x 0.15 x 14 x 125 x 10-4 (100 — 20) tanh 20.7


= 0.35 x 80 x 1
= 28 W

26. One end of a long rod 3.5 cm in diameter is inserted into a furnace with other end
projected outside the furnace in air. After steady state is reached, the temperature
of the rod is measured at two points 180 mm apart and found to be 180°C and 145°C.
The atmospheric temperature is 25°C. If the heat transfer coefficient is 65 W/m2°C,
calculate the thermal conductivity of rod. Assume that the end of the fin is insulated.

Temperature distribution is
0 cosh (m (L — x))
19 0
= cosh mL
I 56 Heat and Mass Transfer

at x = L, Or, — 1
00 cosh mL
OL = 180 — 145 = 35
00 = 180 — 25 = 155
35 _ 1
155 cosh mL

cosh (mL) = 155 = 4.42


35
mL = 2.166
m — 2.166 — 2.166
L 0.180
= 12.03
65 x rd
2 hP 65 x 4 260
m = k x gd 2 = =
kA = 4 k xd kx 35x10-2

k= 260 = 51.33
3.5 x 10-2 x (12.03)2

27. The both ends of a 6 mm diameter copper rod (U — shaped) having k = 330 W/mK are
rigidly connected to a vertical wall as shown in the figure. The wall temperature is
constant at 110°C. The developed length of the rod is 50 cm and is exposed to air at
30°C. The value of h = 30 W/m2K. Find: (i) temperature at centre of the rod, and (ii)
heat transfer by the rod.
(PU — 1996)

T, = 100°C
T,. = 30°C

Guidance: At the centre of the fin, temperature would be constant due to symmetry.
U-shaped fin can be considered to be two fins of length 25 cm each (total length = 50 cm)
which are insulted at tip.
TL — T 1
To — Too — cosh (mL)
TL — 30 _ 1
100 — 30 cosh (mL)
30 x ird — 30 x 4
m2 = hP —
kA 2rd2 330 x d
330 x
4
Heat Transfer from Extended Surfaces I 57

120
330x6x10-3
m= 7.78 m-1
Now mL = 7.78 x 25 x 10-2 = 1.945
70
TL,— 30 =
cosh (1.945)
TL, = 30 + 19.6
= 49.6
Q = 211hPkA (To — Ta)tanh (m • L)

= 2 1130 xxx6x 10-3 x 330 x x (6 x10-2)2 x 70 x tank 1.945

= 9.76 W

28. How can fin effectiveness be increased?

Fin effectiveness is
Qfin A/hP kA (To — Ta)
E=
Qwithout fin hA(To — Tot)
)1/2
kP
— (hhA
Hence, effectiveness can be increased by:
(a) Using material of higher thermal conductivity (k)
(b) Increasing the ratio of perimeter (P) to cross-sectional area (A)
(c) Using lower 'h' i.e. natural convection instead of forced convection.

29. What is the reason for the error in the measurement of temperature of fluid flowing
through a pipe when measured by a thermometer placed in the thermometer well of
the pipe? How can error be reduced?
`T0' at pipe
ii
Thermometer well 11 wall

Liquid

.V1h. 1.% 1121.


I 58 Heat and Mass Transfer

A thermometer well is a small tube welded radically to a pipe through which a fluid whose
temperature is to be measured is flowing. The well is filled with little of other liquid and then
thermometer is dipped in this liquid. The liquid receives the heat from the fluid flowing
whose temperature is being measured.
The heat flows from the fluid to liquid through the wall of the well. The temperature (TL)
recorded by the thermometer is that of the liquid at the bottom of the well and it will be less
than flowing fluid temperature (Ta). The temperature in the well will reduce as we move
towards the top of the well or towards the wall of tube in which the well is located where
temperature is 70'. Hence, heat will flow from the bottom of the well to the wall of the
tube in which the well is located. Therefore, the well can be considered a fin but with a
difference that the heat is flowing from the surroundings to the base (wall of the tube).
Ta

Area =irxdxt
Ta t= thickness of well wall
Perimeter = irxd

Well acts as a fin

The temperature distribution on the wall of the thermometer well is same as that in a fin
of finite length IL
TL —TOO _ 1
To — Ts., h
cosh (mL) + — sinh (mL)
mk
As h sinh mL is small and it can be neglected. Hence, we have
mK
TL — Tso . 1 ..:. 1 ..:__. 1 1 kA
To — T cosh mL mL L 1 hP
kxjrdxt — 1 1k•t
L hxrd LN h
Now the thermometer is measuring II' instead of To.and quantity (TL — T) is a error in
the measurement.
The error in the measurement can be reduced by following ways:
(a) Well material should have moderate thermal conductivity.
(b) Keeping thickness of wall (t) of the well as small as possible.
(c) Keeping length of well as large.
(d) Using this method for measuring temperature of the fluid having low heat transfer
coefficient.

30. Temperature of the air flowing in a tube is measured with the help of a thermometer
placed into a thermometer well filled with oil. The well is made of stainless steel
Heat Transfer from Extended Surfaces I 59

(k = 55.5 W/mK), 12 cm long and 0.15 cm thick. The surface heat transfer for air and
well is 23.5 W/mK and temperature shown by the thermometer is 360°K. Evaluate
the measurement error if the temperature at the well of tube is 315°K.
Temperature distribution is
TL - Too = 1
To - Too cosh mL
1 23.5 x Ird
m = lhP
i kA A 555x7rdxt

_ 235
A; 55.5 x 0.15 x 10-2
= V282.28
= 16.8
mL = 16.8 x 12 x 10-2
=2
360 x T 1 1
315 x Tso cosh (2) - 3.76
= 0.266
360 - Too = 83.79 - 0.266 Too
or 0.734 Too = 360 - 83.79
= 276.21
T - 276.21
- 0.734
= 376.3°K
Measurement error = 376.3 - 360
= 16.3°C

31. A mercury thermometer is placed in a oil well for the measurement of temperature
in a pipe. The well is made of steel (k = 50 W/m°C) and is 14 cm in length and 1.0
mm in thickness. The temperature recorded by well is 100°C while pipe wall
temperature is 50°C. If the heat transfer coefficient between the air and well wall
is 30 W/m2°C, calculate the true temperature of air.
(UPTU - 2004 - 5)
Temperature distribution is
TL - T
= 1 (mL)
To - Too cosh
Ph r h x rcd
mL = kA = k x rcd x t
I 60 Heat and Mass Transfer

= Li — 0.14 J 30 .,
kxt v50x1x10—
= 3.42
Now we have
100 —T ___ 1 _ 1
50 — T cosh 3.42 15.3
1530 — 15.3 To. = 50 — To.
14.3T_ = 1480
To. = 103.5°C
32. A thermometric well is placed in a pipe having diameter of 55 mm. Pipe well
temperature is 100°C and 'h' inside is 300 W/m2K. Thickness of thermowell is 1.2
mm and its length is 50 mm. Thermal conductivity of thermowell material is 30 W/
mK. If the temperature of gas flowing through the pipe is recorded by thermometer
as 210°C, determine the true temperature of gas. If the error in gas temperature is
to be reduced by 60% by increasing the length of thermowell, find the new length
of thermowell.
Temperature distribution is
TL —Ts. 1
To — T., cosh mL

m .I h = l 300
-3
I k•t i 30x1.2x10
= 91.29
mL = 91.29 x 50 x 10-3
= 456.45 x 10-2
= 4.56
210— T. _ 1 __ 1
100 — T — cosh 4.56 48
10080 — 48T_ = 100 — To.
47T = 9980
T.. . 9980
47
= 233.6°C
Error = 233.6 — 210
= 23.6°C
Allowable error 40% of present error = 0.4 x 23.6
= 9.44
Now 71— Too = 9.44
9.44 __ 1
233.36 —100 cosh (91.29 x L)
Heat Transfer from Extended Surfaces I6I

cosh (91.29L) = 133.36


9.44
= 14.13
91.29L = 3.32
or L 91.29 = 0.357 m

33. A thermometer well 2 cm in diameter 1 mm thick is made of steel (k = 30 W/mK and


it is used to measure the temperature of air flowing through a pipe. Calculate the
minimum length of pocket so that the error is less than 1% of applied temperature
difference. Air temperature is 300°C and h = 100 W/m2K.

Temperature distribution is
TL —7' 1
To — Tso cosh mL
Now error = TL — Too = 1% of applied temperature
1 x (To — 7',,,)
= 10
0
1 _ 1
cosh mL 100
or cosh mL = 100
or mL = 5.3
m= 1h
1 k•t

= 1 100
1 30 xlx10-3
= V33.33 x 102
= 57.7
mL = 57.7 x L
= 5.3
or L = 5.3 = 9.19 x 10-2
57.7
= 91.9 mm

34. Determine the energy input required to solder together two very long copper wire
pieces of diameter 1.7 mm. The melting point of solder is 200°C and environment
temperature is 20°C. Take 'h' for environment as 16 W/m2K and k for wire material
as 330 W/m°C.

Guidance: The problem is nothing but a heat transfer by two fins with the solder thickness
acting as hot surface at base temperature at 200°C and two pieces of wire are acting as long
I 62 Heat and Mass Transfer

fins transferring heat to the environment. The heat transfer for one fin is calculated which
is later doubled to find total heat transfer from the solder to the environment through two
long pieces of wire being soldered as shown below:
Solder acts as base
plate at 200°C

Wire Wire

Ta = 20°C

Soldering of wires: two fins

For long fin, heat transfer by it is


Q = . \ I kA Ph (T, - E,‘)

rd2 ir x (1.7) 2
A= x 10-6 = 2.27 x 10-4 m2
4 4
P = ird = x x 1.7 x le = 5.34 x 10-3 m
Q = \1330 x 2.27 x 10-4 x 5.34 x 10-3 x 16 x (200 - 20)
= 1.44 W
Heat lost by both wire type fins is
Qtotai = 2 x 1.44
= 2.88 watt
In order to solder and maintain the temperature of solder at 200°C, we have to supply 2.88
watt of energy to the melted solder.

35. A cutting tool of 60 mm diameter is to be grounded for sharpening. The tool has
length of 1 meter. During grinding 40 watt of energy is dissipated to the tool which
is convected to the air at 20°C. If h = 7.1 W/m2°C and 'le of tool material = 42.4 W/
m°K, find the temperature of the rod when grinding is taking place.

Guidance: The problem can be visualized as a fin consisting of tool which is convecting
heat out through its surface while tip of the tool is at a constant temperature To

tanh (mL) + h
km
Qtool = V hP kA (7's - Tc.c) [
h
1+ — tanh (mL)
m

Grinder Ta = 20°C

Grinder and cutting tool: fin


Heat Transfer from Extended Surfaces I 63

(60 x 10-3)2
A = rd2 = ir x = 2.83 x le m2
4 4
P = ird = ir x 60 x le . 188.5 x 10-3 m

m liP — 17.1 x 1885 x 10-3


_ Il kA 1 42.4 x 2.83 x 10-3
= 3.34 m-1
Now mL = 3.34 x 1 = 3.34
h = 7.1 = 5 x 10-2
km 42.4 x 3.34
Putting these values in the equation of heat transfer, we get
i [ tanh 3.34 + 5 x 10-2
Qtooi = 0.1 x 1885 x 10-3 x 42.4 x 2.83 x 10-3 x (To — 20) x
1 + 5 x 10-2 tank 3.34
40 = 0.4 x (To — 20) [1]
or 40 = 100
To — 20. _
04
or To = 120°C

36. An experimental arrangement for measuring the thermal conductivity of solid


material involves the use of two very long rods that are equivalent in every aspect
(dimension, shape, etc.) except that one is fabricated from standard material of
known conductivity kA while the other is fabricated from the material whose thermal
conductivity kp is desired. Both rods are attached at one end to a heat source of fixed
temperature Tb, are exposed to a fluid of temperature T.. and are instrumented with
thermocouple to measure the temperature at fixed distance 'x', from the heat
source. If the standard material has kA = 200 W/mK and measurements reveal the
value of TA = 75°C and TB = 60°C at x for Tb = 100°C and T..= 25°C, what is thermal
conductivity kp of the test material?
(UPTU — 2007-8)

T,.= 250°C
kA = 200 W/mk
A ..--------

Tb = 100°C
kB
B
x

Temperature distribution is
T — Tc.,
= e-`"X where m = ithP
Tb— Too \i' kA
I 64 Heat and Mass Transfer

For material 'A'


75 - 25 hP
e-mix where m1 =
100 - 25 = kA x A

or 75 = 0.405
mix = log 50
For material fi
60 - 25 - 2x hP
e-m where m2 =
100 - 25 kp x A

or m2x = log 75 = 0.762


35
Now dividing equation (i) to equation (ii), we get
Pit = 0.762 _ 1.88
m1 0.405

. kfl = 1.88
kA
or kp = 1.88 x kA
= 1.88 x 200
= 376 W/mK

37. Define hyperbolic functions.


Hyperbolic functions are:

ex - e-x
1. sinh x =
z
ex ± e-x
2. cosh x =
z
ex _ e-x
3. tank x =
ex + e-x

38. What are the derivatives of the hyperbolic functions?


The derivatives are:

(sinh x) = cosh x
1. dx
d (cosh x) = sinh x

2. dx
x) = 1
3. dx cosh2 x
Heat Transfer from Extended Surfaces I 65

Hyperbolic Functions

x sink x cosh x tank y


0.0 0.000 1.000 0.000
0.1 0.100 1.005 0.100
0.2 0.201 1.020 0.197
0.3 0.305 1.045 0.291
0.4 0.411 1.081 0.380
0.5 0.521 1.128 0.462
0.6 0.637 1.186 0.537
0.7 0.759 1.255 0.604
0.8 0.888 1.337 0.664
0.9 1.027 1.433 0.716
1.0 1.175 1.543 0.762
1.1 1.337 1.670 0.801
1.2 1.510 1.811 0.834
1.3 1.700 1.971 0.862
1.4 1.904 2.151 0.885
1.5 2.136 2.352 0.905
1.6 2.376 2.577 0.927
1.7 2.646 2.828 0.935
1.8 2.942 3.108 0.947
1.9 3.268 3.418 0.956
2.0 3.627 3.762 0.964
2.1 4.022 4.144 0.970
2.2 4.457 4.568 0.976
2.3 4.937 5.037 0.980
2.4 5.466 5.557 0.984
2.5 6.050 6.132 0.987
2.6 6.695 6.770 0.990
2.7 7.406 7.474 0.991
2.8 8.192 8.253 0.993
2.9 9.060 9.115 0.994
3.0 10.018 10.068 0.997
3.5 16.543 16.573 0.998
4.0 27.290 27.308 0.999
4.5 45.003 45.014 0.999
5.0 74.203 74.210 0.999
6.0 201.71 201.72 0.999
7.0 548.32 548.32 1.000
8.0 1490.50 1490.50 1.000
9.0 4051.50 4051.50 1.000
10.0 11013.20 11013.20 1.000
I 66 Heat and Mass Transfer

39. A cylinder of 1 m long and 50 mm in diameter is placed in an atmosphere at 45°C.


It is provided with 10 longitudinal straight fins of a material having k = 120 W/mK.
The height of 0.76 mm thick fins is 1.27 cm from the cylindrical surface. The heat
transfer coefficient between cylinder and atmosphere air is 17 W/m2K. Calculate the
rate of heat transfer and the temperature at the end of fins if the surface
temperature of cylinder is 150°C.
(Annamalai University, 2004 — 5)

5= 0.76 mm

h= 1.27 cm

(a) Front (b) Side view

Considering fin having short length and its end insulated, we have—
T—T cosh m(L— x)
=
To _ T cosh (mL)

= ihP
Now m
N kA
p = 2 x (length of cylinder + 8)
.-- 2 x 1 = 2
A = Lx S
= 1 x 0.76 x 10-3
= 76 x 10-5 m2
1 17 x 2
m=
120 x 76 x10-5

= V3.728 x 102
= 19.3 m-1
Temperature at end when x = L is—
T — 318 cosh 19.3 (L — L)
=
423 — 318 cosh (19.31x 12.7 x 10-2)
T —318 = 1
or
105 1.03
Heat Transfer from Extended Surfaces I 67

or T= 102 + 318
= 420 K
Q = I f hPkA (To — 11,) tanh (mL)
= (17 x 2 x 120 x 76 x 10-5) (423 — 318) tanh (19.3 x 12.7 x 10-2)
= 3.1 x 105 x tanh (2.451)
= 3.1 x 105 x 0.983
= 320 W
Now for 10 fins, heat transmitted is
Qtotal =10 x 320
= 3.2 kW

40. A circumferential rectangular profile fin on a pipe of 50 mm outer diameter is 3 mm


thick and 20 mm long. The thermal conductivity is 45 W/mK. The convection
coefficient is 100 W/m2K. Base temperature is 120°C and the surrounding
temperature is 35°C. Determine: (i) heat flow rate, (ii) fin efficiency and (iii) fin
effectiveness.
(Annamalai University 2007 — 8)

✓ A r/A
i_
T
✓ A II
s
i' nc127—
%/A r/.
-t- k— L —.-1
(a) Side view (c) Equivalent fin

(b) Top view

Circumferential rectangular fin

. ihP
m
l kA
p = 2(7rd + 5) --. 27rd
A = 7rd x 8 = 7rd8
P . 27rd . 2
A ,rdx5 8
I 68 Heat and Mass Transfer

100 2
. IN =
11 45 x ( 5

I 100x 2
=
I 45 x 0.003
= 1/14815
= 38.49 m-1
As fin is short, we can assume that the end is insulated.
Q= VhPkA x (To — Too) tanh (mL)

= V100 x (27r x d) x 45 x (IrchS) x (120 — 35) tanh (mL)


But mL = 38.49 x 0.02
= 0.78
Q = 11100x 2x 45x7r 2 x d 2 x (5 x (85) x (tanh 0.78)

= 119000 x 7r2 x (0.05)2 x 0.003 x 85 x 0.65


= 815 x 10-3 x 85 x 0.65
= 45 W
tanh (mL)
Fin efficiency Thin =
mL

= tanh0.78 = 0.65
0.78 0.78
= 83.33%
tanh(mL)
Fin effectiveness (E) =
/ hA
\" kP

0.65 _ 0.65 _ 0.65



100x8 100 x 0.003 0577
1
N 45 x 2 90
= 1.13

41. A brass rod (k = 133 W/mK), 100 mm long and 5 mm in diameter extends horizontally
from a casting at 200°C. The rod is in air at T = 20°C and h = 30 W/m2K. What is
the rate of heat transfer to air and temperature of the free end of the rod?
(UPTU — 2008 — 9)
Heat Transfer from Extended Surfaces I 69

I hP
m 1 il —
= 1 kA
p = ird = iv x 0.005 m 0.0157 m

A= 5 d 2 = 1.96 x 10-5 m2
4
P ird 4 4
= - - =800
A 7r
—xd 2d 0.005
4

V300 x 800
m=
133
= V1804.5
= 42.49 m-1
mL = 42.49 x .01
= 0.425
As fin is finite length and we can assume that the end is insulated, then-
Q = ,i 1hPkA x (To - Tos)tanh(mL)

f ir
=
AI 300 x (iv x 0.005) x 133 x (-4 x 0.0052) x (200 - 20) x tanh (0.425)
= 0.11 x 180 x 0.405
= 8.036 W
T -To, cosh m(L - x)
Now =
To - T cosh (mL)
Now at x = L, we have
T - 20 = 1
200 - 20 cosh 0.425

T = 20 + 180
1.093
= 184.72°C.

42. Calculate the amount of anergy required to solder together two long pieces of base
copper wire 1.625 mm diameter with solder that melts at 195°C. The wires are
positioned vertically in air at 24°C. Assume that the heat transfer coefficient on the
wire surface is 17 W/m2K and thermal conductivity of wire alloy is 335 W/mK.
(Annamalai University 2002 - 3)
I 70 Heat and Mass Transfer

Assume wire as long fin, then—


Q = ‘IhPkA x (To — To0
Now p = r d = iv x 0.001625
= 5.11 x 10-3 m
gc/ 2
A= = x (0.001625)2
4 4
= 2.07 x 10-6 m2
. Q = \,I17 x 335 x 5.11x 10-3 x 2.07 x 10-6 x (195 — 24)
= 7.76 x 10-3 x 171
= 1.3272 W

43. Two long metal rods of same diameter are placed on a surface with the end
temperature being 300°C. The rods extend to the room where the temperature is
30°C. The temperature at 25 cm from the hot end in one rod is found to be 200°C
while in other rod, this temperature is attained at 400 cm from the hot end. Find the
ratio of conductivities of these rods.
(Annamalai University — 1999 — 2000)

x1 = 25 cm -.1
T, = 300° C T = 200° C T,. = 30° C

-..- x2 = 400 cm
T = 200° C
Two Fins

For rod-1, we have


T —T = mi xi
e
To — Tc,
200— 30 = e 0.25.m1
or
300— 30
or m1 = 1.85m 1
For rod-2, we have
200— 30 . eo4 m2
300— 30
or m2 = 1.567m1
Heat Transfer from Extended Surfaces I7I
Now we have
Q = V hPkA (To - Tj

I hP
= — xkA(To-Tso)
kA
= m x kA(To - TO
Qi = m1 x ki x A(To - Too)
Q2 = M2 X k2 X AM - TO
Now Q1 = Q2
k1
= m2 = 1.567
k2 m1 1.85
= 0.625

44. Calculate the amount of energy required to solder together two very long pieces of
bare copper wire 1.5 mm in diameter with solder that melts at 190°C. The wires are
positioned vertically in air at 20°C. Assume that heat transfer coefficient on wire
surface is 20 W/m2°C and thermal conductivity of wire alloy is 330 W/m°C.
(UPTU - 2009 - 10)
Assume wire as long fin, then
Q = V liPkA x (To - TO
P = ird = ir x 1.5 x 10-3 = 4.71 x 10-3 m
Ord 2 7r x (15 x 10-3)2
A= = = 1.766 x 10-6 m2
4 4

. Q = V20 x 4.71x 10-3 x 330 x 1.766 x 10-6 x (190 - 20)

-
- N15489 •8 x 10-8 x 170
= 74.1 x le x 170
= 1.26 W
Chapter 5
TRANSIENT HEAT CONDUCTION

A LUMPED BODY A TIME CONSTANT


A BIOT NUMBER A RESPONSE OF THERMOCOUPLE
A FOURIER NUMBER A SENSITIVITY OF THERMOCOUPLE
A GEOMETRIC FACTOR A LUMPED ANALYSIS
A CHACTERISTIC LENGTH A HEISLER CHARTS
A SEMI INFINITE SOLIDS A GAUSSIAN ERROR FUNCTION

INTRODUCTION

Transient heat conduction is nothing but the unsteady state heat conduction. In unsteady state,
the heat flow as well as the interior temperature distribution of the system changes continuously
dT dT
with time. It means that — # 0 during transient heat conduction while — = 0 for steady state
dt dt
heat conduction. The transient heat conduction occurs while: (i) heating or cooling of metal
billets, (ii) cooling of IC-engine cylinder, (iii) cooling and freezing of food in canning industry,
(iv) brick manufacturing, (v) vulcanisation of rubber, and (vii) starting and stopping of various
heat exchange units in power installation. If solids have smaller thermal conductivity, then there
is interior temperature distribution while heating or cooling. In case solids have high heat
L
conduction or low internal heat conduction resistance (- while their surface convection
kA

resistance is high I R , then the temperature throughout the solids can be considered to be
AJ
uniform and they form lumped heat capacity systems.
Transient Heat Conduction I 73

1. What is transient or unsteady heat conduction?

When heat energy is either removed from or added to a body, its internal energy changes,
resulting a change in its temperature at each point within the body. This temperature at any
point within the body is a function of time and location of the point in the body in the
direction of heat flow during the transient period. The conduction occurring during the
transient period is called transient conduction or unsteady state conduction. Hence, in
unsteady state—
T = f (x, t)
= function of direction & time

2. Explain the energy balance in a body during transient heat conduction.

During transient heat conduction, the energy balance is—


The net rate of heat transfer from the body to surroundings = the net rate of internal energy
change of the body

. hA(T — T) = —mCp
aT
at
where, h = coefficient of convective heat transfer
A = surface area of body
T = temperature of body
aT = change of temperature of body per unit time
at
C, = sp. heat of material of body
m = mass of body.

3. Explain the main characteristics of unsteady or transient heat flow.

The unsteady state of heat flow has always variation in heat flow and temperature. The
variation can be continuously increasing/decreasing (non-periodic) or periodic. The heat
treatment processes like quenching, annealing and normalizing are such non-periodic
unsteady state of conduction in which heat flow and temperature of the body increases or
decreases continuously till steady state reaches. The heat flow in a building between day and
night is periodic unsteady state involving periodic variation in heat flow and temperature.

4. What is the analytical method of solving transient heat conduction?

If temperature distribution of the body is given at certain instant of time, then we can find
out (i) heat flow inside or outside the system: (ii) rate of change of temperature with time,
and (iii) rate of change of energy within the system. Consider a plain wall with temperature
distribution at any instant is—
T = a + bx + cx2 + dx3
I 74 Heat and Mass Transfer

where a, b, c and d are constants. The general differential equation for one-dimensional heat
conduction is—
a2T qg _ 1 aT
ax2 k a at
where, a = thermal diffusivity of the system
qg = internal heat generation per unit volume.
k = thermal conductivity of the system.

a
q1 q2
k
qg

x= 0 x=L
L

We can now find out the following—


(a) The rate at which heat is entering and leaving the wall is—
(a
qin = —kA
a x Jx=0
gout =— a x d xi
in
k + a x (gin)
(aT
= —kA
a x )x=L
(b) Rate of change of temperature with time is given by—
T [a2 T qg]
=a +
at ax2 k
(c) Rate of change of energy within the wall can be given by—
Estored = (q1 q2) qg(A • L)
where A = area and L = length of the wall.
(d) Maximum temperature within the wall is found out by—
a (aT )
=0
ax at

5. The temperature distribution at certain instant of time in a plane wall, 50 cm thick


is given by the relation—
T = 450 — 500 x +100x2 + 150 x3
Transient Heat Conduction I 75

where temperature T is in degree Celsius and x is in metres measured from hot


surface. The thermal conductivity of the wall material is 10 W/mK. Calculate the
rate of heat energy stored per unit area of the wall at that instant of time.
(UPTU — 2002 — 3)
Temperature distribution is—
T = 450 — 500 x + 100 x2 + 150 x3
a T = —500 + 200 x + 450x2
ax
(aT
Qin = —kA
axl.o
= —10 x 1 (-500 + 200 x + 450 x2 )x.o
= 5000 W

Qout = —kA( a T
ax x=0.5
= —10 X 1 (-500 + 200 x + 450 x2 )x=o.5
= —10(-500 + 200 x 0.5 + 450 x 0.52)
= 2875 W
Heat stored = Qin — Qout
= 5000 — 2875
= 2125 W

6. In previous problem, find the time rate of temperature change at x = 0 and x = 0.5
if a uniform heat generation (qg) of 500 W/m3 is present in the wall and emissivity
of wall a( — k is 6 x 10-3 m25.
p Cp

Time rate of temperature change is—


aT _ a [a2T _E gg ]
at — k (i)
axe
T = 450 — 500 x + 100 x2 + 150 x3
On differentiating, we get—
Now
a T = —500 + 200 x + 450x2
ax
Again differentiating, we get—
a2 T
= 200 + 900 x
axe
Put the values in equation (i)
a T = 6 x 10-3[200 + 900 x + 500 1
at 10
= 6 x 10-3 [250 + 900 x]
I 76 Heat and Mass Transfer

(a_T ) 6 x 10-3 x 250


d tx=0 =
= 1.5°C

= 6 x le [250 + 450]
Cat/ x=03
= 6 x 700 x 10-3
= 4.2°C

7. What is a lumped heat capacity system?

If the internal conductive thermal resistance of a body or system is negligible as compared


to the surface convective thermal resistance, then the system is called a lumped heat
capacity system. For example, if the size of a body is very small, the temperature gradient
exists in the body is negligible, then body can be assumed at constant temperature.

8. Find the expression of cooling of a lumped body.


Or
Find the temperature-time history of a lumped body.
Or
Show that for lumped heat capacity system
hA
T- pC V xT
—e P
- (UPTU - 2002 - 3, 2007- 8, 2009 - 10)

Lumped system analysis means that the temperature gradient in the body is negligible and
body can be assumed to have uniform temperature everywhere at an instant of time. The
analysis of unsteady heat transfer with negligible temperature gradient is called lumped
system analysis.
Consider a lumped body (volume V', surface `A', density 'p', thermal conductivity IC'
and sp heat `Cp') at initial uniform temperature 'Ti' and it is suddenly immersed in a fluid
having temperature To; . The heat is dissipated by convection into the fluid from the surface
of the body. The rate of heat flow out from the body is equal to the rate of decrease of
internal energy of the body.

hA(T — Too) = —MC


aT
paz

= —pVC
aT
Transient Heat Conduction I 77
Now take =T—T
ae aT
at = at
ae —hA
at
= pVCp
On integration, we get—
—hA
or log 0 = t + Cl
p

We know T = Ti at t = 0 or Ot=o = = —
log 0i = C
Putting the value of C1, we get—
hA
log 0 = t + log Oi
pVC p
hA
or log =
Oi pVCp
hA
pvcp t
or e
Oi
hA
T— pVCp t
or
Ti —T = e
The temperature of the body decreases with time as shown below-

E T

lime (t) —.-


Temperature — time history of the lumped body

9. Find the expression for heat transfer from a lumped body.

We know the temperature distribution any instant is given by—


hA
T —T pVC
t
=e P
Ti—T
I 78 Heat and Mass Transfer

The heat transfer at that instant is given by—


qt = hA(T — Too )
Heat transfer for time at is—
qt clt = hA(T — Too)a t
hA
t
But T — Too = ( Ti — T )e PVCP
hA
T
qt at hA(Ti —T )e Pvc P at
Total heat flow from initial instant to time t is—
-hA
„ t
qt at = hAf (Ti — Too) e P' 'P at
0 0
hA
„t
Qt = pVCp(Ti — T) 1— eP"

10. What do you understand from Biot number (Be), Fourier number (Fo) and
geometrical factor (GF)? What is the value of Biot number for lumped system
analysis?

Biot number is the ratio of internal conductive thermal resistance to surface convective
resistance
Internal conductive thermal resistance
B=
Surface convective resistance

(V
A)
_ kA
—1
hA
h • Is
where — = characteristic length of the body (ls)
A
The lumped system analysis can only be applied when Biot number is equal or less than
0.1 i.e. 0.1. This implies that internal resistance to heat flow is very small as compared
to convective resistance to the heat flow.
The Fourier number is the ratio of the rate of heat conduction to the rate of thermal
energy storage in the solid.
Rate of heat conduction
Fo =
Rate of thermal energy storage in the solid
AT
kA
IcAt
Fo =
(AT) p (A x Is)Cp is
pVCp
t
Transient Heat Conduction I 79

at
2 2
P Cp (4) (4)
Fourier number signifies the degree of penetration of heating or cooling effect through
a body.
Geometric factor (GF) is the ratio of volume to area. It is also called characteristic length
of the body. The characteristic length (1s) for certain common body shapes are—
(a) For infinite (large) plate of thickness 'I,' exposed to convection environment on both
sides—
Volume AL_ L
=
=
is Area 2A 2
(b) For a solid cylinder of radius R and length 'L—
K R2 L RL
4=
2K R2 + 2K RL 2(R+ L)
(c) For a long cylinder of radius 'IV where 'I,' » 'R'—
KR2 L R
Is = 2
2 rRL
(d) For a solid sphere of radius 'R'—
3
KR
4= 2
2KR 3
(e) For a cube of side 'L'—
/ L3 L
=
6L2
- 6

11. What is a time constant?

For a lumped body, the temperature difference of a body and fluid decays exponentially as
hA
T —T pVC x t
—e P
Ti —T
hA
The exponent quantity I p xt in the above equation is a dimensionless quantity as
pV C
pV Cp
term has unit of time. In case time period (t) becomes equal in magnitude to
hA
(pVCp
, then exponential temperature decay equation becomes—
hA
T —Too _1 pV C p
=e where =t=
— Too hA
= 0.368
I 80 Heat and Mass Transfer

T— Tso
or 1 — 1 — 0.368
Ti — Tso
—T
or — 0.632
Ti — Tso
or — T = 0.632(Ti — To0
Hence, time constant is time required by a temperature measuring instrument to record
a temperature difference equal to 63.2% of initial temperature difference.

12. What do you understand from the response of thermocouple and the sensitivity of
thermocouple?

pvC
e r where 1-
hA p

i 8
0.632

T1 T2 2-3 T4
-A- Time constant (t)
Transient temperature response to different time constants (r)

Thermocouple is a temperature measuring instrument. The response time of a thermocouple


is defined as the time taken by the thermocouple to indicate the source time. The sensitivity
of thermocouple is defined as the time required by the thermocouple to indicate 63.2% of
the initial temperature difference. The temperature difference is reduced by 63.2% after one
time constant. Hence, a thermocouple is sensitive in case it can have low time constant and
thermocouple can achieve 63.2% of the initial temperature difference quickly. The low value
( pVCp
of the time constant is desirable and it can be achieved by—
hA
V
(a) Decreasing characteristic length ( — of wire of thermocouple i.e. radius of wire is
A
small
(b) Using light metal for wire of thermocouple having low density and low specific heat
(c) Increasing the heat transfer coefficient.

13. Find the expression for temperature of plane wall at any instant using lumped heat
analysis.
Transient Heat Conduction I8I
Consider a plane wall of thickness having one surface subjected to constant heat flux
`q' and other surface is exposed to convective environment. The wall has Biot number (Bi)
0.1
h

E h(T— Teo)
q

x=0 x=L

and lumped analysis can be applied. The wall has initial temperature Ti and convective
environment has temperature T._
Now applying the energy balance for the wall, we have—

A x q — hA(T — Too) =mC


dT
P at

Put 0 =T
ae aT
at = at
d0
A x q — hAO =mC
P a
ad Axq hA0
or
at mcp mcp
ae
or = a — h0
at
or at = ae
a —b0
Axq hA
where a= &b=
mCp mCp
We know 0 = Oi at t = 0
e
f at =
Now
o e, a—b• 0
a—bOi l
log
a+b0
or t— 1
b

a +bOi _bt
or —e
a —b0

14. A copper plate 2 mm thick is heated upto 400°C and then quenched in water at 25°C.
Find the Biot number and verify if the lumped heat capacity analysis is applicable.
I 82 Heat and Mass Transfer

Also find the: (i) time constant, and (ii) time required for the plate to reach the
temperature of 40°C. The data given: h = 100 W/m2K, C,, = 400 J/kgK, k = 385 W/
mK, plate size = 25 m x 25 m, p = 8800 kg/m3.
(UPTU - 2003 - 4)

L • B • (5 (5
Characteristic length is = V where = thickness
A = 2 L•B = 2

= = 1 mm
2
hls 100 x 1 x 10-3
Biot number Bi = k = 385
= 2.5 x < 0.1
Hence, lumped heat capacity analysis can be applied as Bo < 0.1
pVCp pCp x is
Time constant (r) =
hA - h
8.800 x (400) x 1 x 10-3
100
= 35.2 sec
Now exponential temperature decaying equation is-
T - Tso _dr
=e
-
40 - 25 - e-t/35.2
400 - 25
-t
= log 15
35.2 375
or t = 35.2 log 25
= 113.3 sec

15. Balls of 12 mm diameter are anneded by heating to 877°C and then slowly cooling
to 127°C in air at a temperature of 52°C and h = 20 W/m2K. Calculate time
required for cooling process for the balls. Take p = 7800 kg/m3, k = 40W/mK and
C,, = 600 J/kg K
(UPTU - 2005-6)

4 irR3
Characteristic length is = —V_ 3 R
A - 4r R2 3

= 6- = 2 mm
3
Transient Heat Conduction I 83

his 20 x 2 x10-3
Biot No (Bi) =
k 40
= 1 x 10-3 < 0.1
Since Bi < 0.1, lumped heat capacity analysis can be applied
-hA t -h
T -T pCpV = pCp x1, xt
e e
- Tso
xt
127 - 52 75
pCp l,
877 - 52 825
or x t = log 825
pCp is 75
= 2.39
2.39 x p x Cp x ls
t= h

2.39 x 7800 x 600 x 2 x 10-3


20
= 1122.17 sec
= 18.7 min

16. A steel ball 5 cm in diameter is heated to a temperature of 900°C and placed in still
surrounding atmosphere for cooling, if atmospheric temperature is 30°C, calculate
the initial cooling rate of the ball in °C/min. For steel take p = 7800 kg/m3, CP = 2
kW/kg • K Assume h = 30 W/m2°C.
(UPTU - 2004 - 5)
Equating conduction and convection heat transfer-

-mC a = hA(Ti - T-)


P atT

or
aT hA
(Ti Toc) = (Ti Lc)
at VC pcpis
is = characteristic length
=—V=R 5
A 3 = 6 cm
aT 30 (900 - 30)
at 7800 x 2 x103x 5 x10-2
6
30 x 870 x 6
7800 x 100
= 0.2 °C/sec
I 84 Heat and Mass Transfer

= 0.2 x 60 °C/min
= 30°C/min

17. A solid steel ball 5 cm in diameter and initially at 450°C is quenched in a controlled
environment with convective coefficient of 115 W/m2K. Determine the time taken
by centre to reach a temperature of 150°C. Take C,, = 420 J/kg K, p = 8000 kg/m3
and k = 46 W/mK.
(PU 2003)

Guidance: For lumped system, temperature is uniform throughout the body. However,
lumped system analysis is possible in case Biot number is less than or equal to 0.1.
Characteristic length = —V = = 5- cm
A 3 6
. hl 115 x 5 x10-2
B1 = s
k 6 x 46
= 0.021 < 0.1
Hence, lumped system analysis is possible. Now-
T -T _ e m-P" ;
/1 xt

T,- -
—115
2 xt
150 - 90 8000x420x5/6x10-
e
450 - 90 -
8000 x 400 x 5 x 10-2
t= x log 360
115 x 6 60
= 440.4 sec = 7.34 min

18. A person is found dead at 5 pm in a room whose temperature is 20°C. The


temperature of the body is measured to be 25°C when found, and the heat transfer
coefficient is estimated to be 8 W/m2K. Modelling the human body a 20 cm diameter
and 1.7 m long cylinder, calculate actual time of death of the person. Take
thermophysical properties of the body.
k = 6.08 W/mK, p = 900 kg/m3, C,, = 4000 J/kg • K
(NMU - 2002)

Human body can be modelled to


Transient Heat Conduction I 85

Characteristic length is = A
V
2
rR L RL
2 ir(R2 + RL) 2 (R + L)
15 x10-2 x1.7
2(15 x10 -2 +1.7)
= 0.069 m
The body temperature can be considered uniform in case Biot Number 0.1
. hl 8 x 0.069
B1 = =
ks 6.08
= 0.092
Now we can apply lumped system analysis
T —Tpcpisxt
Ti — T e
25 — 20 900 x 0.069 x 4000 x t
=e
37 — 20
17 x 3.23 x 10+5
t = log —
5
= 37943 sec = 10.54 hour

19. An egg with mean diameter of 4 cm is initially at 25°C. It is placed in boiling water
for 4 minutes and found to be consumer's taste. For how long should a similar egg
for same customer be boiled when taken from refrigerator at 2°C. Use lumped
system analysis and take thermophysical properties of egg as: k = 12 W/mK, h = 125
W/m2K C,, = 2000 J/kg K, p = 1250 kg/m3.
(PU — 1997)

Consider an egg being boiled as shown—

2
h = 125 W/m k

- 100°C

Characteristic length /s = V=R


3 = 0.02
3
I 86 Heat and Mass Transfer

Applying lumped system analysis, we have—


T—T epcpisxt
Ti —
—125
0 02 x(4 x60)
T x 100 1250 x2000 x
3
=e
25 —100
T= 100 — 75 x 0.165
= 87.5°C
The consumer requires egg with uniform temperature of 87.5°C.
Applying again lumped system analysis for the egg with initial temperature as 2°C
375 xt
T T = e moo
Ti — T
—375
87.5 —100 xt
= e 5000
2 —100
or t = 10g
98
x 5000
12.5 375
= 275.7 sec
= 4.6 min

20. A ball of 20 cm diameter with initial temperature 400°C is exposed to air of


temperature 25°C. If h = 80 W/m2K, calculate: (i) time required to cool the ball to
85°C, (ii) initial rate of cooling, (iii) instantaneous rate of heat transfer at the end
of one min. Take k = 40 W/mK, p = 7800 kg/m3, C,, = 450 J/kg K.

R = 1 x 10-3
Characteristic length Vs' = V = 3 3
= 3.33 x m
B. = hls 80 x 3.33 x10-3
k 40
= 6.66 x 10-3 < 0.1
Since Biot number is 0.1, lumped system analysis can be applied
—h
T— Too pC 1 xt
= e P'
—T
80 xt
85 — 25 7800 x 450 x3.33x10-'
e
400 — 25 —
7800 x 450 x 3.33 x 10-3
t= x log 375
80 60
78 x 450 x 3.33
x 10-1 x 4.094
8
Transient Heat Conduction I 87
= 5981.5 sec
= 1.66 hour
Now for initial rate of cooling, we have—
dT
—mC = hA(Ti —
P at
dT hA
or
at (pV)Cp(Ti —
V R
Characteristic length `/,' = A 3 = 3.33 x m
dT 80 3 (400 — 25)
at — 7800 x 450 x 3.33 x10-
80 x 375
— 2.567°C/s
78 x 45 x 333
Instantaneous heat transfer rate is—
qin = hA (T — To j
h xt
But T— = (Ti — To je PCP4
h xt
qin = hA(Ti — To )e PCPis
Here t = 60 sec
qin = 80 x 47r x (1 x 10-2)2 x (400 — 25)7800
—80 x 60

x 450 X e 3'33 x1°-3 x 7800 x 450


= 37.68 x e-13.41
= 25 W
Total energy transferred during first minute is—
ht
Cp l,
Q = pVCp (Ti — Toc)(1—e P

80x60

= 7800 x —4 7r x (10-2)3 x 450 x (400 — 25) x [1 _ e7800x450x3.33x10-3


3
= 7800 x —4 x 7v X 450 x 10-6 [1 com] x 375
3
= 7.8 x 0.15 x4 X7r(1 — 0.664) x 375
= 7.8 x 0.15 x 4 x 7V (0.336) x 375
= 1851 W

21. A thermocouple junction is in the form of a 4 mm diameter sphere. The properties


of the material are Cp = 420 J/kg K, p = 8000 kg/m3, k = 40W/mK, h = 40 W/m2K.
The junction is initially at 40°C is inserted in a stream of hot air at 300°C. Find: (i)
time constant, (ii) thermocouple is taken out from the hot air after 10 sec and is kept
I 88 Heat and Mass Transfer

in still air at 80°C. Assuming heat transfer coefficient in still air is 10 W/m2K, find
temperature attained by junction 20 sec after removing it from hot air stream.
(PU — 1955)

— 2 x10-3
Characteristic length is — V — R
A 3 3

h s 40 x3 x10-3
B = i =
k 40
= 6.67 x let 0.1
Hence, lumped system analysis is possible.
T—T h xt
pcp is
T—T = e

pC„Is 8000 x 420 x x 10-3


Time constant r = 3
40
= 56 secs
Temperature (T1) attained by junction after 10 secs when exposed to hot air stream
(Toc = 300°C)
Ti 300 5t6
here t = 10 secs
40 — 300 — e
— 300 _15(6)
e 0.84
40 — 300 —
Tl — 300 = —0.84 x 260
= 300 — 218.4
= 81.6°C
Now temperature attained by the junction in 20 sec when exposed to air at 30°C having initial
temperature of 81.6°C.
h2
T2 — 30 pc 4 xt
81.6 — 30 — e
10x20
8000x420xx10-3
3
=e
= 0.915
T2 = 30 + 51.6 x 0.915
= 30 + 47.21
= 77.21°C

22. A mercury thermometer bulb idealized as a sphere of 0.3 mm radius is used for
measuring the temperature of fluid whose temperature is fast changing. Taking the
Transient Heat Conduction I 89
following data: k = 120 W/mK a = 5 x 10-5 m2/s, h = 10 W/m2K, specify whether
thermometer is able to read the temperature faithfully if time for temperature
change of the fluid is 3 secs. If not, what should be the diameter of a thermocouple
to read the temperature of the fluid taking following data: k = 100 W/mK, a = 120
m2/s, h = 10 W/m2K.

0.1 x 10-3
Characteristic length is — R
3 3
= 0.333 x 10-4 m
hi 10 x 0.333 x10-4
B•= s=
k 100
= 0.333 x le 0.1
Hence, lumped system analysis is applicable
p Cp ( PC p ) k
Time constant = 1
h k h s
1 k
= — X—
a h xis
1 x 100
x 0.333 x 10-4
5 x10-5 10
= 2 x 10 x 0.333
= 6.66 sec > 3 sec
As time constant is more than the time of fluctuation of fluid, the temperature cannot be
recorded faithfully by the thermometer. It is possible with thermocouple with some other
radius having time constant as 3 sec.
=3=— 1 x— k xis
a h
i 3x120 x100
or — 100
= 36
Now is = 11 =36
3
or R = 36 x 3 = 108 mm
= 0.108 m

23. A household iron has a surface area of 0.05 m2 and is made of stainless steel having
weight of 1.4 kg and 500 W capacity. If h = 17 W/m2K on the surface and air at 30°C,
how long it takes iron to reach 110°C. Take p = 8000 kg/m3, C,, = 500 J/kg°C & k
= 20 W/m2C. Find out the Biot number.
I 90 Heat and Mass Transfer

h V hi pA
m
Bd— A—

20x
(8001x.50.05)
20
= 0.0375 0.1
a — b 0i _ e —bt
Now
a —b0

Aq 8c. b _ hA
where a=
mCp mCp
0.05 x 500 17 x 0.05
a= b=
1.4 x 500 1.4 x 300
a = 0.0357, b = 1.2 x 10-3
01 =T; —T = 0
0=T—T = 110 — 30 = 80
0.0357 =
0.0357 — 80 x1.2 x10-3

1.2 x 10-3t = log 0.0603


0.0357
= 0.524
t — 0.524 — 436.67 sec
1.2 x10--

24. A thermocouple junction of spherical form is to be used to measure the temperature


of the gas stream for which h = 400 W/m2K and k = 20 W/mK, C,, = 400 J/kg K,
p = 8500 kg/m3. Find: (i) junction diameter needed for the thermocouple to have time
Transient Heat Conduction I9I

constant of one second, (ii) time required for the thermocouple junction to reach
198°C if junction is initially at 25°C and is placed in a gas stream which is at 200°C.

Characteristic length = A
V = 4/3 x R23 = 1/3 R
4 7r R
pClg
Time constant r = P = 1 sec (given)

8500 x 400 x RI3


or =1
400
400 x 3
or R= — 3.53 x 10-4 m
8500 x 400
Hence, junction diameter d = 2R
= 2 x 3.53 x 10-4 m
= 0.706 mm
Now temperature distribution is—
T—T
=e
Ti — T
198 — 200 I
= e as time constant r = 1 sec
25 — 200
t = log 175
2
= 4.47 sec

25. How the transient heat conduction for a plain wall can be analysed with the help of
Heisler charts.
(UPTU — 2006 — 7)
Or
How is transient heat conduction in solids having finite conduction and convective
resistance (0 < Bi < 100) determined?

All practical heating and cooling of metal objects involve both internal
and external resistance. In lumped system analysis, we consider body
at uniform temperature, thereby neglecting the internal resistance to
heat transfer.
Heisler charts are used to solve the transient heat transfer problems
taking into consideration both internal and external resistance to heat
transfer. The charts are used when Bi > 0.1. Such chart solutions for
x=—L x=L
transient analysis are available for standard objects like rectangular, x=0
cylindrical and spherical shapes. Heisler charts are provided at the end
of this chapter.
I 92 Heat and Mass Transfer

Consider a plane wall of thickness `2L'. It has initial temperature 70' and it is suddenly
exposed to environment at temperature ' . Analytically we have seen that the temperature
of the object depends on Biot number, Fourier number and location of section

0.1
—8

0.01

I I I
1 2 3 4 8 12
as
F0 =
L2
Centreline temperature for a plane wall of thickness (2L)

hL)taking Ton
Heisler had drawn the graphs for different values of Bi = on
hL k To —1:
y-axis and af on x-axis. Centreline temperature can be obtained from this chart.
To determine the temperature at a location other than centreline, other chart is used which

have graphs for different values of taking ";


T' t-
on y-axis and on x-axis where
L
Tat) — hL
T(x, t) = temperature at x at time t, T(0, t) = centreline temperature and x = the distance
measured from centreline.

0.2

I
N H
C 0.5
L

0.001 0.1 1 10 100


1
Bi

Temperature as a function of centreline


for plane wall of thickness 2L

The heat gained or lost by the plane wall can be determined using chart which is plotted with
Transient Heat Conduction I9I

constant of one second, (ii) time required for the thermocouple junction to reach
198°C if junction is initially at 25°C and is placed in a gas stream which is at 200°C.

Characteristic length = A
V = 4/3 x R23 = 1/3 R
4 7r R
pClg
Time constant r = P = 1 sec (given)

8500 x 400 x RI3


or =1
400
400 x 3
or R= — 3.53 x 10-4 m
8500 x 400
Hence, junction diameter d = 2R
= 2 x 3.53 x 10-4 m
= 0.706 mm
Now temperature distribution is—
T—T
=e
Ti — T
198 — 200 I
= e as time constant r = 1 sec
25 — 200
t = log 175
2
= 4.47 sec

25. How the transient heat conduction for a plain wall can be analysed with the help of
Heisler charts.
(UPTU — 2006 — 7)
Or
How is transient heat conduction in solids having finite conduction and convective
resistance (0 < Bi < 100) determined?

All practical heating and cooling of metal objects involve both internal
and external resistance. In lumped system analysis, we consider body
at uniform temperature, thereby neglecting the internal resistance to
heat transfer.
Heisler charts are used to solve the transient heat transfer problems
taking into consideration both internal and external resistance to heat
transfer. The charts are used when Bi > 0.1. Such chart solutions for
x=—L x=L
transient analysis are available for standard objects like rectangular, x=0
cylindrical and spherical shapes. Heisler charts are provided at the end
of this chapter.
I 94 Heat and Mass Transfer

Similar solutions of heat conduction can be obtained for a sphere of radius `r0'. Heisler's
charts can be used for finding temperature at centre of the sphere (radius `r0') at different
instants of time. The charts are drawn with central temperature on y-axis with respect to
a 1
Fourier number ( F0 x t on x-axis for various . The charts are also available for
ro JJ Bi

finding temperatures at other positions as a function of r and also for finding


Po Q0

Heating and cooling of a sphere

27. A solid sphere and a hollow sphere of the same material and size are heated to the
same temperature and allowed to cool in same surroundings. If the temperature
difference between the body and that of the surroundings is T, then—
(a) both spheres will cool at the same rate for small values of T.
(b) both spheres will cool at the same rate for all values of T
(c) the hollow sphere will cool at a faster rate for all the values of T.
(d) the solid sphere will cool at faster rate for all the values of T
(IES 92)
pC„V
Time constant Z =
hA
Volume of solid sphere is higher than the volume of hollow sphere.
Hence, rsolid > Thollow
Therefore, hollow sphere will cool faster than solid sphere.
Option (c) is correct.

28. A large metal plate of thickness 5 cm is initially at 460°C. It is suddenly exposed to


a fluid at 100°C with a convection coefficient of 142.5 W/m2K. Find the time needed
for its mid plane to reach a temperature of 316°C and surface temperature at the
same instant of time. Take k = 21.25 W/mK and a = 1.2 x 10-5 m2/s.
(UPTU — 2007 — 8)

Guidance: Use Heisler charts.


Here 2L = 5 cm or L = 2.5 cm
Transient Heat Conduction I 95

1 k 21.25

Bi hL — 142.5 x 2.5 x 10-2
= 5.96 = 6
The centreline temperature is—
_ 316-100 216 _
06
Tg - — 460 —100 — 360 — .

Now for 1 = 6 and T°'t = 0.6,
Bi Ts —1:
at
We get the value of F, = 2 = 2.2

2.2 x L2 2.2 (2.5 x 10-2)2


t=
a 1.2 x 10-5
= 45.83 sec
0.6

1
=6

2.2 F = at
o 2
L
Centreline temperature of infinite plate

Now x = 1 for surface of the wall,


L
= 6, we get = 0.92
Bi —

0.92

6
I 96 Heat and Mass Transfer

T2.5,45.83 — 100
0 92
460 -100 - .
T2.5,45.83 = 100 + 360 x 0.92
= 100 + 331.2
= 431.2°C

29. Two identical balls made of pure iron and copper having diameter of 6 cm and initial
temperature of 500°C are being cooled in oil having temperature of 100°C and heat
transfer coefficient of 10 W/m2K. It is desired that both balls should reach a
temperature of 150°C at same time. Which ball should be inserted into oil first?
After how much time second ball should be inserted into oil?
Justify the answer and formula used. Properties are—
(a) Pure copper: p = 8954 kg/m3, k = 386 W/mK
Cp = 383 J/kgK
(b) Iron: p = 7897 kg/m3, k = 73 W/mK
Cp = 452 J/kgK
Find Biot number for copper and iron balls.
hls
(Bd copper = is = characteristic length of ball =
3
where R = radius
1, = 3 = 1 cm

hR
Now (B d copper = 3k

10 x3x10-2
= 2.5 x < 0.1
3 x 386
hR 10x 3 x10-2 =
(lid iron = 3k = 3 x 73 13.6 x 10-4 < 0.1
Hence, lumped system analysis is possible in both the cases.
_ h xt
To Tcc pcp4
Now
T -Tcc e
And T = 150
For iron ball
10
500 -100
= e 7897x452x1x10-2 xis
150 -100
= 7897 x 452 x10-2
or t, log 400
10 50
= 3569.44 x 2.079 = 7421 sec
Transient Heat Conduction I 97

For copper ball


8954 x 383 x 10-2 400
t, = x log = 7223 sec
10 50
Now is - tc = 7421 - 7223
= 298 sec
Hence, copper ball has cooling faster and it has to be inserted after the iron ball.

30. A spherical thermocouple junction of diameter 0.706 mm is to be used for the


measurement of temperature of gas stream. The convective heat transfer coefficient
of the bead surface is 400 W/m2K. Thermophysical properties of thermocouple
material are k = 20 WmK, C,, = 400 J/kg K and p = 8500 kg/m3. If the thermocouple
initially at 30°C is placed in a hot stream of 300°C, the time taken by the bead to
reach 298°C is
(a) 2.35 sec (b) 4.9 sec
(c) 14.7 sec (d) 29.4 sec
(GATE 2004)
Temperature of thermocouple will vary exponentially.

----- Thermocouple

—..-
To,= 300°C -.
_,..
c:
To = 30°
d = 0706 mm

h
T -1: pCpl' xt
= e-BiF0 = e_mt = e
To — 1:
V—R = = 0.706 = 0.1176 mm
1c = A 3 6
400 -1x10-3
7:- T
=e 3500x400x0.1176x10-
3 xt
= e
10-3 xt xt =
T_ -To
log 300 - 30
or t=
300 - 298
= 4.9 sec.
Option (b) is correct.

31. A steel ball of 5 cm diameter at 500°C is suddenly placed in a controlled environment


maintained at 100°C. Taking following data, find the time required to maintain
centre point temperature of 150°C in the ball. C,, = 450 J/kg°C, k = 35 W/m°C, h =
10 W/m2K, p = 8000 kg/m3.
(UPTU - 2009 - 10)
I 98 Heat and Mass Transfer

his
Biot number Bi =
k
R
/, for steel ball = — = 25 = 0.833
3 3
10 x 0.833
B1 = = 0.238
35
As Bi > 0.1, hence the lumped heat-system analysis can not be used. Heisler charts are to
be used.
1 1
= 4.2
Bi 0.238
The centreline temperature is-
- _ 150 - 100 _ 50 = 0.125
- 500 -100 - 400

From Heisler chart, for centreline temperature = 0.125 and 1 = 4.2, we have-
Bi
at
Fourier Number F0 = 2 =2
Po

2. rj
t-
a
35
Now a= = 9.7 x 10-6 m2/s
p.Cp 8000 x 450
2 x (25 x 10-2)2
••• t=
9.7 x 10-6
= 1.29 x 102 S.
= 129 s

32. A large metal plate 10 cm thick is initially maintained at a temperature of 500°C.


It is suddenly exposed to a surrounding at 140°C with a heat transfer coefficient of
570 W/m2K. At a later instant, its centre plane reaches a temperature of 350°C. Find
the temperature of a plane at a distance of 2.5 cm from the mid plane at the same
instant. Take k = 170 W/mK, a = 4.78 x 10-5m2/s.
(UPTU - 2008 - 9)
Here 2L = 10 cm
L = 5 cm = 5 x 10-2m
Transient Heat Conduction I 99

1 k 170
— —6
Bi hL 570 x 5 x 10-2
The centreline temperature is—
To,t — Ts. = 350 —140 =
210
= 0.583
Ts — T 500 — 140 360

1
Now from Heisler chart for centreline temperature = 0.583 and — = 6, we get—
Bi

Fourier Number Fo = a2t = 3.8


L
3.8 x L2
or t=
a
3.8 x (5 x 10-2 )2
=
4.78 x 10-5
= 1998

Now to find temperature at a plane at distance x = 2.5 from the midplane.


x 2.5
= = 0.5
L 5

x
Using position correction chart for — = 0.5 and 1 = 6, we get—
L Bi

Tx — T
= 0.95
Ts —1:
Tx —140
or = 0.95
500 —140
or Tx = 140 + 360 x 0.95 a 482°C.

33. What are infinite and semi infinite solids or bodies? Give the examples of semi
infinite body. What is the general criteria for any body to be considered as semi
infinite body?
An infinite body is a body which extends itself infinitely in all directions of space. If an
infinite solid is split in the middle by a plane, each half is known as semi infinite solid. A
semi infinite solid is a body that has a single boundary surface. Which extends to infinity
in one direction as shown in the figure. The earth and any thick slab can be considered to
be a semi infinite body which helps in obtaining the temperature variation in it nearer to its
surface. The general criteria for an infinite body to be considered semi infinite body is—
200 Heat and Mass Transfer

8 0.5
21,1 at
where 8. thickness, a = thermal diffusivity and t = time.
The heat conduction is a semi infinite body occurs is one dimensional direction.

A semi-infinite solid

34. How is temperature distribution in semi-infinite solid found out using Gaussian error
function? Find the expression for: (i) instantaneous heat flow rate at given x-
location, (ii) heat flow at surface, and (iii) total heat flow for a time interval.

Consider a semi infinite solid, initially at uniform temperature T,. The surface of the solid
is now (t > 0) subjected to some boundary condition. The temperature distribution and heat
flow at any position 'x' in the solid with time can be obtained by using the equation—
a 2T 1 aT
— — where 0 x
ax` a at

Transient heat flow in a semi infinite plate

The body is initially at uniform temperature Ti including the surface at x = 0. The surface
temperature at x = 0 is instantaneously changed to and held at T, for all times greater than
Transient Heat Conduction 20 I
t = 0. Thus the boundary conditions are—
T(x, 0) =
T(0, t) = Ts for t > 0
t) = Ti for t > 0
The solution of the differential equation for transient heat conduction as given with the above
three boundary conditions gives the following temperature distribution at any time 't' at a
plane parallel to and at a distance 'x' from the surface-
740 — Ts
_erf x, = erf (0)
Ti — Ts 2V at

where = and erf (0) = Gaussian error function.


2Ia•t
The values of Gaussian error function for various values of '0' are tabulated at the end of
this chapter. Also we have—

erf O = 2 re-edn

where tl = similarity variable = x


2,\Ia•t

x
Now T(x, t) — Ts = (Ti — Ts) exf
2\la•t
On differentiation, we get—

aT [Ti — Ts j —x2
X exp
ax Vir•a•t [4.a•ti

As per fourier's law, the instantaneous heat flow rate at a given x-location within the semi-
infinite solid at any specified time 't' is—
a T.
(x, t) = —k • A.
ax

[ -X2
= —k • A (T, —Ts) x exp
a•t 4. a • t
At the surface we have x = 0, hence surface heat flow is—
kA(Ti — Ts)
Q0,0 =
g • a•t

It is apparent from the above that the surface heat flow diminishes with time. The total heat
flow in time interval from t = 0 to t = t is—
202 Heat and Mass Transfer

—kA(Ti —TS ) ft 1
Qtotal — dt
v r•a .s1t

—2 k • A • (Ti — Ts)
•a

35. What is the expression for the temperature at the centre of cylinder or sphere of
radius `R' in Gaussian error function?

Under similar conditions of heating or cooling, the temperature at the centre of infinite
cylinder or sphere of radius 'IV can be given as—
T — Ts
= erf [ a.2t
Ti — Ts

[ a
The values of function erf for the cylindrical and spherical surfaces can be obtained
/V' J
from the figure below:

1.0

0.8

0.6

0.4

0.3

0.2

0.15

0.1
r 1
0.08

0.06
Infinite cylinder
0.04

0.03

0.02

0.015

0.01
0.1 0.15 0.2 0.3 0.4 0.6 0.8
a.t
R2 —
Error integral for cylinders and spheres
Transient Heat Conduction 203

36. What do you understand from penetration depth and penetration time?

The penetration depth is the depth from the surface (x = 0) where the temperature changes
is within 1% of the applied change in temperature (7; — Ti). Hence
T
= 0.99 = erf (1.8)
Ti — Ts

Penetration depth x= 1.8 x 2 x a• t

Or x = 3.6V a•t

Therefore the penetration time at a given depth indicates the time taken by the heat to get
the penetration of 1% from the surface. Hence—

__ 1 x
= x )2
a 36

37. The ground at a particular location is covered with snow pack at —10°C for a
continuous period of 90 days. The average soil properties at that location are: (i) k
= 0.4 W/mK, and (ii) a = 0.15 x 10-6 m2/s. Assuming an initial uniform temperature
of 15°C for the ground, determine the maximum depth to bury the water pipes from
the surface to avoid freezing.

Ts = —10° C _ earth surface


/ / / / / / / / / /

Soil

Water pipe buried in soil

The temperature distribution in the soil can be given as-

74,0 — 7;
= erf
Ti - Ts 2Vot.t
Here T(x, t) = 0 when water starts freezing
Ts = —10°C
Tl = 150°C
0 — (-10)
Hence = erf x = 0.4
15 — (-10) 2 ' a .t
From the table for erf (0) = 0.4, the value of 0 = 0.37
204 Heat and Mass Transfer

tfr = = 0.37
21/ a •t

or x = 2 x V0.15 x10-6 x (90 x 24 x 60 x 60) x 0.37


= 0.8 m

38. A large block of steel is initially at 35°C. The surface temperature is suddenly raised
and maintained at 250°C. Calculate the temperature at a depth of 2.5 cm after a time
of 30 seconds. The thermal diffusivity and thermal conductivity of steel are 1.4 x
10-5m2/s and 45 W/mK respectively.
(Annamalai University — 2008 — 9)
For semi infinite solid, we have—
T —Ts
= erf [
Ti — Ts 2
11(37
xt

x 30 x 10-2
Now —
21,1a•t 241.4x 10-5 x 30
= 0.61
From Gaussian error function table, the value of—
erf [0.61] = 0.612
T-250
= 0.612
35— 250
T = 118.5°C.

39. A large concrete high way initially at a temperature of 70°C and stream water is
directed on the highway so that the surface temperature is suddenly lowered to 40°C.
Determine the time required to reach 55°C at a depth of 4 cm from the surface.
Annamalai University — (2007 — 8)

T-1 x
= erf [
Ti — Ts 21ji
c
For concrete, a = 8 x 10-7 m2/s from data book.
55-40 15 =
Now erf[ ix ]= = 0.5
2V a • t 70 —40 30
Transient Heat Conduction 205

From the table for erf = 0.5, we have-


[2 \,/ xa • t

= 0.48
2 cc•
2
X 1
x —
2 x 0.48) a

4x10-2 )2 1
(`2x0.48 8x10-7

17.36 x 10-4
8x10-7
= 2170 seconds or 36.2 min

40. A thick steel slab is initially at a uniform temperature of 25°C. When the slab is
exposed to hot flue gases, the surface temperature suddenly changes to 450°C. Make
calculations for the temperature in a plane 250 mm from the slab surface 5 hours
after the operation of change in surface temperature. Find also the heat flowing into
2 square metres of this plane and the total energy flowing through the surface during
5 hours period. Assume h = 160 kJ/m • hr • degree, p = 8000 kg/m3 and C = 0.48 kJ/
kg-degree.

160
a= = 4.17 x 10-2 m2/hr
p• C 8000 x 0.48
T -Ts 0.25
Now = erf ,x j= erf
TTi — Ts a•t 2 V4.17 x 10-2 x 5
= erf (0.274)
From the table of Gaussian error function, it is found out-
erf (0.274) = 0.30
T -T
Hence S = 0.30
Ti - Ts
T - 450
or = 0.30
25-400
or T = 322.5°C
206 Heat and Mass Transfer

The instantaneous heat flow rate of this plane at t = 5 hr is-

- k • A•(T-T,)•exp x2
4 a• t
(.
Q(0.25,5) =
V tr•a•t
[ -0.252
= -160 x 2 (25 - 450). erf
4 x 4.17 x 10-2 x 5
= 1.56 x 10-5 kJ/hr
The total heat flow from this plane in 5 hr is-

Qtotal = x k • A • (Ti - Ts ) x V a
1/7r

-2 f 5
= x 160 x 2 x (25 - 450) x
e tc '\,1 4.17 x 10-2
= 1.68 x 106 kJ.

41. A thick copper slab (a = 1.1 x 10-4 m2/hr, k = 380 W/mK) is initially at a uniform
temperature at 10°C. Suddenly the surface is raised to 100°C. Calculate the heat
flux at the surface 5 and 10 min after raising the surface temperature. How long with
it take the temperature at the depth 5 cm from the surface to reach 90°C.
Heat flux at t = 5 min is-
-k • (T - Ts)
qt =5 min =
-V r.a• t
-380 x (10 -100)
=
Vtrx1.1x10-4 x5x 60
= 106.2 kW/m2
Similarly heat flux at t = 10 min is-
-380 x (10 -100)
qt=lo mm =
~tc x1.1x10-4 x 10x60
= 75 kW/m2
Now temperature distribution is-
TX -TS x
= erf
-Ts 211 a • t
Transient Heat Conduction 207

90 -100
= erf
10-100 a•t

or erf x = 0.11
2,0H
1t.t
From the table of Gaussian error function, we have-
= 0.10
2-fa•t

5x10-2
- 0 10
2111.1x lex t
or t = 568 hrs

42. The surface temperature of a large aluminium slab initially at 200°C is suddenly
lowered to 70°C. What is the total heat removed from the slab per m2 area when the
temperature at a depth of 4 cm has dropped to 120°C. Assume a = 8.4 x 10-3 m2/s
and k = 204.2 W/mK.

Tx - T,
= erf x
- Ts 2v a•t

x 120 - 70
or erf , = 0.385
Va•t 200 -70
From the table of Gaussian error function, we have-
x = 0.36
21 a•t

(0.04) 2
= (0.36)2
4 x 8.4 x 10-' x t

16 x10-4
or t= = 0.37 sec
4 x 8.4 x 10-3 x 0.1296
The total heat removed per unit area is-
2
qtoto = x k x (Ti - Ts) x , t
V a

= -2 x 204.2 x (70 200) x ; 0.37


1/ Jr A; 8.4 X 10-3

= 1988 kW/m2
Heishler chart for temperature history at the centre of a plane wall
1.0
0.7
...am ,Ite-4
0.5
0.4 Mb I. In.
90
0.3
'A4 r 100
HEISLER CHART (PLANE WALL)

0.2 II X 80
0.1
NwiLal NoleL
416.110111. III
Lm misamomhm.
kLIL_
6'0 6:9
IIMINIMEWNICUOS. 111MMILIM11.1.9MIL91=1611=1. v0 v
0.07 'PS
0.05 -44 0
0.04 'tti%\ 0-% 1)
13 0
0.03 a kh 1,„
0 • 9 IIMIN1INIM1 cf3
I- -2- 0.02
0.01
0.007
0.005
0.004
0.003
0.002
Heat and Mass Transfer

0.001
0 1 2 3 4 6 8 10 12 14 16 18 20 22 24 26 28 30 40 50 60 7080 90 100110 130 150 200 300 400 500 600
208
Transient Heat Conduction 209

CORRECTION FACTOR

1.0
XL = 0.2

0.9
0.4
0.8

0.7
0.6
0.6 L

0.5

0.4
0.8

0.3

0.9
0.2

0.1
1.0

0
0.01 0.02 0 05 0.1 0.2 05 1.0 2 3 5 10 20 50 100
1_k
B1 hL

Correction factor chart for temperature history in a plate plane wall


1.0
0.7 ,.Z.4.,,N
, ''------ -,-'--z--
4-- --.--41:1
— -,4 .fir
1,21.-"SISr.V.----4
0.5
0.4
9 It 4,......z.v....,-.4.44, 14141\

Heishler chart for temperature history in a cy linder


0.3 1:111441". NS %%hiN\._ • MN••MENNEMMENNEN■
la .. 011\litiMA\SZN\
0.2
N, M11i'1111U1:118
\
P14 0.1
IawlM\
nil
fl
.... \ limelipL
mh.5001
:,Z,`" 70
+ 0
0.07 RIM ■m1.
LW" 0 *0 mmanominmeugusw.
it 0 26,
>4
0.05
0.04
01 voimanualmw
Nommommunimummi.
C.) I
"Z-5 0.03
• a .a NVOOMMILIMMW&
NOOMMAIIIIMM
0.02
>-‘ IMEEIOMUNM
\\ ASso 1
AO
0.01 k
usImmthuLwitainsam
I sx%
0.007 I INNII=NEMIAILIM
1-4 0.005
0.004
0.003
0.002
2 I 0 Heat and Mass Transfer

0.001
1 2 3 4 6 8 10 12 14 16 18 20 22 24 26 28 30 40 50 60 70 80 90 100 110 120130 140 150 200 300 350
at
F0 - 112
2II
Transient Heat Conduction

1.0
0.7
taIN

Heishler chart for central temperature history in a sphere


0.5
0.4
0.3 'EP •
kitMIN "N`1.411\11111.11L '116.
716.
\\WV \11...
\\
0.2 I MUM '11\\NW
Nikh. (90
S
0.1 M141M1 MIOIM,M,MCM V.
"1110/0/MOMMI.
\ va
,90 \ AS
MIIMMILIMM13.1060.1Welh. .NIZIMMOMMOMMI.
.10611111MILIMIln.
0.07 MittalLINEL LMLW‘k A '11101MIn, c,)0
0.05 ELFWERMWW NEW
0.04 itilLISIMIMInkfl.Wink 'MAWR
0.03
0 0.02
0.01
0.007
0.005
0.004
0.003
0.002
0.001
0 0.5 1.0 1.5 2 2.5
\\\ \\\ \\ k
3 4 5 6 7 8 9 10 15 20 25 30 35 40 45 50 70 90 110 130 170
\\\
210 250
at
F0 = —
L2
2I2 Heat and Mass Transfer

HEISLER CORRECTION FACTOR


1.0
= 0.2

0.9
04
0.8

0.7
06
0.6 r/R

0.5

0.4
08

0.3
09
0.2

0.1
1.0

0
0.01 0.02 0 05 0.1 0.2 0.5 1.0 2 3 5 10 20 50 100
1_k
B1 hR

Correction factor chart for temperature history in a Cylinder.

1.0
r =6:2

0.9
4
0.8

0.7
06
0.6 r/R

0.5

1-13
0.4
0.8
FX H
0.3
0.9
0.2

0.1
1.0

0
0.01 0.02 0 05 0.1 0.2 0 5 1.0 2 3 5 10 20 50 100
1=k
B1 hR
Correction Factor Chart for Temperature History in a Sphere.
Transient Heat Conduction 2 I 3

GAUSSIAN ERROR FUNCTION


erf 0 = ,2- id' e-n2dri
1 gJ0

0 erf 0 0 erf 0 0 erf 0


0.00 0.0 0.42 0.4475 1.35 0.9431
0.02 0.0225 0.44 0.4662 1.40 0.9523
0.04 0.0451 0.46 0.4847 1.45 0.9592
0.06 0.0676 0.48 0.5027 1.50 0.9661
0.08 0.0901 0.50 0.5205 1.55 0.9712
0.10 0.1125 0.55 0.5633 1.60 0.9763
0.12 0.1348 0.60 0.6039 1.65 0.9800
0.14 0.1569 0.65 0.6420 1.70 0.9838
0.16 0.1709 0.70 0.6778 1.75 0.9864
0.18 0.2009 0.75 0.7112 1.80 0.9891
0.20 0.2227 0.80 0.7421 1.85 0.9909
0.22 0.2443 0.85 0.7707 1.90 0.9928
0.24 0.2657 0.90 0.7970 1.95 0.9940
0.26 0.2869 0.95 0.8270 2.0 0.9953
0.28 0.3079 1.0 0.8427 2.10 0.9967
0.30 0.3286 1.05 0.8614 2.20 0.9981
0.32 0.3491 1.10 0.8802 2.30 0.9987
0.34 0.3694 1.15 0.8952 2.40 0.9993
0.36 0.3893 1.20 0.9103 2.50 0.9995
0.38 0.4090 1.25 0.9221 2.60 0.9998
0.40 0.4284 1.30 0.9340 2.80 0.9999
Chapter 6
FORCED CONVECTION HEAT
TRANSFER

A VISCOSITY A REYNOLDS NUMBER


A SHEAR STRESS A PRANDTL NUMBER
A FREE CONVECTION A NUSSELT NUMBER
A FORCED CONVECTION A STANTON NUMBER
A LAMINAR FLOW A RAYLEIGH NUMBER
A TURBULENT FLOW A CRASHOFF NUMBER
A HYDRODYNAMIC BOUNDARY LAYER A SKIN FRICTION COEFFICIENT
A THERMAL BOUNDARY LAYER A FILM TEMPERATURE
A COEFFICIENT OF FRICTION A BULK MEAN TEMPERATURE
A LOCAL HEAT TRANSFER COEFFICIENT A EQUIVALENT DIAMETER
A AVERAGE HEAT TRANSFER COEFFICIENT A DIMENSIONAL ANALYSIS

INTRODUCTION

Thermal convection occurs when a temperature difference exists between a solid surface and
a fluid flowing past it. The convection process is essentially a process of heat energy transport
achieved by the circulation of fluid particles. As heat energy is transported by the transport of
fluid itself, hence transported heat energy depends upon: (i) conduction, (ii) fluid flow, and
(iii) mixing of fluid medium. The flow can be laminar or turbulent. In turbulent flow, irregular
velocity fluctuations are always superimposed on the main stream flow and this provides
additional mixing effect on the flowing fluid particles. Hence, these fluctuations are responsible
for the higher transfer of heat and momentum in turbulent flow. The rate of heat and momentum
transfer in turbulent flow is many times larger than what is possible in laminar flow. The heat
transfer by convection is determined by studying the analogy between heat and momentum
transfer. The convective heat transfer problems may be solved using the empirical correlations
of dimensionless numbers such Nusselt number, Prandtle number and Reynolds number.
Forced Convection Heat Transfer 2I5

1. What do you understand by the fluids?

Fluid is a substance which cannot acquire any static equilibrium under the action of any
shear fore of even a small magnitude. In other words, fluids cannot acquire a static
equivalent deformation (0) in order to achieve an equilibrium with external applied torque.
However, a fluid resists the applied torque by attaining the constant value of the rate of

change of deformation ( d°— and then the acceleration of fluid becomes zero. Hence, a fluid
dt
is a substance which can: (i) attain the shape of the container as it has no definite shape of
its own, (ii) deform continuously under the action of shear force, (iii) attain the equilibrium
rate of deformation, and (iv) the deformation does not disappear on the removal of torque.

2. What is the viscosity of a fluid?


The property which characterizes the resistance which a fluid offers to applied shear force
is called viscosity. The resistance does not depend upon the deformation (0) but on the rate

of deformation d°). When there is a relative motion between different layers of the fluid,
t
then there is a tangential friction force in between the layers. Viscosity is less in gases but
larger in liquids.
When a solid mass slides over a surface, a friction force is developed to oppose the
motion. Similarly, when a layer of a fluid slides over another layer of the same fluid, a friction
force is developed between them opposing the relative motion. The tangential force is called
viscous force. Consider a fluid moving in streamlined manner on a fixed horizontal surface
as shown in figure. The layer of the fluid in the contact of the surface remains stationary
while the velocity of other layers increases with distance from the fixed surface. There is
therefore an internal tangential resistive force acting between two layers which is opposing
their relative motion.

a)

N
_
0

Velocity

Velocity gradient due to viscosity

This resistive force is called viscous force. In order to maintain the flow of the fluid, we
have to apply an external force to overcome the tangential resistive or viscous force.
According to the Newton's equation of viscosity, the tangential shear stress (resistive force
per unit area) existing between two adjacent fluid layers is directly proportional to the
velocity gradient existing in the fluid in a direction perpendicular to the fluid layers.
2I6 Heat and Mass Transfer

Shear stress (r) a


du
y
du
or
=
du
where = Velocity gradient
dy
= Viscosity

--Idu r-- U U„,

u-0

Velocity gradient

Fluids following Newton's law of viscosity are called Newtonian fluids while other fluids
are called non-Newtonian fluids.

3. How is heat transferred in convection?

Heat is transferred in convection by two mechanisms. One is heat transfer from a hot
surface to adjacent fluid by random molecular motion which is called diffusion. The other
one is the transport of heat by bulk movement of the fluid from higher temperature region
to lower temperature region.

4. What is the classification of convective heat transfer?

The convective heat transfer can be classified as natural and forced convection depending
upon how the fluid motion is initiated. The natural convection is a process in which the fluid
motion results due to density changing on heat transfer. Natural circulation of the fluid is
caused by the rise of warmer fluid and the fall of colder fluid.

Hot plate
Natural convection

In forced convection, the fluid is forced to flow over a hot surface. The heat transfer rate
is much higher in forced convection.
Forced Convection Heat Transfer 2I7

Q
Relative velocity
of fluid layers
Fan

Forced convection Hot plate

5. What is a laminar flow (viscous flow)?

Laminar flow is a flow in which flow takes place in layers. There is no mixing of fluid
particles between any two adjacent layers. The shape of lamina (layer) depends upon the
shape of the boundary through which flow is taking place. In laminar flow, the fluid particles
move in unmixing layers or streams and follow smooth continuous paths. The fluid particles
retain their relative positions at successive cross-sections of the flow passage. There is no
transverse displacement of fluid particles. Soldiers marching in orderly manner is an analogy
to laminar flow. The shape of laminae if flow takes place between two parallel flat plates
are plane sheets parallel to each other as shown below. In case the flow takes place through
a circular pipe, the laminae become concentric sheets as shown in figure.

////////////////////

////////////////////
Concentric cylindrical
Layers in plane sheets
layers

6. What are the conditions which help the flow to be laminar?

The flow will be laminar when-


1. Velocity of flow is low
2. Diameter of pipe is small
3. Viscosity of the fluid is high
4. Density of the fluid is less
5. Reynolds number is less than 2300 for flow in pipes and less than 3.5 x 105 for flow
over plates.

7. What is turbulent flow?

The turbulent flow is when the velocity of flow reaches a certain limit such that the fluid
particles no longer move in layers or laminae. Violent mixing of fluid particles now takes
place due to which they move in random manner. As a result the velocity at any point varies
both in magnitude and direction from instant to instant. In turbulent flow, the motion of fluid
particles is irregular. The fluid particles move along erratic and unpredictable path. The
2I8 Heat and Mass Transfer

velocity of fluid particles fluctuates both along the direction of flow and also perpendicular
to the flow. A crowd of commuters on a railway station rushing for boarding a train is an
analogy of turbulent flow. The flow in pipes having Reynolds number above 4000 is always
turbulent.
/////////////////////////////

/////////////////////////////
Turbulent flow

8. What is Reynolds number? How is it useful?


(UPTU — 2002 — 3)

Reynolds number is the ratio of inertia force to the viscous force of the flowing fluid. It
is a dimensionless number and it is given for plate and pipe as—
pVL
(a) Re — for plate where L is the length of plates
pVd
(b) Re = for pipes where d is the diameter of pipe

Reynolds number is very useful for-


1. Predicting whether flow is laminar or turbulent.
2. Finding out the coefficient of friction (f) in order to determine the loss of heat due to
viscosity.

9. What are the parameters on which convective heat transfer depends?

Convective heat transfer strongly depends on: (i) fluid properties such as viscosity, thermal
conductivity (k), density (e) and specific heat (Cr) (ii) geometry and roughness of the solid
surface, and (iii) type of flow (laminar or turbulent).

10. Explain the concept of velocity boundary layer development over a flat plate.

h— Laminar Transition -to— Turbulent —I


U U
u = 0.99 U

(5= thickness of
boundary layer
•••
= laminar
\%\ sublayer
Leading edge 5 5
Re=3.5x10 Re=5x10
Growth of boundary layer on a flat plate
Forced Convection Heat Transfer 2I9

The figure above shows the development of a boundary layer on a flat plate held parallel to
the flow of a fluid having free stream velocity as U. The retardation of the fluid increases
as more and more of the flat plate is exposed to the fluid flow. The boundary layer having
velocity gradient and shear stress thickens as the distance of the fluid from the leading edge
of the plate increases upto certain distance from the leading edge, where the Reynolds
(pux
number where x is distance from the leading edge is less than 3.5 x 105. The flow
in this part of the boundary layer is laminar. In the laminar boundary layer, velocity
distribution is parabolic and shear stress can be given by Newton's law of viscosity. The
laminar boundary layer can grow to certain thickness beyond which it becomes unstable.
Hence, transition from laminar to the turbulent boundary layer occurs in the transition
region. As the name suggests, the flow in the transition region is neither fully laminar nor
fully turbulent. It fluctuates between the two types of flow. In turbulent boundary layer,
there exists always a thin laminar sublayer close to the flat plate surface. In this sublayer,
the flow is always laminar. The velocity distribution in turbulent boundary layer is
logarithmic but it varies similar to laminar boundary layer from zero to free stream velocity.
11. What is hydrodynamic boundary layer?
Or
Describe briefly the hydraulic boundary layer over a flat plate when a free stream
of fluid flows longitudinally over it.
(UPTU — 2003 — 4)
Hydrodynamic boundary layer is a thin layer of fluid in immediate vicinity of boundary
surface in which large velocity gradients and shear stresses exist. When a fluid flows past
a solid surface, the fluid particles on the surface have the same velocity as that of the surface
due to viscosity of the fluid. In case the boundary surface is static, the fluid velocity
du
gradually increases. The velocity gradient (u is fluid velocity in x-direction and y is
dy
perpendicular distance from the boundary surface) varies with y and it has high values close
du
to the boundary. This high velocity gradient gives rise to large shear stress (/ T= I at
dy )
the boundary surface. The term hydrodynamic layer is used to describe the thin layer of fluid
flow on the boundary surface within which the velocity gradient is appreciable magnitude.
Outside the hydrodynamic boundary layer, the velocity of fluid is same as the main stream
velocity. Hydrodynamic boundary layer is defined as the distance away from the surface
where local fluid velocity is 99% of the free stream velocity (u_).

u - 99% uo,

Distance °Hydrodynamic

u- 1111110""
Velocity -.-

Hydrodynamic boundary layer


220 Heat and Mass Transfer

12. Explain thermal boundary layer for flow of a cold fluid on a hot plate.
Or
Describe briefly the thermal boundary over a flat plate with flow of fluid.
(UPTU — 2002 — 3)

If a fluid flows on a surface having different temperature than the surface, the thermal
boundary layer is developed similar to velocity boundary layer.
Consider a fluid at temperature 'Too' flows over a surface at temperature 70', the fluid
particles adjacent to the surface get the same temperature that of the surface. These fluid
particles exchange heat energy with particles in adjoining layers and this continues till the
adjoining layers have no temperature difference which happens at large distance away from
the surface. As a result, a temperature gradient is developed in the fluid layers and
temperature variation occurs in the fluid flow from `To' to 'Too' . The flow region over the
surface in which the temperature variation in the direction normal to the surface is observed
is called thermal boundary layer. The thickness of the thermal boundary layer (8th) is defined
as a distance from the surface at which the temperature difference is 0.99 (To — Too). It can
be appreciated that the heat transfer cannot penetrate further into the free stream as the
distance from the leading edge increases. The convection heat transfer rate anywhere along
the surface is directly related to the temperature gradient at that location. Therefore, the
shape of the temperature profile in the thermal boundary layer leads to the local convection
heat transfer between surface and flowing fluid. The boundary layer thickness is
T —T
8th = — 0.99.
To — Too
Thermal boundary
Y 00
layer

To.
OC

Uo.
T T

Hot plate at To

13. What do you understand from the boundary layer concept?

In the boundary layer concept, the fluid flow field over a body is divided into two regions
as under:
(1) A thin region near the surface where velocity and temperature gradient are large and
it is called boundary layer.
(2) The region outside the boundary layer where velocity and temperature of the fluid is
nearly equal to the free stream values.
The hydrodynamic and thermal boundary layer thicknesses are defined as the distance
from the surface at which the local velocity and temperature of the fluid reach 99% of free
Forced Convection Heat Transfer 22 I

stream velocity and temperature. The heat transfer between a fluid and a surface can take
place when both the velocity and thermal boundary layer exist simultaneously.

14. Explain the formation of simultaneous velocity and thermal boundary layers.

During flow of fluid over a heated surface, both velocity and thermal boundary layers are
developed simultaneously. If the effects of fluid viscosity is stronger than thermal effects,
then the velocity boundary layer is thicker than thermal boundary layer and vice versa. For
liquid metals thermal effects are much stronger than viscous effects and hence Soh > 8. For
oil and greases, the viscous effects are much stronger than thermal effects and hence
8> 8th. For gases, 8th = S.

T
mmik —4— =1,1
eth
To —
VAowiwiwzbviwiwiwit,,w////
Liquid metals Oils and Greases

15. Explain the variation of coefficient of friction (Ctx) and local heat transfer coefficient
(hi) with velocity and thermal boundary layer.

At surface, heat transfer is—


aTI
qo = —1c( = hx(To — Tco)
C '0' y=0
)
n
or = k C a) Y
To — Tso

Now as boundary layer thickness (5th) increases with increasing `x', the temperature
(a T
gradient must decrease as x increases. Hence, the value of h, will decrease as 'x'
Y )31= 0
increases. There is a very significant increase in the heat transfer coefficient in the transition
region due to fast intermixing of the fluid particles. In turbulent region, heat transfer
coefficient (hi) decreases due to added resistance to heat transfer from laminar sublayer and
much thicker boundary layer thickness (5th).

Laminar -Transition,- Turbulent


Variation of heat transfer coefficient
222 Heat and Mass Transfer

16. What is Prandtl Number (Pr)? What is its significance?


(UPTU — 2003 — 3)

Prandtl number (Pr) is defined as the ratio of molecular diffusivity of momentum to the
molecular diffusivity of heat
Molecular diffusivity of momentum
Pr =
Molecular diffusivity of heat
µCp
k
pip
k a

Kinematic viscosity
Thermal diffusivity of heat
It means that Prandtl number is a measure of relative value of momentum transfer and
heat energy transfer in the velocity and thermal boundary layers. In other words, Prandtl
number measures relative thickness of velocity and thermal boundary layers. Therefore we
have
(a) Pr >> 1 then 8» 8th for oil. It means heat diffuses very slowly.
(b) Pr = 1 then S = 8th for gases. It means that momentum and heat diffusion take place
through fluid at the same rate.
(c) Pr << 1 then 8« 8th for liquid metals. It means that heat diffuses very rapidly in the
fluid.

Pr < 1
Pr = 1
Thermal and hydrodynamic
3th boundary layers merge
at Pr = 1
Pr > 1

///////,//// //,/,////
Thermal boundary layers

17. What is Nusselt number (Nu)?


(UPTU — 2002 — 3)

Nusselt number (Nu) is the ratio of rate of heat flow by convection process under a unit
temperature gradient to the rate of heat flow by conduction process under a unit temperature
gradient through a stationary thickness of metres
Heat flux through convection
Nu =
Heat flux through conduction in fluid
Forced Convection Heat Transfer 223

For convection: Qconv — h x (T — Too) h when T — Too = 1


A
Qcond =dT k
For conduction: kx — when dT 1
A L L
hL
Nu = —

Nusselt number is a measure of the convection heat flow occurring at the surface.
Nusselt number in thermal boundary layer is analogus to the friction coefficient (Cf) in the
velocity boundary layer. Bigger the Nusselt number, more is the heat transfer coefficient (hx)
and higher is the convection heat transfer from the surface. It represents the slope of the
temperature gradient curve in the thermal boundary layer thickness. If 'Nu' is unity, it
indicates that there is no convection and heat transfer is by pure conduction in the boundary
layer. Larger value of 'Nu' indicates large convection in the fluid.

18. What is Grashoff number (Gr)?


(UPTU — 2002 — 3)

Grashoff number is used in natural convection and its role is same as that of Reynolds
number in forced convection. It is defined as the ratio of the product of inertia force and
buoyancy force to the square of viscous force
Gr = Inertia force x bouyancy force
(Viscous force)2
(p V2 L2) x (p • • g • AT • L3c )
(tiV L)2
_ p2 • • g • AT • L3
— 2
/I
1
where /3 = coefficient of volumetric expansion = , A T = temperature difference
Tf + 273
+ Too
between surface and fluid, Tf = mean film temperature = To 2 , = significant length
of the body (height for vertical plates and cylinders, diameter for horizontal cylinders and
spheres).
In natural convection, the fluid flow is produced by bouyant effects resulting from a
temperature difference. For free convection, the transition from laminar to turbulent
occures when Gr = 109.

19. What is Stanton number (St)?

It is the ratio of the heat transfer at the surface to that transported by fluid by its thermal
capacity.
Heat flux to the fluid
St =
Heat transfer capacity of fluid
224 Heat and Mass Transfer

h x AT h
pxCp xu_xAT pCpu_
h•L

pu_L ti • Cp
k
Nu
Re • Pr
Hence, Stanton number can be defined as the ratio of Nusselt number to the product of
Reynolds number and Prandtl number.

20. What is Rayleigh number (Ra)?

Rayleigh number is the product of Grashof number and Prandtl number.


Ra = Gr x Pr
g•/3•L3 AT
v•a
Rayleigh number is useful in natural convection heat transfer. The flow is laminar in natural
convection if Rayleigh number is less than 109.

21. What is skin drag?

When fluid flows past a solid surface, it exerts a tangential shear stress or friction drag on
the surface parallel to the direction of flow resulting due to the velocity gradient in the
boundary layer. This is called skin drag. Skin drag is the force exerted by the fluid on the
surface in the direction of the flow.

22. What is skin friction coefficient or friction drag coefficient? How is skin friction drag
force found out?

Skin friction coefficient is the ratio of shear stress at the surface in the direction of flow
to the product of density and velocity head.
(T) y=0
Cfx =
2

P 2
The average skin friction coefficient can be obtained by integrating skin friction
coefficient for entire surface and dividing by surface length.

Cfx • dx
cf
Forced Convection Heat Transfer 225

Skin friction drag force (F) is found out from average skin friction coefficient,
2
U
F = Cf (p =)A
2
where A = area of contact between fluid and surface and tt,,, = free stream velocity of fluid.

23. Find average heat transfer coefficient if hx = C • x-m where C = constant and 'x' is
distance from the leading edge of the plate.
We know—

h • dx
h=

Cx _01
= Jo

c [ x0.9 ix
x 0.9 0

C cx-c"
x 0.9 0.9
= 1.11 hx

24. Heat transfer coefficient for laminar natural convection over a hot vertical plate of
length is given by—
hx = C X X-114
Find average heat transfer coefficient.
The average heat transfer coefficient is
L
J hx • dx
h= o
L

IL
c.x-imax

[X 3/4 t
= C•
•L

C • L3/4
3
71.
. •L
226 Heat and Mass Transfer

4CL 4
3

25. Tabulate forced convection correlation for flow over flat surface (both laminar and
turbulent flow).
All fluid properties are calculated at film temperature (Tf) where—
T° +T
Tf = 2
S.N. Correlation Laminar flow Turbulent flow
Re < 3.5 x 105 Re > 5 x 105
5x 0.381
1. Boundary layer thickness = =
Re (Re)"-
_ 0.664 c = 0.0592
2. Local skin friction coefficient cfx
VRex fx (Re)1/5
r. 1.328 074
3. Average skin friction coefficient Cf =2 .-• fx = C f = 0. 1/5
Al Rex (Re)
4. Local Nusselt number Nux = 0.332 Re, Pr1/3 Nux = 0.0296 x
Re2.8 x Pr1/3
5. Average Nusselt number Nux = 2 Nux = 0.664 x Nu = 0.037 x
Rex. Pr1/3 Reo.8 x pr1/3

In case fluid is initially laminar and thereafter turbulent over remaining part of the surface,
then correlations are-
1. Boundary layer thickness
( 0.381 1.0256 )
for 5 x 105 < Re < 107
Re1/2 Rex
2. Average Nusselt Number
Nu = Pr15 [0.037 Re" — 872]
p •uL hL
where Re = and Nu =

In case of constant heat flux condition, we can have


Nuz = 0.453 Rex/2 • Pr1/2
Average Nu = 0.6795 Rem • Pr 1/2
Forced Convection Heat Transfer 227

26. What do you understand from film temperature?


(UPTU — 2003)
In the boundary layer thickness, the temperature of the fluid is maximum at the surface and
it decreases as the distance of the fluid from the surface increases. At the surface, heat
transfer from hot surface to the fluid takes place purely due to conduction. However, away
from the surface and deep into the fluid, heat transfer occurs due to convection. The
temperature of the fluid varies from surface to fluid upto the thermal boundary layer
thickness. In order to perform convection analysis to determine the heat transfer coefficient,
an average temperature of the fluid and the surface is taken. The temperature is known as
mean film temperature
To + T_
T =
f 2
Note: In case heat transfer between the tube wall and a fluid, the bulk mean temperature
is taken which is mean of inlet and outlet temperature of fluid from the tube.

27. The temperature profile in a thermal boundary layer for flow over a flat plate is given
by:
T —Ts _ 3 y 1 ( Y j
To. — Ts — 2 6th 2 (5th
x
6thx = 4.53 x
Revs P,"3
Find hx and average h.
The temperature y profile is—
T —Ts
= 3/2 Y 1 ( Y )3
T — Ts 8th 2 Sth

aj 1 = (T _ To 3 x 1 1 x 3Y2 c, Ts )
( _
uY/y=o 2 oth 2 (51 25th
— y=o 3(T
raT)
k
a y )y=0 3k k • Re1/2 Pr1/3
Now hx = — — 0.332 —
T,.— Ts 26th x
,x
Hence Nu., — hk = 0.332 Rein Pr15

But h= 1f L = 2(hx)x=i,
o
Nu = 2 Nux
228 Heat and Mass Transfer

28. Explain the velocity profile and boundary layer for fluid flow through a tube pipe.

Boundary layer

Fluid flow
fully developed
H— Entry length —)-1
Parabolic velocity profile for laminar flow

Laminar sublayer

Velocity profile for turbulent flow

Consider a fluid entering a tube. Due to viscosity, fluid layer adjacent to tube surface
immediately becomes stationary and velocity of the subsequent layers increases as distance
from the surface increases. The boundary layer develops from the entrance of tube similar
to the flow over a plane surface. The velocity of fluid in the boundary layer is zero at the
wall of the pipe and it is maximum value at the centre of the pipe after covering a certain
distance which is called entry length. The fluid flow after entry length is fully developed.
The initial uniform region of velocity profile is called core and it shrinks throughout the entry
length. Finally, at last point of entry length of centre at tube, boundary layers from the two
sides merge at core top. Now velocity profile is fully developed inside the tube as flow is
fully pervaded by the viscous action. In case of laminar flow a parabolic velocity is
developed. When flow is turbulent, a much flatter velocity profile is observed.

29. Explain the development of the thermal boundary layer in a tube/pipe.


(UPTU — 2003)

A thermal boundary layer is developed similar to velocity boundary layer when a fluid having
different temperature than that of the tube flows through the tube. The fluid particles in the
layer in contact with the surface of the tube assume the wall surface temperature (To). This
initiates convection heat transfer in the tube and the development of the thermal boundary
layer along the tube. Now temperature gradient exists in the thermal boundary layer. The
thickness of the boundary layer increases along the flow until boundary layer reaches the
tube centre, thereby filling the entire tube. The region of flow in which the thermal boundary
layer develops and reaches the tube centre is called thermal entry region. The thermal entry
length (xth) is related to hydrodynamic entry length (xv) as under—
xth
= Pr for laminar flow
xv
After thermal entrance region, we have thermally developed region.
Forced Convection Heat Transfer 229

30. What criteria is used to identify types of flow in a tube (pipe)?

The Reynolds number for a fluid flow in a tube is given by—

Thermal
boundary layer

Thermally
developed region

6th = thickness of
boundary layer
[4- Thermal entrance region
Thermal boundary layer

pVd
Re =
ii
where d = diameter and V = velocity of flow.
The flow is considered to be laminar when Re < 2300 and it is considered to be turbulent
for Re > 2300.
Reynolds number can also be given in mass flow rate—
pVd
Re =
ii
M = Flow volume x density
= (Lr d2 V) x p

4M
V=
rd 2 p
p( 4Mj d
Re= , x
µ ird`p

. 4M
irdit

31. What is the equivalent diameter pipe (hydraulic diameter)? Find equivalent diameter
for rectangular duct and hollow cross-section pipe.

The concept of equivalent diameter pipe or hydraulic diameter is used to simplify the analysis
when the flow is taking place in the ducts of square or rectangular cross-sectional area.
Equivalent diameter is a circular pipe which would present the same resistance to flow and
heat transfer rate which is being presented by the actual duct.
230 Heat and Mass Transfer

Equivalent diameter (de) — Li


4 x flow cross-sectional area
Wetted perimeter
Rectangular duct has equivalent diameter as worked out below.

T
Depth = D

I-
H— Breadth = B

4A 4(B x D)
de = =
P 2(B + D)
In case of square duct, B=D=a
4 x a2
de=
2 x 2a =a
Hollow tube has equivalent diameter as worked out below.

4A 4 x 1r4 (D4' — di2)


de = =
P r (Do + di)
= (Do — di)
Equivalent diameter for a rectangular duct with a pipe inside can be worked out as under-

--I

I B

4A 4(B x D d2)
de = = 4
P 2(B+ D)-F ird
Forced Convection Heat Transfer 23 I

32. Tabulate forced convection correlations for flow inside a pipe.

1. Laminar fluid flow (Re < 2300)


(a) Friction factor
f = 64
Re
(b) Frictional heat loss h_ f LV 2

f 2. g • d
(c) Frictional pressure drop (AP)
p 2
2
= P .g. h f = f
d
(d) Nusselt number for surface having constant temperature is
Nu = 3.66
(e) Nusselt number for constant heat flux condition is
Nu = 4.364
All fluid properties are calculated at mean bulk temperature (Tb) which is

Tb =
Tinier + Toutlet
2
2. Turbulent flow inside a pipe (Re > 2300).
The Nusselt number has same value for both constant surface temperature and
constant heat flux condition.
Nu = 0.023 Re" x Pr"
where n = 0.4 if fluid is getting heated
= 0.3 if fluid is getting cooled
3. Fluid is liquid metals (Pr << 1)
(a) Constant surface temperature
Nu = 4.8 + 0.0156 Pe0.85 pr 0.08

where Pe = Re x Pr
(c) Constant heat flux condition
Nu = 4.82 + 0.0185 (Re x Pr)°•827

33. For laminar flow inside tube, show—

fo U • r • T • dr
(a) mean temperature T „, =
U • r • dr
J0

where u is velocity at radius r where r = radius of tube


(b) heat transfer coefficient =
A (Ts. — T„,)
232 Heat and Mass Transfer

where Q = rate of heat flow from tube to fluid,


A = tube surface area, and
Ts = wall surface temperature of the tube

r
r + dr

Consider a laminar flow through a tube as shown above. The velocity and temperature
both vary along the radius at a particular cross-section. The velocity is maximum at centre
and it becomes zero at the wall. Hence, velocity is a function of radius
u = f(r)
The mass of fluid flowing in the pipe is integration of flow through an elemental area
having radius `r' and 'dr' is—
M= f (elemental area) x velocity x density
o

= f (27 r r • dr) x 4 x p (i)


o
If 'um' is the mean velocity of flowing fluid through the tube, the mass flowing is—
M= R2 • u ff, • p
Equating equations (i) and (ii), we get—

ir R2 • urP= f 22r r • dr • u • p
o
R
or um = 2 f u • r • dr
1? 0
Now temperature of the fluid also varies with radius. Hence, we have temperature as a
function of radius as—
T = f(r)
Total enthalpy of the fluid passing through the tube is—

Q= f (Elemental area) x velocity x density x sp. heat x tamp.


o

= j (2,nr•dr)xuxpxCp xT
o
rR
= 27rpCp J r • u • T • dr
o
Forced Convection Heat Transfer 233

In case T ff, is the mean temperature of the fluid, then enthalpy of the fluid is—

Q= f (27r r • dr) u p Cp • Tn,


o

= 2irpCpTifi r u • r • dr (iv)

Equating equations (iii) and (iv), we have—


R
f U•r •T • dr
o
T,„ = R
U• r • dr
fo

R
But = 2 u • r • dr
R2 0

Tm = 2R u • r • T • dr
R2 um ,0
Now Q = hA(T, — Tn.)

or h— Q
A(Ts —Tm )

34. Explain heat transfer in a pipe when the flow is laminar and fully developed in the
condition: (i) when [ pipe wall is maintained at constant temperature, and (ii) when

constant heat flux q = ( 2) is given to the surface of pipe.


A s

T8 I, Ts I.— Ts —.1 Ts
////t/+////////1)i /////////// //t///

T.--

//////////// //////:3////7////// //////////


I.._Tffd 1.._Tf2_,1 HTf3H
HT H
Tfi> T T f2 > Tfi Ts > Tf3
f
Tube wall at constant temperature (Ti)

As the fluid at a temperature To, enters in the tube which is maintained at temperature Ts,
heat transfer from the wall to the fluid takes place. The fluid at the surface attains the
temperature at the wall but temperature at centre of the tube remains at T. As the fluid
moves further, the thermal boundary layer grows and temperature at the centre of tube
increases which ultimately becomes equal to temperature of the surface (T,) if tube has
sufficient length. The average temperature (Tm) at a cross-section increases in the direction
of the flow.
234 Heat and Mass Transfer

—0- Distance
Constant wall temperature heating

However, in case of constant heat flux heating, the temperature of the fluid and
temperature of the wall both increase along the flow direction. Hence, the fluid flowing in
the tube can never reach the wall temperature. Both wall temperature and fluid mean
temperature increase.
Ts3 H— 7-84 —P1
//////////// //////////// /////////////// //////

//////////// //////////// //////////////////////


Tf Fdr.- Tf HTfel I Tf 3
~Tf2~

Constant flux heating Tf3 > Tf2 > Tr, > Tf

Co
a)
E
H

x
—0- Distance
Constant flux heating

35. In a constant surface temperature tube, the fluid with laminar flow enters at
temperature Ti and leaves the tube at temperature To, Prove that—

hA, • L
TS — To mC
=e P

Fluid is entering at temperature `Ti' and leaving at temperature 70'. 'T5' is temperature of
the wall. Heat is flowing from the wall to the fluid and mean temperature of the fluid is
increasing in the direction of the flow. Consider a control volume of the fluid of length 'clx'
and apply energy balance on the fluid in this control volume. We have—
Forced Convection Heat Transfer 235

Increase in enthalpy of the fluid in the control volume = Heat convected to the fluid in
control volume from the surface

m • Cp • d Tift = h • (T, — T,p) • dA


where m = mass of fluid flowing
However, dA = P • dx where P = perimeter

dTm h•P
X dx
Ts — Tni m • Cp

On integration, we get—

T° dT„, h • P fL
dx
JTs
T
- T„, m•Cp o

h• P
or [—log (Ts — T,„]To =
Ti mCp
hP
Ts — To m•Cp XL
or
Ts — = e

x dx
x=0
F- L
Heating of constant surface temperature

Variation of mean temperature of the fluid

36. In a constant surface tube, show that the heat transfer rate to the fluid with laminar
flow can be given by
Q= h • As • (AT)L.„,

where A TL„, is the log mean temperature difference which is equal to—
236 Heat and Mass Transfer

) — (TS — To)
(A T)Lin — ( —
Ts —
log
TS — To
Or
Derive an expression for heat transfer coefficient for laminar flow inside a tube,
given that temperature is constant throughout.
(UPTU — 2003 — 4)

The heat transfer rate for the fluid in constant surface temperature tube is—
Q = m • Cp • (To — Ti)

where T = inlet temperature of the fluid


To = outlet temperature of the fluid
= mC[(T, —T1)— (Ts —To)] (i)
where Ts = Tube surface temperature
However, temperature distribution is—
h•P•L
Ts — Ti m .c
Ts —To = e
h•P• L
or
CP T—T
log T:_
To
Putting the value of m • Cp in equation (i), we get—

Q=h•P•L
(Ts — — (Ts — To)
Ti
Ts —T
log
Ts — To
= h • P • L • (AL)Lm

where (AL))LLm = (Ts — — (Ts — To)


— T —T
log
Ts — To

m • cp • ( To — Ti)
or h=
P • L • (AL) L„, P • L • (AL)lin

37. Find the length of the pipe 'I,' required when tube wall at constant temperature 'Ti'
to heat fluid from 'Ti' to `T0' with flow rate of `tn' kg/s.

We have already found out—


h.P.L
Ts — T
= e m•cp
Ts — To

h•P•L Ts —
or =log
M•C TS —To
Forced Convection Heat Transfer 237

m•C T —Ti
or L— P x log s
h•P Ts—To

38. Find the expression for heat transfer in a tube in laminar flow when constant heat
flux is supplied to the wall of the pipe. Show Nu = 4.36.

Consider an element of length `dx' of the fluid and thickness 'dr' at a radius `r' as shown
in the figure.

fi
R

~— dx H

The increase in enthalpy along radial direction of the fluid passing through the thickness
`clr' is equal to the heat conducted through the thickness `dr', i.e. dQ = Q r — Qr+dr

As Qr = k • (27r r • dx) d T

And Qr+dr = Qr dd (Qr) • dr


r

dQ = Qr+dr Qr

= d (k •2,irr • dx • dr )
dr

d dT
= Drk•dx • dr r (i)
dr dr )
Increase in enthalpy can also be calculated as—
dQ = mass x sp. heat x temperature rise
= m • Cp • dT
= (27r r • dr • u • p) • Cp • dT
Equating equations (i) and (ii), we get—

2,7rIc • dx • dr• d lr .dT) = 2,7r r • dr • u • p • Cp • dT


r dr

d (p•Cp ) dT
or = u•r
dr Cr dr k ) dx

u • r dT
a dx
238 Heat and Mass Transfer

But u= umax 1— (
ri )2]
d r dTl umax dT r3
.*.
dr r dr — a dx R2 )
On integration, we get—
dT umax dT (r r 3 j+ Cl
dr — a dx 2 4R2 R
Once again integrating, we get—
dT (r 2 r4
T = umax + Ci log r + C2
a dx 4 16 R2

dT
Now at r=0, 0 and also at r = 0, T = Tc
dr =
where Tc = Temperature at centre of tube. Applying these conditions, we get—
C1 = 0, and C2 = Tc
T = T umax dT R2 [( r )2 1 r )4 1
c a dx 4 0? ) 4 0?
( R)4
Now apply r= R, T = 7's, we get—
u dT 3 2
7; = T + • R
c a dx 16
The bulk mean temperature is—
= Bulk enthalpy
Bulk mass x cp
i?
J (2nr • dr • u • p)Cp xT
o
Cp f 2nr•dr•u• p
o

U•T•r• dr
fo

U•r• dr
fo

Now putting the values of u and T, we get—

R2 r)2
u
r2,1Hma, dT 4
1( r)))
r•umax [1— dr
fo R2 _I -c a dx 4 /?) 4U?
Tin =
"2
r
r • umax(1— 2 dr
JR
Forced Convection Heat Transfer 239

, dT 7
= + umax R- • •
a dx 96
Now heat flow is—
dT
Q= hA(T, — T,n) = kA(
drj r=R

But
(dT) umax R dT
dr L=R a 4 dx

Q= k•A U max R dT
a 4 dx

(dT
k
dr j r=R
Also h—
Ts — T„,

uinax R dT
•k•
a u dx
u dT 3 2 ( u dT 7
T + inax R T + inax x
a dx 16 e a dx 96

= k • 24
R 11
h • R 24
or
k 11
h•d 48
or where d = 2R
k 11
Nu = 4.36.

39. Explain dimensional analysis and apply the same to forced convection heat transfer
process.

Dimensional analysis helps in determining a possible arrangement of variables in a physical


relationship. This is achieved by forming a number of non-dimensional groups free from
units from given number of dimensional quantities in such a way that variables can be
reduced and physical relationship can be expressed in a possible arrangements of remaining
variables. Dimensional analysis is done by balancing the fundamental dimensions viz. mass,
length, time and temperature. Buckingham ' iv' theorem is used in dimensional analysis when
variables are large. The theorem states that if there are 'n' number of variables and 'm'
number of fundamental dimensions in a dimensionally homogeneous equation, then the
variables can be arranged in (n — m) number of dimensionless terms. These dimensionless
terms are called 'n' terms.
Let x1 = .f (x 2, x3... xn)
240 Heat and Mass Transfer

f(xi, x2, x3 ... xn) = 0


If there are 'n' variables with 'm' fundamental dimensions, then equations can be written
as—
fOri, irn,)=
The forced convection heat transfer process depends on the following variables-
1. Fluid velocity (V) = LT-1
2. Fluid density (p) = AIL-3
3. Fluid viscosity (y) = ML-1T-1
4. Fluid thermal conductivity (k) = MLT-3 0-1
5. Heat transfer coefficient (h) = MT-30-1
6. Fluid specific heat (Cr) = L2T-20-1
7. Diameter of pipe (D) = L
We have now n = 7 and m = 4. Hence, `7C' terms = 7 — 4 = 3. Hence, we have—
Arch 1r2, 1r3) = 0
where = Da pb tic kd • h
2T2= Da Pb tic kd V
ir3 = Da pb /lc kd Cp
The above `7C' terms are formed by selecting repeating variables as D, p, II and k.
Now i1 = La(M L-3)b(ML-1T-1)c(MLT-3 0-1)d x MT-30-1 = M°L°T°Q°
On solving we get a = 1, b = 0, c = 0, d = 1
hL
= = Nusselt number.
pVd
Similarly, we get r2 = = Reynolds number

µ Cp
7r3 = — Prandtl number
k
Hence, from above, we get for forced convection heat transfer a relation of dimensionless
numbers as—
= f(r2, r3)
or Nu = f(Re, Pr)

40. Derive an expression for analogy between momentum and heat energy transfer.
An expression for a relation between heat transfer and skin friction is to be developed. Near
the surface, the fluid is stationary. Hence, heat transfer is due to conduction near the
surface. The ratio of heat flux to shear stress in laminar flow is-
dT
-k
qs = dy = —k dT
S
du au du
dy
Forced Convection Heat Transfer 24 I

_ - Cp dT
µCp du
k
.- Cp dT
Pr du
For the identical velocity and thermal boundary layers, we have Pr = 1

. qs =-C dT
zs P du
(qs x1 j
or du= -dT
T s Cp
On integration, we get-
u.c L
( qs • cl j f du = - f dT
Ts p o Ts

or qs u_ = Ts - T_
T s • Cp

or 1 ( qs ) __ Ts __ [ T s j x uc,p
Cp Ts -Ts., u_ 1nu! 2
2'
1 qs 1 Ts
or
Cp • p u_ (Ts - T_ - 2 1 n u!
2r
Ts
Put = cf, = local skin friction coefficient
1
- pu-
2
qs
and - h = local heat transfer coefficient
Ts -T x
We get-
hx _C fx
p•Cp •tic, 2

(h,,x)
or
k ) _ C fx
rps • c,x )r 2
upj
II A k

Nux _ Cf .,
or
(Re)x • Pr - 2
242 Heat and Mass Transfer

The above is called Reynolds analogy which expresses the relationship between fluid
friction and heat transfer for laminar flow on a plate. It is valid only when the Prandtl number
is unity. If the Prandtl number is different than one, Colburn analogy is used which gives—
Nux
= 0.332 Rex1/2
Rex • Pr y 3
or Stx • Pr2/3 = 0.332 Rex1/2

41. What is the expression for the heat flow coefficient across a cylinder for turbulent
flow?

As per the experimental data of Hilpert for gases and Knudsen Kartz for liquids, the average
heat transfer coefficient for flow across cylinder is given by—
hd
Nu = = c • (Re)" • (Pr)1/3
and where 'c' and 'n' are constant which depend on Reynolds number calculated at film
temperature of the fluid.

S.N. Re
1. 0.4 to 4 0.989 0.330
2. 4 to 40 0.911 0.385
3. 40 — 4,000 0.683 0.466
4. 4,000 — 40,000 0.193 0.618
5. 40,000 — 4,00,000 0.0266 0.805

42. Write the correlation for heat transfer from sphere for turbulent flow.

The correlation for heat transfer coefficient for gases and liquids flowing past sphere is
given by Whitaker which is—
\1/4
Nu = 2 + [0.4 Re112 ÷ 0.06 Re2/3)pr0.4 Pcc

The above is valid for 3.5 < Re < 8 x 104 and 0.7 < Pr < 380. Properties are calculated
at free stream temperature (T) except its is found out at surface temperature.

43. A fluid flowing over a flat plate has the following properties—
= 25.10-6kg/ms
Cp = 2 kJ/kg K
k = 0.05 W/mK
If 6 = 0.5 mm, find `8th'.
(GATE 92)

'LC 25.10-6 x 2 x 103


Pr = P
k 0.05
=1
Forced Convection Heat Transfer 243

Now St
(Pr) l/3
0.5 - 0.5 mm
(1)"3

44. Air flowing over a plate with a velocity of 2 m/s. Temperature of air is 10°C. If the
width of the plate is 30 cm and it is at temperature of 65°C, find the following
parameters at a distance of 25 cm from the leading edge—
(1) boundary layer thickness
(2) thermal boundary layer thickness
(3) local friction coefficient
(4) average friction coefficient
(5) total drag force.
(6) local heat transfer coefficient
(7) average heat transfer coefficient
(8) total heat transfer rate from the surface.

Guidance: From data bank, find the properties of air at film temperature of 10 + 65
2
37.5°C. Assume flow at one side.
p = 1.14 kg/m3, k = 273 x 10-2 W/mK,
= 1.005 kJ/kg K,
v = 16 x 10-6 m2/s, Pr = 0.67.
p•V•x V•x 2 x 0.2T
Now Re=
- v - 16 x 106
= 3.125 x 105 < 5 x 105
Hence, flow is laminar.
5x
Boundary layer thickness 6 -
VRex
5 x 0.25
or S=
V3.125 x105
= 0.224 cm
Thermal boundary layer thickness-
0.224
5th = (Pr)1/3 (0.67)1/3
= 0.256 cm
Local friction coefficient is-
C = 0.664 - 0.664
fx Rex V3.125 x105
= 0.12
244 Heat and Mass Transfer

Average friction coefficient is-


Cf = 2 Cfx
= 2 x 0.12 = 0.24
Drag force is-
F=rxA

= Cf x (PV2 ) (0.25 x 0.3)


(1.14 22 )
= 0.24 x (0.075)
2
= 5.04 x 10-3 N
Local heat transfer coefficient is-
hx • x =
Nux = 0.332 Rem'. Pr1/3
k • Re1/2 • Pr1/3
hx = 0.332 x —
x
2.73 x10-2
= 0.332 x x (3.125 x 105)1/2 (0.67)1/3
0.25
= 17.73 W/m2K
Average heat transfer coefficient is-
h = 2 hx
= 2 x 17.73
= 35. 46 W/m2K
Rate of heat transfer is-
Q = hA(T, - 71.0
= 35.46 x (0.25 x 0.3) (65 - 10)
= 146.27 W

45. Air at atmospheric pressure and 200°C flows over a plate with velocity of 5 m/s. The
plate is 15 mm wide and is maintained at a temperature of 120°C. Calculate the
thickness of hydrodynamic and thermal boundary layers and the heat transfer
coefficient at a distance of 0.5 m from the leading edge. Assume that flow is on one
side of the plate.
p = 0.815 kg/m3, µ = 24.5 x 10-5 Ns/m2
Pr = 0.7, k = 0.0364 W/mK
(AMIE 1997)

First to ascertain the type of flow, find Re number.


Forced Convection Heat Transfer 245

V•xxp 5 x 0.5 x 0.815


Rex = -
µ 24.5 x10-6
= 8.3 x 103 < 5 x 105
Hence, flow is laminar.
Boundary layer thickness is-

= 5 x 0.5
V8.3 x103
= 8.67 mm
Thermal boundary layer thickness is-
15 = S 1/ 3
(P
= 8.67
(0.7)1/3
= 9.76 mm
Local heat transfer coefficient is-
k
hx=— x 0.332 x Re f/2 x Pr1/3
x
- 0.0364 x 0.332 x (8.3 x 103)1/2 (0.7)1/3
0.5
or hx = 6.19 W/m2K

46. Water at 10°C is to be heated to 40°C in a tube of 2 cm diameter. The outside of the
tube is wrapped with an electric heating element that produces a uniform flux of
200 W/m3 over the surface and the mass flow rate is 0.2 kg/s. Find-
(i) The Reynolds number
(ii) The heat transfer coefficient
(iii) Length of the pipe
The relevant properties of water at 25°C are given below-
p = 997 kg/m3 C,, = 4180J/kg K
k = 0.608 W/mK µ = 910 x 104 poise
(UPTU - 2003-3)
µ = 910 x 10-6 poise = 910 x 10-7 Ns/m2
40+10
Film temperature Tf = = 25°C
2
246 Heat and Mass Transfer

4m 4 0.02
Now Re = x 910 x 10-7
rd • /I • (0.02)
= 13998.7 > 2300
Hence, the flow is turbulent.
91 x 10-6 x 4180
Now Pr = µCP =
k 0.608
= 0.626
For constant flux with fluid getting heated 'n' = 0.4
Now Nu = 0.023 Re°.8 x (Pr)o.4
= 0.023 x (13998.7)0.8 x (0.626)°A
= 0.023 x 2074.3 x 0.829
= 39.55
hx d
But Nu =
Nu • k
or h=
d
39.55 x 0.608
0.02
= 1202 W/m2K
Now Q = As x q = mCp(To —
or ir•d•Lx q=mCp (To—Ti)
m C (To — Ti)
or L= P
ir • d • q

0.02 x 4180 x (40 —10)


xx 0.02x 20 x103
= 1.996 m

47. Air at 20°C is flowing along a flat plate maintained at 134°C with a velocity of
3 m/s. The plate is 2 m long and 1.5 m wide. Calculate the local heat transfer
coefficient at 40 cm from the leading edge of the plate. Also determine the heat
transferred from the first 40 cm of the plate to air. For air (at 77°C), take following
property data—
p = 0.998 kg/m3, Pr = 0.697, C,, = 1009 kJ/kgK
v = 20.76 x 10-6m2/s k = 0.03 W/mK
(UPTU — 2003)

Find the value of Reynolds number at x = 40 cm.


pxtic,xti
Rex =
Forced Convection Heat Transfer 247

XX

3 x 0.4
20.76 x 10-6
= 5.78 x 104 < 5 x 105
Hence, flow is laminar. For laminar flow, we have—
Cp
Pr=
v • p • Cp
k
20.76 x10-6 x 0.998 x 1.009 x 103
0.03
= 0.697
hx x x
Nu = 0.332 x Re1/2 Pr1/3 =
k
k
hx= — x 0.332 x Re.1,/2 Pr1/3

0.03 x 0.332 x (5.78 x 104)1/2 x (0.697)1/3


0.4
= 0.03 x 0.332 x 2.4 x 102 x 0.887
0.4
= 5.3 W/m2K
h = 2 x hx
= 2 x 5.3 = 10.6 W/m2K
Q = h • A(T, — Lc)
= 10.6 x (0.4 x 1.5) x (134 — 20)
= 725.04 W

48. Water at 20°C enters a 20 mm diameter tube with a flow rate of 0.5 kg/s. The tube
wall is maintained at constant temperature of 100°C. Find the average heat transfer
coefficient and tube length, if the outlet temperature of water is 70°C. Assume that
the flow is hydrodynamically developed. Take following properties for water—
p = 977.3 kg/m3, Pr = 2.524, CI, = 4186 J/kgK
19.= 0.665 W/mK, µ = 4.01 x 10-4g/mc.
(UPTU — 2003 — 4)
Mass flow is—
ird2
m— xVxp
4
248 Heat and Mass Transfer

4m
or V-
re p
4m
p•V•d
P • 1d2p d
Re=
/1 /1

4m 0.5x4
r•dlt x 20 x10-3 x 4.01x10-4
= 7.94 x 104 > 2300
Hence flow is turbulent for turbulent flow, we have-
Nu = 0.023 Re°.8 x Pr OA
= 0.023 x (7.94)0.8 x (2.524)°A
= 276.9
h•d
Nu =
k
h= — k x Nu

0.665 x 276.9
20 x 10-'
= 9207 W/m2K
Now we have-
h pL
Ts -T - emcp
Ts -To
m•C
or L= P log Ts
h•P Ts -To
0.5 x 4186 x lo 100-20
9207x7rx20x10 — g 100 - 70
0.5 x 9186 x 0.98 x103
9207x x20
= 3.55 m

49. A plate is heated and its temperature is maintained at 60°C. Air at 27°C and 10 bar
flows over this plate at 2 m/s velocity. Calculate the heat transferred per hour per
unit width upto 400 mm from leading edge of the plate. For air at mean temperature
of 43.5°C, take v = 17.36 x 10-6m/s, k = 0.02749 W/m°C, Cp = 1.006 kJ/kg°C, h =
287 W/m2K and Pr = 0.7. Assume equation Nux = 0.332 (Re)112 (Pr)1/3.
(UPTU - 2004 - 5)
Forced Convection Heat Transfer 249

We have to ascertain the type of flow by finding Reynolds number.


V• x 2 x 0.4
Rex = =
v 17.36 x 10-6
= 4.608 x 104 < 5 x 105
Hence flow is laminar. Hence, we have—
Nux = 0.332 x (Rex)112 (Pr)113
But Nu = 2(Nu)x
. Nu = 0.664 x (Rex)1/2 (Pr)1/3
= 0.664 x (4.608 x 104)1/2 x (0.7)1/3
= 0.664 x 2.146 x 102 x 0.888
= 126.52
h •x
Now Nu =
k
or h = k • Nu
x
= 0.027 x 126.52
0.4
= 8.54 W/m2K
Q = hA(T, — 71,0
= 8.54 x (0.4 x 1) (60 — 27)
= 112.73 W = 112.73 J/s
Heat transfer in one hour = 112.73 x 60 x 60
= 405.83 kJ

50. Water is heated while flowing through a 1.5 cm x 3.5 cm rectangular cross-section
tube at a velocity of 1.2 m/s. The entering temperature of water is 40°C and tube wall
is maintained at 85°C. Determine the length of tube required to raise the
temperature to 70°C. Properties of water at 55°C are—
p = 985.5 kg/m3, CI, = 4.18 1U/kg K
r = 0.517 x 10-6 m2/s, k = 0.654 W/mK
and Pr = 3.26
Use the equation Nud = 0.023 Re3•8 x Prim
(UPTU — 2006 — 7)

4 x A, _ 4 x Cross - section area


Equivalent diameter (de) = —
P Wetted perimeter
= 4 x (1.5 x 3.5)
2(1.5 + 3.5)
= 4 x 1.5 x 3.5
10
= 2.1 cm
250 Heat and Mass Transfer

Now we have to find Reynold number.


p•V•d V•d 1.2 x 2.1 x 10-2
Re =
0.517 x 10-6
= 4.874 x 104
Also, Pradtl number is—
Pr = 3.26
Given equation—
Nu = 0.023 x (Re)°.8 x (Pr)o.4
= 0.023 x (4.874 x 104)13.8 x (3.26)°.4
= 0.023 x 5627.4 x 1.604
= 207.65
Nu = h d

or h= Nu
0.654 x 207.65
2.1 x 10-2
= 64.668 x 102 W/m2K
hPL
TS — T mc P
Now e
Ts — To
m • cp Ts — T
or log
h•P Ts — To

i7rd2
4 xvxpj •cp log -s
hxird Ts —To

d•V•p•Cp 7:s 7
— log
4h 7 3: To

— 2.1 x10-2 x1.2 x 985.5 x 4.18 x103 35 — 40


log
4 x 64.668 x 102 35 — 70

= 40.13 x 10-1 x log 4


5
= 4.013 x 1.098
= 4.408 m

51. A circular cylinder of radius 12.5 mm is initially at 150°C and is suddenly exposed
to oil at 80°C which moves at velocity of 2 m/s in cross flow over the cylinder. What
Forced Convection Heat Transfer 25 I

is the initial rate of heat loss per unit length of the cylinder? For oil take at T.. =
353°K-
v = 38.1 x 10-6 m2/s, Pr = 501, k = 0.138 W/mK
For oil at T, = 423 K, Pr = 98
(UPTU - 2005 - 6)
Temperature of oil = 80 + 273 = 353°K
d = 2 x (2 x (12.5 x 10-3)
Re- V•
38.1x10-6
= 1312
Now Nu = C • (Re)" • Pr1/3
For Re = 1312, C = 0.683 and n = 0.466
h•d
Nu = = 0.683 (1312)0.466 (501)1/3
= 0.683 x 28.37 x 7.94 = 153.9
Nu • k
Now h=
d
153.9 x 0.138
- 25 x10-3
= 849.5 W/m2K
Q= h • A • (T, - To.)
=hxr•d•L(Ts -T..)
or Q _ h • ir • d(T, - Toj
L-
= 849.5 x 3.14 x 25 x 10-3(150 - 80)
= 4.668 kW/m

52. Air flowing across a 10 mm diameter sphere at 1 atmosphere pressure and 25°C with
a velocity of 5 m/s. An electric coil inside the sphere maintains the surface
temperature at 80°C. Calculate the heat lost by the sphere.
At free stream temperature (T.. = 25°C), the properties of air are-
k = 0.026 W/m°C Pr = 0.708,
po. = 1.85 x 10-5 kg/s-m and v.< = 15.68 x 10-6 m2/s
At surface temperature (T, = 80°), we have
its = 2.07 x kg/ms
p•V•d V•d 5x10x10-3
Now Re =
It- - v. - 15.68 x 10-6
= 3189
252 Heat and Mass Transfer

Now we have-
pr0.4 x N= 1/4
Nu = 2 + (0.4 Re112 + 0.06 Re2/3)
0.4
(1.84 x 10-5
= 2 + [0.4 x (3189)1/2 + 0.06 (3189)2/3] x (0.708)" x
2.07 x10-5
= 32.01
h•d
But Nu = = 32.01
k • 32.01
h= —

0.026 x 32.01
10 x 10-3
= 84.3 W/m2K
Heat transfer rate-
Q = hA(Ts -
= h • (47r r2) (Ts -
2
= 84.3 [4 x (10 x 10-3 ) (80 - 25)
2
= 1.6 W

53. In a hollow cylinder of ID = 4 cm and OD = 6 cm, water is heated while flowing inside
the cylinder by maintaining outer surface at 70°C. The water enters at 10°C and
leaves at 45°C. The flow rate of water is 2 m/s. Find heat transfer coefficient. The
(10 45
properties of water at bulk temperature 27.5°C 2 = 27.5°C) are-
+

Cp = 4179 J/kg K, p = 997 kg/m3


= 855 x 10-6 Ns/m2, Pr = 5.83
k= 0.613 W/mK
The equivalent diameter is-
4 x Cross-sectional area
de -
Wetted perimeter

4x (D4' - Di2 )
4
ir (Do + Di )
= Do - Di
= 6 - 4 = 2 cm
Forced Convection Heat Transfer 253

p•V•de
Reynolds number Re —

997 x 2 x (2 x10)-2 =
46643 < 2300
855 x 10-6
Hence flow is turbulent—
Nu = 0.023 x Re°.8 x
= 0.023 x (46643)0.8 x (5.83)°.4
= 0.023 x 5.43 x 103 x 2.02
= 0.25 x 103
Nu k
But h—
de
0.25 x 0.613
x io3
2 x 10-2
2.5 x 6.13
— io3
2
kW
= 7.66 m2K

54. A liquid metal flowing at the mass flow of 4 kg/s in a 6 cm diameter tube which is
heated with constant flux. The surface temperature is 240°C and outlet temperature
of liquid metal is 200°C. Determine the length of the tube for 25°C rise in bulk
temperature. The properties of liquid metal are—

p= 7.7 x 103 kg/m3, v = 8 x 10-8 m2/s


Cp = 130 J/kg°C, k= 12 W/mK, Pr = 0.011
The equation Nu = 4.82 + 0.0185 Pe11872 where Pe = Re x Pr is to be used.

pVd
First find Reynolds number Re =

4m 4m
ir•d• au ir•d•(vxp)
4x4
x 0.06 x (7.7 x 103 x 8 x 10-8)
= 137870
Now Nu = 4.82 + 0.0185 (Re • Pr)°.827
= 4.82 + 0.0185 (137870 x .011)0.827
= 4.82 + 0.0185 x 427.14
= 4.82 + 7.9 = 12.72
254 Heat and Mass Transfer

hd =
But Nu
k
or h= • Nu

12 x 12.72
6 x10- 2

= 2544 x 102 = 2544 W/m2K


Now Q = mCp x AT = hA(T, -
= 4 x 130 x 25 = 2544 Or x 0.06 x L) (40)
4x130x 25
L= - 0.68 m
2544xxx0.06x 40

55. The ratio of thickness of thermal boundary layer to thickness of hydrodynamic


boundary layer is equal to (Pr)' where 'n' is-
(a) -1/3 (b) 2/3
(c) 1 (d) -1
(GATE)
The relation of thicknesses of hydrodynamic and thermal boundary layers is-

8th = (Pr)" 3
th
or = (Pry1/3
n = -1/3
Option (a) is correct.

56. For flow over a flat plate, the hydrodynamic boundary layer thickness is 0.5 mm. The
dynamic viscosity is 25 x 10-6 m2/s, sp. heat is 2 kJ/kg K and thermal conductivity
is 0.65 W/mK. The thermal boundary layer thickness would be-
(a) 0.1 m (b) 0.5 mm
(c) 1 mm (d) 2 mm
(GATE)
8 = (Pr)v3
th

25x10-6 x2x103
Now Pr = P -
k 0.05
=1
8 = (1)15 = 1
th
Forced Convection Heat Transfer 255

. 8 = 8th = 0.5
Option (b) is correct.

57. For air flow over a flat plate, velocity (u) and boundary layer thickness (8) can be
expressed respectively as-
2
u = 3 y _ 1 y and 3 - 4.64x
uo. 2 6 2 ( S ) V Rex
If the free stream velocity is 2 m/s and air has kinematic viscosity of 1.5 x 10-4
1112/S and density of 1.23 kg/m3, the wall shear stress at x = 1 m is-

(a) 2.36 x 102 N/m2 (b) 43.6 x 10-3 N/m2


(c) 4.3 x 10-3 N/m2 (d) 2.18 x 10-3 N/m2
(GATE 2004)

p•u•x u•x 2x1


For x = 1, Re = - =
/I, x 1.5 x10-5
= 1.34 x 105
(du
To = P d
u

Given u = u—[32Sy 21(121


(5
(du
dyjy=o - u-x 2x8
4.64x - 4.64x1
But (5)x=i =
. 1, 1 2x1
1 1.5 x10-5
= 0.0127
(du) . 3x 2
= 236.2
dy y=0 2 x 0.0127
du
To =
11 (dy jy=o
= (1.5 x 10-5 x 1.23) x 236.2
= 4.36 x 10-3 N/m2
Option (c) is correct.

58. Engine oil at 60°C flows at 0.5 kg/s in a duct with constant surface temperature of
20°C. Assume full developed flow. Find: (i) heat flux at entry, (ii) pressure drop per
256 Heat and Mass Transfer

metre length for 3 cm diameter and for 3 x 1 rectangular duct of equal wall area.
The properties of oil at 40°C are—
p = 876 kg/m3, k = 0.144 W/mK
v = 0.24 x 10-3 m2/s, C, = 1.96 kJ/kg K
Given m = 0.5 kg/s
=pA x V where V= velocity
or 0.5
V = 0.5 —
pA 876 x LT x (0.03)2
4
= 0.81 m/s
V x d = 0.81 x 0.03
Now Re=
v 0.24 x10-3
= 101 < 2300
Hence, flow is laminar.
Hence from correlation for laminar flow, we have—
Nu = 3.66
h •d
But Nu =
k
or h= x 3.66
d
= 0.144 x 3.66 = 17.57 W/m2K
0.03
Heat flux at entry will be—
Q = q = h(T, — To) where To = oil temperature
A
= 17.57 (20 — 60)
= —702.7 W/m2
Now friction factor for laminar flow-
f = 64 64
Re 101
= 0.633
The pressure drop equation is—
AP = f pV 2
L d 2
_ 876 x (0.81)2
— 0.633 x
0.03 2
= 6031 Palm
Forced Convection Heat Transfer 257

In case of rectangular duct of same area, we have—


Area of circular duct = Area of rectangular duct
ir • d • 1 = 2(a + 3a) x 1
ir • d = 8a
7rd
or a=
8
7r x 0.03
8
= 1.18 m
Now if de is the equivalent diameter of the rectangular duct, we have
4A 4xax3a
de =
P 2(a + 3a)
12 a
= = 1.5 x 1.18
8
= 1.77 mm
Now the velocity through the rectangular duct is—
V, = m 0.5
p•A 876 x 3a2
0.5
876 x 3x (1.18 x10-3)2
= 1.37 m/s
V x de
Re =
v
1.37 x (1.77 x 101
0.24 x 10-3
= 100.8 < 2300
The flow remains laminar and from correlation we get—
Nu = 3.96
h • de
But Nu =
k
or h= Nu
de •
3.96 x 0.144
= 32.3 W/m2K
0.177
Now heat flux at entry—
A = q = h(T, — Toc)
258 Heat and Mass Transfer

= 32.3 (20 — 60)


= —1289 W/m2
For a rectangular duct, we have—
68.36
f = Re,

68.36
100.8
= 0.67
Now pressure drop is—
AP f p Vr2 0667
. x 876 x (1.37)2
L de 2 0.177
= 31350 Pa/m
Note: If we calculate pressure drop per unit length for equal heat flux, it can be seen that
rectangular duct results into higher pressure drops requiring more power for fluid flow.
Hence circular ducts are more efficient.

59. Water flowing over a plate of size 100 cm x 100 cm with a velocity of 2 m/s. The plate
is at uniform temperature of 90°C and water temperature is 10°C. Estimate the rate
of heat transfer from the entire plate. Also calculate the length of plate over which
the flow is laminar. The properties of water at 50°C are—
p = 988 kg/m3, v = 0.556 x 10-6 m2/s
k = 0.648 W/mK, Pr = 3.54
Guidance: In case Reynolds number is more than 5 x 105, such problem is solved by
finding firstly the length of plate 'x' where the Reynolds number is equal to 5 x 105 i.e.
critical Reynolds number. The flow having laminar part and turbulent part can be solved by
correlation Nu = Pr1/3 (0.037 Re" — 872)
pV L V L
Re =

2 x100 x10-2
= 3.597 x 106 > 5 x 105
0.556 x 10-6
Suppose x is the distance from the leading edge which gives critical Reynolds number.
•x
Rec = 5 x 105 = V

5 x 105 x 0.556 x10-6


or x=
2
= 0.139 m
Now Nu = Pr1/3 (0.037 Re" — 872)
= (354)1/3 [0.037 x (3.59 x 106)0.8 — 872]
= 8563
Forced Convection Heat Transfer 259

hxL
Now Nu =
k
or h = — x 8563

0.648 x 8563
1
= 5549 Wm2K
Now q =hA(Ts —L)
= 5549(1 x 1) (90 — 10)
= 444 kW

60. When there is a flow of fluid over a flat plate of length 'V the average heat transfer
coefficient is given by—
(a) f hx dx d
(b) — (hx)
o dx
(c) f:hx dx (d) k fNu
L dx (Nux = local Nusselt number)
L 0 x
(IES — 1997)
The average heat transfer coefficient is—
h= —
1 f L dx
o hx
Option (c) is correct.

61. Consider the development of laminar boundary layer for a moving non-reacting fluid
on contact with a flat plate of length 'V along the flow direction. The average value
of heat transfer coefficient can be obtained by multiplying the local heat transfer
coefficient at the tailing edge by the factor.
(a) 0.75 (b) 1
(c) 1.5 (d) 2
(IES — 1996)

hx x L
Local Nusselt: Nux — k

hxL
and average Nusselt: Nu =
But Nu = 2 Nux
hxL 2xhx xL

h = 2 hx
Option (d) is correct.
260 Heat and Mass Transfer

62. The transition Reynolds number for flow over a flat plate is 5 x 105. What is the
distance from the leading edge at which transition will occur for flow of water with
uniform velocity of 1 m/s? (For water v = 0.858 x 106 m2/s)
(a) 1 m (b) 0.43 m
(c) 43 m (d) 103 m
(IES — 1994)

V•x V• x
Re, = 5 x 105 = P

5 x 105 x v
or x=
V
5 x105 x 0.858x10-6
1
= 0.43 m
Option (b) is correct.

63. Atmospheric air at 10°C and at a velocity of 15 m/s flows over a 2 m long flat plate
maintained at a temperature of 40°C. Find: (i) the average heat transfer coefficient
in laminar boundary region, (ii) the average heat transfer coefficient for entire
length, and (iii) total heat transfer rate.

10 + 40
The film temperature = = 25°C
2
The air properties at 25°C are
Pr = 0.708, II = 2 x 10-5 kg/ms,
v = 16.8 x 10-6 m2/K and k = 0.026 W/mK
p•V•L V•L
11,
15 x 2
— 17.86 x 105 > 3 x 10-5
16.8 x 10-2
Hence flow is turbulent. Let 'x' is length for transition Reynolds number.
V •x
Re = 3 x 10- —

3 x105 x16.8 x10-6


or x=
15
= 0.336 m
For laminar flow, we have-
hx
Nu = = 0.664 Rem Pr15
T
Forced Convection Heat Transfer 26 I

or k
h=— x 0.664 Re f/2 Pr1/3
x
0.026 x 0.664 x (3 x 105)1/2 (0.708)1/3
0.336
= 9.99 W/m2K
For entire surface including laminar and turbulent region, we have
Nu = (0.037 Re" - 872) Pr1/3
= [0.037 (17.86)0.8 - 872] (0.708)1/3
h•L
- Nu
k
h
0.026 x [0.037(17.86)08 - 872] x (0.708)1/3
= 32.95 W/m2K
Now heat transfer is-
Q = hA(T, -
= 32.95 x (2 x 1) (40 - 10)
= 1976 W

64. For a forced flow over a flat plate of length '1,', the local heat transfer coefficient 'hi'
is known to vary as x-0.5 where 'x' is the distance from the leading edge of the plate.
Determine the ratio of the average Nusselt number for the entire plate (NuL) to the
local Nusselt number at x = L(NuL).
(UPTU - 2007 - 8)
hx = x-0.5

hL, = —
1 f L h dx
L x

1 f L x-°• 5 • dx
=L o

[x +°5 1L
L 0.5 jo
1 L°5 - 2 L-0.5
L 0.5
Now (hx)x.1, = L-0.5

h•L
Nu
k
(hx)x=z, L
(Nux)x=i,
k
Nu 2L-°5
=2
(Nux)x=L (hx)x=z, L °.5
262 Heat and Mass Transfer

65. Air at 34°C is flowing over a flat plate maintained at 120°C with a free stream
velocity of 5 m/s. The plate is 10 m long and 1 m wide. Calculate the local heat
transfer coefficient at 0.5 m and 5 m from the leading edge of the plate. Also
determine the heat transferred from the first 50 cm of the plate to air. For air take
Pr = 0.697, v = 20.76 x 10-6 m2/s, k = 0.03 W/mK.
(UPTU - 2007 - 8)
1. At x = 0.5
p•V•x V•x
Rex = -
/I, v
= 5 x 0.5
20.76 x 10-6
= 1.2 x 105 < 3.5 x 105
Hence, flow is laminar. Hence, we have-
Nux = 0.332 Re1/2 Pr1/3
= 0.332 x (12 x 104)1/2 x (0.697)15
= 0.332 x 3.464 x 0.886
= 1.02
xx
Nux = hk = 1.02

x 1.02 = 0.03 x 1.02


or (hx)x=o.5 = xk
0.5
= 0.0612 W/m2K
2. At =x =5m
5x5
Rex =
20.76 x 10-6
= 1.2 x 10+6 > 5 x 105
Hence, flow is turbulent. Hence, we have
(Nu)x = 0.0296 x Re°.8 x Pr1/3
= 0.0296 x (1.2 x 10+6)0.8 x (0.697)15
= 0.0296 x 73003.72 x 0.8866
= 1916
h xx
Nu = x
x k
Heat transfer coefficient at x = 5 m
k
or (hx),5 = - • Nux
x
= 0.03 x 1916
5
Forced Convection Heat Transfer 263

= 11.496 W/m2K
Now heat transfer coefficient at x = 0.5
(hx)x.0.5 = 0.0612
hay = 2 x 0.0612 = 0.1224
Q = 0.1224 (0.5 x 1) (120 — 34)
= 5.2632 W

66. Water at 40°C with a flow rate of 1 kg/s is heated in a tube to 60°C. Surface of the
tube is maintained at 100°C. Find the length if the diameter of tube is 25 m.
For water (at 50°C) — Cp = 4.81 J/kg.K, µ = 5.48 x 10-4 kg/ms, k = 0.643 W/mK,
Pr = 3.56
For water (at 100°C) — µ = 2.79 x 10-4 kg/ms
(UPTU — 2007-8)
Properties of water at bulk temperature
40 + 60
— 50°C
2
4m 4x1
Velocity V=
p • d2 1x103 x x (25 x10-3)2
= 2.038 m/s
p•V•d 1x103 x 2.033x 25x10-3
Re =
5.48 x 10-4
= 9.33 x 104 > 2300
Hence, flow is turbulent. Hence we have—
Nu = 0.023 x Re" x pro.4
= 0.023 x (9.33 x 104)0.8 x (3.56)0.4
= 0.023 x 9460 x 1.66
= 361.6
hd
But Nu = k

or h= k x Nu
0.643 x 361.6
25x 10 3
= 9.3 x 103
Perimeter P=7cd
h.P.L
T, — _ mC
e P
T, — To —
264 Heat and Mass Transfer

or L= m CP 10g Ts—
h•P Ts — To
1 x 4181 lo 100 — 40
9.3x103 xxx 25x10— g 100-60
4181 x 0.405
9.3xivx25
= 2.322 m

67. Match List-I and List-II and select the correct answer.

List-I List-II
(Analogy between) (Number)
A. Momentum and Heat transfer 1. Stanton number
B. Momentum and mass transfer 2. Lewis number
C. Mass and heat transfer 3. Sherwood number
Codes: A B C
(a) 1 3 2
(b) 1 2 3
(c) 3 1 4
(d) 3 2 1
Option (a) is correct

68. Match List-I and List-II and select the correct answer.

List-I List-II
(Number) (Ratio of)
A. Biot number 1. Buoyance force to thermal diffusivity
B. Reynold number 2. Momentum diffusivity to thermal diffusivity
C. Prandtle number 3. Inertia force to viscous force
D. Grashof number 4. Conductive to convective resistance of slab
Codes: A B C D
(a) 4 1 2 3
(b) 4 3 2 1
(c) 2 1 4 3
(d) 2 3 4 1

Option (b) is correct


Forced Convection Heat Transfer 265

69. Match List-I and List-II and select the correct answer.

List-I List-II
A. Reynold number of pipe 1. Film coefficient, diameter and thermal conductivity
B. Prandtle number 2. Flow velocity, acoustic velocity
C. Nusselt number 3. Heat capacity, dynamic viscosity, thermal conducivity
D. Mach number of pipe 4. Flow velocity, diameter and kinematic viscosity
Codes A B C D
(a) 4 1 3 2
(b) 4 3 1 2
(c) 2 3 1 4
(d) 2 1 3 4
Option (b) is correct
Chapter 7
FREE CONVECTION

KEYWORDS AND TOPICS

A COEFFICIENT OF THERMAL EXPANSION A ENERGY EQUATION


A BUOYANCY FORCE A DIMENSIONAL ANALYSIS
A BOUNDARY THICKNESS A RAYLEIGH NUMBER
A GRASHOF NUMBER A NUSSELT NUMBER
A PRANDTL NUMBER A FILM TEMPERATURE
A SIGNIFICANT LENGTH A VON KORMAN INTEGRAL TECHNIQUE
A CONTINUITY EQUATION A CONVECTION CORRELATION
A MOMENTUM EQUATION A LAMINAR & TURBULENT

INTRODUCTION

The heat flows from hot body to the surrounding fluid due to the temperature difference. The
fluid in vicinity of hot body gains temperature and its density decreases. Due to the difference
in densities of the warm fluid in vicinity of the hot body and the cold fluid away from the hot
body, the warm fluid begins to flow in upward direction. The force making the warm fluid to
flow upward is called buoyancy force. Whenever heat transfer is caused due to the motion of
fluid by densities difference only, it is called free or natural convection heat transfer. The
circulation pattern consisting of upward movement of the warm fluid and downward movement
of cold fluid is called convection currents. These currents are set up naturally due to gravity alone
and these are responsible for heat convection. Free convection would be non-existent if there is
no gravitational force. Heat transfer with free convection is insured in many applications such
as: (i) transmission lines, (ii) cooling of transformers, (iii) heating of houses, and (iv) cooling of
core of the nuclear reactor.

1. What is free convection? How does it differ from forced convection?

In forced convection, heat transfer by the fluid motion is induced by a pump, fan or a
blower, whereas in free convection, fluid motion is induced by a charge of density of the
Free Convection 267

fluid resulting from heating or cooling. It is similar to hot air balloon which rises as hot air
inside the balloon has lower density as compared to surrounding air.

Pair

Hot air balloon

The fluid motion is set up as a result of the buoyancy force caused by density difference
which takes place due to temperature difference. The heat transfer by convective motion
as a result of buoyancy forces is called free convection.

iffr
A
Turbulent
Vertical Flow
Fluid motion hot plate (Ts )
on heating

Laminar
flow

////////////////////////
Boundary layer on
T I Fluid at To<

Horizontal hot plate


fluid motion vertical plate
Hot plate: Natural convection

Consider a hot plate placed vertically in a fluid at rest whose temperature (T) is lower
than that of hot plate (T5 ). The fluid in contact with the hot plate will be heated up due to
conduction of heat from the hot plate. A temperature gradient will be established in the fluid
in which fluid in contact is at the highest temperature and fluid temperature decreases as
fluid is located away from the hot plate. The temperature gradient within the fluid generates
a density difference. The hot fluid has lower density resulting more buoyant force acting
upwards and hot fluid moves up. It is replaced by colder fluid having higher density and
higher gravitational force. The fluid motion is set up and the heat transfer takes place from
hot plate to fluid due to convection motion which is called free convection.
268 Heat and Mass Transfer

2. Compare and explain the possible quantity of heat transfer by free and forced
convection.

Heat transfer in free convection is quite small as compared to forced convection. The flow
velocity of the fluid in free convection is much smaller than that of forced convection. The
fluid motion in free convection is set up as a result of buoyancy forces caused by density
difference which are small and heat transfer by free convection is therefore small.
The fluid motion in forced convection is comparatively larger resulting higher transfer of
heat by convection.

3. What is the coefficient of thermal expansion or volumetric coefficient of thermal


expansion?

The fluid adjacent to the hot surface gets heated and this results in thermal expansion of the
fluid. The thermal expansion of the fluid reduces the fluid density as compared to the fluid
density prevalent far from the surface. The difference in densities generates a buoyancy
force which acts on the fluid causing fluid motion in which hot fluid moves up and it is
replaced by cold fluid.
The coefficient of thermal expansion is the ratio of increase of volume per unit
temperature to the original volume at constant pressure.
(dV)
a T P = Const
13 — V (i)

But 1 where p = density


V=
On differentiation, we get—
av _ 1 aP
aT p2 oT

la V
a T1p=Const
But - V

1 (ap
P aT)p=Const

(ap
P a T)P=Const
For ideal gas, we have—
PV = RT

or V = RT
Free Convection 269

or (ay) =R
a T P=Const P

Putting the values of V and ( a VT ) in equation (i), we have-


a P=Const

la v1
— a T )p= Const
ig
V
R
_ P
- RT
P
_1
- T
4. Find the expression for the buoyant force per unit mass of the fluid.
The coefficient of thermal expansion is-

fi= i(ap
P a I')P=Const
The fluid at surface is heated to temperature T and has density as p, while fluid at distance
from surface has temperature at T.. and density as p...
(a pj P - P-
.%
a T P=Const - T -T
# _ 1 rp—p-)
pT-L
or p/3 (T - To.) = po, - p (i)
The buoyancy force per unit volume of fluid is-
Fb per unit volume = g (p0. - p)
= g x p/3 (T - L) from equation (i)
= g pI3 AT where A T = T - T..
Now buoyant force per unit mass of fluid is-
g p PAT
Fb per unit mass -
P
= g/3 • AT

5. What are the parameters on which buoyancy force of the fluid is dependent?
270 Heat and Mass Transfer

The buoyancy force per unit mass = g • 13 • AT


Hence, buoyancy force is dependent on-
1. Larger is temperature difference in the fluid, larger is the buoyancy force.
2. Larger is the coefficient of thermal expansion (/3), larger is the buoyancy force.

However, 13 = —1 and it decreases with increase of absolute temperature of the fluid.


Larger is the buoyancy force, stronger will be natural convection currents resulting into
higher heat transfer rate.

6. What is Grashof number?


The flow in free convection is characterized by a dimensionless number called Grashof
number. Grashof number is the ratio of buoyancy force to viscous force acting on the fluid.
Gr = Buoyancy force
Viscous force
g • /3 • AT, m
where v = kinematic viscosity
pV 2
g • /3 • AT • V
112 where V = volume

Now volume V = Lc where Lc = significant length


g#(T — T )L3
Gr = ' where Lc = Surface area
112 Perimeter

7. Explain the significant length for: (a) vertical plate, (b) vertical cylinder, (c) horizon-
tal cylinder, (d) sphere, (e) rectangular duct (horizontal), and (1) circular disc of dia-
meter (d).
The characteristic length is given as—
LC =
Surface area A,
Perimeter
The significant length for various bodies is taken as—
(a) Vertical plate — vertical height
(b) Vertical cylinder — height of cylinder
(c) Horizontal cylinder — diameter of cylinder
(d) Sphere — diameter of sphere.
(e) Rectangular duct horizontal: — 1 =1 1
Lc Lv LH
where Lv = vertical side and LH = horizontal side
Free Convection 27 I

A, ir2
(f) Horizontal disc lc = — = — , while for vertical disc = d where d= diameter
Ps 4 7rd 4
of disc.

8. What do you understand by the: (i) continuity equation, (ii) momentum equation,
and (iii) energy equation for laminar flow in a boundary layer?

The continuity equation signifies that the mass entering a controlled volume is equal to mass
leaving the controlled volume i.e. mass is conserved in the controlled volume. The continuity
equation is—
v
= 0 where u and v are velocities in x and y directions
ax a y
The momentum equation is based on Newton's second law i.e. charge of momentum is
equal to the applied force. The charge of momentum in a control volume of fluid results in
variation of velocities in 'x' and 'y' directions while applied forces are viscous force, gravity
force and pressure force.
Change of momentum of fluid
= Viscous force + pressure force + gravity force

or p(u
au+ Vau) =a2u2 aP
ax ay ay a X— g
where P = pressure, II = viscosity and p = density
Energy equation is based on the conservation of energy in a control volume. Energies
involved are: (1) convected heat energy, (2) conducted heat energy, and (3) viscous work
done. On energy balance, we get energy equation which is—
aT ,,aT a2T = (au 2
u + aye
ax v ay pCp pCp j

or u
aT aT
+v =a
a2T+ v (al
ax ay ay2 Cp ay
au
In case viscous force is small i.e. = 0, then—
ay

= a a2T
aT aT
u +v
ax av ay2

9. Show that in free convection, the momentum equation can be modified to—
au +v a v = gli(T — T.0 +
u
ax P
ay2 U2
The general momentum equation in laminar flow in a boundary layer is—
272 Heat and Mass Transfer

au av aP — g + „ a2u
U +v = (i)
ax ay pax pay 2
At the edge of boundary layer, we can apply boundary condition i.e. u = 0 at boundary edge
where p =
Hence, 1 aP =0
g ax

or
aP = xg
ax
Substituting the equation (ii) in equation (i), we get-
u av g
u ax v ay = p (P- P)+ p ay2
2U
The changes in fluid temperatures and density are interrelated as-

1(Po. - = (To. - (iv)


Now substituting equation (iv) in equation (iii), we get-
ati , 2
u +v = gig (T - To0 + ay2 'u
ay ay p

10. Why is the problem of free convection more complicated than that of forced
convection?
The energy equation for free and forced convection is same but momentum equation in
forced convection is independent of temperature while in free convection it is dependent on
temperature. The momentum and energy equations in free convection are interconnected or
coupled. A solution for velocity profile in the boundary layer also demands a knowledge of
the temperature distribution. Hence, two equations have to be solved simultaneously in free
convection unlike in forced convection. This makes the problem of free convection much
more complicated as compared to the forced convection.

11. Using dimensional analysis to free convection, show-


Nu = f(Gr • Pr)
The variables affecting free convection are: (1) buoyant force: gl3 • AT(Lt-2), (2) height
of plate 'x'(L), (3) density of fluid ' (ML-3), (4) viscosity of fluid 'it' (ML-1 1_1)9 (5)
sp. heat Cp(MLt-37-1), (6) heat transfer coefficient 'h' (M'37-1) and (7) thermal
conductivity (MLt-37-1). Here variables n are n = 7 and fundamental dimensions m are
m = 4. Hence, number of 'n' terms are m - n = 7 - 4 = 3. Now choose repeating variables
as (g/3 AT), 'it', and 'x'. Hence, terms are-
xi = x b • kd] x p = M°L°t°T°
[(g/3
7r2 = ••• kdi C = MOLOtOTO
[(g/3 Ana
7r3 = Ana • x b • • kcl ] x h = M°L°t°T°
[(g 13
p
Free Convection 273

On solving, we get—
p2(gfiAT)x 3
= 2 = Grashof Number
Gr
/.1C p
7C2 = k = Prandtl number
= Pr
hx
7r3 = — = Nusselt number

= Nu
Now we have—
= f(x2, r3)
Nu = f(Gr x Pr)

12. Explain Von Korman integral technique as applied to natural convection on a vertical
heated plate. What are assumptions made in this technique?
The assumptions are—
(a) flow is laminar, steady and density variation exists within boundary layer.
(b) the buoyancy effects are confined within the boundary layer. Velocity perpendicular to
plate is zero.
(c) Prandtl number is unity i.e. 8th = 8.
(d) viscous force is negligible.
The general momentum equation in free convection is-
au a
u +v = fig(T — 71.) + ila 2u
p x
The general energy equation is—
aT +v aT = a a2T
u
ax ay y2

J pudy+ d Vpudyixdx
o8 dx

Control volume
274 Heat and Mass Transfer

On incorporating above assumptions, the momentum and energy equations reduce to—
d µ (ayu)
u2dy = fi • g (T — To ) dy—
dxJ
0
p y=o

and f u(T dy = a[ a (T T_)]


dx ay y=o
o
The velocity profile will have following boundary conditions—
(a) u = 0 at y = 0
(b) y = 0 at y = 8
au
(c) — = 0 at y = 8
ay
The temperature profile will have following boundary conditions—
(a) T=T at y = 0
(b) T=T at y = 8

(c) a (T — 71.) = 0 at y = 8

Assuming temperature profile as quadratic and velocity profile as cubical, we have-

(a) T = + C2 i( + C3 (;) 2

2
(b) u = ui[di+ a2W+ a3(w) + a 4( .67 where u1 = u(x)
.11
The temperature and velocity profiles in free convection can be found out as—

T— y )2
(a) =
TS — (5)
2
u =Y Y wh ere = uixgx[3x(Ts—L)52
(b)
u, 4xv

au
Using velocity profile, the maximum velocity (umax) can be found out by equating g =0

1 4y 3y2
or S 2+ 83 = "

Y = 6 or
3
Free Convection 275

Also u = i (i _1)2 = Au
' 3 3) 27 x
Now individual terms in momentum and energy equation become—
6 v 4
(a) 5 u2dy = 5 u02 x (5' 2 x(1— —
Y(5 ) dy
o o

=
4 (5
105
(5

(b) f (T — To )dy = (T — Lc) x 8


o
Uo
(u)y 0 = [u x Y (1_ Y )1 2 —
(c) OY = OY ° ( 5 (5 ) (5
y=0

(5 (5 2 2
u0 (Ts —L)3
5 ) x (Ts —L)(1— i) dy =
(d) f u(T — ToOdy = f u0 x El — 1
:
o o ( 30

The momentum equation reduces to—


t,(5) = /I x uo
+ fi • g(T — To0 • 8
dx 105 p (5
The energy equation reduces to—
a Fuo (Ts — Tjsi = 2a(Ts —L)
ax L 30 (5
To solve momentum and energy equation, we assume u., and (5 are functions of 'x'.
LI, = C1 x'n and (5 = C2 • Xn

Solving we get, 21 and n = —


m=— 4

C1 = 5.17. v [ 20 ± v l1/2 g • [3(Ts —L)11/2


L21 aJ v2
im Fg p (Ts _ T )dia -1/2
C2 v [20 ± vi
= 3.93 .
.

ry
21 aJ L 2
v i a
Using C2, velocity boundary thickness is-
5
= 3.93(0.952 + Pr)1/4 x Grua 1x pr1/4
x
276 Heat and Mass Transfer

The heat flux q is-

q=k
(a T =
2k(T-T) =h(TS-T)
dY)y=o
2k
or h=
Hence, Nusselt number is-
2k
hx x x o '
Nux = _
k k
2x
= = 0.508 Pr1/2 (0.952 + Pr)-1/4 x Gr1/4
(5
As found out above, we have-
8 = Cixn where n =
4
or S « x 114 i.e. thickness of boundary layer increases as 'x' increases.
Also we have seen that h=
2k 1 which gives hx « x-1/4. Hence, local heat transfer
or h « —
coefficient decreases with increase of 'x'.
Average h = 1f xdx
L oh
= 1 IL c, x-1/4 dx

4
[Cix314]x=L
3L

= hL
3 ,
Average Nu = 0.677 pr 1/2 (0.952 + pr)-ita x Grim
Some other relations extracted for above equations are-
27
(a) tt mean = 48 Umaximum
711/2,
(b) ttx = local velocity = 5.17 x ( 2? + Pr) 1/ 3 [ gP(Ts 2 x 112

4
(c) ttmax = 27 ttx
(d) Mass flow through boundary is-

Gr
M = 17 p [ L I/ 4
Pr- (Pr + 0.592)
Free Convection 277

+
Note: All properties are to be calculated at film temperature Tf — 2

13. What is criteria to identify types of flow in free convection?

In free convection, the type of flow is decided by a dimensionless number known as


Rayleigh number. The Rayleigh number is the product of Prandtl and Grashof number
Ra = Gr • Pr
If the value of Rayleigh number is—
(a) less than 109 (Ra < 109), the flow is laminar
(b) higher than 109 (Ra > 109), the flow is turbulent.

14. Tabulate correlation for free convection for various geometry and Rayleigh number
for constant wall temperature.

The Nusselt numbers for various geometry and Rayleigh number are tabulated as under.

S.N. Geometry Rayleigh Number (Gr. Pr) Correlation


1. Vertical plates and cylinders 104 < Ra < 109 Nu = 0.59 Ralm
109 < Ra < 1013 Nu = 0.13 Ravi
2. Horizontal cylinders 104 < Ra < 109 Nu = 0.53 Ralm
109 < Ra < 1013 Nu = 0.13 Ravi
3. Horizontal plates
(a) Hot surface facing up 105 < Ra < 107 Nu = 0.54 Ralm
107 < Ra < 1011 Nu = 0.15 Ravi
(b) Hot surface facing down 105 < Ra < 1011 Nu = 0.27 Ralm
4. Sphere 1 < Ra < 105 Nu = 2 + 0.43 Ralm
105 < Ra < 108 Nu = 2 + 0.5 Ralm

Ts + T„
Note: All fluid properties at film temperature Tf— . Heat transfer is increasing with
2
increase of Rayleigh number. Heat transfer from horizontal plate facing up is more than that
of facing down.

15. A vertical pipe 7.5 cm in diameter carrying hot water is located in atmospheric air
at 10°C while the temperature of the outer surface of the pipe is 70°C. The pipe is
2 m in height. Find the heat loss from the pipe per hour. Properties of air at mean
temperature of 40°C are—
g p p 2c
p
= 4.5 x 1010 and k = 2.75 x 10-2 W/mK
µk
(UPTU — 2003-4)
278 Heat and Mass Transfer

It is given-
g p p2 c
P = 4.5 x 101°
auk
2
or (g1313
2 ) x( i1CP ) = 4.5 x 101°
ti

(g•P•p2xx3xATj (µCp) =
or 1.12 x 4.5 x 101° x x3 x AT

But AT = 70 - 10 = 60 and x = height = 2 m


Gr x Pr = 4.5 x 101° x 23 x 60
= 2.16 x 1013
From the correlation for Rayleigh number = 2.16 x 1013, we have-
Nu = 0.13 Ra t/3
h•x
But Nu =
k
h = — Nu
x
2.75 x 10-2
x 0.13 x (2.16 x 1013)1/3
2
= 49.8 W/m2K
Heat transfer rate is-
Q= h • A • AT
= h(irdL) • AT
= 49.8(ir x 7.5 x 10-2 x 2) • 30
= 1406.8 W
Heat transfer for one hour is-
(Q)t=1 hour = 1406.8 x 60 x 60
= 5.07 x 106 Joules per hour

16. A vertical plate 0.2 m in height and 1 m wide is maintained at a uniform temperature
of 110°C. It is placed in quiet ambient air at 30°C. Find the heat loss from the two
surfaces of the plate. Properties of air of 70°C are: p = 1.029 kg/m3, µ = 20.6 x 10.6
Poise, Pr = 0.694 and k = 0.0296 W/mK.
(UPTU - 2002 - 3)
Given
Pr = 0.704
Free Convection 279

We know
3
X XgXP(Ts—L)
Gr =
v2
Ts + Toa 110 + 30
Tf — -
— 2 2
= 70°C
=1= 1 _ 1
T 273 + 70 343
For vertical plate x = L = 0.2 m

(0.2)3 x 9.81 x ( 343)(110 — 30)


Gr = 2
20.6 x 10-7
( 1.029

= 8 x 10-3 x 9.81 x 2.92


-12
x 10-3 x 80
4.01 x 10
= 4.57 x 10 9
Ra = Gr • Pr = 4.57 x 109 x 0.709
= 3.24 x 109
From the correlation, for Ra = 2.9 x 109, we have—
Nu = 0.13 Ra1/3
= 0.13 x (3.24 x 109)113
= 0.13 x 1.48 x 103
= 0.192 x 103
h •x
Now Nu = where x = 02 m
k
or h = x • Nu = 0'0296 x 0.192 x 103
0 .2
= 28.4 W/m2K
Heat transfer is—
Q = 2[h x A(Ts — T5)]
= 2 x 28.4 x (0.2 x 1) x 80
= 909 W

17. A hot plate 30 cm high and 1.1 m wide at 120°C is exposed to an ambient of still air
of 20°C. Using the approximate solution, calculate the following:
1. Maximum velocity at 15 cm from the leading edge of the plate
2. Boundary layer thickness at 15 cm from the leading edge of the plate
3. Local heat transfer coefficient at 15 cm from the leading edge of the plate
280 Heat and Mass Transfer

4. Average heat transfer coefficient


5. Total mass flow through the boundary
6. Heat loss from the plate
7. Temperature rise of air.
(NMU - 1999)
T +T 120 + 20
Film temperature Tf = s - =
' 2 2
= 70°C
The properties of air at 70°C from data bank.
p = 1.029 kg/m3, v = 20.02 x 10-6 m2/s
Pr = 0.694, h= 0.0296, 13= 1 = 2.9 x 10-3 lc-1
70 + 273
k = 29.6 x 10-3 W/mK
1. Local velocity at x= 0.15 m
1/2
ux = 5.17 x v (pr +
20)-1/2[#xgx(Ts-T1 X X1/2
21 v2
In 1/2 . x10-3x 9.1 x 1/2
= 5.17 x 20.02 x 10-6(0.694 + `") x (0.15)1/2
21 2 9 (20.2 x 1086)2 100
= 2.63 m/s
2. Maximum velocity at x= 0.15 m
u = 4 = 4 x 2.63 = 0.39 m/s
Imre 27 ux 27
3. Boundary layer thickness is-
1/4
S = 3.93 x (Pr)-1/2 (0.952 + Pr)1/4 (Grx) X x
g • P • AT x x3
Grx = .9.81x 2.9 x 10-3 x 100 x (0.15)3
v2 (20.02 x 10-6)2
= 2.41 x 107
S = 3.93 x (0.694)-1/2 (0.952 + 0.694)1/4 x (2.41 x 107)-1/4 x 0.15
= 11.5 x 10-3 m
4. Nusselt number is-
Nux = 0.508 x Pr+1/2 (0.952 + Pr)-1/4 th .1/4

.2 • x _ 2x0.15
S 11.5 x10-3
= 26.3
Free Convection 28 I

h xx
Nu = x
x k
Nu x k
or hx - '
x
26.3 x 29.6 x 10-3
=
0.15
= 5.189 W/m2K
hx(L )-114
Now hx=L = —
x
1/4
= 5.189 x ( (13 )
0.15
= 4.4 W/m2K
5. Average value of heat transfer coefficient is-
h = L1 f 0L hdx
..., = 3h,_L
...- ,

= 4 x 4.4 = 5.8 W/m2K


3
6. Mass flowing through the boundary is-
11/4
M = 1.7 x p x v[ 2 Gri,
(Pr) (0.592 + Pr)]

GrL = Gr.„(')3
x
3 x10-2 )3
= 2.41 x 107 x (
1.5 x 10-2
= 193 x 106
1/4
193x106
m= 1.7 x 1.029 x 20.02 x 10-6 [
(0.694)2 (0.592 + 0.694)
= 4.5 x 10-3 kg/s
7. Heat transfer from both sides of the plate is-
Q = 2 xhxAx(Ts -To,)
= 2 x 5.8 x (0.3 x 1.1) (120 - 20)
= 385 W.
8. Temperature rise of the air is-
Q = m • Cp AT
282 Heat and Mass Transfer

or OT = Q 385
-
m • Cp 4.5 x10-3 x 1005
= 87°C.

18. A 0.003 m long glass plate is hung vertically in the air at 27°C while its temperature
is maintained at 77°C. Calculate the boundary layer thickness at trailing edge of the
plate and average heat transfer coefficient. If the same plate is placed in a wind
tunnel and air is blown over it at a velocity of 4 m/s, estimate the boundary layer
thickness at its trailing edge and the average heat transfer coefficient. Properties of
air at 52°C.
/3 = 3.07 x 10-3 K-1, k = 28.15 x 10-3 W/mK
v = 18.41 x 10-6 m2/s, Pr = 0.7
Use for free convection
(5 = 3.98 • x • (0.952 + Pr)1/4 x 1
Pr1/2 x Grx1/4
Nux = 0.508 Pr 112 (0.952 + Pr)-114 x GrY4
Use for forced convection
Nu = 0.664 Re?2 x Pr 1/3
(UPTU - 2006 - 7)
Film temperature is-
T Ts - F T„ 77 + 27 104
f= 2 - 2 - 2
= 52°C
x3 xgx/3x(Ts -TO
Grx - v2

= (0.03)3 x 9.81 x 3.07 x 10-3 x (77- 27)


(18.41 x 10-6)2

= 27 x 10-6 x 9.81 x 3.07 x 10-3 x 50


338.9 x 10-12
= 1.199 x 105
Now 3 = 3.93 x x x (0.952 + Pr)1/4 x 1
Pr 1/2
U2 Grx1/4

= 3.93 x 0.03 x (0.952 + 0.7)1/4 x 1


(0.7)1/2 x (1.999 x 105)1/4
_ 3.93 x 0.03 x 1.34
0.836 x 18.6
= 1.016 x 10-2 m
Free Convection 283

= 10.16 mm
Nux = 0.508 x Pr1/2 x (0.952 + Pr)-1/4 x Gr1/4
= 0.508 x (0.7)1/2 (0.952 + Pr)-1/4 x (11.99 x 104)1/4
- 0.508 x 0.836 x 18.6 - 5.89
1.34
hx • x
Nux =
k
k
or hx = x Nux

28.15 x 10-3
= x 5.89
0.03
= 5.527 W/m2K
Now hx= L = hx

It = ir dx = lh
L 0 hx 3 x= L
=—4 x 5.527 = 7.37 W/m2K
3
For forced convection, we have-
pVL VL
Re = -
µ v
4 x 0.03
18.41 x 10-6
= 6.52 x 103 < 5 x 105
5• x
8=
.‘i Re
_ 5 x 0.03
V6.52 x 103
5 x 0.03
80.74
= 1.86 x 10-3 m
Nu = 0.664 x Re1/2 x Pr1/3
= 0.664 x )16.52 x103 x (0.7)1/3
= 0.664 x 80.74 x 0.888
Nu = 47.60
Now h= • Nu
x

- 28.15 x 10-3 x 47.6


0.03
284 Heat and Mass Transfer

= 44.66 W/m2K
Note: The heat transfer coefficient is always higher in forced convection as compared to
free convection. In this problem it is almost six times higher.

19. A rectangular plate of size 0.3 m x 0.5 m is maintained at temperature of 100°C is


placed in surrounding air at 30°C. Find heat transfer rate: (i) when the plate has
vertical height as 0.3 m, (ii) when the plate has vertical height as 0.5 m.

T +T 100 + 30
Film temperature Tf = 8 =
2 2
= 65°C
Now = 1 1 — 3 x 10-3 K-1
t- T1+ 273 — 65 + 273
From data bank, properties of air at 65°C
v = 1.8 x 10-5 m2/K, k = 30.1 x 10-3 W/mK
Pr = 0.703
Now Grashof number is—
gx/3xATxx3
Gr =
v2
Case 1: 0.3 m is vertical height
Gr = 9.81 x 3 x 10-3 x (100 x 30) — 0.33
(1.8 x 10-5)2
= 1.7 x 108
Rayleigh No., Ra = Gr x Pr
= 1.7 x 108 x 0.703
= 1.2 x 108 < 109
The flow is laminar and from correlation we get—
Nu = 0.59 (R01/4
= 0.59 x (1.2 x 104)
= 0.59 x 104.7
= 61.75
h
Nu = x

or h = Nu x k
x
61.75 x 30.1 x10-3
0.3
= 6.195 W/m2K
Free Convection 285

Heat transfer is—


Q= 2 x hA(T, — 71,) = 2 x 6.195 x (0.3 x 0.5) (100 — 30)
= 160 W
Case 2: 0.5 m side is vertical height
g• • AT • x3
Gr = 2
V

9.81 x 3 x 10-3 x 70 x (0.5)3


(1.8 x 10-5)2
= 7.8 x 108
Ra = Gr x Pr = 7.8 x 108 x0.703
= 5.5 x 108 < 109
The flow is laminar.
Nu = 0.59 (Ra)1/4
= 0.59 x (5.5 x 108)1/4 = 90.35
k 30.1 x 10-3 x 90.35
h= x Nu =
x 0.5
= 5.44 W/m2K
Q = 2 x hA(T, — To0
= 2 x 5.44 x (0.5 x 0.3) x 70
= 114.24 W
Note: For same area plates, the plate having lower height would have higher heat transfer
rate for all other conditions remaining same. The reason is that local heat transfer coefficient
(hi) decreases as 'x' increases from the leading edge.

20. A hot plate of size 30 cm x 50 cm is kept in air at 30°C. The temperature of the plate
is initially at constant temperature at 200°C. Find: (i) heat transfer coefficient, (ii)
initial rate of cooling of the plate, (iii) time to cool plate to 50°C. Given mass of plate
= 15 kg, Cp = 420 J/kgK and plate has side 30 cm in vertical position.
Film temperature is-
200 + 30
Tf = = 115°C
2
Properties of air at 115°C
Pr = 0.688, k= 0.0321 W/mK, v = 23.18 x 10-6m2/s
Now 1 1 -3 K-1
Q = Tf 115 + 273 2.58 x 10
x = 0.3
286 Heat and Mass Transfer

1. Heat transfer coefficient


g• AT • x3
• 9.81 x 2.58 x 10-3 x 170 x 0.33
Gr =
v2 (23.18 x 10-6)2
= 2.16 x 108
Ra = Gr x Pr
= 2.16 x 108 x 0.688
= 1.48 x 108 < 109
Flow is laminar,
Nu = 0.59 (Ra)1/4 = 0.59 x (1.48 x 108)1/4
= 65.08
65.08 x 3.21 x 10-2
h-
0.3
= 6.96 W/m2 K
2. Initial rate of cooling
Rate of heat convected initially from both sides
= Rate of decrease of internal energy initially
dT
2x
hA(Ts-T-)=mCP dt
dT 2 x hA(Ts -L)
or
dt m• Cp
2 x 96 x (0.3 x 0.5) (200 - 30
15 x 420
= 0.0563°C/s
3. Time to cool to 50°C.
The hot plate is considered a lumped body.
hA hA
T- T - - pVC p xt = mcp xt
e e

Ti -Tc,
—6.96x(2x0.5x0.3) xt
50 - 30 15x420
=e
200 - 30
or t = 6448 s
= 107.5 min

21. A square and horizontal duct of 50 cm x 50 cm carrying cooled air so that its outside
temperature is maintained at 5°C while surrounding air is at 35°C. Find heat gained
per unit length by the duct.
Film temperature is-
5 + 35
T= = 20°C
f 2
Free Convection 287

Air properties at 20°C are-


p = 1.205 kg/m3 C = 1006 J/kgK
v= 1.506 x 10-'5 mfis, k= 2.56 x 10-2 W/mK
Pr = 0.71
For rectangular duct, the value of characteristic length 'x' is-
1= 1 + 1
x L. L,
or x = 4L.+
x L. _ 0.5 x 0.5 - 0.25
4 - 0.5 + 0.5 1
. . AT . x3 9.81x 20 + 273 x (35.5) x 0.253
Gr = 6 t'
V2 - (1.506 X 10-5)2
= 6.9 x 10 7
Ra = Gr x Pr - 6.9 x 107 x 0.71
= 4.9 x 107 < 109
The flow is laminar and using correlation of horizontal cylinder, we get-
Nu = 0.53 (Ra)1/4
= 0.53 x (4.9 x 107)1/4
= 0.53 x 83.66
= 44.34
Nu =
h• x
k
or h = k- x Nu
x
2.56 x 10-2
= x 44.34
0.25
= 4.54 W/m2K
Q =hxAx(Ts -To,)
= 4.54 x [2(0.5 + 0.5) x 1] [35 - 5]
= 4.54 x 2 x 30
= 272.4 W

22. A circular disc heater 0.2 m in diameter is exposed to ambient air at 25°C. One side
is insulated and the other side is maintained at 130°C. Find heat transfer when disc
is placed: (i) horizontally and hot surface facing up, (ii) horizontally with hot surface
facing down, and (iii) vertical.
Film temperature is-
130 + 25
T f = Ts + -
T
- = 77.5°C
2 2
288 Heat and Mass Transfer

The properties of air at 77.5°C are—


k = 0.03 W/mK, Pr = 0.697, v = 2.08 x 10-5m2/s
R= 1 = 1 1
350.5 — 2.85 x 10-3 K-2
Tf 77.5+ 273
Case 1: Horizontal and facing up
g• AT • x3

Gr =
v2
2
A 7r/4d d — 0 2 = 0.05 m
x— =
P rd 4 4
Gr = 9.81 x 2.85 x 10-3 x (130 — 25) x (0.05)3
(2.08 x 10-5)2
= 7.85 x 107
Ra = Gr • Pr
= 7.85 x 105 x 0.697
= 5.47 x 105 < 109
Flow is laminar and hot surface facing up, therefore—
. Nu = 0.54 (R01/4
= 0.54 (54.7 x 104)1/4
= 0.54 x 2.72 x 10
= 14.68
h• x
Nu —

or h= —k x Nu (here x = )
4
=0.03 x 14.68
0.05
= 8.81 W/m2 K
Q = hA.(Ts — Toc )
= 8.81 x x d2 (130 — 25)
4
8.81x x 0.202 x 105
4
= 29 W
Case 2: Horizontal facing down
Nu = 0.27 (Ra)1/4
= 0.27 x (5.47 x 105)1/4
= 7.5
Free Convection 289

h= x Nu = 0.03 7.5
x (here x =
0.05 4
= 4.5 W/m2K
Q = hA(T5 — 71.)
= 4.5 x x 0.22 ) (130 — 25)
4
= 14.8 W
Case 3: Disc is kept vertical.
Nu = 0.59 x (Re)1/4
= 0.059 x (5.47 x 105)1/4
= 46.3
Now h = —k x Nu (here x = d)
0.03 x 46.3
0.2
= 6.94 W/m2K
Q = hA(T, — Too)

= 6.94 x (ir x 0.22 (130 — 25)


4
= 23 W
Note: Heat transfer by free convection is maximum when the disc is kept vertical. In
horizontal position, heat transfer is more when it faces up as compared to that when it faces
down.

23. What is the characteristic length for: (i) horizontal plate, (ii) inclined plate,
(iii) hollow cylinder in vertical position and horizontal position, (iv) hollow sphere, (v)
two parallel plates in vertical and horizontal?
1. Horizontal plate: The characteristic length is 'I,' as shown below.

/ /-
2. Inclined plate: Characteristic length is 'I,' and correlation to find Nusselt number is
same as that of vertical plate but with only difference is that `g cos 0' to replace `g'
in Gr number.
290 Heat and Mass Transfer

3. Hollow cylinder:
(a) Hollow vertical cylinder: characteristic length is length 'I,' as shown below.


(b) Hollow horizontal cylinder: Characteristic length is
2

— d.
4. Hollow sphere: The characteristic lengthd0isI as shown below.
2

d,
5. Parallel Plates:
(a) Horizontal plates: The separation 's' is characteristic length.

(b) Vertical plates: The characteristic length is 's' as shown below.


Free Convection 29 I

24. Estimate the heat transfer rate from a 100 W incandescent bulb at 140°C to ambient
air at 24°C. Approximate the bulb as 6.0 cm diameter sphere. Calculate the
percentage of power lost by natural convection.
Correlation is given Nu = 0.60 (Gr Pr)1/4
Properties of air at 82°C are-
v = 21.46 x 10-6m2/s, k1 = 30.38 x 10-3 W/mK
Pr = 0.699
(MU - 2002)
Geometric shape is approximated as shown in figure.
T + Tso 104 + 24
T= = 82°C
f 2 2
= 1 = 1 1 = 2.8 x 10-3 K-1
= 355
Tf. 82 + 273

d= 6.0 cm
Approximation

Bulb Sphere

Characteristic length for the sphere is-


= d = 0.06 m
T„ )d 3
Gr =
v2
9.81 x 2.8 x 10-3 (140 - 24) x 0.063
(21.46 x 10-6)2
9.81 x 2.8 x 116 x 0.216 x 10-3 x 10-3
460 x 10-12
= 1.503 x 106
Ra = Pr • Gr
= 0.699 x 1.503 x 106
= 1.05 x 106
Nu = 2 + 0.5 (Ra)1/4
= 2 + 0.5 x (105 x 104)1/4
= 2 + 0.5 x 3.2 x 10
= 2 + 16 = 18
h=— k x Nu
292 Heat and Mass Transfer

= 30.38 x 18 x 10-3
0.06
= 9.114 W/m2K
Q conv = hA(Ts — T
= 9.114 (4 x jr x 0.032) (140 — 24)
= 11.95 W
Percentage power lost by natural convection—
Qconv x 100 — 11.95
lid x 100
Q
= 11.95%

25. Find the heat lost by a human body by convection to surrounding air. The body can
be approximated to a vertical cylinder of 25 cm in diameter and 175 cm in height. The
air is at 13°C and body is at 37°C.
Geometric shape is

Approximation E

1 >

T+T 37+13
T= s — = 25°C
2 2
The properties of air from data book at 25°C
v = 15.71 x 10-6 m2/s, k = 2.614 x 10-2 W/mK
Pr = 0.707
Now
= 1 = 1 1
= 3.35 x 10-3 K-1
Tf. 25 +373 298
= h = 1.7 5 m
g 13 • AT x 1.75 9.81 x 3.35 x 10-3 x (37 —13) x 1.75)
Gr = 2
V (15.71 x 10-6)2
= 1.72 x 1010
The Rayleigh number is—
Ra = Gr • Pr
= 1.72 x 1010 x 0.707
= 1.2 x 1010 > 109
Free Convection 293

Flow is turbulent
Using correlation for above Ra, we get-
Nu = 0.13 (Ra)1/3
= 0.13 x (1.2 x 1010)
= 0.13 x 2.29 x 103
= 298
Now h=— k x Nu and x = 1.75
x
2.614 x 10-2
x 298
1.75
= 4.45 W/m2K
Now Q = hA(T, - Toj
= 4.45 x (ivDL) (37 - 13)
= 4.45 x (iv x 0.25 x 1.75) (24)
= 146.7 W

26. Two concentric spheres of diameter 20 cm and 30 cm separated by air. The surface
temperature of two spheres are 47°C and 7°C respectively. Find heat transfer to
outer sphere.
Using relation Nu = 0.228 Ra0.226. The properties of air at 27°C are: v = 1.57 x 10-5 m2/s,
Pr = 0.712 and k = 2.6 x 10-2 W/mK
Film temperature is-
To -F T 47+7
Tf = = - 27°C
2 2
f4 = 1 = 1 =
1
T1 27 + 273 300
= 3.3 x 10-3 K-1
d0 30 -
x = characteristic length = -
2 = 2 20 = 5 cm
g • (3 • ATx3
Gr =
V2

9.81 x 3.3 x 10-3 x 40 x (0.05)3


(1.57 x 10-5)2
9.81 x 3.3 x 40 x 125 x10-4
2.46 x 10-10
= 65.8 x 10+4 = 6.58 x 105
Ra = Gr x Pr = 6.58 x 105 x0.712
= 4.7 x 105
294 Heat and Mass Transfer

From the relation given-


Nu = 0.228 x (Ra)o.226
= 0.228 x (4.7 x 105)0.226
= 4.37
Now h = —k x Nu
x
- 2.6 x 10-2 x 4.37
= 2.27 W/m2K
0.05
Q = hAAT
= 2.27 x (47 r r?) x 40
= 2.27 x 4ir x (0.1)2 x 40
= 11.4 W.

27. Determine the coefficient of heat transfer by free convection and maximum current
density for a nichron wire 0.5 mm in diameter. The surface of the wire is maintained
at 300°C. The wire is exposed to still air at 20°C and resistance per metre length of
the wire is 6 ohm/m. Use relation Nu = 1.18 (Gr Pr)1"8. Properties of air at 160°C are-
k = 3.61 x 10-2 W/mK, Pr = 0.687
v = 30.35 x 10-6 m2/s
(Annamalai University 2003)
Film temperature is-
Ts -F 1._ 300+20
Tf -- - =160°C
2 2
Now fi = 1 = Q • 31 x 10-3 k-1
160 + 273
Characteristic length x = 5 x 10-4 m
g•i3x ATd3 = 9.81x 2.31x10-3 x 280 x(5 x10-4)2
Gr =
v2 (30.35 x 10-6)2
= 0.86
Now Ra = Gr • Pr
= 0.86 x 0.687
= 0.591
Given relation is-
Nu = 1.18 (Re
= 1.18 x (0.591)1/8
= 1.105
Now h = —k x Nu
x

- 3.61 x 10:2 x 1.105


5x
Free Convection 295

= 79.77 W/m2K
Q = hA(T, — Lc)
= 79.77 x (iv x 5 x x 1) x (300 — 20)
= 35 W/m
Heat generated by wire—
Q = I2R but R = 6 ohm/m
or /2 = Q = 35 = 5.833
R 6
or I = V5.833
= 2.42 ampere

28. The water in a tank at 30°C is heated by passing the steam through a pipe of 50 cm
long and 5 cm in diameter. If the pipe surface temperature is maintained at 80°C,
find the heat loss from the pipe per hour. If the pipe is kept vertical, what will be
the heat loss per hour from the pipe.
Film temperature is—
T+T 80 + 20
T= =
2 2
= 50°C
Properties of water at 50°C are—
p = 988.1 kg/m3, v = 0.556 x 10-6 m2/s
= 0.55 x 10-3 kg/ms, C, = 4200 J/kgK
k = 0.64 W/mK, 13 = 5.1 x 10-4 K-1
0.55 x 10-3 x 4200
Pr= 11CP =
k 0.64
= 3.60
Case 1: Pipe is horizontal. Hence, x = d = 0.05
g•PATxx 3 = 5.1 x104 x 9.81x 60 x (0.05)3
Gr =
v2 (0.556 x 10-6)2
= 1.21 x 108
Ra = Gr Pr
= 3.60 x 1.21 x 1011
= 4.26 x 1011
From the correlation, we get—
Nu = 0.53 (Ra)1/4
= 0.5 x (4.26 x 1011)1/4
= 75.7
296 Heat and Mass Transfer

h= —k x Nu - 0.64 i x75.7
x 5 x 10-2
= 968 W/m2K
Q = h(ivdL) (T, - TO
= 968 Or x 5 x 10-2 x 0.5)(80 - 20)
= 4.55 kW
Case 2: Pipe is vertical
Now characteristic length x = L = 0.5 instead of d = 0.05
Hence, Ra = 4.26 x 1014
Nu = 0.13 (4.26 x 1014)1/3
= 978.2
h = —k x Nu = 0'64 x978.2
x 0.5
= 1250 W/m2K
Q = h(ivdL) (T, - TO
= 1250 (iv x 0.05 x 0.5) (60)
= 5.89 kW
Note: The convective motion is more when tube is vertical resulting in higher heat
transfer.

29. Match List-I and List-II and select the correct answer using the codes given below
the lists.
List-I List-II
(Flow Pattern) (Situation)

1. Heated horizontal plate

2. Cooled horizontal plate


Free Convection 297

(C) 3. Heated vertical plate


///////////////////

(D) 4. Cooled vertical plate


///////////////////

Codes: A B C D
(a) 4 3 2 1
(b) 3 4 1 2
(c) 3 4 2 1
(d) 4 3 1 2
(IES — 1995)
In flow pattern (A), air is moving up which is possible when air gets heat from plate and
its density reduces to facilitate upward motion. The plate has to be hot.
In flow pattern (B), air has to be cooled down and gain density to move down. Plate has
to be cold.
In flow pattern (C), air has to be heated up so as density reduces to facilitate upwards
motion. Hence, plate has to be hot.
In flow pattern (D), air has to be cooled down so as move down to plate surface due to
high density. Hence, plate has to be cold.
Option (C) is correct.

30. Assertion (A): A slab of finite thickness heated on one side and held horizontal will
lose more heat per unit time to cooler air if the hot surface faces upwards when
compared with the case where hot surface faces downwards.
Reason (R): When the hot surface faces upwards, convection takes place easily
whereas when hot surface faces downwards, heat transfer is mainly by conduction
through air.
Mark the answer as:
(a) If both (A) and (R) are true and (R) is the correct explanation of (A)
(b) If both (A) and (R) are true but (R) is not the correct explanation of (A)
(c) If (A) is true but (R) is false
(d) If (A) is false but (R) is true.
(IES — 1996)
The convective motion for plate facing up and down as shown below:
298 Heat and Mass Transfer

Plate facing up Plate facing down

The convective motion is possible when hot plate is facing up as air heated up with reduced
density can move up facilitating higher rate of heat transfer. The upward motion of heated
air is not permitted when hot plate is facing down and heat transfer is mainly by conduction.
Both assertion (A) and reasoning are correct and reasoning is the correct explanation of
assertion
Option (a) is correct.

31. Two vertical plates each 120 mm high and maintained at 85°C are placed in water at
15°C. Find minimum spacing which will prevent interference of free vertical
o.i
8 th /15 [ 1 + 0.494(Pr) 2/3
boundary layer. Use relation = 0.565 (Pr)-8 for turbulent
Gr
flow.
Guidance: The problem is to find the thickness 8th of boundary layer at trailing edge of
the vertical plate. The spacing between two plates has to be double of this thickness.

H 6th —b-h— 3th -H

85 +15
Tf = — 50
2
For water at 85°, the properties are—
k = 0.674 W/m°C, v = 0.566 x 10-4 m2/s
Pr = 3.54
1 1
Now Q = T f. + 273-323 and x = 120 x 10-3 K-1

g [3 AT x 3 9.81x x 70 x 120 x 10-3


Gr = 323
2
V (0.566 x 10-4)2
= 11.9 x 109
Free Convection 299

Ra = Gr x Pr = 11.9 x 109 x 3.54 = 42.1 x 109 > 109


Flow is turbulent and as per correlation given-
0.1
St, [1 + 0.494 x Pr,,,
= 0.565 x (Pr)-8/15
Gr
,,, 0.1
1 + 0.494 x (3.54)—
= 0.565 x (3.54)-8/15 [
42.1 x 109
S
t' = 3.06 x 10-2
or 8th = 120 x 10-3 x 3.06 x 10-2
= 3.67 x 10-3 m
Now spacing S = 28th
= 2 x 3.67 x 10-3
= 7.34 x 10-3 m
= 7.34 mm

32. A vertical 0.8 m high and 2 m wide double pane window consists of two sheets of glass
separated by 2 cm air gap at atmospheric pressure. If temperature of outside and
inside glasses of air gap are at 12°C and 2°C, find heat transfer through the double

pane. Use relation Nu = 0.197 Ralm (f! 9 where H = height of plate.


c

Guidance: The problem is to be solved considering two vertical plates separated with a
spacing 'x' which is the characteristic length to be used for calculating Grashof number
Ti= 2°C

To = 12.5

To + Ti 12 +
Film temperature T = = 7°C
f 2 22
Properties of air at 7°C
Pr = 0.717, k = 2.46 x 10-2 W/mK
v = 1.4 x m2/s
R 1 1 _ 1 K-1
Now
Tf 7 + 273 - 280
300 Heat and Mass Transfer

g• • AT • x3
Gr = 2
V

9.81 x 280 x 10 x (0.02)3


(1.4 x 10-5)2
= 14.3 x 103
Now Ra = Gr x Pr = 14.3 x 103 x 0.717
= 10.25 x 103

Nu = 0.197 (Ra)1/4 x ( H ) 119


X

= 0.197 x (10.25 x 103)1/4 x


(0 202) 9
= 1.3
The effective thermal conductivity of double pane is
keff = Nu x k = 1.3 x 2.46 x 10-2
= 3.2 x 10-2 W/mK
keff A(To - Ti) 3.2 x 10-2 x (0.8 x 2) x 10
Q=
x 0.02
= 25.6 W
k x Nu, keff = k x Nu.
Note: While h = —
Chapter 8
RADIATION HEAT TRANSFER

A PHOTON A SPECULAR AND DIFFUSE REFLECTION


A MAXWELL'S ELECTROMAGNETIC THEORY A IRRADIATION
A QUANTUM THEORY A RADIOSITY
A STEFAN-BOLTZMANN'S LAW A MAX. PLANCK'S LAW
A PROVOST THEORY OF EXCHANGE A SOLID ANGLE
A SPECTRAL EMISSIVE POWER A STERADIAN
A BLACK BODY A INTENSITY OF RADIATION
A ABSORPTIVITY A SHAPE FACTOR
A REFLECTIVITY A RECIPROCITY THEORY
A TRANSMISSIVITY A SURFACE RESISTANCE
A DIFFUSE SURFACE A SPACE RESISTANCE
A GREY SURFACE A GREY BODY FACTOR
A OPAQUE SURFACE A GREENHOUSE EFFECT
A KIRCHHOFF'S LAW A SOLAR RADIATION AND CONSTANT
A WIEN'S DISPLACEMENT LAW A SKY TEMPERATURE

INTRODUCTION

The radiation is the transmission of heat energy between two bodies having no physical contact
between them. The transmission of heat energy as radiation does not affect or heat up the
medium between the heat source and the receiver. The transmission of heat energy by radiation
does not infact require any intervening medium and this transmission can take place even when
there is vacuum between the bodies. Infact transmission of heat energy by radiation occurs most
effectively in vacuum and it is reduced or eliminated in the presence of different medium
materials. Energy transmitted by radiation is not continuous but it is in the form of discrete
packets of energy called photons which are successively released by hot bodies. The photons
propagate through space as electromagnetic waves. Whenever these photons in the waves strike
302 Heat and Mass Transfer

a receiver body, then the conversion of wave motion into heat energy takes place. The heat
energy on conversion is partly absorbed, reflected or transmitted on the surface of the receiver
body.

1. What is radiation?

Radiation is the transmission of heat by a body due to its temperature. Every hot body of
a temperature above absolute zero emits thermal radiation. Radiation unlike conduction and
convection does not require any medium. In fact, radiation is most effective in vacuum.

2. What are the basic theories to explain the mechanism of radiation heat transfer?

There are two basic theories to explain the mechanism of radiative heat transfer which are—
(a) Maxwell's electromagnetic theory. According to this theory, heat transfer in the form
of electromagnetic waves takes place from a body when its temperature is above
absolute zero.
(b) Max Planck's concept or quantum theory. According to this theory, heat transfer in the
form of photon or quanta of energy takes place from a body when its temperature is
above absolute zero. The photon has energy = hv, where h = Planck's constant and
v = frequency of photon.

3. Explain types of radiation on the basis of wavelengths.

The radiative heat transfer takes place in the form of electromagnetic waves emission by
a body above absolute temperature. The wavelength of the electromagnetic emission may
vary from zero to infinity. However, the bulk of thermal emission has wavelength varying
from zero to 100 1.tm (thermal emission from sun has wavelength from 0.1 lam to 4 gm).
Types of radiation on the basis of wavelength is as tabulated below.
S.No. Type of Radiation Wavelength
1. Gamma Ray 4x 10-7 to 1.4x 10-4 µm
2. x-rays 1 x 10-5 to 2 x 10-2 pm
3. Ultraviolet rays 0.02 to 0.4
4. Visible radiation 0.38 to 0.76 1.1,m
5. Thermal radiation 0.1 to 100 1.1,m
6. Solar radiation 0.1 to 4µm
7. Radio waves 92 to 2 x 101° 1.1,m

4. What is the relation of speed, wavelength and frequency of electromagnetic


radiation?

The relation between speed of light (C = 3 x 108 m/s), frequency (f) and wavelength is
C = f•A
Radiation Heat Transfer 303

5. What are the parameters deciding the type of thermal emission?

The thermal radiation can have wavelength varying from 0.1 to 100 pm and wavelength of
emission depends upon nature, temperature and state of the emitting surface. Emission of
radiation in gases depends upon the thickness of gas layer and the pressure of the gas.

6. What is quantum theory?

According of quantum theory, the thermal radiation propagates in the form of discrete
quanta or photon. Each photon has energy which is—
E = hv
where h = Planck's constant
= 6.625 x 10-34 Js
v = frequency of photon
Each photon is a particle having energy, mass and momentum similar to the molecule of
a gas. Hence, radiation can be considered a photon gas moving from one location to another.
The mass and momentum of a photon are—
E = mq? = hv
hv
mass m = CZ
o

and momentum = m • Co = hv2 x Co


Co

hv
Co

7. What is Stefan-Boltzmann equation?

As per Stefan-Boltzmann equation, the flux of heat energy emitted by a black body is
proportional to the fourth power of its absolute temperature
qb = 0- 74
where qb = quantity of energy emitted per unit area and per unit time
a = Stefan-Boltzman constant
= 5.669 x 10-8 W/m2 K4
T = absolute temperature

8. What is Provost theory of exchange?

According to Provost theory of exchange, all bodies radiate thermal radiation at all
temperatures. The amount of thermal radiation radiated per unit time depends on the nature,
area and temperature of the emitting body. The rate of heat transfer is faster at higher
temperature. Besides emission, a body also absorbs a part of thermal radiation which is being
emitted by other bodies and which falls on it. Three conditions may prevail—
304 Heat and Mass Transfer

(a) If a body radiates more than what it absorbs, its temperature falls.
(b) If a body radiates less than what it absorbs, its temperature rises.
(c) If a body radiates and absorbs same amount of thermal radiation, its temperature
remains constant.

9. What is emissive power (E)?

The emissive power of any body is defined as the energy emitted by the body per unit time
and per unit area. It is expressed as W/m2. The emissive power of a black body is
proportional to the fourth power of its absolute temperature as per Stefan-Boltzmann
equation.

10. What are total emissive power (E) and monochromatic (spectral) emissive power
(Ea)? How are they related to each other?
(UPTU — 2006 — 7)

The amount of radiation emitted per unit wavelength varies at different wavelengths. Hence,
monochromatic emissive power is defined as the rate of energy radiated per unit area of
body and per unit wavelength. Total emissive power is—

E = f EbAAA,
o
where A = wavelength of emission
Eb = monochromatic emission power of black body at wavelength A.

11. What do you understand from a black body?

A black body is an ideal hypothetical body which can—


(a) absorb all incident radiation from all directions at all wavelengths. Hence, there is no
reflection, transmission and scattering of incident radiation
(b) emit maximum amount of radiation as compared to a real body at any temperature.

12. Why term black is used for black body?


(UPTU — 2003)
Or
What do you understand by coloured surface?
(UPTU — 2005 — 6)

As black body absorbs all incident radiation including visible light spectrum therefore, the
black body appears black to human eyes.

13. Is ice a black body?

Most of black coloured surfaces have high value of absorptivity and they are classified as
black body. However, ice appears white but ice also absorbs all incident radiation. Hence,
ice is also a black body.
Radiation Heat Transfer 305

14. How can an artificial black body be created?

An artificial black body can be created by making radiation to enter through a small aperture
in a hollow sphere. The radiator undergoes many reflections within the sphere till complete
radiation is absorbed inside. Here aperture can be considered as black body.

Small
aperture

Black body

15. Explain absorptivity, reflectivity and transmissivity.


Or
Explain why radiation is usually treated as a surface phenomenon.
(UPTU — 2006-7)

When a radiation is incident on a body, three things happen which are: (i) a part is reflected
back, (ii) a part is transmitted through, and (iii) the remainder is absorbed. In other words,
we have

Incident
Reflected Q

Absorbed Q,

Transmitted Q.,

Incident radiation = Absorption + Reflection + Transmission


Q = Qa+ Qp Q,
Absorptivity (a) is the fraction of incident radiation which is absorbed by the body
Qa
a =
Q
Reflectivity (p) is the fraction of incident radiation which is reflected by the body
Qp
P=
Q
Transmissivity ('r) is the fraction of incident radiation which is transmitted by the body
306 Heat and Mass Transfer

16. Prove p + + a = 1.

Now incident radiation is—


Q=Qp+ar+Qa
Now dividing by Q, we get—
QP ar + Q. = Q
Q Q Q Q
p++a=1

17. What do you understand from: (i) white body, (ii) opaque body, (iii) diffuse grey body,
and (iv) transparent body?
(UPTU — 2002-3, 2003, 2007-8)

1. A body is called a white body if it reflects all incident radiations. Hence, for a white
body, we have—
p = 1 and a= =0
2. A body is called an opaque body if it does not transmit any part of incident radiation.
Hence, for an opaque body, we have—
r = 0 and p+a=1
3. Diffuse surface: For real surfaces, the spectral emissivity EA is a complex function of
wavelength 'A,' and emissivity calculation for all wavelengths become very difficult.
Hence, grey and diffuse approximations are commonly used in finding emissivity. A
surface is called to be diffuse surface, if its properties are independent of direction of
radiation.
4. A body is called grey body if its monochromatic emissivity is independent of wave-
length. The emissivity of the grey body does not change with temperature. Thus, a grey
body has a characteristic emissivity value which does not vary with temperature and
the value of emissivity of a grey surface is less than one.
5. A body is called transparent if radiation passes through it fully i.e. 1" = 1 and a = p = 0.

18. What do you understand from emissivity and emission from real body?

Emissivity (E) is the ratio of the emissive power of any real body to the emissive power
of a black body at the same temperature. It is the ability of a real body to radiate heat in
comparison to a black body at the same temperature
Emissive power of the real body
Emissivity = E =
Emissive power of black body
E
Eb
Radiation Heat Transfer 307

The emission power of a real body is given by using emissivity of the body and emission
power of a black body as per Stefan-Boltzmann equation
E = E [6A T4]

.1\ Black body

Grey body

Real body

Wavelength (X) -.-


Spectral emissivity versus wavelength

Note: Spectral emissivity is for a particular wavelength i.e. E A = while emissivity


(total hemispherical emissivity) is average emissivity in all directions and wavelength i.e.
= — E . The average emissivity of a black body, grey body and real body is as shown
Eb
below.
19. What is Kirchhoff' s law?
(UPTU — 2003, 2005 — 6)
According to Kirchhoffs law, the ratio of total emissive power to absorptivity is constant
for all bodies which are in thermal equilibrium with the surroundings. Therefore, it means
that emissivity of a body is equal to its absorptivity. When body attains thermal equilibrium
with its surroundings, it means—
E1 E2 = E3 Eb
al a2 a3 ab
But a black body has absorptivity ab = 1, hence we
_ E2 E3 Eb
al -a2 -a3 1
El E2 E3
or = = a2 and = a3
Eb Eb Eb
But as per definition:
E2 E3
= E3
Eb - E 19 Eb = E 29 Eb
Hence, al = E 1, a2 = E 2 and a3 = E 3
Kirchhoffs law can be also stated that the emissivity (E) and absorptivity (a) of a real
surface are equal for radiation with identical temperature and wavelength.
308 Heat and Mass Transfer

20. What is Wien's displacement law?


(UPTU — 2003, 2003 — 4)

According to Wien's displacement law, the product of maximum wavelength of a radiation


from the body and the absolute temperature of the body is constant.
Amax x T = Constant
The above is the relationship between the temperature and the wavelength at which the
maximum value of monochromatic emissive power occurs.

21. Explain the types of reflection of radiation.

Reflection can be: (1) specular reflection or (2) diffuse reflection. In specular reflection, the
incident radiation and reflected radiation make equal angle with the normal to the surface
at the point of incident. Polished and smooth surfaces give specular reflection.

Incident Normal Reflected


radiation radiation

/////////// ////////////
Surface
Specular reflection

In diffuse reflection, the incident radiation is reflected in all directions. The rough surfaces
give diffuse reflection.
Incident
radiation

Reflected radiation

//////////// ////_,/////////////
Surface
Diffuse reflection

22. What is irradiation (G)?

It is total amount of radiant energy which is incident on a surface per unit area and per unit
time.
Radiation Heat Transfer 309

23. What is radiosity (I)?


(UPTU — 2005 — 6)

It is the total amount of radiant energy leaving a surface per unit area and per unit time.
Radiosity (J) comprises: (1) emission (E) from the surface, and (2) reflected part of
irradiation (p x G) as shown below.

pxG

Emission and reflection

Radiosity J = E p x G where E = emission, p = reflectivity, G = irradiation.


For black body, p = 0 and J = E
For non-black body J = E Eb p x G.

24. Explain Max-Planck's law for spectral emissive power (EbA).

The energy emitted by a black body varies with wavelength and temperature. For a
particular wavelength, a body tends to radiate more heat at a particular temperature.
According to Max-Planck's law, the spectral emissive power (EbA) at any temperature and
wavelength is—

EbA = 1AT
— 1)
(ec2
where Cl and C2 are constants, A = wavelength in p,m and T is temperature in absolute.
The variation of spectral emissive power with wavelength and temperature is as shown
below—

gt 17
1.-----
1 Amaxx T = constant
'N Ti> T2 > T3 > T4
E to
T2 a
15
ai
o.
Band emission
r
=i EbAdA.
0

0 Ai A2
Wavelength (A.)
Spectral black body emissive power

Following are the observations from the graph—


(a) Emissive power at any temperature increases with increase of wavelength upto some
value and after this it decreases.
3I0 Heat and Mass Transfer

(b) At any wavelength, emissive power increases as temperature is increased.


(c) At elevated temperatures, maximum emission takes place at shorter wavelengths.
Hence, most of emission is taking place at elevated temperatures in a narrow band on
both sides of wavelengths at which emission is maximum.
(d) Connecting 'A ' points on graph for various temperatures gives Wien displacement
law i.e. A x T = constant
Ai E
(e) Total radiation EB is r —Eb,d,. while band emission is Ebo, Ao = f bAdA.
o o

25. Prove Stefan-Boltzmann law using Max-Planck's equation.

The emissive power of a black body is as per Max-Planck's equation is


Eb = f EbA dA
o

=f
- cl A,-5
10 ec2 /AT _ 1 clA

Now substitute A-1 =x


or -A-2 dA = dx on differentiation
or dA = - —1 dx
x2
r 5
Ci X ( 1 )
Eb = dx
c` ec2xiT -1 x2
[ c2 x x
-1
3
= Ci J«X X e T —1] dx
o
—c2 x —2(c2 x) —3(c2 x
c` 3
=C1 f X X [e T +e T +e T + ...1dx
o
—c2 xnxx
3
= c1 f X X [e T jdx where n = 1, 2, 3
o
Now applying integral formula
in 1
x e dx
m+i•
fo m —ax — a

3! 6 c1
Eb = Ci = 4 4
n)3+1 c2 x n
(C2 X —
T T4
6 c1 T4
= 4 4
C2 X n

= 6 1 4 [1±1
4
C2 14 2 4
Radiation Heat Transfer 3II

6c1. T4 (7r 4
4 90

6ci x7r4
— x T4
c2 x 90
= a x T4
iir 4
where a= = 5.67 x 10-8 W/m2K
c2
6cx 90

26. Prove Wien's displacement law using Max-Planck's equation.


According to Max-Planck's equation—

A5
Ebb =
(ec2 — 1) — ec2''' —1

C2
Put =x
AT
c2
or
xxT
x x5 x T5
EbA =
cl (ex —1)

On differentiating and putting a (E1,) = 0, we can get maximum spectral emissive


ax
power for a particular wavelength.
a a r clx x5x T5 ) o
ax (Eb), = 0, ax el, (ex —1)
ci x T5 5x4(ex _ 1) _ x5(ex)
or 5 — 0
C2 (e x —1)2

Ci X T5 X4
or x , [5ex — 5 — x x eX] =0
,5
E.,2 (ex 1)2
4
or Ci X X(5 — x) — 5] = 0
c3 (e x —1) [e
4
or (ex (5 5)
(ex —1)
3I2 Heat and Mass Transfer

The value of x is found out by trial error as—


x = 4.96
C2
or — 4.96

or Lax 6 = 0.0029
— 4.9
or Lax = constant

27. What do you understand from solid angle?

A solid angle is defined as a portion of the space inside a sphere enclosed by a conical surface
when the vertex of the cone is located at the same location as the centre of sphere. Consider
an area dAs on the surface of sphere (radius r) whose centre is at 'c'. The area dAs subtends
an angle 'of at c as shown in the figure which is called solid angle.
dAs
w= 2
r

dAs
r

Solid angle

Consider a sphere is located at 'r' which has radius as 'r' and centre at 'c'. Then solid angle
is—
dils 4 nr 2
CO = = = 4n.
r2 r2
In case we consider hemisphere with radius 'r' and centre at 'c', solid angle is
dAs 2 nr 2
CO = = = 27C
r2 r2
The unit of solid angle is steradian (Sr).
Radiation Heat Transfer 3I3

28. Explain intensity of radiation (I).


(UPTU — 2003, 2007 — 8)

Intensity of radiation is the rate of heat radiation in a given direction from a surface per unit
solid angle and per unit area of the projection of the surface on a plane normal to the direction
of radiation.

dA2 = area on sphere surface


= r 2 sin 0. de • drp

dA1

de
Intensity of radiation

Consider radiation from an elementary area dA, located at centre of sphere having radius
`r' as shown in the figure. The radiation is absorbed by other elementary area dA2 located
on the hemispherical surface.
Now we have—
(a) Projected area of dA1 on a plane perpendicular to the line joining dA1 and dA2 which
is equal to dA1 cos 0
(b) Solid angle subtended by dA2 is—
dA2
dA2 = r 2

As per definition, we have—

Intensity of radiation = d Q12


dA2
(dA1 cos 19) X

where dQ12 = rate of radiant heat transfer from dA1 to dA2.


However, dA2 = rde(rs: 0)d0 = r2 sin 0. de. deP
I— dQ sin 0 • dO. d0
x
dAi x cos° r2
dQ12 r2
dA1 sin 0 • cos 0 • dO d0
or dQl2 =Ix
= dA1 xsin0xcos0x dex dO
3I4 Heat and Mass Transfer

29. Find total radiation or emissive power from a black surface in terms of intensity of
radiation.

The radiation from a surface will take place through a hemisphere which has origin at the
surface.

dQ = I x dAi sin 0 cos 0 dO x dtp


For the hemisphere, we have
0=z12 0=27r
f dQ = IdA1 sin 0 • cos 0 • dO • thip
e=o (p=0
0=z12
or Q = IdAi 2xxxsinecose• de
0=0
0=z12

= X dAi X Ir f sin2 0 • dO
e=o

= x I x dAi [cos2 r/2


2 0
=rx/xdAi
or Q = x I
dA1
or E= x I
The total emissive power of a black surface is equal to times its intensity of radiation.

30. How is emissive power defined?

Emissive power denotes the energy radiated per unit area and per unit time and per unit solid
angle along the normal to the area.

Normal
Radiation Heat Transfer 3I5

dQ
E—
dAxdcoxdt
where E = emissive power
dQ = heat energy radiated
d w = solid angle
dt = time
dA = area.

31. What is shape factor?


(UPTU — 2003, 2007 — 8)

Shape factor is defined as the fraction of radiation energy leaving one surface that falls on
the other surface.
A fraction of radiation energy leaving one surface that reaches the other surface because
of the following reasons—
(a) orientation of emitting and absorbing surfaces
(b) characteristic of emitting and absorbing surfaces.
(c) medium in between emitting and absorbing surfaces

n2

ni

dA2

dA1
Radiant heat exchange

The surface A l will emit radiation


Qi = 6A1 T4
Out of this Qi radiation, a part of radiation (Q12) will be falling on surface A2 depending upon
orientation. The fraction reaching surface A2 depends upon shape factor (F12) as given
below
Q12 = Q1 X F12 where F12 = shape factor
2
and F12 = 1 f f cos 01 X cos 02 X dAl x 2dA
A2 A2
irr
3 I 6 Heat and Mass Transfer

32. What is reciprocity theorem?

According to reciprocity theorem, net radiant heat exchange between two surfaces may be
evaluated by computing shape factor from either of the two surfaces. In other words, the
shape factor (F12) from surface A l to surface A2 has definite relation with shape factor (F21)
from surface A2 to surface A1 which is given by—
A1F12 = A2F21

33. Evaluate shape factor and prove reciprocity theorem.

Consider two black bodies A1 and A2. Heat transfer is taking place between these two bodies
in non-participating medium. Consider two elemental areas dA1 and dA2. Let `r' be the
distance between the two areas and line joining the elemental areas make angle 01 and 02
with respective normals to the areas as shown in the figure.
The projected area of dA1 in the direction of radiation is dA1 cos 01. The radiation energy
leaving from area dA1 and falling on area dA2 is—
dQ1_2 = Intensity x projected area x solid angle.
dA2 cos 02 )
dQ1 _2 = Il x (dA1 cos 01) x
r2

nl

dA1

Radiant heat exchange

However, intensity = Ebl = a 7i4


Il

a 77 4
(cIA2 cos 02 )
dQ1 _2 '1 x (dA1 cos 191)
7r r2
Now on integration, total radiation from body A l is—
a it f f cos° i x cos 02 x dA x dA2
Q12 = JA2 2
r
= F12 x (6A1T14)
1 f r cos OiX cos 02
where F12 = A
— .1Aj2 X dA1 X dA2
Al AI r2
= Shape factor (i)
Radiation Heat Transfer 3I7

Similarly, radiation leaving A2 and absorbed by Al can be evaluated as under-


6T2 f f COS 91 X COS 92
Q21 = 2 xdA1 xdA2
JA,JA2 r
= F2_1 (6A2 T24)
I r cos 91X cos 92
where F2-1 = A 2 X dA1 x dA2
2 JAIJA2 r
From equations (i) and (ii), it is apparent—
A1F12 = A2F21

34. Explain shape factor of a surface with respect to itself (F1_1). What is F1_1 for a
concave, flat and convex surface?
(UPTU — 2003)
Consider a concave surface. The fraction of radiant energy leaving the concave surface is
intercepted by another part of the same concave surface. The shape factor of a surface with
respect to itself is denoted by F1_1. The shape factor F1_1 for concave surface is not zero
while it is zero for flat and convex surface.

Concave surface

F11
Flat surface F11

Convex surface surface

35. Show shape factor from a radiating surface to a subdivided receiving surface is simply
the sum of individual shape factors.
Consider A l radiating surface and A2 surface consisting A3 and A4 as receiving surfaces.

A2 = A3 + A4

F1
Receiving surface
1-4

Radiant surface
Relation between shape factors
3I8 Heat and Mass Transfer

For surface Al and A2, applying reciprocating theorem


Al F12 = F2_1 A2 = Q124 = C1 X Q12
a Ti

Also Al F13 = F31 A3 = Q134 = C1Q13


a Ti

and A l F14 = F41 A4 =Q14 = „1


, y
a r, — Q14

Total Q12 = Q13 + Q14


... Al F12 = Al F13 ÷ Al F14
or F1.2 = F1.3 ÷ F1A

36. Find the relationship in shape factors in case radiating surface (A1) is subdivided in
to subareas A3 and A4.
Consider radiating area Al is subdivided to subareas A3 and A4.

Receiving
F4_2 surface

Radiating
surface

Q1-2 = Al F1_2
Q3-2 = A3 F3_2
Q4-2 = A4 F4_2
But Q12 = Q3-2 + Q4-2
A l F12 = A3 F3_2 + A4 F4_2

37. Find relationship of shape factors for an enclosure consisting of n surfaces.


The enclosure has n surfaces and heat leaving one surface is absorbed by all n surfaces
Q = Qi_i + Q1-2 + Q1-3 • • • + Ql-n
= Q x Fi _ i + Q X Fi2 Q X Fi3 Qx
or F1 _1 + F12 + F13 ••• F1.n = 1
Similarly, we can write—
F2-1 + F2-2 + F2-3 • • • F2n = 1
F3_1 + F3_2 + F3_3 ... F3n = 1

Fn_1 Fn_2 Fn _3 Fnn = 1


Radiation Heat Transfer 3I9

Enclosure with n-surfaces


j=n
We can write Fij = 1 where J = 1, 2, 3, ... n. It is called summation rule.
j=i

38. Find various shape factors when a black body is inside a black enclosure.
Consider a black body (A1) inside a black enclosure (A2)

All radiation emitting from black body Al is intercepted by the enclosure A2. Hence, shape
factor—
F12 = 1
Since two bodies are interacting radiation, we have from reciprocity rule—
A1 F1 _ 2 = A2 F21
AlE,
F2_1 = A r 1-2
2

Al
A2
Also heat omitting from A2 is absorbed by surfaces Al and A2.
F21 + F22 = 1
Al
But F21 = ri2
Al
F22 = 1 A2
320 Heat and Mass Transfer

39. Find shape factors of a tube with a cross-section of an equilateral triangle as shown
below.

Using the relation of an enclosure with three surfaces, we have—


F1.1 ÷ F1-2 ÷ F1-3 = 1
However, surface 1 is flat, hence—
F11 = 0
Also from symmetry—
F12 = F13
F12 ÷ F13 = 1
or F12 = F13 = 0.5
Similarly, F21 = F23 = 0.5
and F31 = F32 = 0.5

40. Find shape factors for hemispherical surface and a plane surface as shown below.

Using summation rule w.r.t. surface 1, we have—


F11 + F12 = 1
But by reciprocity rule, we have—
F1 _ 2 A1 F2 _1 x A2
Using summation rule w.r.t. surface 2, we have—
F21 + F22 = 1
But F22 = 0 as it is a flat surface.
F21 = 1
A2
F12 = F21 x
Radiation Heat Transfer 32 I

= A2
Al
. F11 = 1 — F12

= 1 — A2
A1
However, A2 = iv r2
A 1 = 2ir r2
A2 1
.'. — = — or F12 = 0.5 and F11 = 0.5
Ai 2
Hence, shape factors are—
F11 = 0.5, F12 = 0.5, F22 = 0 and F21 = 1
Note: F2_1 = 1 means that radiation of flat surface is completely intercepted by
hemispherical surface. F12 and F11 equal to 0.5 each means that radiation from hemisphere
is equally intercepted by flat surface and hemisphere.

41. Find the shape factors of cylindrical cavity as shown below.

Using summation rule w.r.t. surface 1, we have—


F1-1 + F12 = 1 (i)
Also summation value w.r.t. surface 2, we have—
F21 + F22 = 1
But F22 = 0 as surface 2 is flat surface
F21 = 1
Now using reciprocating rule
Al F1 _ 2 = A2F21
A
or F1-2 = — :x1
A
322 Heat and Mass Transfer

Jr
— 4 , x1
7rd`
7rdh+


d
4h+ d
From equation (i)
Fll = 1 - F12

=1 d
4h+ d
4h
= 4h+d

42. Find shape factors for conical cavity as shown below.

Using summation rule w.r.t. surface 2, we have—


F22 ÷ F21 = 1
But F21 = 0, as surface 2 is flat
.% F21 = 1
Using reciprocity rule, we have—
A1 x F12 = A2 F21
A2
or F12 = — X F21
Ai

7rd2
= 4 x 1 where 1= slant height
irdl
d d
4h2+(12 -%/4h2+d2
2
Radiation Heat Transfer 323

Using summation rule w.r.t. surface 1, we have—


F11 ÷ F12 = 1
or F11 = 1 — F12
=1
4h2 + d2

43. Find shape factors for a sphere (diameter = d) kept in a cubical box of side 'el' as
shown below.

Since the sphere is within the cube, hence—

F12 =1
Now using reciprocity rule
A1 F12 = A2 F21

d
Sphere in box

Al
or F21 = A2
— X F12

2
= rd = ,r
6d2 6
Using summation rule—

F11 + F12 = 1
or F11 = 1 — F12
=1—1=0
Also F21 + F22 = 1
or F22 = 1 — F21
= 1 — it/6
324 Heat and Mass Transfer

44. Find shape factors for diagonal partition within a square duct as shown below.

Using summation rule—


F11 ÷ F12 ÷ F13 = 1
But F11 = 0, as surface 1 is flat
F12 ÷ F13 = 1
Due to symmetry F12 = F13 = 0.5
Using reciprocating rule—
F12 X A1 = F21 X A2
Al E,
F21 = r 12
A2

1 x‘
— . 12,a x 1
2 x 0.5
xaxl
2
= x 0.5 = 0.71.

45. Calculate the shape factor for a very small disc dAi and a large disc A2 kept parallel
at a distance 'I?' directly above the smaller disc.
A2 and dA I are discs at vertical separation of height `h'. Consider an elemental area

at radius 'x' and thickness `dx' on disc A2. The elemental area dA2 is—
dA2 = 27rx • dx
Radiation Heat Transfer 325

The shape factor of area dAi w.r.t. dA2 is—


1 f COS 91 • COS 92 • dAl ' dA2
F1 _2 = 1
dAl jAl jA2 7r R 2
Here 01 = 02 = 0
1 f 1 COS2 9° dAi • dA2
••• F1-2 =
dAl jAl jA2 7r R 2
However, i = dA1 as area is small
fA, dA
C 0 S 2 9 dA2
... F1_2 =
5A2
7r R2
But R 2 = X2 ÷ b2

and c o s2 0 =
112 h2
=
R2 h2 +x 2
h2 2xx•dx
F12 = IA h
2 + x2 X X x (b2 + x2 )
2x
= f0r 11h2
2 + x2 h2 + x2 dx
2
= h 2r+ r2

46. Calculate shape factor between two opposite sides of a hollow cube if shape factor
between two adjacent sides of it is 0.2, as shown below.

Applying summation rule w.r.t. surface 1, we have—


F11 + F12 + F13 + F14 + F15 + F16 = 1
Given F12 = F16 = F14 = F15 = 0.2
Fll + 0.2 + F13 + 0.2 + 0.2 + 0.2 = 1
F11 + F13 = 0.2
However, F11 = 0 as surface is flat
F13 = 0.2
Similarly, we can find out other shape factors
F2_4 = 0.2
F5_6 = 0.2
326 Heat and Mass Transfer

47. Explain Hamilton and Morgan charts for evaluating shape factors for simple
configurations.

Hamilton and Morgan charts are available for finding shape factors of two surfaces in the
following configurations—
(a) Aligned Rectangular Parallel Plates

Plates 1 and 2 (size 'X' and 'Y') and spacing `L'. The graphs are drawn for F1_2 versus
LC for various
L
(b) Perpendicular rectangular plates with common edge

I
Plates 1 and 2 of sizes (X x Y) and (X x Z) perpendicular to each other with one
common edge. The graphs are drawn for shape factor (F1_2) versus for various .
X X
(c) Coaxial parallel discs

Discs 1 and 2 with radius r1 and r2 and interspacing . The graphs are drawn for
L r2
shape factor (F1_2) versus — for various .
L
Radiation Heat Transfer 327

0.9 I I I I I I
— Y/L = 20
0.8 WIC
— L: , . 10 _
0.7
_ —
0.6 .---
— A011
..0
111111F-
, 011P-
110111 —
0.5 AA 4
1.5
F1-2 — 1.0
0.4
_ _
0.8
0.3
— alP"--
iIIIIII 0.6 —
0.2 AiAiii
0.2 —
0.1 AIWW111"--
0.1
P d."----
0
10 20 30 4.0 50 60
X/L
Shape factor for aligned parallel plates

0.5
Y/X = 0.02

0.0
0.4
0.1

0.2
I 0.3
F1-2

0.2
0

1
0.1

0
01 0.2 04 06 08 2 4 6 8 10
Z/X
Shape factor for perpendicular rectangles with common edge
328 Heat and Mass Transfer

1.0
r2

0.8
ri
4

3
r2IL
0.6
2
15 1.25
F1-2
1.0
0.4
0.8
06

0.2 0.4

03

0
01 0.2 0.4 06 08 1 2 4 6 8 10

Shape factor for coaxial parallel discs

48. What do you understand by equivalent electrical network for radiative heat transfer?
or
Derive an expression for surface resistance of a grey body.
(UPTU — 2003)

Consider an opaque non-black surface or diffused grey surface with radiosity (J) and
irradiation (G). Let E be emissive power of surface and Eb the emissive power of black
surface at same temperature.

p.G. J = (E = p.G)

////////////////////// ///////////////
Radiosity (J)

J=E+ p• G.
= E Eb p• G where E = emissivity
Radiation Heat Transfer 329

Now for opaque surface, transmissivity z = 0. Hence, p + a = 1 or p = (1 — a)


J= EEb +(1— a) • G
According to Kirchhoff s law E = a
J= EEb + (1 — E )G.
E Eb
or G —
1— E
The net heat transfer from the surface is equal to the difference of radiosity and irradiation
J—G= Q
or Q = A (J — G)
A(j J — E
=
1— E

(J—JE—J+EEb)
—A
1—E
A E (Eb
1—E

Comparing with electric circuit:


1— E
Now Q is analogus to current (i), (Eb — J) is analogus to potential difference (AV) and
AE
E
is analogus to resistance (R) . The term 1A is called surface resistance. The equivalent
E
electrical circuit and surface resistance network are-
0 1\ A& 0
Eb 1—E J
AE

49. Derive the expression for heat exchange between two non-black surfaces. Draw the
equivalent electric circuit for this heat flow.
or
Find grey body factor.
330 Heat and Mass Transfer

Consider two non-black surfaces with radiosity J1 and J2. Now out of all radiation emitted
by first surface, a fraction J1 A1 F1.2 is received by second surface. Similarly, out of all
radiation emitted by second surface, a fraction J2A2 F2.1 is received by first surface.
Therefore, net heat transfer between first and second surface is—
Q12 = Jlx A1 x F1.2 — J2 x A2 x F2.1
Applying reciprocity rule, A l F12 = A2F21
Q12 = Al X Fi.2(Ji — J2)

— 1
Al X Fi_2
The equivalent electric circuit and space resistance are-

1
Al x F12
Two grey surfaces Electric circuit Space resistance

The term 1 is called space resistance which depends upon geometry and distance
Al fi-2
between the surfaces. Besides space resistance, both surfaces have their respective surface
resistances as shown below.
N
\\.\\\\\\\\\\\\\\\\\\\
\\\\\\\\\\\\\\\\\\\\\

A1F12

[ 1 — £2]
A2E2
Surface and space resistance

The equivalent electrical circuit is—


Eb1 Eb2

Ebl — Eb2 Ebi — Eb2


Q1-2 — — 1 — Et + 1 ± 1 — E2
X Al
El f1-2 E2 A2
Radiation Heat Transfer 33 I

A 0.(T4 _ 4)
i i T2

= 1 — + 1 1 — E2 X A2
El F1-2 E2 Al
= Fg 1 _2 Al a ( Ti4 — T24)

where Grey body factor = Fg1_2 — 1


1—E1 1 + 1— E2 x Al
El F1-2 E2 ti2

50. Find the grey body factor for: (i) two infinite parallel plates, (ii) two concentric
spheres of radius r1 and r2, (iii) two black surfaces, and (iv) A very small body in a
large enclosure.
Case 1: Infinite parallel grey plates
\\\\\\\\\\\\\n)
N\N\\\\\.\\\\

= A2

If two grey surfaces are infinite parallel plates, radiation emitted by first surface is fully
absorbed by second surface and vice versa. Hence, F1.2 = F21 = 1 and A l = A2.

Fgi _2 = 1
1 1— E2
X Al
El F1-2 E2 A2
1
—1±1±k-1
El E2
1
— 1
— —1—1
El E2
Case 2: Concentric grey spheres of radius r1 and r2
332 Heat and Mass Transfer

The radiation emitted by first sphere is completely absorbed by second sphere. Hence,
F1_2 = 1 and Al = 42r r?, A2 = 4 ir d
Fgi _2 = 1
1 — el + 1 + 1 — G 712
2X ..2.
El 1 E2 12

= 1
2
1 + (1 _1)11
El E2 r22
Case 3: Two black surfaces
In this case E1= E 2 = 1
... 1
Fg 12 = 1 — 1 1 1-1 =F1.2
+
1 + F12 1
or Fg12 = F12
For black bodies, grey body factor is same as shape factor.
Case 4: A very small body in a large enclosure

0A1 A2 A2 >>> Al

Ai
All radiation emitted by Al is intercepted by A2. Hence, F12 = 1. Also A —> 0
Al

Fg _2 = i _ 1 1
1 Ei + 1 + 1 — E2 l = 1 — el 2
XA +1+1—G X0
E1 F1-2 E2 A2 el ez
= E1

51. What is the purpose of radiation shield? Find the ratio of heat transfer between two
infinite parallel surfaces with radiation shield and without radiation shield.
(UPTU — 2002 — 3)
Heat transfer between two surfaces can be reduced by placing a radiation shield between
them. The heat transfer is reduced as placing a radiation shield between two surfaces offers
additional resistance in the heat flow

//
/
\\\\\\\\\\\\\\

\\\\\\\\\\\\\\

T1 ,,.
/ T2 T1
//
//
/
81 i82 ei
i
e3
Without shield
With shield
Radiation Heat Transfer 333

Case 1: Without shield.


For infinite parallel surfaces, the grey body factor is-
Fgi _2 = 1
i1 + 1 _ 1)
E]. E2

Ai o- (Ti4 — T24)
Q1-2 =
( 1 + 1 _ 1)
E1 E2
Case 2: With shield
Q1-3 = Q3-2
a ( 4 _
Al Ti T34 ) Al a (T34 — T24)
( 1 + 1 _ ij (1+1_
El E2 E3 E2 ij

4 ( 1 + 1 _ ij + 4 ( 1 + 1 _1)
T1 T2
E3 E2 El E3
Also T 34 —
(1 + 1 _ 1)+ (1 + 1 _
E3 E2 El E3 1)

Ai a (114 — T24)
Q13 =
( 1 + 1 _ ij+ ( 1 + 1 _
E3 E2 El E3 1)
( 1 + 1 -1)
Q1-2 Ej. E2
=
Q1-3 ( 1 + 1 —1)+( 1 + 1 _ 1)
El E3 E3 E2

52. If two infinite parallel plates have equal emissivity, find heat transfer with and
without shield having the same emissivity.

( 1 ± 1 _1)
Q1-2 El E2
Q1-3 ( 1 + 1 _ ij+ ( 1 + 1 _ i)
E1 E3 E3 E2
Now given El = E2 = E3 = E
( 1 + 1 - 1)
Q1-2 E E )
=
a_3 (1 ± 1 _0+ p
+1-11
E E I k,E E )
334 Heat and Mass Transfer

__ 1
__1
1 +1 2
Hence, heat transfer is reduced to half with the use of shield of same emissivity.

Also ..T 34 = T14 + T24


2

53. Find total resistance in case 'n' radiation shields of same emissivity are used. Also
find the ratio of heat transfer with 'n' shields and without shields. Assume
temperatures of surfaces remain at T1 and T2.
When 'n' shields are used, we will have—
(a) (n + 1) space resistance but F1_3 = F3_4 = F4_5 = 1
(b) (2n + 2) surface resistance
Total resistance is—
1
ER =[(2n+2)(1 — E )+(n+1)xlix-
E A
Aa (Ti4 — T24)
Qwith'n'shields =
(2n +2)( —1)+n+1
1-

= Aa(T 4 — r24,)
2(n +1)
(n + 1)
E

= Aa (Ti4 — T24)
(n + 1) r-
2 —1)
E

A CT (Ti4 — T24)
Q(i.e.
without shield =
n=0) 2 —1
E
Qwith shields _ 1
Qwithout shields n +1
Note: The equation of heat flow for parallel plates by n shields with emissivity E si,
E s2 ... E Sri as
A a ( Ti4 — T24)
Q12 =
1 + 1+ 2L,
÷ 1
El E2 n=1 Esn
Radiation Heat Transfer 335

54. Show heat transfer by radiation from a cavity is—

[ — Fil
Q = AiE Ti4
1— Fll(1 —Ei)
Consider a cavity as shown below which is at uniform temperature T1. Out of the emitted
radiant heat energy, a part of this energy is intercepted by the surface itself. Certain portion
of this intercepted energy by the surface is further absorbed by the surface and the
remaining is reflected to the surroundings. Let us chalk down emission, absorption and
reflection of the radiant energy.
(a) Emission of radiation from the surface— Al E 1a Ti4

Cavity

(b) The amount of radiant energy intercepted by the surface


= F11 x (A E l a T 14)
(c) The amount of intercepted radiant energy absorbed by the surface—
= E l x (Fil Ai x E l X aT14) as (al = E 1)
(d) The amount of energy reflected—
= (1 —E 1)(F11 XAi xE i xaTi4)
(e) The amount of reflected energy which is again intercepted and absorbed—
= (1 — E 1) (Fi? Ai E 1 X a Ti4)
(f) The amount of reflected and intercepted energy which is again reflected—
= (1 — E 1)2 (Fl ? A iE 1 X a T4)
(g) Reflected energy which is again absorbed—
= (1 — E 1 )2 (A iE ? F1? a T4)
(h) The fraction of this intercepted energy which is again reflected
= (1 — F11 a T14)
E 1)3 (A 1E 1

Keeping the above absorption and reflection in view, the net rate of emission from the
surface is—
Q = First emission — total absorption
336 Heat and Mass Transfer

= Ai E la Ti4 - [E ? Fn. Ala Ti4 ÷ (1 - E OE ? Fli Ala T4


+(1 - E 1)2 E1 Fil Ala Ti4 ÷ ...1
= A1E 1 676"[1 - E 1F11 - E 1(1 - E 1)Fii - E 1(1 - E 1)2 F1?...]
= AiE la T4[1 - E iFidl ÷ (1 - E OFii + (1 - E 02 Fli]

= AiE la Ti4{1 E1 F11 }


1- F11 (1- E1)
1-F11 }
= AlE la T4{
1 - Fn. (1 _ Ei)

55. For two infinite parallel planes with emissivity El and E 2, the interchange factor for
radiation from surface 1 to surface 2 is given by-
E1E2 1 1
(a) (b) — + —
El + E2 - E1 E2 E1 E2
(c) E1 + E2 (d) E1E2
(IES 93)
1 but F12 = 1
Fg12 = 1 - E1 + 1 1 - E2
+
El F1-2 E2

1
1 -1+1+ —
-— 1 -1
Ei E2
E1 E2
El ÷ E2 - E1 E2
Option (a) is correct.

56. What is the net radiant interchange per square metre for two very large plates at
temperatures 800 K and 500 K respectively? (The emissivity of the hot and cold plates
are 0.8 and 0.6 respectively. Stefan-Boltzmann constant is 5.67 x 10_8 W/m2K4.)
(IES 96)

Fg1. 2 - El E2
El ± E2 - E1 E2

= 0.8 x 0.6 = 0.48


0.8 + 0.6 - 0.8 x 0.6 0.8 + 0.6 - 0.48
= 0.52
Q

A = F8120(1'14 - T24)
= 0.52 x 5.67 x 10-8 (8004 - 5004)
Radiation Heat Transfer 337
= 29484 (4096 - 625) x 10-8 x 108
= 40260 W
= 10.260 kW/m2

57. Match List-I with List-II and select the correct answer using the codes given below
the lists.
List-I List-II
(A) Infinite parallel plates 1. E
(B) Completely enclosed body large compared to 2. E 1 E 2
enclosing body (subscript for enclosed body)
(C) Two rectangles with common side perpendicular 3. 1
1
— —1 - 1
E1 E2

(D) Concentric cylinders 4. 1


1+ 1
E1 A2 c2
Codes:
A B C D
(a) 1 2 4 3
(b) 3 1 4 2
(c) 2 1 3 4
(d) 3 1 2 4
(IES - 95)

1
Fg1-2 = 1- E1 + 1 + 1 - e2 Al
El F12 E2 A2
For parallel plates, F12 = 1 and Al = A2
Fg 1 _ 2 - 1
1 +1-1
E1 E2
For concentric cylinders, F12 = 1
Fgi _ 2 =
1
1 Ai 1 )
-A 1
E1 112 E2

For complete enclosed body, Al <.< A2 and - A -> 0 and F12 = 1


112
Fg _2 = 1 _ 1
1 e 1 — E2
1 +1+ x0
E1 E2
=c1
338 Heat and Mass Transfer

For two rectangles, we have


Fg1_2 = E l E 2
Option (d) is correct.

58. Consider two infinitely long black body concentric cylinders with the diameter ratio
D2/Di = 3. The shape factor for the outer cylinder with itself will be—

(a) 0 (b) 1/3


(c) 2/3 (d) 1
(IES — 97)

1
Fg12 =
El — 1 1 E2-1 Al
E1 + F12 + E2 A2
Al dl L d1 1
Now F12 = 1, A2 = gc12 L — d2 = 3 &E 1 =E 2 = 1 for black bodies.

1
F81-2 = 0 +1+ 0 = 1
Option (d) is correct.

59. Two large parallel plates having emissivity of 0.5 each exist at temperature of 1200 K
and 300 K: (a) Find the heat transfer rate per m2 of the plate. (b) Find heat transfer
rate if a radiation shield with an emissivity of 0.5 on both sides is placed between the
two plates.
(UPTU 2002 — 3)

(a) The grey body factor is—

Fg1 _ 2 = 1 (E 1 = E 2 = 0.5 also A l = A2)


El 1 E2 Al
El + F12+E2 X A2
Radiation Heat Transfer 339

= 1 = 1 = 1
+0.5+1+0
055 +1+1+1 3
0.5 0.5
Q1-2
= Fg1_2 x a (Til — T24)
A
_13 x 5.67 x 10-8 x (12004 — 3004)

_13 x 5.67 x 10-8 (20736 — 81) x 108

= 39.038 kW/m2
(b) The grey body factor with shield—
Fg1_2 = 1
( 1 + 1 _ i) + ( 1 + 1 _ 1)
E1 e3 e3 e2

= 1
(1 1 )
0.5 + 0.05 — 1)+(005 + 0.5 1
1
= 2(2 + 20 —1)

= 1
42
Q1-2
= Fgi_2 x a x (Ti4 — T24)
A
= 4 x 5.67 x 10-8 (12004 — 3004)
2
= 4 x 5.67 x (20655)
2
= 2.79 kW/m2

60. Calculate the energy emitted by a grey surface of emissivity 0.45 at a temperature
800°C. The surface measures 5 m x 6 m.
(UPTU — 2003 — 4)
Energy emitted by a grey surface is—
Q = EA6T4
= 0.45 x (5 x 6) x 5.67 x 10-8 (800 + 273)4
= 1.01 x 103 kW
340 Heat and Mass Transfer

61. A flat plate 5 m2 receives normally radiant energy with an intensity of 660 W/m2. The
absorptivity of the plate is 2 times its transmissivity and 3 times its reflectivity. Find
the energy absorbed, transmitted and reflected in watts.
(UPTU — 2003 — 4)
Given
(a) absorptivity 2 times its transmissivity
a = 2r or r = a
2
(b) absorptivity 3 times its reflectivity
a = 3p or P a
Now a+p+ = 1
or a
a + (±1 + — = 1
2 3
or
6a +3a +2a = 1
6
or a= 6
11
r = a =3
2 11
=a=2
' 3 11
Radiant energy intercepted = 660 W/m2
(a) Absorbed energy = a x 660 x area
6 x 660 x 5
= 11
= 1.8 kW
(b) Energy transmitted = 660 x p x area
= 660 x 2 x5
11
= 600 W
(c) Energy reflected = z x 660 x area
3 x 660 x 5
= 11
= 900 W

62. Two parallel rectangular plane black surfaces of 1 m x 2 m each, face each other and
are located at a distance of 4 m. The temperature of one surface is 200°C and the
other is 100°C. Calculate the net heat exchange between the two surfaces.
(UPTU — 2002-3, 2004-5)
Radiation Heat Transfer 34 I

Guidance: The problem is to be solved with Hamilton and Morgan chart for aligned
rectangular parallel plates by finding shape factor F1-2.

0 x 'N

Here L . 4, x = 1 and Y =2
X =— 1 = 0.25
L 4
Y 2
L = 4 = 0.5
From the graph for calculated X and Y
L L
Fg12 = 0.05
Q12 = Fg1_2 x A a (Ti4 - Ti)
= 0.05 x (1 x 2) x 5.67 x (4734 - 3734) x 10-8
= 0.05 x 2 x 5.67 x (500.5 - 193.5)
= 174 W

63. The outer surface of a tube of 100 mm diameter is maintained at 120°C by the
passage of steam through its interiors. A radiation shield is installed around the tube,
with an air gap of 10 mm between the tube and the shield. The shield is at a
temperature of 35°C. The tube and shield are diffuse grey surfaces with emissivity
of 0.8 and 0.1 respectively. What is the radiant heat transfer from the tube per unit
length?
(UPTU - 2005 - 6)
The arrangement of tube and shield is-
Shield (d2 = 0.12 m)

Tube (d1 = 0.1 m)


T1 = 120°C
T2 = 35°C

Fg1 _2 = 1
1+(1 ) Al
-1
El e2 A2
342 Heat and Mass Transfer

= 1
1 _, ( 1 x1
1 ) irdi
0.8 ' 0.1 ) xd2 x 1

= 1
1.25 + (10 1) 0.1
0.12

= 1
1.25 + 7.5

= 1 = 0.1143
8.75
Q = Fgi _2 x A x ax (Ti4 - Tit)
= 0.1143 x 5.67 x 10-8 x (iv x 0.1 x 1) (3934 - 3084)
= 0.203 x 10-8 x (238.5 - 89.99) x 108
= 0.203 x 148.51
= 30.14 W/m

64. A small convex object of area A1, temperature T1 and emissivity E l is enclosed within
a larger enclosure at temperature T2 and emissivity E 2. Derive an expression for the
net heat exchange between the two.
(UPTU - 2007 - 8)

1
F
g12 = 1-El 1 1- E2 Al
+ + X
E1 F1-2 E2 A2

T2
£2

For convex surface F11 = 0, hence F12 = 1


Ai
Also A2 >» A l and A -> 0
Al
1
F$1_2 = 1 = c1
— -1+1
cl
Q = Fgi _ 2 x Ai a (Ti4 - T24)
= E illi a (Ti4 - Th

65. Two large, diffuse, grey, parallel surfaces (emissivity for both the surfaces is 0.8) are
separated by a small distance. A thin radiation shield is used to reduce the radiation
Radiation Heat Transfer 343

heat transfer rate between the two surfaces to 10% of the original. Find the
emissivity of the shield.
(UPTU — 2007 — 8)
Case 1: Without shield, shape factor is
1
Fg1-2 1 1
—1
El E2
1
= 1 + 1_
0.8 0.8
1 = 1
2.5 —1 1.5
= 0.67
Qws = Fg1.2 A a (T14 —
= 0.67 x A x a(Ti4 —
Case 2: With radiation shield, shape factor is
1
Fg1_2 =
1 + 1 _].)+( 1 + 1 —1)
E3 E3 E2
1
= 1 1 1
+- -1
E3 E3
1
2.5 — 2 +
E3

1
0.5+
E3
Qwith shield = Fg12 xAx a (Ti4 — T24)
1 xA x a (Ti4 — T24)
0.5 +1'
E3

Given 0
= 100
Qwith shield
1 Qws
x A x a (7,14 ) = 0.1 x 0.67 xA x a (Ti4 — T24)
E3

or 1 = 0.067
0.5 + 2
E3
344 Heat and Mass Transfer

or 1 = 0.0335 + 0.134
E3
or 0.134 = 0.9665
E3

or E 3 = 0.134
0.965
= 0.14

66. A thin aluminium sheet with an emissivity of 0.1 m on both sides is placed between
two very large parallel plates that are maintained at uniform temperature T1 = 800 K
and T2 = 500 K and have emissivities 0.2 and 0.7 respectively. Determine the net rate
of radiation heat transfer between the two plates per unit surface area of the plates
and compare the result to that without the shield.
(UPTU — 2006 — 7)
The set-up of plates and shield is—
Aluminium shield
= 0.1

T1 = 800 K T2 = 500 K
= 0.2 E2 = 0.7

1. Case 1: Without shield, the shape factor is


1 1
Fg12 = 1 1 _ 1— 1 + 1 _ 1
Ei +E 2 0.2 0.7
1 1
5 +1.43 —1 5.43
= 0.184
Qws
A
= Fg12 x a x (71 — Tit)
= 0.184 x 5.67 x 10-8 (8004 — 500
= 1.04 x 10-8 (4096 — 625) x 108
= 3.61 kW/m2
2. Case 2: With shield, the shape factor is
1
Fg1-2shield = + 1) + + 1)
E1 E3 E3 E2
Radiation Heat Transfer 345

1
( + _ i) + ( + 1. )
0.2 0.1 0.1 0.7 -
1
(5 + 10 - 1) + (10 + 1.43 - 1)
1
14 + 10.43
1
24.43
= 0.041
Qshield
Fg1-2shi.eid a x (Ti4 - T24)
A
= 0.041 x 5.67 x 10-8 (8004 - 5004)
= 0.041 x 5.67 x 10-8 3471 x 108
= 807 W/m2
Qshield - 807 - 22.35%
Qws 3610

67. A 20 cm diameter spheric ball at 800 K is suspended in the air. Assuming that the
ball closely approximates a black body, determine-
(i) the total black body emissive power
(ii) the total amount of radiation emitted by the ball in 5 min.
(UPTU - 2006 - 7)
Q =A x ax T 4
A = ird2 = x 0.22
= 0.1256 m2
Q = 0.1256 x 5.67 x 10-8 x 8004
= 2.917 kW
Heat dissipated in 5 min.
Qtote = 2917 x 5 x 60
= 875.1 kJ

68. What is greenhouse effect? Why is it a matter of great concern among environmen-
tal scientists?
(UPTU - 2006 - 7)
Earth has atmosphere around it for many kilometres which is mainly air with pollutants like
carbon dioxide, water vapour and methane, etc. These pollutants are transparent for shorter
wavelength radiation but these pollutants are opaque for longer wavelength radiation. The
earth receives solar radiation in day time and solar radiation has shorter wavelength
facilitating solar radiation to pass through the atmosphere. Solar radiation reaches the earth
and it warms up the earth on absorption.
346 Heat and Mass Transfer

As sun has very high temperature, hence solar radiation has short wavelength. During
night earth cools down by radiating its heat energy into atmosphere. As earth has relatively
low temperature, hence earth emits radiation at longer wavelength in the infrared region. The
pollutants do not permit longer wavelength radiation emitted from the earth to pass through
the atmosphere and this radiation is absorbed in the atmosphere. This absorbed radiation is
again reradiated to the earth surface. On account of this interception of energy in lower
atmosphere and reradiation towards earth, the temperature of lower atmosphere and earth
increases. Such warming of lower atmosphere and earth due to interception of radiation
from earth is called greenhouse effect. Due to greenhouse effect, there is no significant
change between day and night temperatures. Increase of pollutants in atmosphere results
from (1) excessive deforestation (more carbon dioxide in atmosphere), and (2)
industrialization and burning of fuels (concentration of CO2, in ethane and CFCs in
atmosphere).

Solar radiation can


pass through due to
short wave length

Infrared radiation is
Atmosphere
reflected by pollutants with pollutants
in atmosphere
Greenhouse effect

Greenhouse effect is of great concern to environment scientists due to the following


adverse consequences from it—
(a) It heats up the earth surface and evaporates the surface water.
(b) Global warming affects food production.
(c) Global warming melts ice of polar regions and glaciers, thereby increasing sea level.
Increase of sea level leads to flooding of low coastal areas of the world.
(d) Carbon dioxide undergoes photochemical reactions at higher atmosphere to form
carbon monoxide. Carbon monoxide is a toxic gas.

69. What is solar radiation? What is the effective sky temperature?


(UPTU — 2002-3, 2006-7)

The fraction of energy emitted by sun which reaches earth is called solar energy. Sun can
be considered a nuclear reactor in which heat is being generated continuously by fusion
reaction i.e. hydrogen atoms forming helium atoms. The reason for a fraction of solar
energy reaching earth is that the sun subtends only at an angle of 32 min at the earth. The
earth is located at a mean distance of 1.496 x 108 km from the sun. The intensity of solar
radiation reaching the earth is almost constant and it is called solar constant (G5). The solar
constant (G5) is the rate at which solar radiation flux is received on earth's surface normal
to sun rays. Considering sun as black body and solar radiation is evenly distributed on the
Radiation Heat Transfer 347

surface area of the sphere formed by earth's orbit around the sun, we have solar constant
as
r )2
Solar constant G, = ( '4'1 a Ts = 1353
rorbit
Due to eccentricity of earth's orbit, the distance of earth from sun is more in winter season
as compared to summer season. Hence, the value of solar constant `GS' varies as per earth's
orbit around sun. Maximum quantity of solar energy reaches the earth surface when: (1)
rays are normally incident on earth surface, (2) rays travel a smaller distance (seasons of
year), and (3) there is less absorption.
The solar radiation reaching the earth surface is the sum of direct solar radiation (Gd) and
diffuse solar radiation (Gdf) as shown in the figure. The scattered radiation is received by
earth from all directions. Diffuse solar radiation varies from low value on clear day to
maximum on totally cloudy day.
Diffuse solar radiation (Gdf)

Atmosphere ,- Direct solar


radiation (Gd)

Earth

The solar radiation passes through atmosphere and it undergoes absorption and scattering
in the atmosphere. The atmosphere also contains pollutants like carbon dioxide, water
vapour and other suspended particles. The emission of heat energy from the atmosphere to
the earth surface is called atmospheric emission. The atmospheric emission depends upon
effective atmospheric absolute temperature. The atmospheric emission constant `Gsky' is
given by
G sky = aTs2ky W/m2K
Effective atmospheric temperature (Tay) ranges from 285 K in warm and cloudy day to
230 K for cold and clear day.

70. If solar constant on earth surface is 1353 W/m2, find the temperature of sun
considering it is a black body. Sun's diameter = 1.4 x 106 km and mean distance of
the sun from the earth = 1.5 x 108 km.

The solar radiation is spread over a sphere of radius (r) = 1.5 x 108 km

..
,,
t ,
I
L 0 Earth
l 1
,‘ Sun i
, I
,,,
348 Heat and Mass Transfer

Qsun = Asun x a x 71un

G = Qs = Asun ) Tx Ts4un
r 2
grorbit grorbit
2 4
rxds xax Tsw,
or 1353 =
4 x — rorbit

(1.4 x 109)2 x 5.67 x 10-8 x Tstn


4 x (1.5 x 1011)2

4 x 1353 x 2.25 x 1022


or sun Q = 1096 x 1012
T = 1.96 x 10" x 5.67 x 10—
or Tsun = 5762 K.

71. A furnace has temperature of 2000°C and it emits radiation. Find: (1)
monochromatic emissive power at 2 pm wavelength, (2) wavelength having
maximum emission, (3) maximum emissive power, and (4) total emissive power. Take
C1= 0.374 x 10-15 and C2 = 1.4388 x 10-2

1. Using Max-Planck's law, for black body


c
Eb A — 1
c2
e —1

6.374 x 10-15 x (2 x 10-6)-5


Eb b = 1.4888 x10- 2
e 2 x10-6 x2273 —1
= 5.2 x 1010 W/m2
2. As per Wein's displacement law
2898 x 10-6 2898 x 10-6
Amax = 7 - 2273
= 1.28 pin
3. Maximum spectral emissive power
0.374 x 10-15 x (1.28 x 10-6)-5
(FbA,)max
1.4888x10-2
1
e 12,8x10-6 x 2273

-
3.74 x 10-5 x 0.29 x 10"
= 6.5 x 1012 W/m2
167 -1
Radiation Heat Transfer 349

4. Total emissive power per m2


E = a T4
A
= 5.67 x 10-8 x (2273)4
= 1.513 x 106 W/m2

72. The radiative heat transfer rate per unit area (W/m2) between two plane parallel grey
surfaces (E = 0.9) maintained at 400 K and 300 K is-
(a) 992 (b) 812
(c) 464 (d) 567
(Stefan-Boltzmann constant a = 5.67 x 10-8 W/m2K4)
(GATE - 93)
Shape factor 1 _ 1
Fg1-2 = 1 + 1 _1 1 + 19 1
El E2 09 0
= 0.818
Q = F g 12 a (ril - Ti)
A
Q = 0.818 x 5.67 x 10-8 (4004 - 3004)
A
= 812 W/m2
Option (b) is correct.

73. For the circular tube of equal length and diameter as shown below, the view factor
F13 is 0.17. The view factor F12 in this case will be-

(a) 0.17 (b) 0.21


(c) 0.79 (d) 0.83
(GATE - 2001)
Using the summation rule
F11 + F12 + F13 = 1
350 Heat and Mass Transfer

Since surface A l is flat, hence F11 = 0


F12 ÷ F13 = 1
or F12 = 1 — F13 = 1 — 0.17 = 0.83
Option (d) is correct.

74. Match List-1 (type of radiation) with List-II (characteristic) and select the correct
answer using the codes given below the lists.
List-I List-II
(Type of radiation) (Characteristic)
A. Black body 1. Emissivity does not depend on wavelength
B. Grey body 2. Mirror-like reflection
C. Specular 3. Zero reflectivity
D. Diffuse 4. Intensity same in all directions
Codes
A B C D
(a) 2 1 3 4
(b) 3 4 2 1
(c) 2 4 3 1
(d) 3 1 2 4
(GATE)

A. Black body has a = E = 1 and p = 0 (option 3 in List-II)


B. Grey body has emissivity which is independent of wavelength (option 1 in List-II)
C. Specular has mirror-like reflection (option 2 in List-II)
D. Diffuser has emissivity independent of direction (option 4 in List-II)
Hence, code is — 3, 1, 2, 4
Option (d) is correct.

75. Sun emits maximum radiation at A = 0.52 IL Assuming sun as black body, find the
surface temperature of the sun and emissive power at that temperature.

According to Wien's displacement law, we have


Amax x T = 2898
As Amax = 0.52 hence-
T = 2898
0.52
= 5580 K

Now Q = a 74
= 5.67 x 104 X (5580)4
= 5.67 x 107 W/m2
Radiation Heat Transfer 35 I

76. A spaceship launched from the Earth to the Venus. The distance of sun to Venus is
108 x 106 km while to Earth it is 150 x 106 km. If temperature of spaceship at earth
is 300 K, find the temperature at Venus.
Guidance: The surface area of spaceship remains same and the solar radiation intercepted
depends upon the distance from the Sun to Earth and Venus respectively.

Venus

Earth

If Q is total emission from Sun, then 2 = Solar constant at Earth = crAspace X Ti


47r rsun_earth

Similarly - Solar constant at Venus = 6 Aspace X Ti


4 7trs2un-venus
2 4
rsun-venus = 1
r h T2
(los x 106 300 )4
150 X 106 - T2
1
or
T2 (150)2
300 - 108)
= 1.18
or T2 = 1.18 x 300
= 354 K

77. Sun's surface at 5800 K emits radiation at wavelength of 0.5 A furnace at 300°C
will emit through a small opening a radiation at a wavelength of nearly-
(a) 10 p. (b) 5 p.
(c) 0.25 p. (d) 0.025 p.
(IES 97)
Using Wein's displacement law-
x Ts = X Tf

A
mf - X Ts
T1

- 0.5 x 10-6 x 5800 = 5.06 x 10-6


573
352 Heat and Mass Transfer

= 5.06 gm
Option (b) is correct.

78. Assertion (A). In a furnace, re-radiation from the walls has the same wavelength as
the incident radiation from the heat source.
Reason (B). Surfaces at the same temperature radiate at the same wavelength.
Mark the answer as—
(a) If both (A) and (R) are true and (R) is correct explanation of (A)
(b) If both (A) and (R) are true but (R) is not the correct explanation of (A)
(c) If (A) is true but (R) is false
(d) If (A) is false but (R) is true
(IES — 98)
As per Wein's displacement law—
An, x T = constant
1
or T
m
The wavelength of radiation depends upon temperature of surface.
Assertion is true as all walls of furnace are at same time.
Reason is true as surfaces at same temperature radiate at same wavelength and it is a correct
explanation of assertion.
Hence, option (a) is correct.

79. Two long parallel surfaces each of emissivity 0.7 are maintained at different
temperatures and accordingly have radiation heat exchange between them. If is
desired to reduce 75% of this radiant heat transfer by inserting thin parallel shields
of same emissivity on both sides. The number of shields should be—
(a) one (b) two
(c) three (d) four
(IES — 92)

Qwith shield = 1
Qwithout shield n+1
Qwith shield = 25% Qwithout shield
1
= 0.25
n+1

or n+1=
0.25 = 4
or n=3
Option (c) is correct.
Radiation Heat Transfer 353

80. A source of radiation has an intensity of 800 watts/m2. Find the number of photons
per sec per sq metre represented by this intensity if the wavelength is 500 n.m (use
speed of light = 3 x 104 m/s and Planck's constant = h = 7 x 10r34 J/s)
(a) 10.4 x 1021 (b) 6.8 x 1021
(c) 4.4 x 1021 (d) 2.0 x 1021
(IES 88)

Energy of a photon is
E=h•v
= 7 x 10-34 x c
A
3 x 10 4
= 7 x 10-34 x
500 x 10-12
= 21 x r.-20
1U = 4.2 x 10-20
5
If there are n photons per m2
n x E = 840
840
or n—
4.2 x 10-20
= 2 x 1021
Option (d) is correct.

81. Mark the answer as—


(a) If both (A) and (B) are true and (R) is the correct explanation of (A).
(b) If both (A) and (R) are true but (R) is not the correct explanation of (A).
(c) If (A) is true but (R) is false.
(d) If (A) is false but (R) is true.
(1) Assertion (A): The nose of aeroplanes is painted black.
Reason (R): Black body absorbs maximum heat which is generated by aerodynamic
heating when plane is flying.
(IES — 96)
(2) Assertion (A): In an air-conditioned room, the reflective coating should be on the inside
of the window.
Reason (R): Window pane glass is transparent to solar radiation
(IES — 96)
(3) Assertion (A): Solar radiation is mainly scattered or transmitted but not absorbed by the
atmosphere.
Reason (R): Absorptivity of atmosphere is low.
1. The nose of aeroplane: The nose of aeroplane houses the radar equipment and any
colour not hampering electromagnetic transmission form radar can be given to nose.
354 Heat and Mass Transfer

The assertion is false. The reason that black body absorbs maximum heat is true.
Hence, option (d) is correct.
2. Air-conditioned room: In air-conditioned room, the temperature inside the room is
maintained at low temperature. The glass is transparent to solar radiation and solar
radiation heats up the cooled space inside the air. To avoid this, reflective coating is
provided on the inside of the window. Hence, both assertion and reasoning are true and
reasoning is the correct explanation of assertion.
Hence, option (a) is correct.
3. Solar radiation: Atmosphere is transparent for solar radiation and radiation consists of
both scattered and transmitted radiation. Assertion is therefore true. The reasoning is
also correct and reasoning is the correct explanation of the assertion.
Option (a) is true.

82. Match List-I with List-II and select the correct answer using the codes given below
the lists.
List-I List-II
(A) Window glass 1. Emissivity independent of wavelength.
(B) Grey surface 2. Emission and absorption limited of wavelength
(C) Carbon dioxide 3. Rate at which radiation leaves a surface
(D) Radiosity 4. Transparency to shortwave radiation
Codes:
A B C D
(a) 1 4 2 3
(b) 4 1 3 2
(c) 4 1 2 1
(d) 1 4 3 2
(IES — 96)

(a) Window glass: They are transparent to short wave radiation—(option 4 of List-II)
(b) Grey surface: Their emissivity is independent of wavelength—(option 1 of List-II)
(c) Carbon dioxide: their emission and absorption limited of wavelengths (option 2 of
List-II)
(d) Radiosity: It is the rate at which radiation leaves a surface (option 3 of List-II)
Hence option (c) is correct.

83. A square room 3 m x 3 m has a floor heated to 27°C and has a ceiling at 10°C. The
walls are assumed to be perfectly insulated. The height of the room is 2.5 m and the
emissivity of all surfaces is 0.8. Find: (i) the net heat exchange between the floor and
ceiling, (ii) wall temperature. Assume shape factor of ceiling to floor as 0.25.

The room is depicted as below.


Radiation Heat Transfer 355

0Surface = ceiling
T 0Surface = floor
2.5
0Surface = wall

Room

Applying summation rule


F11 + F12 ÷ F13 = 1
But F11 = 0 as ceiling is flat surface
••• F13 = 1 — F12 = 1 — 0.25 = 0.75
Similarly,
F22 ÷ F21 ÷ F23 = 1
F22 = 0, F21 = F12 = 0.25
F23 = 1 — 0.25 = 0.75
The equivalent circuit diagram for space resistance when two interacting surfaces are
connected with third surface which acts as refractory wall is—

The total space resistance between surfaces 1 and 2 is


1 ) 1, ▪ , 1
Rspace 1 1
Al F12 Al F13 ▪ A2 F23
1 ± A2 2A1 fi2
or Rspace =
1 A2 Al F122
Ai a(Ti4 - T24)
Q1-2 = 1—E1 + 1—E2 Al + ± A2 — 2A1 F12 but Al = A2
El E2 A2 A2 —Al F2

Ai a (Ti4 — T24) Ai a(Ti4 — T24)


e)+ 2(1— F12)
E J) (1 F122) 2[(1
E J)+ 1-1F12
356 Heat and Mass Transfer

3x3x 5.67 x 10-8 (2.884 x3004)


— —_296 W
1— 0.88 1
2 +
[( 0.8 ) 1+0.25
Heat will flow from floor (surface-2) to ceiling (surface-1) as sign is negative. And wall
temperature is—
T3 = + V2I) = (2884 + 3004)
= 294 K

84. Liquid oxygen at —153°C is in a spherical vessel (diameter = 20 cm). A spherical


container (diameter = 30 cm) enclosed the vessel concentrically and space in between
is evacuated. Vessel surface and container surface have same emissivity of 0.04 and
temperatue of container surface is 27°C. Find: (i) heat transfer rate, and (ii) rate of
evaporation of liquid oxygen if latent heat = 209 kJ/kg.
The vessel and container are as shown below.

Liquid oxygen

Vessel

Container

Fgi _2 = 1
1-E1 1 + (1- E2 x Al
El r1-2 E2 A2
But F1_2 = 1 as container is completely enclosing the vessel.
Fgi _ 2 = 1 1
1 1 - 1) 1 +( 1 1) d2
El e2 A2 0.04 0.04 )4,
1
25 + (25 — 1) x 022
0.32
1 1
25+24x4 25+10.67
9
1 = 0.028
35.67
Q12 = Fgi_2 x A x cr(Ti4 — T24)
= 0.028 x (n x 0.22) x 5.67 x 104(1204 — 3004)
= —1.58 W
Radiation Heat Transfer 357
Negative sign shows that heat is flowing into liquid oxygen and liquid oxygen will
evaporate. Rate of evaporation per min is-
1.58 x 60
Mevap
209 x 103
= 0.45 x 10-3 kg/min

85. Two parallel plates are radiating to each other having temperature of 500 K and
300 K. Find heat transfer rate: (i) assuming the plates as black body, (ii) plates
having emissivity of 0.4. Each plate has surface area = 2 m2.
Case 1: Plates are black bodies
Q1-2 = F12 x a x A(Ti4 — Ti)
As plates are black F12 = 1
Q12 = Fi2, X a X A(Ti4 — T24)
= 1 x 5.67 x x 2 (5004 — 3004)
= 6.17 x 103 W
= 6.17 kW
Case 2: Plates are grey bodies.
1
Fg1-2 = 1— e + 1 1 — e2 X Al
e1 F12 e2 A2
Now A1 = A2 and F12 = 1
1 = 1
Fgi _2 =
1 + 1 _1 1 + 1 1
El E2 0.4 0.4
= 0.25
Q1_2 = 0.25 x 2 x 5.67 x 10-8 (5004 — 3004)
= 1.54 kW

86. Three concentric spheres of diameter 20, 30 and 40 cm are as shown. Innermost and
outermost spheres have temperatures of 200 and 800 K respectively. Find
temperature of intermediate sphere. Assume perfect vacuum for annular spaces and
emissivity = 0.2 for all spheres.
The set up is
358 Heat and Mass Transfer

Case 1: Fg12 = 1 1
1 + Al 1 1 + 0.2 2 ( 1 _ 1)
El -2 -2 0.2 0.32 10.2 -1/
= 0.148
Fg2.3 = 1 1
1 + A2 1 -1J 1 + 0.32 ( 1 1)
E2 A3 e3 0.2 0.42 10.2 )
= 0.138
Q12 = Q23
Fg12 X Ai x a(T4- T24) = Fg2.3 X A2 x a (T24 -
Tl - = Fg2 3 X A2 X (71 -
Fg1-2 Al
0 138 X 0.32 'pet 74\
ki 2 - 31
0.148 0.22
8004 - T24 = 2.1(T24 - 2004)
or 3.1 T24 = 8004 - 2004
= (4096 - 16) x 108
= 4086 x 108
4 4086 x 108
or T -
3.1
= 1318 x 108
Tl = 603 K

87. Two long parallel plates of same emissivity 0.5 are maintained at different
temperatures and have radiation heat exchange between them. The radiation shield
of emissivity 0.25 placed in middle will reduce radiation heat exchange to-
(a) 1/2 (b) 1/4
(c) 3/10 (d) 3/5
(ESE - 2002)

A a (Ti4
_ T24)
1 —1 - 1

Q El E2
A a (7,4 _ 7,4)
Qshield 1 2
(1 ± 1 _ 1) ± ( 1 ± 1 _ 1)
El E3 e2 E 3
Radiation Heat Transfer 359

(1 + 1 _ i.) + ( 1 + 1 —1)
E1 E3 E2 E2
(0.5 1 —1) + ( 1 —1)
= = +0.25 0.5 +0.25
1 1
—+——1
E2 E2 (01.5 + 0.5 1)

(2+4-1)+(2+4-1)
=
(2+2-1)
_5+5__10
3 3
Qshield_ 3
Q 10
Option (c) is correct.

88. Two large parallel plates with a small gap, exchange radiation at the rate of 1000 W/
m2 when their emissivities are 0.5 each. By coating one plate, its emissivity is
reduced to 0.25. Temperatures remain unchanged. The new rate of heat exchange
shall become—
(a) 500 W/m2 (b) 600 W/m2
(c) 700 W/m2 (d) 800 W/m2
(ESE — 2002)

Qi a (T4 - T24, )
A —1+— 1 —1
E1 E2

1000 = a (T14 - T24)


1 1 1
0.5 ' 0.5 —
or a (T i4 — T24) = 1000(2 + 2 — 1)
= 3000 W/m2

Now G - Th
_ a (7i4
A 1 +1 -1
El E2

_ 3000 3000
— 1 ± 1 — 1 — 2 + 4 —1
0.5 0.25
= 600 W/m2
Option (b) is correct.

89. Match List-I and List-II and select answer using the codes—
360 Heat and Mass Transfer

List-I List-II
(A) Black body 1. Emissivity does not depend on wavelength
(B) Grey body 2. Mirror-like reflection
(C) Specular 3. Zero reflectivity
(D) Diffuse 4. Intensity same in all directions
Codes
A B C D
(a) 2 1 3 4
(b) 2 4 3 1
(c) 3 4 2 1
(d) 3 1 2 4
(IES — 2002)
A. Black body: It has zero reflectivity (option 3 of List-II)
B. Grey body: Its emissivity does not depend in wavelength (option 1 of List-II)
C. Specular: It has mirror like reflection (option 2 of List-II)
D. Diffuse: In diffuse radiation, intensity is same in all directions (option 4 of List-II)
Option (d) is correct.

90. An enclosure consists of the four surfaces 1, 2, 3 and 4. The view factors for radiation
are F11 = 0.1, F12 = 0.4 and F13 = 0.25. The surface areas Al and A4 are 4 m2 and 2 m2
respectively. The view factor F4_1 is—
(a) 0.75 (b) 0.5
(c) 0.25 (d) 0.1
(ESE — 2001)
Applying summation rule
F11 + F12 + F13 + F14 = 1
0.1 ÷ 0.4 + 0.25 + F14 = 1
or F14 = 1 — 0.75
= 0.25
Using reciprocity law
A1F14 = A21 F41
4 x 0.25 = 2 X F41
or F41 = 0.5
Option (b) is correct.

91. Solar radiation of 1200 W/m2 falls perpendicularly on a grey opaque surface of
emissivity 0.5. If the surface temperature is 50°C and surface emissive power
600 W/m2, the radiosity of that surface will be—
(a) 600 W/m2 (b) 1000 W/m2
(c) 1200 W/m2 (d) 1800 W/m2
Radiation Heat Transfer 36 I

Equivalent electric circuit is—


— Q
Eb 0--V\A—ci j
I—E
AE

Q Eb — J
A — 1— E
E

Q J
= 600 = 1200 — — 1200 J
A 1 — 0.5
0.5
or J = 600 W/m2
Option (a) is correct.

92. A spherical aluminium shell of inside diameter 2 m is evacuated and used as a


radiation test chamber. If the inner surface is coated with carbon black and
maintained at 600° K. The irradiation on small test surface placed inside the chamber
is (a = 5.67 x 10_8 W/m2 K4)
(a) 1000 W/m2 (b) 3400 W/m2
(c) 5680 W/m2 (d) 7348 W/m2
(ESE — 1999)
The set up is—

As per summation rule F11 + F12 = 1

But F11 = 0 for black body


F12 =1
Q = F12 X T4
A
= 1 x 5.67 x 10-8 x 6004
= 7348 W/m2
Option (d) is correct.

93. A...body reflects entire radiation incident on it—


(a) transparent (b) black
362 Heat and Mass Transfer

(c) grey (d) white


Option (d) is correct.
Note: It is beneficial to wear white clothes in summer.

94. The monochromatic emissivity of a white body at all wavelengths and temperatures
is equal—
(a) zero (b) 0.1 to 0.4
(c) 0.6 (d) unity
Option (d) is correct.

95. "All bodies above absolute zero temperature emit radiation". This statement is based
on—
(a) Stefan's law (b) Planck's law
(c) Provost theory (d) Wien's law
Option (c) is correct.

96. Total emissive power E of a diffuse surface is related to radiation intensity I as E


equal to—
(a) 4x/ (b) it x /
(c) 7r2 x i (d) 47r x /
Option (b) is correct.

97. The absorptivity of a white washed wall is—


(a) 0.1 (b) 0.3
(c) 0.5 (d) 0.9
White washed wall has maximum reflectivity and minimum absorptivity.
Option (a) is correct.

98. A body at 1000°C in black surroundings at 500°C has an emissivity of 0.42 at 1000°C
and emissivity of 0.72 at 500°C. Calculate the rate of heat loss by radation per m2—

(i) When the body is assumed to be grey with e = 0.42


(ii) When the body is not grey.
Assume that the absorptivity is independent of the surface temperature.
(a) 20.6 kW, 18.5 kW (b) 32.6 kW, 28.5 kW
(c) 54.393 kW, 47.962 kW (d) 68.96 kW, 52.9 kW
Radiation Heat Transfer 363

Case 1: Body is grey with E = 0.42


The grey body emissivity remains constant and e = a
Net Q = E a (VI - T24)
A
= 0.42 x 5.67 x 10-8(12734 - 7734)
= 54.893 kW/m2
Case 2: Body is not grey
Hence, body will emit with emissivity 0.42 at 1000°C and absorb with absorptivity
0.72(e = a) at 500°C
Q
= 0.42 x 5.67 x 10-8 x 12734 - 0.72 x 5.67 x 10-8 x 7734
= 62538 - 14576 = 47962 W/m2
= 47.962 kW/m2
Option (c) is correct.

99. A small body at 100°F is placed in a large heating oven whose walls are maintained
at 2000°F. The average absorptivity of the body varies with temperature of the
emitter as follow:
Temperature 100°F 1000°F 2000°F
Absorptivity(a) 0.8 0.6 0.5
What is the rate at which radiant energy is absorbed by the body per unit surface
area? (a = 0.1714 x 10-8 BTU/hr ft2 K4)

(a) 2.11 x 10+4 BTU (b) 3.38 x 10+4 BTU


hr-ft2 hr-ft2

(c) 13.7 x 104 BTU (d) 3.16 x 10+4 BTU


hr-ft2 hr-ft2
(GRE)


Q = Heat absorbed at 2000°F - Heat emitted at 100°F
= a (a x T14 - E Th
= 0.1714 x 10-8(0.5 x 24604 - 5604 x 0.8)
= 0.1714 x 10-8(18.31 - 0.08) x 1012
= 3.16 x 10+4 BTU/hr-ft2
Option (d) is correct.
100. The earth receives solar radiation at a rate of 3.2 J/m2 min. Assuming that earth
radiates like a black body, calculate the surface temperature of the sun. Angle
subtended by the sun on the earth is 0.53° and a = 5.67 x 10-8W/m2K.
364 Heat and Mass Transfer

Earth is rotating about sun as shown below.

Angle subtended w =

or = 0.53 x 180 = 9.25 x 10-3 radian


Qs„,i = irD2 x a x T4
Qs„,, is distributed evenly on the sphere surface of radius R. Hence, radiation received at
earth surface (distance R) per unit area is—
aarth 7rD2 X X T 4
A 4irR2
0.7,4 ( 2 8.2
= (given)
4 R) = 60
5.67 x 10-8 a 8.2
x (9.25 x 10-3)2 x T- =
4 60
T4 . 8.2 x 4
or
60 5.67 x 10-8 x (9.25 x 10-3)2
T = 5800 K

101. A black body of surface area 10 cm2 is placed inside an enclosure having constant
temperature 27°C. The black body has constant temperature 327°C by maintaining
by heating by electricity. Find electric power required to maintain temperature.
a = 5.67 x 10-8 W/m2K.

Guidance: The black body emits radiation at 327°C and absorbs radiation falling on it
which is emitted at 27°C. Net heat flowing out is the difference of two.
Q = Aa(rit —
= 0.1 x 5.67 (6004 — 3004) x 10-8
= 69 W
Hence, 69 W electric power is to be given.

102 A piece of charcoal and steel of same area are heated to same temperature and left
to cool. Which will cool faster?

Charcoal acts like a black body and emission is—


Q b = Aa(Tia — 7,4
surrounding)
Radiation Heat Transfer 365

Steel if acts like a grey body with emissivity E„ the emission is-
Qg = E s A a (Ti4 - nunounding)
Since E s < 1, hence charcoal will cool faster.

103. Find the shape factor for surface Al for the following configuration: Al and A2 are
squares. All sides are 1.0 m each. Distance between Al and A2 is 1.0 m.
F.— 1 m

Guidance: Take area A2 + A3 = A4 and find shape factor F41 from Hamilton and Morgan
graphs. Similarly, find out shape factor F31. Now F41 x (A4) = - F2-1 x A2 ÷ F3_1 x A3 and
F21 can be found out.
1. Shape factor F4_1

= = = =
X 1 ' X 1 2
Hence, from graph F4_1 = 0.1.
2. Shape factor F3_1

Z _ _
X ' X —
From graph F31 = 0.2
366 Heat and Mass Transfer

Now F4_1 x A4 = F31 x A2 + F21 A3


0.1 X (1 X 2) = 0.2 x (1 x 1) + F21 X (1 X 1)
F21 = 0.2 - 0.2 = 0

104.Find F1_2 for configuration of two offset squares of area A as shown in figure-

A5

Let A5 = A2 ÷ A4, A6 = Al ÷ A3
A1 F12 = A6 F6_5 - Al F1_4 - A3 F32 - A3 F3_4
Due to symmetry A1 F12 = A3 F34
= A6 F6_5 - A3 F32 - Al F12
or 2 A1 F12 = A6 F6_5 - A3 F32
1 A E,
or F1_2 = 2A1 / 6-5 - A3 F32)
Now A i = A3 = A and A6 = 2A
1
F12 = 2 (2 x F6_5 - F32)
From Hamilton and Morgan graph,

F6 _5 (for = nd =- =0.25ndF32 ( — =1, — =1 = 0.2


)
F12) =0.5(2x0.25-0.2)=0.15

105.A hollow enclosure is formed between two infinitely long concentric cylinders of radii
1 m and 2 m respectively. Radiative heat exchange takes place between the inner
surface of the larger cylinder (surface-2) and the outer surface of the smaller
cylinder (surface-1). The radiating surface are diffuse and the medium in the
enclosure is non-participating. The fraction of the thermal radiation leaving the
larger surface and striking itself is-

Surface —1

Surface — 2
Radiation Heat Transfer 367

(a) 0.25 (b) 0.5


(c) 0.75 (d) 1
(GATE — 2008)

F11 ÷ F12 = 1
But F11 =0
F12 = 1
Now AiFi2 = A2 F21
or Or • di • L) F12 = (iv • d2 • L) F21
or F12 = 2 X F21
1
or F21 = — X F12 = 0.5
2
Now F22 ÷ F21 =1
or F22 ÷ 0.5 =1
or F22 = 0.5
Option (b) is correct.

106. Two parallel plates are at temperature T1 and T2 and have emissivities € 1 = 0.8 and
€ 2 = 0.5. A radiation shield having the some emissivity E3 on both sides is placed
between the plates. Calculate the emissivity E3 of the shield in order to reduce the
radiation loss from the system to one tenth of that without the shield.
(UPTU — 2009 — 10)

1
F g12 = 1 1 = 1 1
+ —1
E2 0.8 + 05 —1
1 1
= 0.444
1.25+2-1 2.25
1
As Qshield = — Q, hence, (Fg12),hield =—1 x Fg12 = 0.0444
10 10

1
(F g 12)shield =
1 + 1 -1)±i + -1)
E1 E3 J E3 E2 )

1
0.0444 =
1 1 1
0.8 E3
368 Heat and Mass Transfer

= .
1.25+2-2+ 1.25+
E3 e3
2 1
or 1.25 + = = 22.52
E3 0.0444
E3 1
or
2 = 21.27
or e3 = 0.094

107. Two concentric spheres of diameter D1 = 0.8 m and D2 = 1.2 m are separated by an
air space and have surface temperatures of T1 = 127°C and T2 = 27°C. Find the net
rate of radiation exchange between the spheres if: (i) the surfaces are black, and (ii)
the surfaces are diffuse and gray with E1 = 0.5 and E2 = 0.05.
(UPTU — 2008 — 9)
If surfaces are black, then Fg 12 = F12 = 1
Surface —1

Surface —2

Concentric spheres

If surfaces are gray, then


1
Fg 12 =
1—E1 1 1—E2 di
+ + X A2
E1 F12 E2 142
1

1-05 1 1-0.05 1.22
++ x
0.5 1 0.05 0.82
1
1+1+2.138

= 1 = 0.242
4.138
T1 = 127 + 273 = 400 K and T2 = 27 + 273 = 300 K
Radiation Heat Transfer 369

A = 4 Kr? = 4 x x x 0.42 = 2.01 m2


Qblack = a A Fg 12 = (T14 — T24)
= 5.67 x 104 x 2.01 x 1 x (4004 — 3004)
= 5.67 x (256 — 81) x 2.01
= 1994.42 W
Quay = 5.67 x 10-8 x 2.01 x 0.242 x (4004 — 3004)
= 482.65 W

108. A cylindrical cavity of diameter 10 cm and depth 20 cm is maintained at 60°C. Find


the heat transfer rate from this cavity to atmosphere at 30°C. Assume cavity and
atmosphere to be black bodies.
(UPTU — 2008 — 9)

d 10 10
F1_2 = = =
4h+ d 4x20+10 90
= 0.111
Q= Al X F1 _ 2 X a X (T14 — T24)

7rd2 )
= rdh+ x 0.111 x 5.67 x 10-8 x (3334 — 3034)
4

= n x 0.1x 0.2 + x (112 x 0.111 x 5.67 x 10_8 (123 — 84.3) x 10-8


( 4
= (0.063 + 0.008) x 0.63 x 38.7
= 1.73 W
Chapter 9
HEAT EXCHANGERS

KEYWORDS AND TOPICS


A HEAT EXCHANGER A DOUBLE PIPE HEAT EXCHANGER
A DIRECT CONTACT HEAT EXCHANGER A SHELL AND TUBE HEAT EXCHANGER
A INDIRECT CONTACT HEAT EXCHANGER A OVERALL HEAT TRANSFER COEFFICIENT
A FOULING A LOG MEAN TEMPERATURE DIFFERENCE
A PARALLEL-FLOW A HEAT CAPACITY
A COUNTER-FLOW A EFFECTIVENESS
A CROSS-FLOW A NUMBER OF TRANSFER UNITS
A EVAPORATOR A REGENERATOR
A CONDENSER A CORRECTION FACTOR

INTRODUCTION
Heat exchanger is a thermal device which is used for the heat transfer from hot fluid to cold fluid.
Temperature of each fluid changes as they pass through a heat exchanger. The heat exchanger
can be: (i) direct contact heat exchanger in which heat is exchanged between the fluids by coming
into direct contact, (ii) recuperator heat exchanger in which fluids are separated by a wall, and
(iii) regenerator heat exchanger in which each fluid is made to flow alternately or periodically.
The heat exchange in the heat exchanger may be occurring in the form of latent heat or sensible
heat or the combination of both. The heat exchangers are also classified as: (i) parallel flow, (ii)
counter flow, and (iii) cross flow depending open the direction of motion of fluids. Heat
exchangers are used in many applications such as: (i) boilers, (ii) automobile as radiators, (iii)
pasteurising plant as milk chillers, (iv) refrigeration plant as condensers and evaporators, (v)
water heaters, (vi) air heaters, and (vii) power plants as condensers.

1. What is a heat exchanger?

Heat exchanger is a device which is used in transferring heat from one fluid to another.
The objective of a heat exchanger is either to cool or heat a particular fluid by other fluid.
Heat Exchangers 371

Boilers, condensers, cooling towers, radiators and preheaters are common heat
exchangers used in industry. Heat exchange may take place in most of applications from
hot water to cold water or hot water to cold air or vice versa in a heat exchanger.

2. What do you understand from direct contact heat exchanger?

In direct contact heat exchanger, hot and cold fluids are made to come in direct contact
so that transfer of heat can take place. Such exchangers have to have fluids in different
states such as one is liquid and other is gaseous. Cooling tower of air-conditioning plant
is commonly used direct contact heat exchanger in which hot water is cooled by
surrounding air.
Hot water

Pump
Cold air Cold air

Cold water

Cooling tower: Direct contact heat exchanger

3. What do you understand by indirect contact heat exchanger?

Hot and cold fluids do not come in contact in indirect contact heat exchanger. Hot and
cold fluids through the exchanger and heat transfer takes place through the walls
separating the flow of the fluids. Such type of indirect contact heat exchangers are called
regenerators. There is other type of indirect contact heat exchangers which are called
recuperators. In this type hot and cold fluids flow alternatively through the same space.
The heat is accumulated during the flow of hot fluid in the walls of the exchanger is
transferred to the cold fluid when it is made to flow through the exchanger after the flow
of hot fluid.

4. What is fouling? How is it taken care of?


(UPTU — 2007 — 8)
The walls separating the fluids in a heat exchanger do not remain clean after the exchanger
has been in use for some time. The walls become fouled with deposits resulting from
impurities in fluids and rust formation. Due to fouling the thermal resistance in the transfer
of heat from hot fluid to cold fluid increases and overall heat transfer coefficient of the
exchanger decreases. In order to take care of this adverse effect of fouling, an additional
thermal resistance called fouling resistance or fouling factor is used while designing the heat
exchanger.

5. How can heat exchanger be classified according to flow arrangements of fluids in the
heat exchanger?
372 Heat and Mass Transfer

or
Show temperature distribution in counter-flow and parallel-flow heat exchangers.
(UPTU — 2002 — 3)
The flow arrangement of fluids in the heat exchanger can be—
(a) Parallel-flow
(b) Counter-flow
(c) Cross-flow
Parallel-flow: In parallel-flow heat exchangers, hot fluid and cold fluid flow in the same
direction inside the exchanger. The temperature difference of the fluids of the inlet of the
exchanger is maximum and temperature difference reduces as fluids flow through the
exchanger. The temperature difference is minimum at the outlet at the exchanger.

Fluid-1 => c> Fluid-1


(hot)

Fluid-2 => 1=> Fluid-2


(cold)

Th1
lot fluid

AT1 The AT2


C2
Cold fluid
Tc1
Distance from inlet
Parallel—flow

Counter-flow: In counter-flow heat exchanger, hot fluid and cold fluid are made to flow in
opposite or counter directions. The inlet of hot fluid and outlet of cold fluid is at one end
while the outlet of hot fluid and inlet of cold fluid is at other end of the exchanger. The
difference of temperatures of hot and cold fluids remains almost constant throughout the
length of the exchanger.

Fluid-1 E Fluid-1
(hot) (hot)

Fluid-2 <=I Z' Fluid-2


(cold) (cold)

Distance from inlet


Counter—flow
Heat Exchangers 373

Cross-flow: In cross-flow exchanger, hot fluid and cold fluid are made to flow at right angle
to each other in the exchanger.

Fluid-2

Fluid-1 Fluid-1

Fluid-2

6. What do you understand from evaporator and condenser?

Evaporator: As the name suggests, cold fluid is evaporated at constant temperature by using
the heat of hot fluid. Temperature of hot fluid decreases from inlet to outlet.

—.- Distance from inlet


Evaporator

Condenser: As the name suggests, hot fluid is condensed at constant temperature by


transferring the heat to the cold fluid. The temperature of cold fluid increases from inlet to
outlet.

Condenser

7. Explain construction features of double pipe heat exchanger and shell and tube heat
exchanger.

Double pipe heat exchanger: Double pipe heat exchanger consists of two concentric pipes
and each pipe is carrying one of the fluids so that heat transfer can take place.
Shell and tube heat exchanger: In shell and tube heat exchanger, a bundle of circular tubes
are mounted in a cylindrical shell parallel to the cylindrical axis. Heat exchange takes place
when one fluid flows in tubes while other fluid flows through the shell. Baffles are provided
374 Heat and Mass Transfer

to guide the flow of fluid in the shell and also to generate turbulence in the flow for better
heat transfer. The shell can be one or two and tubes can have 2, 4, 6, etc. passes. The heat
transfer area increases with number of tube passes.

Fluid-1

Fluid-2 =>

Double pipe heat exchanger

Tube inlet Shell outlet

Tube
header Tube header
Tube outlet
Shell inlet
Shell and tube heat exchanger

8. What are various thermal resistances in the path of heat flow from hot fluid to cold
fluid in a double pipe heat exchanger? Find the overall heat transfer coefficient (u)
of the heat exchanger.

The thermal resistances in the path of heat flow from hot to cold fluid in a double pipe heat
exchanger are—

(a) Convection resistance for heat flow at inside surface of the inner tube = 1
hi Ai

log do
(b) Conduction resistance for heat flow as imposed by the inner tube material =
2irkL

(c) Convection resistance at outer surface of the inner tube = 1


ho Ao
(d) Scale resistance due to deposit of scales at inner and outer surface of the inner tube
-
Ao
Heat Exchangers 375

////////////////////////z/zz zz/
di
//i.///.///////////////////%71///!
`/

Double tube heat exchanger

Total thermal resistance in the path of the heat flow is—


do
log
Fi Fo
ER = 1 + di.+ 1 + +
hi Ai 2 xkL Ito Ao Ai Ao
The overall heat transfer coefficient based on outside surface area of inner tube can be given
as—
1
uo =
A0 ER
1
do
log
A_ [ 1 + di + 1 + Fo
+F`
u hi Ai 2 xkL ho Ao Ai Ao

Now Ai = irdiL & Ao = irdoL

tio =
1
do ( do 1 do
— + — log do + — + — x F + Fo
di hi 2k di ho di

9. What are the parameters on which the overall heat transfer coefficient of a heat
exchanger depends?

The overall heat transfer coefficient depends upon—


(a) Flow rate of both fluids in the exchanger.
(b) Properties of the fluids on which heat transfer coefficients 'hi' and 'Ito' depend.
(c) Thickness of inner tube.
(d) Inside and outside surface conditions of the inner tube.
(e) The geometric configuration of heat exchanger.

10. What is log mean temperature difference (LMTD) for a heat exchanger?

The temperature difference between the two fluids continuously changing from the inlet to
the outlet of the heat exchanger. This difference has to be averaged over the entire length
376 Heat and Mass Transfer

of heat exchanger so that heat transfer rate can be evaluated by simply multiplying surface
area and overall heat transfer coefficient of the exchanger to the LMTD (A TO
Q= u • A • AT,„

11. How is the analysis of heat exchanger done?

The analysis of heat exchanger is done by—


(a) Finding heat transfer rate from LMTD
Q = uAAT,„
(b) Funding heat transfer rate from mass flow rate of hot fluid and difference of
temperature at inlet and outlet of the exchanger
Q = rhh x Ch X (Thi — Th2)

(c) Finding heat transfer rate from mass flow rate of cold fluid and difference of
temperature at outlet and inlet of the exchanger
Q = th, x Co x (T,2 — Tod

12. Derive an expression for LMTD of parallel-flow heat exchanger?

Temperature distribution in parallel-flow for hot and cold fluid is as given below.

AT1

dA
-1.- Distance from inlet
Parallel-flow

Consider an elemental area VA' of heat exchanger and heat transfer dQ from elemental
area is—
dQ = u • dA • AT (i)
Now equating this heat `dQ' to heat transfer from hot fluid is—
dQ = —rhh x Ch dTh

Also equating this heat transfer to heat transfer from cold fluid is—
dQ = —rho x Co x dTh
Heat Exchangers 377

Now total 'AT' can be equated as—


A T = Th Tc

On differentiating 'A T', we get—


d AT = dTh — dTc
On putting the value of `dTh' and `dTc' from equations (ii) and (iii)
dQ dQ
d AT —
MhCh Mc Cc

1
= —dQ x R where R =
MhCh M Cc
Now putting the value of `d Q' from equation (i)
d AT = —u x dA x AT x R

or AT = u x R • dA

For complete surface of the exchanger, and temperature variation of hot and cold fluid
at inlet (ATI) and outlet (AT2), we have on integration-
2 A
dAT __
uxRx fdA
AT
er,
AT2
or log = uxR xA
ATI
Putting the value of R, we have—
AT2 [ l + 1 (iv)
log = —uA ff Mc Cc
ATI
Now total heat transfer for the exchanger is—
Q = —Mh Ch(Thi — Th2) = McCc(Tc2 — Tc1)

or MhCh = and MS C, =
(Thi — Th2) Tc2 Tc,
Putting the value of MhCh and Mc Cc in equation (iv), we have—
uA
log AT2 — —Q [(Th1 — Th2) + (Tc2 — Tc1)]
ATI
+uA
[(Th1 — Tci ) — (Th2 — Tc2)]
378 Heat and Mass Transfer

uA
= — [ATi — AT2]
Q
ATI -AT2
Q = uA
AT
log 1
AT
= uA AT,„
ATI — AT2
where AT ff, = LMTD — AT.
log AZ

13. Derive an expression for LMTD for counter-flow heat exchanger.

Temperature difference curve for hot and cold fluid is as shown below.

—..-1 dA I-4—
Distance from inlet

It can be seen that unlike parallel-flow, ATi and AT2 are—


ATI = Thi — Tc2
AT2 = Th2 — Tc1
We can work out as for parallel-flow—
[ATI— AT2
Q = uA = uA AT,„
log ATI
AT2

ATI -AT2
where AT. = LMTD — AT
log AT2

The expression for LMTD for parallel and counter-flow is same but value of A Ti and AT2
is different in each case.

14. What happens to LMTD expression in case ATI. = AT2? How to overcome this
problem?
Heat Exchangers 379

I
LMTD = AT — Al
A
log TI
AT2
when A — AT2 = 0, then-
LMTD = ° — i.e. LMTD is indeterminate.
log 1 0
This type of problem can be solved by employing 'V Hospital's rule and LMTD becomes
average of inlet and outlet temperature differences.
I + AT2
LMTD — AT
2

15. Explain the method of finding LMTD for multi-pass and cross-flow heat exchangers.

Analytical method of finding LMTD for multi-pass and cross-flow heat exchanger is
very complex and difficult to solve. Hence, LMTD for such heat exchangers is found
out by using LMTD of single pass counter-flow heat exchanger with correction
factor (F).
Q= F(u • A • ATift)
where A T ff, = Log mean temperature difference of counter-flow double pipe heat exchanger
ATI — A T2
log AT1
A ,
L112

F = correction factor
Expressions for the correction factor (F) for various cross-flow and shell-and-tube
arrangements have been developed. A most convenient method of representing the
correction factor (F) is to present it in the form of graphs. In these graphs, following
motions are used—
(a) Subscript 's' is used for shell fluid and subscript 't' is used for tube fluid.
(b) Subscript '1' and '2' for inlet and outlet. Hence, T51 and Ts2 are inlet and outlet
temperatures for fluid in the shell while T51 and Tt2 are inlet and outlet temperatures of
fluid in the tube or tubes.
(c) Letters 'IV and 'P' stand for
T —T
R= s2 and P = Tt2 7;1
Tt2 Tti Ts,— Tt,
The correction factor graphs are drawn taking correction factor on y-axis and `P'
on x-axis for various 'IV values. Three graphs are reproduced for different heat
exchangers.
380 Heat and Mass Transfer

Correction Factor

1.0
Correction factor (F)

0.9

0.8 • I r I = Tt2 -Ttl


P Tsl -Tti
0.7 • t
Tel -
0.6 R= rr
t2- Tt
0.5
0.1 0.2 0.3 0.4 0.5 0.6 0.7 0.8 0.9 10
p ►
One shell pass and 2, 4, 6 - tube passes
Correction factor (F) -Y.

0.4 0.5 0.6 0.7 0.8 0.9 1 0


P
Two shell pass and multiple tube passes

1.0
Correction factor (F) —]..-

0.9

0.8
Tel - Tt1
0.7 Cold fluid

0.6 Hot fluid


0.5
0.1 0.2 0.3 0.4 0.5 0.6 0.7 0.8 0.9 .0 ,1
P -.-
Cross-flow heat exchanger

16. What is the value of correction factor for: (i) counter-flow heat exchanger, (ii) cross-
flow heat exchanger, and (iii) multipass heat exchanger.

The values of correction factor are-


1. Counter-flow heat exchanger: F = 1
2. Cross-flow heat exchanger: F < 1
3. Multipass heat exchanger: F < 1
Heat Exchangers 381

17. What do you understand from heat capacity of a fluid?

The product of mass and specific heat of a fluid is called heat capacity.
Heat capacity = C = thc

18. What is the effect of heat capacity on temperature difference while heating and
cooling?

Heat capacity C = m x c
Q=rilxcxAT
= C x dT

or Q
dT = —
For same heat transfer (Q), large heat capacity leads to smaller temperature difference
of the fluid from outlet to inlet (d7). The temperature difference of the fluid from outlet to
inlet (A T) is larger for smaller heat capacity of the fluid.

> C2 > C3
AT3 > AT2 >ATI
AT3 > AT2 > (Tc2)3

ATe
(T,2)2
(T,2)1 AT3
AIT2
(Th2).1
T AT 2
AT3 1,[
(Th2)2
Cl
(Th2)3

Distance from inlet —0-- Distance from inlet


Cooling of fluid Heating of fluid

It is apparent that the value of (Th1 — Th2) is maximum in case hot fluid has minimum
heat capacity and (Th1 — Th2) is minimum in case hot fluid has maximum heat capacity.
Similarly, (Tc2 — Tc1) is maximum in case heat capacity of cold fluid is minimum and
(Tc2 — Tc1) is minimum in case heat capacity of cold fluid is maximum.

19. What happens in a counter-flow heat exchanger of infinite area when heat capacity
of cold fluid (G) is minimum?

Here Ch> Cc and cold fluid will have a large difference of temperature from outlet to inlet
i.e. (Tc2 — Tc1) is very large resulting in the outlet temperature of cold fluid attaining the inlet
temperature of the hot fluid in the counter flow (Th1 = Tc2)
The maximum rate of heat transfer is—
Qmax = Cc x (Thi — Tci)
Since Cc < Ch, we can say Cc = Cmin
Qmax = Cmin X (Thi Tc1)
382 Heat and Mass Transfer

Distance from inlet


Counter-flow heat exchanger

In case heat exchanger does not have infinite area, the temperature Tc2 does not approach
Thi and heat transfer is—
Q = Cc(Tc2 — Tc1)

20. What happens in a counter-flow heat exchanger of infinite area when heat capacity
of hot fluid (Ch) is minimum?

Here Ch < Cc and hot fluid will have a large temperature difference from inlet to outlet i.e.
(Thi — Th2) is very large resulting the outlet temperature of hot fluid attaining the inlet
temperature of the cold fluid in the case of the counter-flow heat exchanger (Th2 = Tc1)

Th1

= Th2

► Distance from inlet


Cross-flow heat exchanger

The maximum rate of heat transfer is—


Qmax = Ch(Thi — Th2)
But Th2 = Tc1
Qmax = Ch(Thi — Tc1)
= Cmia(Thi — Tc1) here Ch = Cmin
In case heat exchanger does not have infinite surface area, the temperature Th2 does not
approach Tc1 and heat transfer is
Q = Ch(Thi — Th2)
Heat Exchangers 383
21. What is effectiveness of a heat exchanger?

The effectiveness of a heat exchanger (E) is defined as the ratio of actual rate of heat
transfer to the maximum possible of rate of heat transfer
Effectiveness (E) = Actual rate of heat transfer
Maximum possible rate of heat transfer
Actual rate of heat transfer is—
Q = Ch (Thi — Th2) = Cc(T,2 — Tc1 )
Maximum rate of heat transfer is—
Qmax = Cmin(Th1 Tc1)
Here Cmin = Ch in case Ch < Cc
= Cc in case Cc < Ch
Effectiveness can also be given as—
Ch(Thi — Th2 ) (Tc2
E=
Cmin (Thi Cmin(Th, — Tc1 )

22. What is the advantage of using effectiveness of heat exchanger?

The effectiveness is defined as—


= Q
- max
or Heat transfer Q = E X Q,
=EXC • X(Th1 —Tc1)
The heat transfer depends upon—
(a) effectiveness
(b) minimum heat capacity of the fluids
(c) inlet temperature of hot and cold fluids which are known.
As there is no requirement to find out outlet temperatures of hot and cold fluids,
specifying effectiveness provides a simpler method to evaluate heat transfer by the heat
exchanger. Hence, this method can effectively replace the LMTD method where the outlet
temperatures of hot and cold fluids are unknown.

23. What do you understand by NTU (number of transfer units)?

The NTU (number of transfer units) is the ratio of the heat capacity of the heat exchanger
to minimum heat capacity of the fluid flow
Heat capacity of the exchanger
NTU =
Minimum heat capacity of the fluid flow
uxA
Cmin
384 Heat and Mass Transfer

where u = overall heat transfer coefficient


A = area of the heat exchanger
Clnin = (ri2C) Inin
or NTU oc A as u and C • are constants
Hence, NTU is a dimensionless number and it is a measure of heat transfer area of the
exchanger (size of exchanger). If NTU is large, then larger the heat transfer area is required
for the heat exchanger. In other words, if larger is the value of NTU, then closer is the heat
exchanger reaches its full effectiveness of operation.

24. Find the expression for effectiveness of a parallel-flow heat exchanger.


Or
Show that for parallel-flow heat exchanger-
uA (1+ C„,h,)
1 exp
C„,h, C„,..
e—
1 + Cmin
C.
(UPTU — 2003 — 4)
In parallel-flow, temperature distribution is-

dTh

2 AT2
Tc2 t

1-t— x-1-1 dx F4— Area


Parallel-flow

Consider heat transfer between the cold and hot fluids for a differential element of length
dx, we have—
dQ = —1:11hChdTh = —Ch dTh where Ch = thhCh
dQ
or dTh = —
Ch

Similarly, dT = —
c CcQ

d(Th — Tc) = —dQ


l l (i)
Ccc ch
But dQ = u x dA x AT
where A T = Th — Tc
dQ = u x dA x (Th — Tc)
Heat Exchangers 385

Putting the value of dQ in equation (i), we get-


d(Th - Tc) = -u x dA x (Th - Tc) x H +
ch Cc
2 \
or
a (Th— ic)
= u 1 + jx f dA
Tc -Tc 1-12 1,c

or log The -T`2 = -uA 1 + f.,1


Thi - Tc, Ch ,,,c
Now let Ch = C • and C, = C

. log The — Tc2 = — uA 1+ 1


Th, — Tc, Cmin Cmax

-uA + Cm.)
Cmin I\ Cinax

But as per definition: NTU = uA


Cmin

log
— TC2 =
NTu (1 +
C.
Th, — Tc, Cmax
— Tc2 11
or The = exp [ NTU (1 + )1
Th, - Tc, Cmax
Now effectiveness of exchanger is
Ch(Th, Cc(Th2 -
E =
Cmin (Th, Tc,) Cmin (Th, — Tc, )
E Cmin (Thi— Tc1)
Tc2 = Thi
Ch
E Cmin (Thi— Tc1)
and Thi = Tci +
Now substituting the value of Tc2 and Th2 in equation (iii), we have

Th, Tc, c' h

1 [(Th. T ) E =
Cmiexn (pT[hi NTTc1)U + C+, rn;))11
c ax

or [1+ E Cmin I 1 +1
1, ; = exp -NTU 1+ Cmin 11
L•11 %-• J J Cmax JJ
386 Heat and Mass Transfer

1— exp [— NTU(1 + Cmin )1


max
or
Cinin +)
lCh Cc
But as per assumption, Ch = Cmin & Cc = Cmax

1— exp [— NTU(1 + cCmin )1


max
E—
l
c

Note: In case Ch > Cc, then Cc = C • and Ch = C and effectiveness is

1— exp[—NTU(1+ C
c," )1
min
E—
c 1 + 1
rni
n Cmax

25. What is capacity ratio? Write the expression of effectiveness of a parallel-flow heat
exchanger in terms of NTU and capacity ratio.
The dimensionless parameter giving the ratio of C • to C is called the capacity ratio.
C •
Capacity ratio = R = mm
max
Now the effectiveness of a parallel-flow heat exchanger is
1— e-N(1+ R)
E =
1 +R
where N= NTU and R= capacity ratio.

26. Find the expression for effectiveness of counter-flow heat exchanger.


The variation of temperature is-

4 Th —4—
ATi
T C2
z
Th2
AT2
Tc1
dTc

x —A- dx Area —0-


Counter-flow
Heat Exchangers 387
dQ = u • dA(Th — Tc)
Also dQ = —thhchdTh = me cc dTc
= —Ch dTh = —Cc dTc
—dQ —dQ
dTh = and dT =
Ch c G
or d(Th — Tc) = —dQ
(1 1
Ch Cc

= dQV —
Now putting the value of dQ—

d(Th — Tc) = u X dA x ( 1 1 (Th — Tc)


Ch Cc
2 j,„,
or
akih — ic)
= u (1 1
X dA
Th—Tc "
Ti.,z — 1 1
or log Tc1 =uxAx ( cc )
Tit, — Tr z Ch
Do observe that AT2 is now Th2 — Tci and ATI = Th1 — Tc2 as flow is counter

or Tk — Tc1 = exp [uA (— ci — c—


l-,
Th, — Tc2 c h
Now effectiveness is—
Ch (Th, — Cc (Tc2
E=
Cmin — Tc,) Cmin (4, — Tc1 )
E Cmin (Thi Tc1 )
Th2 = Thi
Ch
E Cmin (Thi
Tc2 = Tc1
Cc
Substituting the values of Th2 and Tc2
Cmin ( Th, — Tc,
E T,
[ Th,—
Ch
\ —Exp[uAH — - )1
E Cmin(Th, — Tc,)] Cc Ch
Thi —[ Tc,+
Cc

E X Cmin
1
— Ch
or (T — Tc,) 1 E Cmin
= Exp [uAH
Cc Ch
)1
i
Cc
388 Heat and Mass Transfer

EX C •
1
or Ch
=Exp
[uA ri _ cc )] (i)
1
E Galin cc, Ch )
Cc
uA
1. Case 1: In case Ch > Cc, then C, = C and Ch = C and „ - NTU = N, and
Cmin

R = Cmin - heat capacity. The equation (i) becomes-
max
1— E R
- e N(1- R)
1— E
1- e N(1-R)
or E =
R- eN(1-R)
N(1-R) _ 1
e
N(1-R)
e -R
2. Case 2: In case Cc > Ch, then Ch = Cmin and C, = Cm. Equation (i) becomes
1- E X Cmin
Cmin
= Exp [-N (1 Cmin j]
1 E Cmin Cmax
Cmax
N(1-R) _ 1
e
or E = N(1-R) R
e

27. What is the value of effectiveness of a condenser?


In a condenser, the hot fluid condenses at constant temperature. Hence, we have-
Ch = = Q — = oc = C max
Thi -Th, 0
as Thi = Th2 = condensing temperature.
Cmin = 0
Capacity ratio R = Cmin
Cmax oc

Th1

Condenser

1-e -N(l+R)
Now E= for parallel-flow
1+ R
Putting R = 0, E = 1 — e —N
Heat Exchangers 389

N (1— R)
e
Similarly, E= N (1— R) R for counter-flow
e
Putting R = 0, E=1 — e —N

Hence, for both types of flow, we have effectiveness of a condenser as—


E=1—

28. What is the value of effectiveness of an evaporator?

In an evaporator, cold fluid evaporates at constant temperature.

Thi

Th2

Tci T 2

Evaporator

Q =cc(Tc2—Tc1)
or C — =C as Tc2 = Tcl
c = Tc,—Tc, 0
R = Cinin = Cinin = 0
oc
E=1 — e —Isi both for parallel and counter-flow.

29. What is the value of effectiveness for a regenerator?


In case of regenerator, C • = Cm

R = Cmin — 1
C„„,,
Hence, for parallel-flow regenerator-
1 — e -2N
E=
2
For counter-flow regenerator if we put R = 1
1— e — N(1-1)
E= —N(1-1) 1-1 or it is
indeterminate-
1—1xe 1—1 0
To find the value of effectiveness, L. Hospital's rule has to be used for regenerator when
R —> 1
[i—eNG-R1
aR
E = Limit R —> 1
a [1— Re-(1-R)]
N
aR
390 Heat and Mass Transfer

Ne-N(1-R)
=
Re-N(1-R) x (—N)
—N
—1—N

1+N

30. How can effectiveness of heat exchangers be found out by graphical method?

The effectiveness of heat exchangers can be found out from the graphs prepared by Kays
and London. The graphs are plotted for NTUm versus effectiveness (E) for different heat
capacities (R).
Following graphs are included—
(a) Effectiveness for parallel-flow heat exchanger
(b) Effectiveness for counter-flow heat exchanger
(c) Effectiveness for 1 — 2 parallel counter-flow heat exchanger
(d) Effectiveness for two shell passes and 4, 8, 12 tube passes
(e) Effectiveness for cross-flow heat exchanger with both fluids unmixed
Hot fluid

-0- Cold fluid


100

1 80
Effectiveness

60

40

20

0
0 2 3 4 5

NTU [ UA ]
cmm
Parallel-flow heat exchanger
Heat Exchangers 39 I
Hot fluid

Cold fluid ..
100
. .
cii,inicmax 7 0.25
0.50
80 0.75
1.00
Effectiveness

60

40

20

1 2 3 5
NTU
Counter-flow heat exchanger

Shell fluid
F Tube fluid

One shell
one shell pass
2, 4, 6 etc, tube passes

2 3 4 5
NTU
One shell and 2, 4, 6 etc, two passes exchanger
392 Heat and Mass Transfer

100
amidCmax = 0 !2 ...........

80

1 60 Shell fluid

a) Tube fluid
Ii
w
40 I
2 Shells

20 I Two shell passes


4, 8, 12, etc., tube passes

1 2 3 4 5
NTU
Two shell passes and 4, 8, 12, etc., tube passes heat exchanger

100

80

Cold fluid

Hot
fluid

40
w

20

1 2 3 4 5
NTU
Cross-flow heat exchanger
Heat Exchangers 393

31. A counter-flow heat exchanger is used to heat water from 20°C to 80°C by using hot
exhaust gas entering at 140°C and leaving at 80°C. The log mean temperature
difference for the heat exchanger is—

(a) 80°C (b) 60°C


(c) 110°C (d) not determinable as zero/zero is involved
(IES 96)
Temperature distribution is

80

AT2

20

AT1 = Thi — Tc2 = 140 — 80 = 60°


A T2 = Th2 — Tc1 = 80 — 20 = 60
ATI — AT2
ATff, = LMTD =
AT
log A 1
L1T2
60 — 60 0
Indeterminate and hence—
log 60 0
60
ATI + AT2 60 + 60
LMTD = 60
2 2
Option (b) is correct.

32. A counter-flow shell and tube exchanger is used to heat water with hot exhaust gases.
The water (c = 41 80J/kg°C flows at a rate of 2 kg/s while the exhaust gas (1030 J/
kg°C) flows at the rate of 5.25 kg/s. If the heat transfer surface area is 32.5 m2 and
the overall heat transfer coefficient is 200 W/m2°C, what is the NTU for the heat
exchanger?

(a) 1.2 (b) 2.4


(c) 4.5 (d) 8.6
(IES 95)
ux A
NTU =
Cmin
Cc = thw X Cwater = 2 x 4180 = 8360 .1/°C
Ch = rith x ch = 5.25 x 1030 = 5407.5 J/°C
Now Ch < Cc
Cmin = Ch = 54075 J/°C
394 Heat and Mass Transfer

200 x 32.5
NTU = = 1.2
5407.5
Hence, option (a) is correct.

33. Water at the rate of 1 kg/s is heated from 35°C to 75°C by oil in a double pipe
counter-flow heat exchanger. Oil has a specific heat of 1.9 kJ/kgk and it enters the
heat exchanger at 110°C and 2.5 kg/s. The overall heat exchanger coefficient is 300
W/m2K. Calculate the area of heat exchanger tube.
(UPTU — 2002 — 3)
Temperature distributions

—4— Th1
AEI
7-,2 The--
AT2
Tci

Applying energy balance—


Heat lost by hot oil = Heat gained by cold water
inhCh(Thi — Th2) = me cc(Tc2 — Tc1)
2.5 x 1.9 (110 — Th2) = 1 x 4.8 (75 — 35)
Th2 = 74.8°C
Now A Ti = Thi — Tc2 = 110 — 75 = 35
and A T2 = Th2 — Tc1 = 748 — 35 = 39.8
2 — ATI _ 398.35
LMTD =Tm = AT
AT2 — log 39.8
log
ATI 35
= 37.4°C
Now Q = 1 x 4.2 x (75 — 35)
= 168 ld
Now Q =uxAxAT,„

or A—
u x AT„,

168x103
300 x 37.4
= 14.97 m2

34. In a counter-flow heat exchanger, water is used as a coolant for hot oil from a
temperature of 160° to 60°C. Water enters the heat exchanger at a temperature of
25°C. The mass flow rates of water and oil are each 2 kg/s. The diameter of the tube
Heat Exchangers 395
is 0.5 m and the overall heat transfer coefficient is 250 W/m2K. Calculate the length
of the heat exchanger. Take specific heat for oil and water as 2.035 and 4.187 14/kgk.
(UPTU — 2003 — 4)
Temperature distribution is—

rhi
ATI
c2 2
T AT2

ci t

Counter-flow

First is to find the outlet temperature of the water


Heat lost by oil = Heat gained by water
2 x 2.035 x (160 — 60) = 2 x 4.187 x (Tc2 — 25)
Tc2 = 73.6°C
AT, = Thi — Tc2 = 160 — 73.6 = 86.4°C
AT2 = 60 — 25 = 35°C
AT, — AT2
Now ATff, = LMTD —
AT
log AT2

86.4 — 35
log 86.4
35
= 57°C
Now Q= u • A • AT„,
= u • Or • d • 1) • AT.

_
u•ir • d • AT„,
2 x 2.035 x 103 x (160 — 60)
250x x 0.5 x 57
= 18.2 m

35. 1000 kg/hr of oil (C,, = 2.09 kJ/kg°C) is to be cooled from 80°C to 400°C in oil cooler
by using water flow of 1000 kg/hr at 30°C. Give your choice for a parallel-flow or
counter-flow heat exchanger, if the overall heat transfer coefficient is 24 W/m2C. For
water Cp = 4.18 kJ/kg°C.
(UPTU — 2004 — 5)
396 Heat and Mass Transfer

First to find out outlet temperature of the water


Heat lost by oil = Heat gained by water
1000 x 2.09 x (80 — 40) = 1000
3600 x 4.18 x (Tc2 — 30)
3600
Tc2 = 50°C

80 —1— 80
ATI
50 40
T 5° AT2
40 30 I

30

Parallel-flow Counter-flow

Since outlet temperature of cold fluid is higher that outlet temperature of the oil, hence
parallel flow is impossible. Therefore counter-flow cooling is possible
ATI = 80 — 50 = 30
AT2 = 40 — 30 = 10
ATI — AT2
LMTD = ATff, = AT
log '
AT2

30 —10
log
10
= 18.2°C
Now 1000
3600 x 2.09 x 103(80 — 40)
Q=
Also Q = u • A • AT.

A—
u • AT„,
10
00 x 2.09 x 103 x 40
_ 3600
24 x 18.2
= 53.2 m2

36. A double pipe counter-flow heat exchanger is used to cool 10,000 kg/h of an oil
(C,, = 2095 Joules/kg k) from 80 to 50°C using 8000 kg/h water (Cp = 4180 J/kg k)
entering at 25°C. Find the heat exchanger area required with an overall heat transfer
coefficient is 300 W/m2.
(UPTU — 2005 — 6)
Heat Exchangers 397

First to find outlet temperature of water.


Heat lost by oil = Heat gained by water
10,000 8000
x 2095 x (80 — 50) = 360o x 4180 x (Tc2 — 25)
3600
. Tc2 = 43.8°C

k 80
AT1
1-43.8
50
AT2
25

Counter-flow

ATi = 80 — 43.8 = 36.2°C


AT2 = 50 — 25 = 25°C
ATI — AT2
Now LMTD = ATff, =
AT
log 1
AT2
= 36.2 — 25
log36.2
25
= 30.3°C
Now Q =uxAxAT,n
Q
or A—
ux AT,n
10,000 x 2095 x (80 — 50)
3600
300 x 30.3
= 19.2 m2

37. Water is to be heated from 20°C to 90°C by using a single pass heat exchanger. For
this purpose, saturated steam at 120°C is condensing on the outer tube surface of
a single pass heat exchanger. The heat transfer coefficient is 1800 W/m2K. Calculate
the surface area of a heat exchanger capable of heating 1000 kg/h of water from 20°C
to 90°C. Also calculate the rate of condensation of steam. Take hfg = 2200 kJ/kg.
Guidance: The temperature of steam (hot fluid) in the heat exchanger remains constant
at 120°C and heat is passed to cold fluid i.e., water by condensing steam (i.e. latent heat
of evaporation.)
398 Heat and Mass Transfer

Hot fluid

120°C 120°C —
AT2
ATI 90°C —

Cold fluid
J— 20°C

Parallel flow

Now AT1 = 120 — 20 = 100°C


AT2 = 120 — 90 = 30°C
ATI — AT2
LMTD = —
ATI
log
A2
100 — 30
log 100
30
= 50.14°C
Now Q =uxAxAT,„

or A=
u • AT„,

1000 x 4180 x (190 — 20)


_ 3600
1800 x 50.14
= 0.776 m2
Heat is lost by steam by the way of condensing thereby losing heat of evaporation (hfg).
If m, is steam which has condensed, then
1000 x 4180 x (90 — 20)
m5 = Q = 3600
hfr 2200
= 37 x 10-3 kg/s

38. In a certain double pipe heat exchanger, hot water flows at a rate of 5000 kg/hr and
gets cooled from 96 to 65°C. At the same time 50,000 kg/hr of cooling water at 30°C
enters the heat exchanger. The flow conditions are such that the overall heat transfer
coefficient remains constant at 2270 W/m2K. Determine the heat transfer area
required and the effectiveness, assuming two streams are in parallel flow. Assume
for the both specific heat = 4.2 kJ/Kg k.
(GATE — 97)
First to find outlet temperature of cold fluid.
Heat Exchangers 399

Heat lost in cooling hot fluid = Heat gained in cold water


0 x 4.3 x (95 — 65) — 50,000 x 4.2 x (Tc2 — 30)
3600 3600
or
Tc2 = 3° + 10°
= 33°C

65
TAT2
33 -

Parallel-flow

6,T1 = 95 — 30 = 65
6,T2 = 65 — 33 = 32
65 — 32
LMTD = 6,7,„ = 65 = 46.6°C
log 32

5000 x 4.2 x (95 — 65) = 175 x 103 W


Q 3600
Now Q= u • A • AT,„
or A— Q
u • AT„,
175 x 103

2270 x 46.6
= 1.65 m2
Now Q. will be when hot fluid is cooled to the inlet temperature of cold fluid
Q. = mh Ch(Thl — Tc1)
= x 4.3 x 103(95 — 33)
3600
= 361.7 x 103 W
Qaman 175 x 103
E=
Qmax 361.7 x 103
= 0.48

39. A very long, concentric tube heat exchanger is having hot and cold water inlet
temperature of 85°C and 150°C. The flow rate of the hot water is twice than that of
the cold water. Assuming equivalent hot and cold water specific heats, determine the
400 Heat and Mass Transfer

water outlet temperatures and the heat exchanger effectiveness for the following
modes of operation: (i) counter-flow, and (ii) parallel-flow.
(UPTU — 2007 — 8)

Guidance: The heat exchanger is very long and two situations arise depending upon type
of flow. In case of parallel-flow, both liquids will come out at the same temperature `T'.
In case of counter-flow, the fluid having low heat capacity will have larger temperature
difference from inlet to outlet. Since cold water in this problem has lower heat capacity,
hence it will have larger temperature difference, resulting cold water to attain the inlet
temperature of hot fluid.

85°C 85

rc 2

15°C

Parallel-flow Counter-flow

1. Case 1: Parallel flow


heat lost = heat gained
m h x ch x (Th i = rhc cc (T Tc1)
2 x cw (85 — 7) = cw(T — 15)
3 T = 185
T = 61.67°C
Q 2 Cc (85 — 61.67)
E=
Qmax G(85 —15)
2 x 23.33
= 0.67
70
2. Case 2: Counter-flow
heat lost = heat gained
mh • Ch (1' hi — Th2) = me Cc(Tc2 Tc1)
2 cw(85 — Th2) = cw(85 — 15)
or The = 85 — 35
= 50°C
Q 2 Cc (25 — 50) 2 x 35
E=
Qma. G (85 —15) 70
=1

40. If heat capacities of hot and cold fluids are equal (thhch = cc) in a counter-flow heat
exchanger, show that ATI. = AT2 = AT at any section and temperature profiles of the
fluids are linear and parallel.
(GATE — 96)
Heat Exchangers 40 I

The temperature profile of a counter-flow heat exchanger is—

H— x —0-1 dx H
Counter-flow

For differential element of heat exchanger, heat flow is—


dQ = —MhChdTh= —Mc Cc dTc = u x dA x AT
But thhCh = thcCc
.% dTh = dTc
or d(Th — Tc) = 0
or Th — Tc = constant
. ATi = AT2 = AT = constant
Now —mh x ch x dTh = u x dA x AT
dTh —u AT
or = = constant
dA nth ch
—u AT
Similarly, = = constant
dA
dA thc cc
dT
As ATi = AT2 = AT and = constant for fluids temperature profiles, hence these lines
dA
are linear and parallel.

41. If ATI. = AT2 in a counter-flow heat exchanger, show AT„, = ATI. = AT2.

i — AT2
AT. = LMTD — AT
ATI
log
AT2
Assume ATi = c i.e. a constant temperature, ATi — AT2 = x
AT2 = c — x i.e. it is variable
c —(c — x) x
AT. = =
log c log c
c—x c—x
Now as ATi —> AT2, x —> 0
402 Heat and Mass Transfer

0
limit (ATm) = limit = Indeterminate and using L' Hospital rules we have—
x—>o x—>° log C 0
c—x
a (x)
ax
ATff, =
lixn2>icl
(log c
ax c—x
1
= limit [
x —>0 C — x c
c x [(c — x)2
= limit [c — x] c
x-30
ATm = ATI = AT2
42. In a certain exchanger, both the fluids have identical mass flow rate and specific heat
product. The hot fluid enters at 76°C and leaves 47°C and cold fluid enters at 26°C
and leaves at 55°C. The effectiveness of heat exchanger is—
(a) 0.16 (b) 0.58
(c) 0.72 (d) 1.0
(GATE — 1997)
Temperature distribution is-

76°C

55°C 47°C

26°C

Counter-flow

Heat transferred Q = rhh x ch(Thi — Th2)


= mh x ch(76 — 47°)
Qmax = Cmin x (Thi — Tc1) = Cmin(76 — 26)
Cmin = mh ch = the cc
Q _ rithch(76 47)
= Qmax nth ch (76 — 26)

— 0 58
—*
— 50 —
Option (b) is correct.
Heat Exchangers 403

43. A hot fluid at 200°C enters a heat exchanger at a mass flow rate of 104 kg/hr. Its
specific heat is 2000J/kg k. It is to be cooled by another fluid entering at 25°C with
a mass flow rate 2500 kg/hr and specific heat 400 J/kg k. The overall heat transfer
coefficient based on outside area of 20 m2 is 250 W/m2K. Find the exit temperature
of the hot fluid when the fluids are in parallel-flow.
(GATE — 1999)
Temperature distribution is-
200°C

25°C

Guidance: As only inlet temperatures are given, NTU method is to be used.


104
= mhCh = 3600 x 2000 = 5.55 kJ
Ch
2500
C = thccc = x 400 = 0.278 kJ
3600
C • = Cc = 0.278 kJ
C = Ch = 5.55 kJ
uA 250 x 20
N =NTU = — 17.98
Cmin 0.278 x 103
Cmin _ 0.278
R — — 0.05
C 555
In parallel-flow, effectiveness is—
1— e-N (1+ R) 1—e '
-17 98(1+0.05)
E=
1+ R 1 + 0.05

1 — e-1838
E=
1.05
1 — 6.32 x 10-9
1.05
= 0.952
Ch(Th, Th2)
E=
Cmin (Th, Tcd
555 (200 —
0.952 =
0.278 (200 — 25)
404 Heat and Mass Transfer

= 200 0.952 x 175 x 2.278


Th2
555
= 200 — 8.35
= 191.65°C

44. In a certain heat exchanger, both the fluids have identical mass flow rate-specific
heat product. The hot fluid enters at 76°C and leaves at 47°C and the cold fluid
entering at 26°C and leaves at 55°C. The effectiveness of the heat exchanger is—
(a) 0.16 (b) 0.58
(c) 0.72 (d) 1.0
Ch = Cc = Cmin
C h (Thi — Th2 ) (T — Th2 ) 76 — 47
E Cmin (Th, — (Th, — — 76 — 26

E= = 58
50 0.
Option (b) is correct.

45. In a condenser, water enters at 30°C and flows at the rate 1500 kg/hr. The condensing
steam is at a temperature of 120°C and cooling water leaves the condenser at 80°C.
Specific heat of water is 4.187 kJ/kg k. If the overall heat transfer coefficient is 2000
W/m2K, the heat transfer area is—
(a) 0.707 m2 (b) 7.07 m2
(c) 70.7 m2 (d) 141.4 m2
(GATE — 2004)
Temperature distribution is-

120°C 120
FT2
ATI 80

30

Parallel-flow

ATI = 120 — 30 = 90°C


AT2 = 120 — 80 = 40°C
ATI — AT2 90 — 40
LMTD = Tm =
ATI 90
log 47)
log A
L1T2
Heat Exchangers 405

= 50 = 61.73
0.81

Q = me x c, x (80 — 30) — 3600


1500 x 4187 x 50
= 87229 W
Now Q =uxAxAT,„

or A=
u x ATni
87229
2000 x 61.73
= 0.707 m2
Option (a) is correct.

46. In a counter-flow heat exchanger for the hot fluid, the heat capacity = 214/kg k, mass
flow rate = 5 kg/s, inlet temperature = 150°C, outlet temperature = 100°C. For the
cold fluid, heat capacity = 4 kJ/kg k, mass flow rate = 10 kg/s, inlet temperature =
20°C. Neglecting heat transfer to the surroundings, the outlet temperature of cold
fluid is—
(a) 7.5 (b) 32.5
(c) 45.5 (d) 70
(GATE — 2003)
Temperature distribution is-

150

100°C

200°C

Counter-flow

Heat lost by hot fluid = Heat gained by cold fluid


5 x 2 x (150 — 100) = 4 x 10 x (T,2 — 20)
5x 2x 50
or Tc 2 — 20 = 4 x 10 = 12.5°C
Tc2 = 20 + 12.5 = 32.5°C
Option (b) is correct.

47. Air enters a counter-flow heat exchanger at 70°C and leaves at 40°C. Water enters
at 30°C and leaves at 50°C. The LMTD in deg (°C) is—
406 Heat and Mass Transfer

(a) 5.65 (b) 14.43


(c) 19.52 (c) 20.17
(GATE — 2000)
Temperature distribution is-

1 70°C
AEI
T 50°C 40°C I
472
30°C
T
Counter-flow

ATI = 70 — 50 = 20°C
AT2 = 40 — 30 = 10°C
ATI — AT2 20 — 10
LMTD = =
ATI l0 20
log A 7,
CAA 2 10
10 = 14.43
0.693
Option (b) is correct.

48. Two fluids A and B exchange heat in a counter-current heat exchange. Fluid A enters
at 420°C and has a mass flow rate of 1 kg/s. Fluid B enters at 20°C and also has a
mass flow rate of 1 kg/s. Effectiveness of heat exchanger is 75%. Determine the heat
transfer rate and exit temperature of fluid B (specific heat of fluid A is 1 kJ/kg k and
that of fluid B is 4 kJ/kg k).
(Gate — 99)
Guidamce: As only inlet temperatures of both fluid have been given, we have to use NTU
method.
Ch = thh ch = 1 x 1 = 1 kJ
Cc = cc = 1 x 4 = 4 kJ
• = Cam, = 1
Cc = Cmax = 4
Cc (Tc, — 4 X (Tc,— 20)
E
Cmin (4, — 1 X (420 — 20)
4 x (Tc, — 20)
0.75 =
400
Tee = 20 +
400
4 x 0.75
Heat Exchangers 407

= 20 + 75 = 95°C
Q= Mc X Cc X (95 — 20)
= 1 x 4 x 75
= 300 kW

49. A heat exchanger is designed to cool hot water from 83°C to 45°C with flow rate of
5 kg/mm. Cold fluid is flowing at 9 kg/min has inlet temperature of 25°C and specific
heat of cold liquid is 2.85 kJ/kg k. Specific heat of hot water is 4.18 kJ/kg k. Explain
the type of heat exchanger required for the above purpose.
Heat lost by hot fluid = Heat gained by cold fluid

rhh ch(Thi — Th2) = rhc cc(Tc2 — Tc1)

x 4.18(83 — 45) = 60
9 x 2.85(Tc2 — 25)
60
. Tc2 = 56°C
Since Tc2 (= 56°C) is higher than Th2(= 45), only counter-flow arrangement is possible.

85 85
56°
45 56 45

25 25

Parallel flow Counter-flow

50. A heat exchanger is used to cool engine oil from 60°C to 40°C by cold liquid at inlet
temperature of 10°C and outlet temperature of 30°C. The total heat is 100 kW and
overall heat transfer coefficient based on outer surface area of tube is 75 W/m2C.
Estimate the heat transfer area for single pass parallel flow and counter flow heat
exchanger.

60°C 60
T
AT1
40A—c2 3
0
40°

OW
1 10°
10°C

Parallel-flow Counter-flow

1. Parallel-flow
ATi = 60 — 10 = 50°C
AT2 = 40 — 30 = 10°C
408 Heat and Mass Transfer

I
AT,n = LMTD = AT — AT2
AT2
log
AT2
= 24.9°C
Q = uA AT,n
Q 100 x 103
or A = x 24.9 — 53.6 m2
uAT,n 75
2. Counter-flow
ATi = 60 — 30 = 30
AT2 = 40 — 10 = 30
As ATi = AT2, them
I + AT2
AT,r, = AT
2
30 + 30
= 30
2
Q 100 x 103
A=
u • AT„, — 75 x 30
= 44.44 m2
Note: Area for counter-flow is less. Hence, counter-flow is more effective.

51. A counter-flow heat exchanger having heat transfer area 10 m2 is used to cool engine
oil from 100°C by water available at 20°C. The mass flow rate of oil and water is 2.4
kg/s and 1 kg/s. The overall heat transfer coefficient is 300 W/m2°C. Calculate the
exit temperature of water and total heat transfer rate. Specific heat of oil and water
is 2050 J/kgk and 4180 J/kgk.
Guidance: Since inlet temperatures of hot and cold fluids are given, we have to use NTU
method
C, = inc cc = 1 x 4180 = 1480 W/°C
Ch = 'hitch = 2.4 x 2050 = 4920 W/°C
Cc < Ch
Cc = Gain
Au 10 x 300
NTU = = 0.72
Cmin 4180
Capacity ratio R = Cmin _ 4180
Cma, — 4920
= 0.85
Effectiveness for counter-flow is-
1 — e- N (1—R)
E=
1— R e —N(1—R)
Heat Exchangers 409

1 e-032(1-0.85)

1— 0.85 x e —032(1-0.85)
1_e-0.108 1-0.897
1— 0.85 x e-0108 — 1— 0.763
0.103 — 0.43
0.237
Q = E Cmin(Th — Tc1
)
= 0.43 x 4180 x (100 — 20)
= 143.8 kW
Exit temperature of water = Tc2
Tc2 = Tci ER(Th1 Tc1)
= 20 + 0.43 x 0.85 (100 — 20)
= 20 + 29.24
= 49.24°C.

52. In a tubular counter-flow heat exchanger, 1080 kg/hr of water is heated from 40°C
to 80°C by hot flue gas (sp. heat = 10 kJ/kg k). Flue gas enters at 200°C and leaves
at 100°C. The overall heat transfer coefficient is 200 W/m2K. Find the area of
exchanger: (i) by LMTD-method, and (ii) NTU-method.
Case 1: LMTD method

4 200°C

AT1

180°C 100°C I
AT2
40°C T

Counter-flow

ATi = 200 — 80 = 120°C


AT2 = 100 — 40 = 60°C
AT2 — AT2 120 — 60
OTm —
AT
log 1 to 120
AT2 60
= 86.6°C
Q = A IcCc(Tc2 — Ti) = 1:0 x 1 x 103 x (80 — 40)
= 50.2 kW
4I0 Heat and Mass Transfer

Q =tixAx 6T,„
50.2% x 103
A = 2
200 x 86.6 — 2.9 m
Case 2: NTU method.
C = 1080 x 4.18 x 103 = 1255 W/°C
c 3600
Ch x (200 — 100) = Cc x (80 — 40)
Ch = Cc X 40 = 0.4 x Cc
100
= 502.0 W/°C
Now G > Ch
C•• = Ch = 502
Q Thi ) 200 —100
E=
Qmax Crain (Thi— 200 — 40
= 0.625
Crain
R = capacity ratio =
cinax
502 = 0.4
1255
For counter-flow, effectiveness is
1— e-N(1-R)
E
1 — Re-N(1-R)
1 _ e-N(1-0.4)
0.625 = N
1— 0.4e- (1-0.4)
1 _ e-0.6N
1-0.4e-o.6 N
0.625 — 0.25 e-0.6N = 1 _ e-0.6N
0.75 x e-"N = 0.375
e+0.6N = 0.75 = 2
0.375
or 0.6 N= log 2.0 = 0.693
N 0.693
or
0.6
= 1.155
u
Now N —A
Crain
Heat Exchangers 4II

N x Cmin
or A—

1.155 x 502
200
= 2.9 m2

53. A tube type exchanger is used to cool hot water from 80°C to 60°C. The task is
accomplished by transferring heat to cold water that enters the heat exchanger at
20°C and leaves at 40°C. Should this exchanger operate under counter-flow or
parallel-flow conditions? Also determine the exit temperatures if the flow rates of
fluids are doubled.
Guidance: The outlet temperature of cold fluid is less than the outlet temperature of hot
fluid. Hence, exchanger operates as parallel-flow heat exchanger.
Ch = thh Ch and C, = th c,
Heat lost = Heat gain
thh ch (80 — 60) = th • c,(40 — 20)
Ch = Cc = C • = Cmaac
C
Capacity ratio R = c,mm — 1
1..max
Thz 80 — 60
E— = 1/3
Thl —Tc, 80 — 20
For parallel-flow, effectiveness is—
1+ e-N(l+R) 1— e-2N
E=
1+R 2

1 x 2 = 1 — e-2N
3
or e-2N = 1 — 0.67
= 0.33
or N = 0.55
Now the flow rate is doubled which will change the value of NTU without changing R.
Cmin)new =
= 2x th h C x C•
u uA
Now (NTU)new
(cminInewh — 2 X Cmin
1
=— 1
x NTU = — N
2 2
1
= x 0.55 = 0.275
2
4I2 Heat and Mass Transfer

1 — e-
N(1-FR) 1 e-0.275X2
(E )new =
1+ R — 1+1
E new = 0.211
(Th)new = Th1 E (Th1 Tc1)
= 80 — 0.211 (80 — 20)
= 67.3°C
(Tc2)new = Tc1 + E (Th1 Tc1)
= 20 + 12.67
= 32.67°C

54. A tubular heat exchanger consists of 195 tubes each of 2.2 cm outer diameter and
5 m length. The flow of fluids are in counter-flow. If overall heat transfer coefficient
is 325 W/m2K, find outlet temperatures of the fluid and total heat transfer rate.
Given inlet temperatures of hot and cold fluids are 125 and 22°C and mass rate of
hot and cold fluids are 21 kg/s and 5 kg/s. Also ch = 2100 J/kg k and cc = 4100 J/kg k.

Ch = x ch = 21 x 2100 = 44.1 x 103 J/kg°C


Cc =mc x cc = 5 x 4100 = 20.5 x 103 J/kg °C
Now Cc < Ch C • = Cc = 20.5 X 103
20.5
R = Crain = 0.47
C 44.1
uA 325 x x 2.2 x 10-2 x 5 x 195)
NTU = N =
Crain 20500
N = 1.07
For counter-flow, effectiveness is—
1— e
-N(1-R)
E=
1— Re-N(1-R)
1— C
1.07(1-0.47)
1—
0.47e-1.07(1-0.47)
= 0.59
E X Crain (Th1 Tc1)
Tc1 = Tc1 +
Crain
0.59 x (125 — 22)
= 22 +
1
= 22 + 60.2 = 82.2°C
Crain
Th2 = Th — E (Th i — Tci)
Cmax
= 125 — 0.5 x 0.47 (125 — 22)
Heat Exchangers 4I 3
= 96.7°C
Q = E Cmin(Thi Tc1)
= 0.59 x 20500 (125 — 22)
= 1245 kW

55. A heat exchanger cools a liquid flowing at 8 kg/s from 100 to 50°C. Specific heat is
3850 K/kg°C. The cooling water has flow rate 10 kg/sec in tube side and entry
temperature of 10°C. Overall heat transfer coefficient is 400 W/m2C. Find the heat
transfer area: (i) for parallel-flow, one shell and tube pass, (ii) for counter-flow, one
shell and tube pass, (iii) for one shell pass and two tube pass, (iv) for cross-flow, both
fluids unmixed.
Heat lost = Heat gained
rhh ch(Thi — Th2) = rhc cc(Tc2 Tc1)
8 x 3850 (100 — 50) = 10 x 4180(Tc2 — 20)
Tc2 = 46.8°C
Q = thhch(Thi — Th2) = 8 x 3850 x 50 = 1540 kW
1. Case 1: Parallel-flow

50°C I
AT2
46.8°C

ATI = 100 — 10 = 90°C, ATI = 50 — 46.8 = 3.2


AT2 90 —3.2
ATni = LMTD = — 90 = 25.7°C
log A log
L1T2

=
1540 x 103
A=
u • AT„, 420 x 25.7
= 150 m2
2. Case 2: Counter-flow

—4— 100°C
Ari
46.8°C 50°C I
AT2
10°C 1-
4I4 Heat and Mass Transfer

ATI = 100 — 46.8 = 53.2°C


A T2 = 50 — 10 = 40°C
AT 53.2 — 40
n., = = 46.3°C
log ziic

Q 1540 x 103
A=
u • ATni — 420 x 46.3
= 83.2 m2
3. Case 3: One shell pass and two tube pass.
Here tube side temperatures are
Tti = 10°C, Tt2 = 46.8°C
Shell side temperatures are—
Tsi = 100 & Ty2 = 50°C
12 — Th _ 46.8-10 —
P—T 0 41
Ts, — Tt , - 100 —10 — .
s,— Ts2 100 —50
R=T = 46.8-10 = 1.36
Tt2 - Ttl
For P = 0.41 and R = 1.36, from graph correction factor (F) is—
F = 0.88
A Tff, for counter-flow = 46.3°C (as found out above)
.. Corrected mean temperature for the exchanger is—
A Tm = F X (A Tm)counter flow
= 0.88 x 46.3
= 40.7°C
Q 1540 x 103
A= =
u x ATm 400 x 40.7
= 94.6 m2
4. Case 4: For cross-flow, both fluids unmixed.
Correction factor for P = 0.41 and R = 1.36. The correction factor is—
F = 0.9
Hence, corrected mean temperature is—
ATm ATwcross
= (.-- 1 flow X 0.9
= 46.3 x 0.9
= 41.6°C
Heat Exchangers 4I 5

1540 x 103
A=
u x AT„, — 1500 x 41.6
= 92.5 m2

56. In a parallel flow heat exchanger, oil is cooled by water from 135°C to 65°C. The cold
water has inlet and outlet temperature as 20°C and 50°C. Find: (i) exit temperature
of fluds for counter-flow, (ii) minimum outlet temperature of oil by increasing tube
length in parallel-flow.
Heat lost = Heat gained
thhch(Thi — Th2) = thccc(Tc2 — Tc1)
thhCh(135 — 65) = thccc(Tc2 — Tc1)
thhCh x 70 = rilc cc x 30
Hence, Ch < Cc

R= C = 3° = 0.43
C 70
The effectiveness of parallel-flow heat exchanger is

E=
135-65 — 0.61
135 — 20
1— e —N(l+R)
But E=
1+ R
1 _ e—N(1+1)
0.61 =
1+1
Or N = 1.43
Case 1: Counter-flow
1 e- N (1— R)
E counter = N (1— R)
1— R

1 — e-1.43(1-0.43)
1.43(1-0.43)
1 —1.43 x e-
= 0.69
Now Th2 = Thi — E (Th1 — Tc1)
= 135 — 0.69 (135 — 20)
= 55.9°C
Tc2 = Tc1 -F E R(Thi — Tc1)
= 20 + 0.69 x 0.43(135 — 20)
= 54°C
4I6 Heat and Mass Transfer

Case 2: When heat exchanger is too long in parallel-flow, the outlet temperature of both
fluids is same

Th1

Ch x (Th1 - = Cc(T - Tc1)


T - Tci = Cc (Thi -
= 0.43 (Th1 - T)
0.43 x Th1 + TC , 0.43 x135+ 20
T
1.43 1.43
T = 54.5°C

57. Calculate the overall heat transfer coefficient (u0) based on outer surface of a pipe
with outer diameter of 4 cm and inner diameter of 3 cm. Thermal conductivity (K)
is 53 W/m°C. Heat transfer coefficient hi and 1/0 are 2000 W/m2°C and 1000 W/m2°C.
Fouling factors are Fi = 14 x 10-5 m2°C/W and F0 = 15 x 10-5 m2. ovv.
Overall heat transfer coefficient is-
1
uo =
d1
o do do 1 do
= — +—log—+—+=x+F F o
d ( hi ) 2k di ho I
1
4 ( 1 + 0.04 log 4 + 1 + 4 x 14 x 10-5 +15 x10-5
3 2000 2 x 53 3 1000 3
= 459 W/m2°C

58. Water at the rate of 3.783 kg/s is heated from 37.78 to 54.36°C in a shell and tube
heat exchanger. On the shell side one pass is used with water as heating fluid, 1.892
kg/s entering the heat exchanger at 93.33°C. The overall heat transfer coefficient is
1419 W/m2K and the average water velocity in 1.905 cm diameter tubes is 0.366 m/s.
Because of space limitations the tube length must not be longer than 2.438 m.
Calculate the number of tube passes, the number of tube per pass and length of
tubes.
(UPTU 2006 - 07)
Guidance: The surface for heat transfer is the product of perimeter, length of tube,
number of tubes per pass and number of passes.
Given: Tci = 37.78, Tc2 = 54.36°C, rh = 3.783 kg/s
Heat Exchangers 4I7
Thi = 93.33°C, rith = 1.892 kg/s, u = 1419 Wm2K
d = 1.905 cm, V = 0.366 m/s, L = 2.438 m
Heat gained = Heat lost
inc x cc x (Tc2 - Tc1) = rhh x Ch X (Th1 - Th2)
3.783 x (54.36 - 37.78) = 1.892 x (93.33 - Th2)
- 3.783 x 16.58
or 93.33 -
Th2 1.892
= 33.15
or Th2 = 93.33 - 33.15
= 59.18°C
P .T2 - Ti Here Tsi = Thi, Ts2 = Th2, Tri = Tci and Tt2 = Tc2
Ts, - T11

54.36 -37.78
. P-
93.33-37.78
= 16.58 _ 0.3
55.55
55
T 93.33-59.18
R - si- Ts2 -
Tt2 - Tti 54.36 -37.78

.34.15 _ 2
16.58 -
Using P = 0.3, R = 2 for one shell on 2, 4, 6... tube passes, correction factor is-
F = 0.975
Now OT,„ for a simple counter-flow exchanger

193.33°C
Ail
T 54.36°C

59.18°C1
AT2
37.78°C-t-

Counter flow

ATi = 93.33 - 54.36 = 38.97°C


AT2 = 59.18 - 37.78 = 21.40°C
- AT2 38.97 - 21.4
OT,n
A Ti = log 39.97
in = log A
LIT2 21.4
4I8 Heat and Mass Transfer

= 17.57 = 28.2
0.624
(A Tift)c,,,rected = 28.2 X 0.975 = 27.5°C
Q 3.783 x 4187 x (93.33 — 59.18)
A= —
U • (ATm)corrected 1419 x 27.5
A = 13.86 m2
Now th, = 3.783 kg/s
Cross-sectional area of tubes to handle flow
me 3.783
chow. — p x V —
103 x 0.366
= 10.3 x 10-3 m2
If there are 'n' tubes in tube side, then number of tubes is—
atow 10.3 x 10-3
n= i—
re x x (1.905 x 10-2)
4

.10.3 x 4 x 10-3 .10.3x40


x x 3.63 x 10" x x 3.63
= 36.146
Take n = 36
Now we require surface area of 13.86 m2 for ensuring heat transfer which has to be
provided by 36 tubes of 1.905 cm dia and length `l'. Each tube can have 2 or more pass.
First take 2 pass
13.89 =ir x dx Ixnx2
or /= 13.86
xx1.905 x10-2 x 36 x 2
= 3.218 m
But length has to be not more than 2.438 m. Hence, we take now 4 pass. Then length of
tube is—
l= 3.218 — 1.609 m < 2.438 m

Therefore, the heat exchanger has—


(a) number of tubes per pass = 36
(b) number of tube passes = 4
(c) length of tube = 1.609 m

59. In a counter flow heat exchanger, hot fluid enters at 60°C and cold fluid enters at
30°C. Mass flow rate of the hot fluid is 1 kg/s and that of the cold fluid is 2 kg/s. Sp
Heat Exchangers 4I 9
heat of hot fluid is 10 kJ/kg K and that of the cold fluid is 5 kJ/kg K. The LMTD for
heat exchanger in °C is—
(a) 15 (b) 30 (c) 35 (d) 45
(GATE — 2007)

60°C
i Th y
ATI
—1—Tc2 Th2 .4.
LiT2

i
Tci= 30

Counter flow

Counter flow

Heat capacity of hot fluid = mh ch


= 1 x 10
= 10 kJ/ks
Heat capacity of cold fluid = me cc
=2x5
= 10 kJ/ks
As mh ch = me cc
••• ATI = AT2= x
or 60 — x = 30 + x or x = 45

A T ff, = ATi + AT2


2
15+15
=
2
= 15
Option (a) is correct

60. Hot oil is cooled from 80 to 50°C in an oil cooler which uses air as the coolant. The
air temperature rises from 30 to 40°C. The designed heat exchanger uses a LMTD
value of 26°C. The type of heat exchanger is—
(a) parallel flow (b) double pipe
(c) counter flow (d) cross flow
(GATE — 2005)
420 Heat and Mass Transfer

80 80

50 50
40
40
30
------' 30

Parallel flow Counter flow

For parallel flow—


A Ti — AT2 (80 — 30) — (50 — 40)
A T ff, — —
AT log 80-30
log 1
AT2 50-40
= 24.85°C
For counter flow-
AT1 — AT2 — (80 — 40) — (50 — 30)
A Tn., —
AT log 80-40
log 1
AT2 50-30
= 28.85°C
As ATff, in counter flow is greater than the designed LMTD, hence counter flow heat
exchanger is suitable.
Option (c) is correct.

61. The LMTD of a counter flow heat exchanger is 20°C. The cold fluid enters at 20°C
and the hot fluid enters at 100°C. Mass flow rate of the cold fluid is twice that of the
hot fluid. Sp heat at constant pressure of the hot fluid is twice that of the cold fluid.
The exit temperature of the cold fluid is—
(a) 40°C (b) 60°C
(c) 80°C (d) cannot be determined
(Gate — 2008)
h1 = 100
—4—
AT1
+ Tc2 Th2
AT2
Tc1 -t-

As mh • ch = mc' cc
A Ti = AT2
Heat Exchangers 42 I

LMTD = m = ATi-FAT2
2
or ATI = AT2 = OTm = 20
Exit temperature of cold fluid (Tc2) = Th1 — AT,„
= 100 — 20
= 80°C
Option 'c' is correct.

62. A concentric tube heat exchanger uses water, which is available at 15°C, to cool
ethylene glycol from 100°C to 60°C. The water and glycol flow rates are same
at 5 kg/s. Determine The effectiveness of heat exchanger. Take Cp (water) =
4178 J/kg K, Cp (ethylene glycol) = 2650 J/kg K. Can you comment, whether the heat
exchanger is working in parallel flow or counter flow modes of operation?
(UPTU 2008 — 9)

Th1

Th2

TC1

Given: Th1
= 100, Th2 = 60 and Tci = 15°C

Ch = Mh Ch = 5 x 2650 = 13250 kJ
Cc = mc Cc = 5 x 4178 = 20890 kJ
Now Ch < Cc Cmin = Ch
Also heat given = heat gained
Ch x (Th1 — Th2) = Cc x (Tc2— Tc1)
or 13250 x (100 — 60) = 20890 (Tc2 — 15)
13250 x 40
or Tc2 = 15 +
20890
= 15 + 25.37
= 40.37°C
Q Cmin Th2 )
Effectiveness E=
Qmax — min (Thi Tc,)

100 — 60 40
100 —15 85
= 0.47
422 Heat and Mass Transfer

Since Tc2 (outlet temperature of water) is greater than Th2 (outlet temperature of ethylene
glycol), hence parallel flow is used in the heat exchanger.

63. In a cross flow, both fluids unmixed heat exchanger has water at 6°C flowing at rate
of 1.25 kg/s. It is used to cool 1.2 kg/s of air that is initially at a temperature of 50°C.
Calculate the exit temperature of air and exit temperature of water. Assume overall
heat transfer coefficient is 130 W/m2K and area is 23 m2.
(Annamalai University — 2007 — 8)
C, = me Cpc
= 1.25 x 4.18 = 5.225 kJ/sK
Ch = 1.2 x 1.05 = 1.26 kJ/sK
Here Ch < Cc
. Ch = C • = 1.26 x 103 J/sK
U•A
Now NTU =
C.
130 x 23
1.26 x 103
= 2.37
1.26
Now R — Cmin —
C 5.225
= 0.241
Now for NTU = 2.37 and R = 0.241, effectiveness for counter flow heat exchanger can
be determined from the graph
E = 0.83
Th 2)
Ch (Th, — SO — Th2
Now E= — 0.83
Cmin TO 50-6
. Th2 = 50 — 44 x 0.83 = 13.48°C
Cc (Tc2 — TO 5.225 (Tc2 — 6)
Similarly E= X
Cmin Th TO 1.26 (50 — 6)

4.146 x (Tc2 — 6)
or 0.83 =
44
or Tc2 = 6 + 8.81 = 14.81°C.

64. A counter flow concentric tube heat exchanger is used to cool engine oil (C = 2130 J/
kg K) from 160°C to 60°C with water available at 25°C as the cooling medium. The
flow rate of cooling water through the inner tube of 0.5 m is 2 kg/s while the flow
rate of oil through the outer annulus OD = 0.7 m is also 2 kg/s. If h is 250 W/m2K,
Heat Exchangers 423

how long must be the heat exchanger to meet its cooling requirement.
(Annamalai University 2004 — 5)
Heat lost by oil = Heat gained by water
MhCh(Thi — Th2) = McCc (Tc2 — Tc1)
or 2 x 2130 x (160 — 60) = 2 x 4186 x (Tc2 — 25)
2130 x 100
or Tc 2 = 25 +
4186
= 25 + 50.88
= 75.88°C
Now Q = 2 x 2130 x 100
= 426 x 103 W
1 160
AT1
7508 60 A
1 AT2
25

Counter flow
Counter flow

Now AT1 = 160 — 75.8 = 84.2°C


and AT2 = 60 — 25 = 35°C
AT1 —AT2 84.2 — 35
... Q. =_ 84.2
togATi log
AT2 35
= 49.2
0.878
= 56.05°C
Now Q =hxAx Q,
426 x 103 = 250 x A x 56.05
A — 426 x 103
or
250 x 56.05
= 30.4 m2
Now A= 7r xD1 x L
30.4
or L=
nx0.5
= 19.36 m
424 Heat and Mass Transfer

65. Match List I (Heat Exchanger Process) with List II (Temperature Area Diagram) and
select the correct answer using the codes given below the Lists.

List I (Heat Exchanger Process)


(A) Counter flow sensible heating
(B) Parallel flow sensible heating
(C) Evaporating
(D) Condensing
List II (Temperature Area Diagram)

1 2

t
T

A A —P.-

A A
1.
5

A —3-

Codes AB CD
(a) 3 4 1 2
(b) 3 2 5 1
(c) 4 3 2 5
(d) 4 2 1 5
Option (c) is correct.
Chapter10

CONDENSATION AND BOILING

A CONDENSATION A POOL BOILING


A FILMWISE CONDENSATION A FORCED CONVECTION BOILING
A DROPWISE CONDENSATION A SUBCOOLED BOILING
A VAPORIZATION A SATURATED BOILING
A LAMINAR FILM CONDENSATE A REGIMES OF SATURATED
A NUSSELT' S THEORY OF FILM POOL BOILING
CONDENSATION A NUCLEAT BOILING
A FILM THICKNESS A UNSTABLE FILM BOILING
A FILM HEAT TRANSFER COEFFICIENT A STABLE FILM BOILING

INTRODUCTION

Condensation and boiling are nothing but special convective heat transfer processes which are
associated with phase change of a fluid. Condensation is a process which involves a change of
phase from vapour to liquid while boiling involves a change of phase from liquid to vapour. These
processes are widely used in many applications such as: (i) boilers and condensers of power
plants, (ii) evaporators and condensers of refrigeration plants, (iii) heating and cooling, (iv) melting
of metals in furnaces, and (v) heat exchangers used in refineries and sugar mill. Since there is a
phase change in these processes, hence the heat transfer is large without much change in fluid
temperature. During a phase change there is: (i) liberation of latent heat by the fluid in condensation
process in condenser, or (ii) absorption of latent heat by the fluid in boiling process in boiler.

1. What is condensation? Discuss the physical mechanism of condensation on a vertical


plate.
(UPTU — 2005-6)

The condensation is a process in which vapour phase of substance changes into liquid phase
by releasing its latent heat of vaporization. Vapour changes into liquid phase when it comes
426 Heat and Mass Transfer

into contact with any surface having temperature lower than the saturation temperature
corresponding to the vapour pressure. During condensation, temperature does not charge
as vapour is converting into liquid by releasing its latent heat of vaporization.

2. What are types of condensation?

The condensation process can be classified as-


1. Filmwise condensation
2. Dropwise condensation

3. What is filmwise condensation? Why is it less effective but still preferred?

In the filmwise condensation, the vapour condensation takes place on the cold surface. The
condensate wets the surface and tries to spread out due to gravitational force. Ultimately
the condensate falls down from the surface on increasing thickness of the liquid film due
to gravity.

Ts < Tsaturation
Vapour

Film of liquid formed by vapour


condensation on cold surface
Cold
surface
(Ts)

Filmwise condensation

Filmwise condensation is not preferred as layer of liquid is deposited on the cold surface
having high resistance to heat flow which hinders the transfer of heat from the vapour to
the cold surface. The filmwise condensation leads to low transfer of heat from the vapour
to the cold surface and hence formation of filmwise condensation is less effective but
preferred in most of the applications as it can be easily achieved in applications. Filmwise
condensation takes place on clean serface and when vapour is having no impurity like
organic substance or fatty acid.

4. What do you understand by dropwise condensation?


or
Which type of condensation has a higher heat transfer film coefficient and point out
the reason thereof.
(UPTU — 2006-7)

In dropwise condensation, condensate does not spread on the cold surface to form liquid
film. Instead condensate forms liquid droplets which grow in size and ultimately they break
away from the cold surface. The condensation of vapour takes place most effectively as
condensate does not form liquid film on the cold surface. The cold surface remains
continuously exposed to the vapour to facilitate effective condensation. Such type of
Condensation and Boiling 427

condensation takes place when the cold surface is highly polished or has coating of teflon
or zinc. Impurities like organic substances or fatty acids in the vapour are also helpful for
dropwise condensation.

0 0 0 0
0 0 0 0 Tsurface < Tsaturation

0 0 0 0
Dropwise condensation

5. What type of condensation is assumed in designing condensers?


(UPTU — 2006-7)

Although heat transfer rate in case of dropwise condensation is much higher than what is
possible in case of filmwise condensation, but it is very difficult to achieve dropwise
condensation. Therefore, in spite of low heat transfer rate possible in filmwise condensation,
film condensation is assumed for designing condensers.

6. What do you understand by boiling and condensation?

Boiling and condensation are also heat transfer processes like conduction, convection and
radiation but main difference is that boiling and condensation involve a phase change of the
substance. In boiling, liquid phase of the substance changes into vapour phase while in
condensation, vapour phase changes into liquid phase. Boiling and condensation are heat
transfer processes which take place at constant temperature. The heat transfer rate in boiling
and condensation is very high as compared to other modes of heat transfer due to latent heat
associated with phase change of the substance.

7. Differentiate between dropwise and filmwise condensation.


(UPTU — 2003-4 and 2006-7)

S.N. Dropwise condensation Filmwise condensation


1. The vapour condenses into small liquid The vapour condenses as liquid film on
droplets of various sizes. These droplets surface and stays on the surface for a lon-
fall down from the surface in random ger time.
fashion. Hence, condensate as droplets
tends to leave the surface quickly without
forming a film due to gravity.
2. No additional thermal resistance to the Additional thermal resistance due to for-
heat flow from vapour to the surface. mation of liquid film.
3. Heat transfer is 5 to 10 times more than Heat transfer is hindered by liquid film.
filmwise condensation.
4. Occurs either on highly polished surfaces Clean surface and no impurity
or vapour contaminated with impurities
like organic substances and fatty acids.
5. Difficult to achieve or maintain. Easy to achieve
428 Heat and Mass Transfer

8. Derive an expression for velocity distribution in the liquid condensate film over a
vertical plate for laminar film condensation.
(UPTU — 2002 — 3)

Nusselt's theory of film condensation is applied for vapour condensation occurring on a


vertical plate. Nusselt's analysis of film condensation is based on following simplifying
assumptions—
(a) Vapour is pure, dry and saturated
(b) The flow of condensate is due to the action of gravity and it is laminar.
(c) The liquid film has attained surface temperature at contact with the cold surface and
saturation temperature exists at liquid-vapour interface.
(d) Viscous, shear and gravitational forces act on the fluid in the film along the surface.
Normal viscous and inertia forces are neglected.
au
(e) No velocity gradient at liquid-vapour interface i.e.( — = 0 where u = velocity,
ay j y.5
y = distance from surface and S = film thickness.
(f) Heat transfer in the film is by conduction and temperature distribution in the film is
linear.
(g) Heat transfer is taking place under steady state.
Consider condensation of vapour on a vertical plate as shown in the figure. Take origin
`o' at upper end of the plate, x-axis along the vertical surface and y-axis at perpendicular
to it. Take height of plate = L, width = b, thickness of film = S at distance 'x' from the origin.
The film thickness is zero at upper and of the plate and it increases gradually till lower end
of the plate where thickness is maximum.
0

Viscous
dy
force

du d2u
du x dx dx µ +µ dy]b x dx
dy dy
[ dY

Viscous
Velocity force
profile Bouyant I I
force = pv.g. (b.dxdy) Body force
= (b.dx.dy)

Filmwise condensation and Force balance in


element of volume in film element of volume

Now consider the equilibrium of gravity, buoyant and viscous forces acting on the
element of volume (b • dx • dy) in the liquid film
du du d2u
pi, • g • (b • dx • dy) — pv • g • (b • dx • dy) = µ— (b • dx) — 2 • dy)(b • dx)
dy dy dy
Condensation and Boiling 429

or d 2u — (PL -Pv)g
dy e ti
On integration, we get—

u = —(PL — Pv) (Y2). g + co, + C2


il, 2
Apply boundary conditions—
At y = 0, u = 0

and at y = 8, =0
du
y
We get the value of c1 and c2 as—

ci = (PL — Pv)• g • O and c2 = 0


µ
The velocity profile on putting the values of c1 and c2 is—

u = PL — Pv [ (5y Y 2 1
µ L 2J

= PLPV (5 2 [y2 1 (y ')21


o 2

The mean velocity in the liquid film at a distance y from the plate is—

umean = I- J.° u • dy
(5 0

= 1 I*6 PL — Pv g o(.2,[y
— —1 ( —
y )1dy
2
8J o µ 8 2 8

(PL — Pv) • g • 52

9. Find the expression for the change of mass flow rate of condensate through the
laminar liquid film formed on the vertical plate.
m
The mass balance in the element of volume in the liquid film formed
on a vertical plate is as shown in the figure— -,- dy1-0-
Now m = density x flow area x mean flow velocity (um) dx
1
(PL — P g) • g • 32
= PL x (b x 8) x
3µ 1 m+ dm
Mass balance
430 Heat and Mass Transfer

As film thickness '8' increases as 'x' increases when vapour moves down condensing on
the vertical plate, the mass flow rate of the condensate is therefore dependent or a function
of distance 'x'. As the condensate flows from 'x' to `x + dx', the film thickness grows from
`8' to '8+ d8'. The growth of film is resulting due to additional condensation of the vapour
on the cold surface. The mass of condensate added to the film from distance 'x' to
`x + dx' can be found out by differentiating mass flow (m)—

dm = d [pL(pL- Pv)• g • b • 8 3 1c18 •dx


d8 3µ dx

= [PL(PL — Pv)• g • b • 82 i ds

10. Find the expression for film thickness (3) in the liquid condensate forming laminar
film on a vertical plate.

The heat balance in the element of volume in the liquid film formed on a vertical plate is as
shown below—

dy

dQ = hA AT dx dQ - hfe dm

dQ = KA ST

The rate of heat flow (dQ) in the film element is equal to the rate of latent heat release
on condensation of vapour at the surface. Hence,
dQ = latent heat of vaporisation x change of mass flow rate
= hfg x dm

= [PL(PL — Pv)• g • b • 82 1 (i)


hfg d8

The mode of transfer of heat (dQ) after condensation is pure conductance across the film.
Hence,
k(b • dx)
dQ — (Tsar — Ts)
Equating equation (i) and (ii), we have—

k • b dx
hfg PL (PL Pv). g b 32 d6 = (Tsar — Ts)
c

or 83 x d8 = kxµx(Tsat—Ts)xdx
p L (pL — pv)• g • h fg
Condensation and Boiling 43 I

Now on integration and considering `S' varies with `x', we get-


84 k x tix(Tsat — Ts)
x -i- ci
4 PL (PL — Pv) • g • hfg
Now boundary conditions are-
8 = 0 at x = 0, ci = 0
Hence, the film thickness is—
xitm
[4xkxtix(Tscit—Ts)
8=
PL(P L — Pv) x g x h fg
The film thickness (8) increases as the fourth root of the distance (x) from the top of the
plate.

11. Find the expression for the heat transfer coefficient for the film on the vertical plate.

The heat flow from the vapour to the surface is assumed to be taking place by only
conduction through the condensate film. Hence,
k • (b . dx)
dQ = (Tsat — 7's) (i)
(5
Now heat transfer at the surface due to only convection is—
dQ = hx(b • dx) (Tsai — Ts) 00
where hx = local heat transfer coefficient at distance 'x' from the top of the plate
Equating equations (i) and (ii), we have—
k • (b • dx) ,,,,
ki sat — Ts) = hx(b X dx) (Tsat — Ts)
(5
k
or hx = 75
1/4
4 x k x µ(Tsa, — T
But S— s) x x
P L(P L — p v) x g x hfg 1
1/4
_[ PL (P L — pv)X k 3 X g X hfg 1
hx
4 11(7'sat — Ts)X x

Now h = 1 rhxdx
Lo
i
1/4
1 IL PL (PL — PV) X k3 X g X hfg
X dx
L Jo 4 11(7'sat — Ts ) X x

1/4
1 X PL (PL -PV)k 3 ' g • hf g Xf L X
-1/4 dx
=
L L 4 au (Tsai — Ts ) JJ o
432 Heat and Mass Transfer

4 [PL (PL Pv) x k 3 x g x h fg 11/4


3 4x ttxLx(Tsat —Ts)
ill4
Pv) x k 3 X g
[PL(PL— h fg
Also hz, = (hx)x.L, —
4xµxLx (T„t — Ts)

. h = —4 x hL
3
Hence, average heat transfer coefficient is 4/3 times the local heat transfer coefficient at
the trailing edge (x = L) of the plate.
The average heat transfer coefficient is generally given as-
1/4
PL (PL — Pv)x k3 x g x hfg
h = 1.13 [
µ x L x (Tsai — Ts)
In case the surface is inclined '0' with horizontal, then average heat transfer coefficient is-
1/4
h = 113 [PL (PL — Pv) x k3 xgxhf g x sin 0
ti x L x (Tsat — Ts)

12. Write the expression for average heat transfer coefficient for laminar film
condensation on: (i) a horizontal cylinder, and (ii) horizontal tube banks.

Average heat transfer coefficient for laminar film condensation on a horizontal cylinder is—
i1/4
rr
[ PL (PL Pv) • k 3 g • hfg
h = 0.725
eilD(Tsat Ts)
where D = diameter of cylinder.
Average heat transfer coefficient for laminar condensation on horizontal tube banks is—
• 1/4
pv). k3. g hfg
h = 0.725 [ PL (PL
D '(Tsat — Ts)
where D = diameter and N = number of horizontal tubes in a vertical tier so that condensate
from one tube flows directly to next tube below it. For a square array, the total tube is a2
while N = a.

13. What is boiling heat transfer?

Boiling is a convective heat transfer process and it differs from other modes of heat transfer
as it involves a phase change from liquid to vapour. As the phase change from liquid to
vapour requires a large quantity of heat as latent heat of vaporization, possible heat transfer
by boiling is large as compared to what can be transferred by other modes of heat transfer.
Boiling occurs when a liquid comes into contact with a hot surface whose temperature is
Condensation and Boiling 433

higher than the saturation temperature of the liquid. Heat transferred to the liquid is—
Q=11)<Ax(T,—Tsat)

14. What are different forms in which boiling heat transfer phenomenon may occur?

The boiling heat transfer phenomenon may occur in any of the following forms—
(a) Pool boiling: Pool boiling occurs when the liquid above the hot surface is stagnant and
liquid motion above the hot surface occurs due to free convection currents resulting
from density variations on heating of the liquid. The liquid particles in contact with hot
surface evaporate and form bubbles. On detachment, these bubbles tend to move up
to the free surface. The hot surface is now exposed to fresh particles of the liquid
which get heated and form bubbles on transfer of heat from the hot surface. Pool
boiling occurs in steam boilers which are designed on natural convection.
(b) Forced convection boiling: Forced convection boiling occurs when the liquid is made
to flow over the hot surface by external mean. The liquid is pumped and forced to flow
over the hot surface, thereby creating forced convective currents. This type of boiling
takes place in water tube boilers involving forced convection.
(c) Sub-cooled or local boiling: Local boiling occurs when the liquid is below saturation
temperature and boiling takes place only near the hot surface. The vapour bubbles
formed at hot surface can travel a short distance in the liquid before collapsing and
vanishing. The energy gained by the bubbles is insufficient to take them to the free
surface.
(d) Saturated boiling: In saturated boiling, the temperature of the liquid exceeds the
saturation temperature. Vapour bubbles are formed at hot surface and these bubbles
have sufficient energy to move to the free surface and finally leave the free surface.

15. Explain the various regimes of the saturated pool boiling.


(UPTU — 2002 — 3)

There are various distinct regimes of pool boiling which have different mechanisms of heat
transfer. The different regimes of boiling are as shown in the graph given below. The graph
showing temperature difference (z Te = T, — Tsat) on x-axis and heat transfer rate [Q =
voltage x current) is obtained from electrically heated wire submerged in the liquid (pool
boiling), thereby obtaining different values of Q (by varying current and voltage) and
corresponding values of the surface temperature of heated wire is measured. There are six
regimes of saturated pool boiling as shown in the graph and explained in succeeding
paragraphs.
1. Free Convection Boiling (Curve '0' to 'A'): It occurs when A T, < 5°C as shown in
regime I. The convection current takes place due to density difference between liquid
particles at hot wire surface and liquid particles at free surface. Heat transfer can be
found out by correlation of free or forced convection.
2. Nucleat Boiling (A to B and B to C): It occurs when 5 < A T, < 50°C as shown in
regime II and BI In regime II, temperature of hot wire surface is not high (z Te < 10°C)
434 Heat and Mass Transfer

Radiation
-II— —..-.— NUL:lei:IL Liming —...-4.— Unstable —1.--N- -1.-4- a -..-
convection film film film boiler
Regine I —.---.— II —.—.— Ill —,•—• IV V . VI —'.-
E
5
csr 10

a)
co B
35
u, 10

H
a) A D
1 2
10

0
10
0 5 10 50 120 1000
Temperature difference (ATc = Ts — Tsat)

Pool boiling curve for water

resulting into isolated bubbles are formed at few nucleation sites on the hot wire. These
bubbles are detached with insufficient energy, thereby collapsing and disappearing in
the liquid without reaching free surface. Regime II is called unstable nucleat boiling
(curve A — B). On further increase of hot wire temperature, the bubble formation is
rapid and bubbles have sufficient energy to reach to the free surface. Bubbles formation
and motion results into liquid agitation and mixing. This promotes considerable heat
transfer from wire to liquid resulting into high boiling heat transfer coefficient. Nucleat
boiling exists upto A T, = 50°C (curve A — C). The maximum heat flux at point 'C' is
called critical heat flux or burnt out point.
3. Film Boiling (curve CD): On further increase of wire temperature, the formation of
bubbles increases and few bubbles are incapable to get detached, thereby forming an
unstable film of vapour. The vapour film formed such on the wire has low thermal
conductivity than the liquid itself, thereby acting as insulation film to reduce the heat
transfer from the wire. Hence, heat transfer from wire falls as compared to the regime
of nucleat boiling. In regime IV, the vapour film is unstable which is seen to collapse
and reform rapidly. However, with further increase of the temperature, vapour film
becomes stable which considerably lowers the heat transfer. This is regime V which
ends at point `D'.
4. Film boiling with radiation (curve D — E). In regimes VI, film boiling continues but heat
transfer increases due to a large amount of heat is lost by the wire due to radiation. The
temperature of the wire is very high at point 'E' which may result into the melting of
the wire.
Condensation and Boiling 435

16. Why is nucleat boiling preferred instead of stable film boiling with radiation in pool
boiling?

The heat transfer increases with nucleat boiling upto critical heat flux or burnt out point
(point 'C' of pool boiling). The film boiling starts after point 'C' and heat transfer reduces
due to vapour insulation blanket formed on the wire which is undesirable. Therefore, nucleat
boiling gives higher values of heat transfer or heat transfer coefficient at considerably lower
temperatures. Though at very high temperature in film boiling, there is marked increase of
heat transfer resulting from a large amount of heat being radiated out from the wire due to
radiation, but there is a large risk about the melting of wire at high temperatures.

17. Condensation of steam at atmospheric pressure occurs over a 5 cm diameter and


1.4 m long tube when: (a) the pipe is kept horizontal, and (b) when the pipe is kept
vertical. Assume all other properties of steam remain the same. For the same
temperature difference, find out which position transfer more heat and by how much.
1/4
[Ic3 p 2 . g • h
h, = 0.943
;IL (Ts — Tsar) i
11/4
[ k 3 . p2 • g • h fg
hh = 0.725
µ • d • (Ts — Tsat)
(UPTU — 2003 — 4)

Guidance: The heat transfer coefficient for horizontal pipe is dependent on the fourth root
power of diameter while for the vertical pipe, it is dependent on the fourth root power of
length.

Vapour Vapour
Tube
Condensate Pipe
Condensate

Horizontal tube Vertical pipe

1/4
[ k3p2 • g • h fg
by = 0.943
(Tsat — Ts)
1/4
2. g • h fg]
hh = 0.725 k 3. p
d (Tsat — Ts)

by _ 0.9431- dill4
hh — 0.725 L l J
436 Heat and Mass Transfer

0.943 x [5 x10 -2 1114


0.725 1.4
= 0.565
or h, = 0.565 hh
But h Q
Q, = 0.565 Qh
Hence, rate of heat transfer is less when the tube is kept in vertical position having length
greater than diameter.
In this case, Qh = 1.77 a, i.e. heat transfer in horizontal position is greater than heat transfer
in vertical position by a factor of 1.77.

18. Saturated steam at 90°C and 70 kPa is condensed on outer surface of 1.5 m long and
2.5 m diameter tube which is maintained at uniform temperature of 70°C. Assuming
filmwise condensation, calculate the heat transfer rate on the tube surface. Use
properties of water at 80°C.
hig = 2309 kJ/kg, p = 974 kg/m2, k f = 0.668 W/mK and µ = 0.355 x kg/ms.
(PU — 2003)

The heat transfer coefficient for laminar filmwise condensation on a vertical tube surface
is—

[ k3 •p2 . g • h fg 114
= 0.943
/I • L • IT — T)Ts)
1/4
[(0.668)3 x (974)2 x 9.81 x 23091
= 0.943
(0.355 x 10-3) x 1.5 x (90 — 70)
Here Ts, = 90° from steam tables
h, = 4670 W/m2K.
The heat transfer is—
Q = h, x A(Tsat — Ts)
= 4670 x AL) (90 — 70)
= 4670 x 7r x 2.5 x 1.5 x 20
= 1099.8 kW

19. An electrical wire of 0.2 mm diameter and 100 mm length is submerged horizontally
at atmospheric pressure. The wire has current = 6 amp and applied voltage = 30 V.
If the wire is maintained at 200°C, find: (i) heat flux, and (ii) boiling heat transfer
coefficient.

Heat transfer from wire is equal to energy supplied to the wire.


Q = voltage x current
= 30 x 6 = 180 W
Condensation and Boiling 437

Heat flux q=
A
180 180
gX0.2X10-3 X100X10 -3
= 2.86 x 10+6 W/m2
Now q = h X (Ts - Tsat)
h= q 2.86 x 10+6
(Ts - Tsai) - (200 -100)
Tsat = 100 at 1 atmosphere for water
h = 2.86 x 10+4 W/m°C
= 28.6 kW/m°C

20. An electric wire of 1.25 mm diameter and 250 mm long is laid horizontally and
submerged in water at 7 bar (saturation temperature of water at 7 bar is 165°C). The
wire has an applied voltage of 2.2 volt and carries a current of 130 amperes. If the
surface of wire is maintained at 200°C, make calculation for the heat flux and boiling
heat transfer coefficient.
(UPTU - 2006 - 7)
Heat transferred is equal to the electric energy given to the wire.
Q= voltage x current
= 2.2 x 130
= 286 W
Area =7rXdX1
= x 1.25 x 10-3 x 250 x 10-3
= 981 x 10-6 m2
q = — = 286 = 0.29 x 106 W/m2
A 981 x 10—
Now q = h(Ts - Tsat)
q 0.29 x 106
or h=
Ts - Tsai 200 -165
= 8.28 x 104 WIm2°C
= 82.8 kW/m2°C

21. A vertical plate 350 mm high and 420 mm wide is maintained at 40°C. It is exposed
to saturated steam at one atmosphere. Find: (i) film thickness at trailing edge of the
plate, (ii) maximum velocity at trailing edge, and (iii) total heat flux for the plate.
Film thickness at trailing edge is given by one-fourth power of distance (x = 350 mm)
11/4
[u • k • it • (Tsai Ts) X X
6=
gPL, (PL, - pv)hfg
438 Heat and Mass Transfer

Now pv << A and it is neglected. Also film temperature is—


s + Tsai — 40+100
Tf— T = 70°C
—2 2
Properties of steam and condensate from tables of 70°C—
hfg = 2257 kJ/kg, II = 406 x 10-6 kg/ms
k = 0.668 W/m.k., p = 977.8 kg/m3
1/4
[4 x 0.668 x 406 x 10-6 x (100 — 40) x 0.351
SL =
9.81 x (977.8)2 x 2257 x 103
= 1.82 x 10-4 m = 0.182 mm
The velocity distribution is—

u—
PL. g. [ (5 Y —Y22 1
eu,
The velocity is dependent on '8' and maximum velocity is at trailing edge where 8= 8L and
Y = SL
p L xgx[(5 L ]
UmaX

(977.8) x 9.81(0.182 x 10-3)2
umax
2 x 406 x10-6
= 0.391 m/s
Now heat transfer coefficient is—
[pi • g • h fg • k 3 T/4
h = 1.13
pL(Tscit — Ts)
1/4
[(977.8)2 x 9.81 x 2357 x 103 x (0.668)3
= 1.13
406 x 10-6 x 0.35 x (100 — 40)
= 5960 W/m2°C
Now heat flux is—
q = h(Tsas — Ts)
= 5960 (100 — 40)
= 357.5 kW/m2

22. A vertical flat plate in the form of a fin is 50 cm in height and it is exposed to steam
at atmospheric pressure. The plate is maintained at 80°C. Find: (i) film thickness of
trailing edge, (ii) heat transfer rate, (iii) condensation mass flow rate. Assume—
hfg = 2275 kJ/kg, Pv = 59.6 x 10-3 kg/m3
Condensation and Boiling 439

PL = 971.8 kg/m3, 19- = 6.74 x 10-1 W/mK


= 355.3 x 10-6 N's/m2
(NMU — 1999)
The vertical plate with film thickness S varying with distance 'x' from leading edge is as
shown below).
y

50 cm

1/4
4 k(Tsa, —Ts ) x x x
8x=0.5 =
[ PL (PL — pv) x g x h fg

4 x 0.674(100 — 60) x 355.3 x 10-6 x 0.5 11/4


[
971.8(971.8 x 0.0596) x 9.81 x 2257 x 103
= 1.74 x m = 0.174 mm = SL
Now local heat transfer coefficient is-
0.674 — 3874 W/m2K
hx=z, = =
8L, 1.74 x 10-4
The average heat transfer coefficient is—
4
hay = — x =—4 x 3874
3 3
= 5165 W/m2K
Heat transfer rate is—
Q = h X A, X (T„, — 7's)
= 5165 x (0.5 x 1) (100 — 60)
= 103 kW/m
Condensate mass flow rate is—
Q = m X hfg

or m= Q
= 103x103
h fg 2257 x 103
= 4.57 x 10-2 kg/s

23. In the last problem, find heat transfer coefficient if the plate is inclined at 30 degree
with horizontal.
440 Heat and Mass Transfer

Guidance: When the plate is inclined 30° from horizontal, the value of gravity acceleration
`g' is replaced to `g sin 0' in the value of heat transfer coefficient. As g > g, x sin 0, hence
the value of heat transfer coefficient is lesser for inclined plate.

Inclined
plate

01/4
hinchned = hhorizontal X (sill
= 5165 x (sin 30)1/4
= 5165 5165
(2)1/4 — 1.189
= 4344 W/m2C

24. A single pass shell and tube type steam condenser is to be designed to condense
500 kg/hr of dry and saturated steam at a pressure of 0.1 bar. A square array of
400 tubes each of 6 mm diameter is to be used. The tube surface temperature is
to be maintained at 24°C. Find the heat transfer coefficient and the length of each
tube.
Guidance: Total tubes are 400 in square array which will give horizontal tubes in vertical
tier i.e. N = V400 = 20. For steam at a pressure of 0.1 bar, steam table gives—

vg = 14.67 m3/kg, hfg = 2393


Tsat = 45.8°C,
1 = 1 = 0.06
p=
v vg 14.67
The film temperature is—
45 8 + 24
Tf = ' — 35°C.
2
Hence, properties of water at 35° C is—
PL = 993.95 kg/m3, k = 62.53 x 10-2 W/m°C
µ = 728.15 x 10-6 kg/m.s.
Now tubes are in horizontal and average heat transfer coefficient is
1/4
h = 0.725[ PL (PI' — gv)k3. g.ilfg 1
N • d • µ(Tsat — Ts)
Condensation and Boiling 441

2 1/4
993.95(993.95 x 0.0676) x (6253 x 10-2)2 x 9.81 x (2.393 x 1011
= 0.725[
20 x (6 x 10-3) x 728.15 x 10-6 x (45.8 — 24)
= 5.359 W/m2°C
Now heat transfer is—
Q = h • As(Tsa, — Ts) = m x hfg
or = 5359 x(rxdx/xN2)(45.8 — 24)=mx 239 x 103
where 500 kg/sec
m = 500 kg/hr = 3600

500 x 2393 x 10 -3
=
3600
5359 xxx 6 x10-3 x 400 x21.8
= 37.74 cm.

25. A surface condenser is designed for a condensation of vapour at the rate of 60 kg/
hr. It has 81 tubes in a square array of 9 x 9. Outside diameter of tube is 10 mm and
its length is 1 m. What will be the condensation rate in vertical position instead of
horizontal position?
For horizontal arrangement-
1/4
rpi • g x h fg x 1
hh = 0.725
Nxdxµx(Ts — Tsat)

Vapour

Tube in horizontal array

Vapour

Tube in vertical array


442 Heat and Mass Transfer

In vertical array, heat transfer coefficient is


3 1/4
[ pi x g x h fg x k
hi, = 0.943
Lx /ix (Ts —T„t )
Now heat transfer is proportional to heat transfer coefficient i.e. Q cc h. Also condensation
mass flow is proportional to heat transfer, i.e. Q = m x hfg. Hence, condensate mass flow
is proportional to heat transfer coefficient
i/4
Mh
— hh 0 725 [ L
Mv — h, -- 0.943• n•d i
11/4
0.725 [ 1
0.943 9 x 0.01
= 1.404
Mh 60
Now MV = = = 42.75 kg/hr
1.404 1.404
Chap ter 1 1

MASS TRANSFER

KEYWORDS AND TOPICS


A MASS TRANSFER A MOLAR FLUX
A DIFFUSION A FICK'S LAW
A MASS CONCENTRATION A DIFFUSION RESISTANCE
A MOLAR CONCENTRATION A GENERAL MASS DIFFUSION EQUATION
A PARTIAL PRESSURE A STEFFAN' S LAW
A MASS AVERAGE VELOCITY A EQUIMOLAR COUNTER DIFFUSION
A MOLARAVERAGEVELOCITY A SCHMIDT NUMBER
A MASS DIFFUSION VELOCITY A LEWIS NUMBER
A MOLAR DIFFUSION VELOCITY A SHERWOOD NUMBER
A MASS FLUX

INTRODUCTION
If there exists a difference in the concentration or density of any species in a mixture, then there
exists a natural tendency of its mass to be transferred in the direction to minimise the
concentration difference. The transport of mass of the species or constituent takes place from
the region of higher concentration to the region of lower concentration. This phenomena is
known as mass transfer. The mass transfer depends upon the concentration gradient of the
species in the mixture. The rate of mass transfer depends upon the difference of concentration
between two regions. The direction of mass transfer is from higher concentration to lower
concentration. The mass transfer ceases when concentration gradient becomes zero. The mass
transfer also depends upon molecular spacing of matter. Hence, diffusion occurs more readily
in gases as compared to liquids and more rapidly in liquids as compared to solids. The process
of mass transfer is also accompanied with heat transfer. This aspect of heat transfer along with
mass transferred is used in many industrial applications such as: (i) humidification of air in
cooling tower, (ii) evaporation of petrol to make air-fuel mixture in the carburettor of a petrol
engine, and (iii) evaporation of liquid ammonia in the atmosphere of hydrogen in electrolux
refrigerator.
444 Heat and Mass Transfer

1. What is mass transfer?


Mass transfer is a process of transfer of mass due to the concentration difference of species
in a mixture. The mixture may have one or more species whose concentrations vary from
point to point. The species having such variation of concentrations start transporting
themselves from the region of higher concentration to lower concentration. So long there
exists concentration difference of a species, mass transfer will occur.
2. Give examples of industrial applications where mass transfer takes place.
The examples of mass transfer in industrial applications are-
1. Humidification of air in cooling tower or cooler. Air is cooled by using its heat for the
evaporation of water.
2. Evaporation of petrol in carburettor. The liquid petrol is evaporated by sucked air at
pressure lower than atmosphere.
3. Evaporation of liquid ammonia in the atmosphere of hydrogen in electrolux refrigerator.

3. Give examples of mass transfer which are occurring in day-to-day life.


The examples of mass transfer occurring in day-to-day life are-
1. Dissolution of sugar added to a cup of tea.
2. Separation of various components of a mixture by distillation.
3. Diffusion of smoke through a tall chimney into the atmosphere.
4. What are different modes of mass transfer?
The modes of mass transfer are-
1. Mass transfer by diffusion.
2. Mass transfer by convection.
3. Mass transfer by change of phase.
5. What do you understand by mass transfer by diffusion?
It is transport of a substance at molecular or microscopic level. The mass transfer takes
place from higher concentration region to lower concentration region of any substance in
a mixture or system. It occurs when a substance diffuses through a layer of stagnant fluid.
In case diffusing substance is in turbulent motion, then eddy diffusion occurs and mass
transport of the substance increases as compared to diffusion by only molecular diffusion.

6. Explain mass transfer by convection.


The mass transfer by convection takes place when: (i) moving fluid passing over a surface,
or (ii) two immiscible fluids are moving together. Mass transfer depends on: (i) properties
of fluid, and (ii) laminar or turbulent nature of the flows of the fluids.
7. What do you understand from the mass transfer by the change of phase?
Mass transfer takes place by the change of phase specially from liquid to vapour. Interacting
fluids are liquid and gaseous which results into transport of liquid fluid into gaseous fluid
by the change of phase of liquid.
Mass Transfer 445

8. Discuss the analogy of heat and mass transfer.


(UPTU — 2007 — 8)

Heat transfer Mass transfer


1. The heat transfer takes place from higher 1. Mass transfer takes place from higher
temperature to lower temperature. concentration to lower concentration of a
substance.
2. The heat transfer stops when there is no 2. Mass transfer stops when there is no con-
temperature gradient exists. centration gradient of the substance exists.
3. It is a molecular phenomenon in case of 3. Diffusion mass transfer is also a molecular
heat transfer by conduction. phenomenon.
4. Convective heat transfer is dependent on 4. Convective mass transfer also depends
the flow properties of the fluids. on the flow properties of the fluids.

9. Explain: (i) mass concentration, (ii) molar concentration, (iii) mass fraction, mole
fraction, and (iv) partial pressure.

1. Mass concentration or mass density: The mass concentration of component 'A' in a


multicomponents mixture is defined as the mass of species 'A' present per unit volume
of the mixture. It is denoted for the component 'A' as pA with unit as kg/m3.
Mass of component `A' mA
PA = Volume of the mixture V
2. Molar concentration: The molar concentration of the component 'A' is defined as the
number of moles of species 'A' present in the unit volume of the mixture. It is denoted
as CA and has unit of kg mol/m3. It is also called molar density.
Number of moles of component 'A '
CA —
Volume of the mixture
nA
=V
Mass of component `A'
Since number of moles nA —
Molecular weight of component `A'
MA
MA
nA
Hence, CA =
V
MA
MA PA
V MA
3. Mass fraction: The mass fraction is defined as the ratio of mass concentration of the
component `A' to the mass density of the mixture. The mass fraction of component
`A' is

(Mf)A = pP
4. Mole fraction: The mole fraction is defined as the ratio of number of moles of a
446 Heat and Mass Transfer

component 'A' to total moles of all species of the mixture. The mole fraction of
component `A' is
CA
XA
5. Partial pressure: Partial pressure of a component gas is the pressure exerted by the
component gas when it is allowed to occupy the total volume of the mixture of gases
at the same temperature. The total pressure of the mixture of gases is the sum of the
partial pressures of component gases.
P = P1 + P2 ± P3
Hence, concentration of various components of gaseous mixture can be expressed as
partial pressure of each component.
P = P1 + P2 ± P3 • • •
or P1—
1= — P2 -F P3 = XA + XB + Xc
P P P

10. Write summation rules for binary mixture of components 'A' and 'IV.

The summation rules for binary mixture can be written as—


(a) pA + pB = p ... mass density
(b) CA CB = C . .. concentration density
(c) (Mf)A + (Mf) B = 1 ... mass fraction
(d) XA XB = 1 . .. molar fraction
(e) PA + PB = P ... partial pressure
In case of gaseous binary mixture, applying perfect gas law—
PAV = nA •R •T
where, R = universal gas constant
nA PA
or
✓ R•T
nA = PA
But
✓ = `- R•T
v PA IR • T PA
Also
• —A
CA PIR • T P

CA CB = C = PA -F 1 B _
R•T R•T R•T

11. Explain: (i) mass average velocity, (ii) molar average velocity, (iii) mass diffusion
velocity, and (iv) molar diffusion velocity.

1. Mass average velocity: It is the ratio of the sum of mass velocity of all components
Mass Transfer 447

to total mass of the mixture.


PA UA + P BUB
Umean
PA + PB
PA UA + P BUB
P
= (Mf)A UA (Mf)B UB

2. Molar average velocity: It is the ratio of summation of molar velocity of each


component to the total concentration of all species.
UA CA UB CB
Umolar
CA ± CB

UA CA UB CB
C
= XA UA XB UB

3. Mass diffusion velocity: It is the velocity of a component relative to mass average


velocity of the mixture. If uA and umean are the velocities of component `A' and mixture,
then mass diffusion velocity of component 'A' = uA — Umean.
4. Molar diffusion velocity: If the velocity of a component relative to molar average
velocity of the mixture. If uA and umolar are the velocities of component 'A' and mixture,
then molar diffusion velocity of component 'A' = uA — umoiar.

12. Explain: (i) Mass flux, and (ii) Molar flux.

1. Mass or diffusion flux: It is defined similar to heat flux (heat transfer per unit area and
unit time). The mass flux (J) is the mass of a species (mA) that passes through a unit
area (A) per unit time relative to a stationary coordinate system. Therefore, mass flux
of a component 'A' can be given as—
j=
Here A = area perpendicular to flow.
A
It can be appreciated that absolute flux is summation of
mass flux of the component `A' and bulk flux of the mixture
as seen by a stationary observer. UA
u
pA uA = absolute flux = diffusion
flux + bulk motion flux
mA + pAu = J +
pA X U
A
Hence, diffusion flux is—
= PA(UA — 14)

2. Molar or diffusion flux. Molar flux is defined as the number of moles of a species
passing through unit area per unit time with respect to stationary coordinate system.
Diffusion molar flux = 11A
— where = number of moles passing area 'A' per unit time
A
448 Heat and Mass Transfer

nA
or NA =
A
Now absolute flux as seen by an stationary observer is—
CAUA = diffusion flux + bulk motion flux
= NA + CA X U
NA = CA(UA — u)

13. A container has binary mixture of oxygen and carbon dioxide gases at 21°C and 1.52
bar. The mean velocity of oxygen is 0.08 m/s and carbon dioxide is —0.02 m/s. Also
molar fraction of oxygen is 0.4. Find: (i) molar fraction of carbon dioxide,
(ii) molecular weight of mixture, (iii) molar concentration of oxygen and carbon
dioxide, (iv) mass concentration of both gases, (v) mass fraction of both gases, (vi)
mass and molar diffusion of both gases, (vii) molar and mass fluxes of both gases.

Let oxygen and carbon dioxide be `A' and 'B' components. Given molar fraction of 02 as
XA = 0.4

Now XA XB = 1
or XB = 1 — XA = 1 — 0.4 = 0.6

Molecular weight of mixture is summation of the product of molar fraction and molecular
weight of each component
M = XA MA XB MB
= 0.4 x 32 + 0.6 x 44
as MA = 02 = 32 and MB = CO2 = 12 + 32 = 44
M = 39.2
We know that molar concentration is dependent on partial pressure.

XA = PA and PB
anal = —
P
PA = XA X P = 0.4 x 152 = 60.8 kPa
PB = XB X P = 0.6 x 152 = 91.2 kPa

Now concentration is related to partial pressure.

CA = PA =
60.8
RxT 8.314 x 294
= 0.025 kg mol/m3
PB 91.2
CB — —
RxT 8.314 x 294
= 0.037 kg mol/m3
Now mass concentration is related to molar concentration.
pA = CA X MA = 0.025 x 32 = 0.793 kg/m3
pB = CB X MB = 0.037 x 44 = 1.64 kg/m3
Mass Transfer 449

Total mass of mixture is—


p = pA + pB = 0.793 + 1.64
= 2.453 kg/m3
Mass fraction can be found out from mass concentrations.

0.793 =n
(mf)A = PA = v.3.3
p 2.433

(mf)B = P
B
= 1'64 = 0.67
p 2.432
Mass diffusion velocity of the mixture is the summation of mean mass velocity of the
components.
u = (mf)A x uA + (mf)B x uB
= 0.33 x 0.08 — 0.67 x 0.02
= 0.0126
Now mass diffusion velocities are—
uA = 0.08 — 0.0126
= 0.0674 m/s
uB = —0.02 — 0.0126
= —0.0326 m/s
Molar diffusion velocity of the mixture is the summation of mean molar velocity of the
components.
V = XAUA XBUB
= 0.4 x 0.08 — 0.6 x 0.02
= 0.02 m/s
Now molar diffusion velocities are—
VA = uA — v = 0.08 — 0.02 = 0.06 m/s
vB = uB — v = —0.02 — 0.02 = —0.04 m/s
Now molar flux is—
NA = CA X UA
= 0.025 x 0.08
= 2 x 10-3 kg mol/m2 s
NB = CB X UB
= 0.037 x (-0.02)
= —7.40 x 10-4 kg mol/m2 s
Similarly, mass flux is—
= pA uA = 0.793 x 0.08
= 0.0634 kg/m2 s
.113 = PB X uB = 1.64 x (-0.02)
= 3.28 x 10-2 kg/m2 s.
450 Heat and Mass Transfer

14. Explain Fick's law of diffusion.


(UPTU — 2002 — 3, 2004 — 5)
Fick's law relates to the mass flux or molar flux to concentration gradient of a species in
a mixture. It states that the mass flux of a constituent per unit area is proportional to the
concentration gradient.
Consider a binary system with two components `A' and 'B' separated by a thin partition
as shown in figure. When partition is removed, two components diffuse through each other
until they reach at an equilibrium when concentration of both components become uniform
throughout.

Partition

Concentration
Component Component
'A 'B'

O Distance
Diffusion of components in binary mixture

Now as per Fick's law, the mass flux of the component 'A' (mA) per unit area 'A' is

proportional to the concentration gradient of component 'A' ((IPA along 'x' axis.
dx

ThA
—DAB PA ' where DAB = Mass diffusion coefficient
dx
The negative sign indicates that the mass diffusion is taking place in the direction of
decreasing concentration.
Similarly, diffusion rate of component 'B' is

/its dpB
- -DBA
A dx
The diffusion coefficient DAB and DBA are equal for binary mixture having equimolar
diffusion i.e. DAB = DBA = D. Fick's law can also be stated in term of molar flux NA
(dCA )
and molar concentration gradient
dx
dCA
NA = -DAB d x
dCB
and NB = —DBA
dx
15. Explain the physical mechanism of mass diffusion in a binary system.
(UPTU — 2005 — 6)
Consider the diffusion of components 'A' and '13' of a binary mixture as explained in last
question. The concentration of component 'A' is greater on left side when the partition is
removed. Higher concentration shows that there are more molecules of component 'A' per
Mass Transfer 45 I

unit volume at left side and more molecules of component 'A' start moving left to right on
removal of the partition. Hence, there is a net mass transfer of component 'A' from a region
of higher concentration to that of low concentration.

r
Concentration (CA)

AAlA DA B ddPA

•X
Distance -•
Diffusion and Concentration

16. Enumerate important aspects of Fick's law.

The important aspects of Fick's law are-


1. Fick's law is stated on the basis of experimental results.
2. It is valid for all states of all matters.
3. Mass diffusion rate depends upon concentration gradient of components which may
be caused by temperature gradient or pressure gradient or any external force.
4. Diffusion of any component is in the direction of decreasing concentration.
5. Diffusion coefficient (DAB or DBA) depends upon temperature, pressure and the nature
or state of system components.

17. Compare Newton's law of viscosity, Fourier's law of heat conduction and Fick's law
of diffusion.

Fick's law of diffusion is analogus to Fourier's law of heat conduction and Newton's law
of viscosity as shown below.

dT
(a) Heat flux = A = q = —k ... Fourier's law of heat conduction
dx

(b) Shear stress = 'r = —11 ud ... Newton's law of viscosity

MA dP A ... Fick's law of diffusion


(c) A = DAB d x

18. What is mass diffusivity? What are its dimensions?


(UPTU — 2002 — 3)

The constant of proportionality DAB in Fick's law is known as mass diffusivity or mass
452 Heat and Mass Transfer

diffusion coefficient. The value of mass diffusivity varies understandably from place to
place.
mA d pA
A = —"AB dx

mA
A
or DAB — —
ciP A
dx

kg
s • M2
kg
m4
= na2is

m
19. State Fick's law in: (i) Mass flux per unit are A) (ii) Molar concentration (CA),
and (iii) Partial pressure (PA).

1. Case 1: Mass flux per unit area: Fick's law is—


MA dpA
= -DAB
A dx
2. Case-2: Molar concentration: Fick's law is—
dCA
NA = - DAB
dx
3. Case-3: Partial pressure: Fick's law can be expressed in partial pressures by using
perfect gas equation.
PA VA = mA RA T where RA = gas constant
RT
or PA— Pa
mA
where R = universal gas constant
MA = molecular weight and
PA = partial pressure

or ( RikixAT )
PA=PA

Now A1A -- -DAB dPA


A dx

= -DAB • (Pi o< MA


dx RxT
Mass Transfer 453

= —DAB x MA d
R xT dx(PA)

Similarly, MB = —DBA „ MB ZNAPB)


A R x T ddx

20. Explain semi-empirical equation as given by Gilliland for mass diffusivity for a binary
mixture of two gases.
As per Gilliland, mass diffusivity of two gases A and B can be given on the basis of kinetic
energy as—
T312
DAB cc p

T3/2 1 1
or DAB = 435.7 +
P(VA1/3VB1/3 X
MA MB
where MA and MB = molecular weights,
VA & VB = molecular volume of gas A and B,
P = total pressure of mixture, and
T = absolute temperature.
Hence as per Gilliland equation, mass diffusivity depends upon pressure, temperature and
other molecular properties of gases. This helps in finding the value of mass diffusivity on
different pressures and temperatures.
T312

3/2
(Ti (P2
or -
D2 T2 ) Pi

21. A gas is diffusing through a wall of 10 mm thickness. The molar concentration of the
gas is 2.1 kg mol/m3 and 0.1 kg mol/m3 across the wall. Find the diffusion rate of the
gas if its diffusivity coefficient is 0.4 x 10-12 m2/s.
The molar diffusion is by Fick's law is—
dCA
NA = D x
dx

= 0.4 x 10-12 x CAI — CA2


L
x (2.1 — 0.1)
= 0.4 x 10-12
10 x10-3
= 0.4 x 10-12 x 2
10-2
= 0.8 x 10-10 kg mol/m2 s.
454 Heat and Mass Transfer

22. Find the diffusion coefficient for CO2 in air at 25°C and atmospheric pressure. Given
Mai, = 28.9, Vair (Molecular volume) = 29.9 cm3/gm mole, —
M co2 = 44, Vc02 = 34 cm3/gm
mole.
(UPTU - 2003 - 4)
The diffusion coefficient of mixture is-
435.7 x T3/2 i 1 + 1
DAB =
P(V1/3 + 43) lMA MB
Here A = CO2 and B = Air

(298)3/2 \i 1 + 1
. D = 435.7 x 44 28.9
101.3 x103 (341/3 + 2992)
= 0.132 cm2/s
= 13.2 x 10-6 m2/s

23. Find the expression for steady state diffusion of gases and liquids through solids or
plain membrane.
Consider fluid 'A' which is diffusing through a solid thickness L as shown in the figure. Let
CA1 and CA2 are concentration of the fluid at front and back surface of the solid.

Fluid 'A'

I.— L —H

Applying Fick's law, we have-


MA n dCA
NA = = -1-"AB
A dx
(
L CA2
or MA Sdx = DAB f dCA
Ao
CAI

MA „ T n fe r. \
A i., = L'ABk‘--Al - %-i121
A
MA (CA1 -CA2)
or = DAB
A L
Mass Transfer 455

= Cm — CA2

(DAB)

The term L is called diffusion resistance.


DAB

24. Using the analogy of thermal and diffusion resistance, write the expression for the
diffusion rate for cylinder and sphere.
Consider a cylinder of inner and outer radius of ri and r2 and length L.

ri
r2

L
Cylinder

log r2
ri
Thermal resistance is-
27r•L•k

log r2
Hence, diffusion resistance is ri
2 KL• DAB

C Al — C A2
. Diffusion rate MA = r2
log —
/1
2 irL• DAB

2 7rL• DAB(Cm — CA2)


=
log r2
Ti

_ DAB (Cm — CA2) An,


(T2 — 11)

2n L(r2 — ri )
where Am —
log r2
ri
Consider now a hollow sphere of radius r1 and r2.
456 Heat and Mass Transfer


P2—
Thermal resistance is
rri r2 k
r2 —
Diffusion resistance is
4 ivr1 r2 DAB
CA1 — CA2
MA —
4ivr1 r2 DAB
DAB x 4ir ri r2 x(Cm. — CA2)
r2 — r1
DAB (Cm — CA2)
m
(r2 — ri)
where Am = 47r ri r2

25. Derive general mass diffusion equation.

Guidance: Mass transfer by molecular diffusion is analogus to heat transfer by


conduction. It is also similar to momentum transfer in laminar flow. We have already seen
a2
1 aT
that heat conduction can be given by Fourier conduction equation a e + =
x k a at
a2 T
which reduces to = 0 for q = 0 and steady state. On the same line, general mass
a x2
diffusion equation is—
a2CA/a x2 = 0
Consider a differential volume, of sides dx, dy and dz of component as given in the
figure below. The mass balance of component 'A' diffusing through the control volume of
stagnant component '13' is—
(a) Net rate of diffusion fluxes entering the control volume from x, y and z direction
= NA, x(dy • dz) + NA,y(dx • dz) + NA, z(dx • dy)
(b) Net rate of diffusion fluxes leaving the control volume in x, y and z directions
= NA,x+dx(dy • dz) + NA, y+dy(dx • dz) + NA,z+dz(dx • dy)
Mass Transfer 457

Now molar flux is—


a
NA,x+dx = NA,x + —
__, (NA z)dx
ux
NA,y+dy = NA,y + (NA,Ody
ay

NA,z+dz = NA,z + (NA z)dz


aZ
Hence, the mass of component of accumulated in control volume due to diffusion is the
difference of net rate of flux in and net rate of flux out

or Mass accumulated = —
, a a Ai
Ax A (i)
L ax + ay NAY + a Z
iv
z

If rizA, g is mass generation rate of component A per unit volume, then net rate of mass
generation of component A in control volume is—

mA, g (dx • dy • dz)

Mass concentration of component A in control volume will change due to mass


accumulation [equation (i)] and mass generation [equation (ii)]. The rate of change of molar
concentration of component A is—
a CA
(dx • dy • dz)
at

NA, z + dz

NA, x
A,x +dx

z
x
N 3,
A,

Control volume and mass diffusions

Now mass balance of control volume is-


ac
- [— N Ax + — NA +— NAz 1(dx • dy • dz) + rhA,g x (dx • dy • dz) = A (dx • dy • dz)
ax ay Y aZ at
Now as per Fick's law, we have—
a CA
NAx = DAB
-

x
458 Heat and Mass Transfer

-a -A
acAl ar D aCA l a D aCA )- F tit _ a CA
ax " ax ) a)A AB ay ) " az) A'g - at

or a2 CA a2CA a2CA + MA,g 1 a CA


axe aye az2 DAB DAB at

Without any mass generation mA,g = 0, for steady state a sCA = 0 and one dimension, the
governing mass diffusion equation becomes—
a2
cA _0
ax 2 —

26. Explain the molecular diffusion when isothermal evaporation takes place through a
stagnant gas or air.

Guidance: The evaporation of water and other volatile liquids such as benzene, acetone,
gasoline and carbon tetrachloride and further their diffusion through stagnant air are called
molecular diffusion through a stagnant gas.
Consider water surface exposed to air and evaporation of water from its free surface
which diffuses through a stagnant air as shown in the figure.

Air A

Pw
Water vapour

Evaporation of liquid in stagnant air

Assumptions made for simplifications—


(a) System has constant pressure and temperature
(b) System has achieved steady state and concentrations remain unchanged at various
locations with time.
(c) Water vapour and air are ideal gases.
On evaporation from the free surface, water vapour diffuses upward through air. During
steady state, the upward movement of water vapour is balanced by downward movement
of air so that concentration at any location 'x' remains constant. The diffusion of air
downward can be given as—
DAB x Ax MA dPA
inA = — — (i)
RxT dx
Mass Transfer 459

where A = cross-sectional area of the tank


MA = molecular weight of air

The content of water vapour or vapour pressure (Pw) decreases in air along x-axis. Hence,
total pressure is sum of partial pressure of air (PA) and water (Pw)
or P = PA Pw
Since P is constant, we can say
dPA dP dP
w as =0
dx dx dx
It means that a decrease of air pressure is compensated with an increase of vapour
pressure. It is also known that concentration of component depends upon partial pressure.
At the surface of water, air cannot move downward. Hence, there has to be a bulk mass
movement in upward direction with a velocity sufficient to balance the mass diffusion of
air occurring in downward direction.
Mass transfer of air in upward direction = —pA x 14A x A

= 12A X M A ) X U A x A
-( RT

PA
where PA = X MA
M
RT
and uA = bulk mass velocity for upward movement.
Equating equation (i) and (ii)
— DAB xAxMA dPA
x = - PA x A xMA X UA
RxT dx RxT
DAB dPA
or uA =
PA X dx
Now the mass of water which is diffusing upward is-
-DAB xAxM„, dPw
mw =
RxT X dx
Also water vapour which is carried by bulk mass velocity (uA) taking place in upward
direction is—
(mw) bulk = Pw X A X uA

=(_Pw x Mw ) x A x uA
RxT
DAB dPA
But we have already found out that uA = x
PA x
P
(mw) bulk = I — w x Mw x Ax DAB x dPA
R xT PA dx
460 Heat and Mass Transfer

The total water mass transport rate is the sum of Mw and (Mw)bulk•

(kw)total = Mw(kw)bulk

—DAB xAxMw dPw Pu xMw xAxDAB dPA


RxT dx PA xRxT dx

—DAB x Ax Mw x (PA + Pw) dPw dPA —dPw


— _ x as
RxT PA dx dx dx

—DA xAxMw P dPw


— _ x x asPA +Pw .P and PA =P—Pw
RxT P—Pw dw

The above relation is known as Stefan's law. Now on integration, we get—


x2 Irt
(m) DAB xAxMw xf, dPw
wtotal dx x
R XT r P—Pw
x, P
DABxAxMw (P—Pc,)
w)total X (1C2 xl) = xPx log
RxT (P — Pw)

DAB xAxMw
or (mw )total - xPxlog ( PA2
RxT EA1

The variation of partial pressures of water vapour and air with distance from the free
surface (x) is as shown below.

P PW + PA
Partial pressure

x
(Distance from free surface)

Partial pressures of air and water vapour vary


with distance from free surface

The above equation can also be expressed in terms of concentrations as under—


NA DAB X C ( C — CA2 moihn2s
log
A C— 1
Mass Transfer 46 I

27. What are the limitations of Fick's law?


(UPTU — 2003 — 4)

Fick's law has the following limitations—


(a) It is valid only when diffusion occurs due to concentration gradient. It cannot be
applied when diffusion occurs due to temperature gradient, pressure gradient or
external force.
(b) The mass or molar fluxes are measured relative to coordinates which move with some
average velocity of the mixture. Fick's law cannot be applied if flux is expressed
relative to a fixed set coordinates.

28. Show that for equimolar counter-diffusion

MA _ A A [PAZ
X — PAl]
Ro T Ax
(UPTU — 2002 — 3)

Guidance: Equimolar counter-diffusion in a binary gas mixture means that each molecule
of component A is replaced by each molecule of component B and vice versa. Also diffusion
is isothermal and the mass flux of one gaseous component is equal to but acting in opposite
direction from other gaseous component.
Consider two large chambers A and B which are interconnected as shown in the figure.
Due to interconnection, molecules of component A and B are replacing each other. The total
pressure P(P = PA + PB) is maintained throughout. Also total concentration of species is
maintained in each chamber.

Chamber Chamber
A B
NA NB PB
PA
CA CB
x
x
2

PAS PD
La2

PBS PA1

P = PA+PB

Equimolar counter diffusion


462 Heat and Mass Transfer

Molar diffusion rates of component A and B can be given by Fick's law as-
d CA
NA = -DAB
dx

and NB = — DBA dx
dx
For steady state condition, we have—

NA + NB = 0
or NA = - NB

and DAB = -DBA

dCA d CB
Therefore, =
dx dx
dCA
Now take NA = -DAB
dx
x2 CA2
or NA f dx = - DAB f dCA
x, cA,
or NA(x2 - x1) = DAB(CA1 - CA2)
DAB(C Ai - CA2)
or NA -
(x2 - x1)

nA PA _ PA
But CA = = - -
V MA RT

and NA = mA
A

ABx1) x 1417, (PAl - PA2)


NA = ma - (x2D-
il

DAB A (P
v- Al - PA2)
or
MA = Ta' (-7C2 - xl)

DBA • A (PAi — PAD


Similarly,
mB = R • T (x2 — x1)

29. A pipe containing NH3 gas is vented to ambient air by lapping the pipe and inserting
a 4 mm diameter tube which extends to 10 m into atmosphere so as to maintain
pressure of one atmosphere. The temperature is maintained at 30°C. If ammonia-
air mixture has diffusion coefficient (D) = 0.4 x le m2/s, find: (i) mass flow rate
of ammonia to atmosphere, and (ii) mass flow rate of air in the pipe both in kg mol/
s and kg/s.

Guidance: It is a problem of equimolar counter diffusion in which mass flux of ammonia


Mass Transfer 463

is equal but opposite to mass flux of air as shown in the figure. Ammonia diffuses into
atmosphere from present partial pressure of 1 atm to zero.

IAmmonia
(A)
/ Air (B) ',
TB - 25°C I,
TA = 25°C
PB = 1 atm l
PA = 1 atm

Diffusion of ammonia and air

N. =MA = DAB[PA1 -PA21


A A R T x2 — x1
Here Pm = 1 atm = 1.01 x 105 N/m2
PA2
DAB =
=0 x m2/s
R = 8314 N/kg.mol
T = 30°C = 30 + 273 = 303 K

A=
7Cd 2 x (4 4 10 3)2
= 12.56 x 10-6 m2
4 4
0.4 x10 4 [1.01X105 -01
MA = 8314 x 303 x12.56 x10-6
L
0.4 x 12.56 x 1.01 x 10-5
8.314 x 3.03 x 10-6
= 2.014 x 10-12 kg mol/sec
Now molecular weight of NH3 is 17.
mA = 17 x 2.014 x 10-12
= 34.238 x 10-12 kg/s
As we know for equimolar counter diffusion—
NB = — NA
mB mA
A A
or mB = —2.014 x 10-12 kg molls
= —2.014 x 10-12 x 29 (molecular wt. of air = 29)
= 58.406 kg/s

30. Explain dimensionless parameters in convective mass transfer.


The dimensionless parameters are generally used to correlate convective mass transport data
to thermal and momentum data.
464 Heat and Mass Transfer

1. Schmidt number: It is the ratio of momentum diffusivity to mass diffusivity


S c — Momentum diffusivity
Mass diffusivity
/1//, v
DAB DAB
Schmidt number compares the relative thickness of velocity and concentration
boundary layers.
2. Lewis number: It is the ratio of thermal diffusivity to mass diffusivity.
= Thermal diffusivity
Le
Mass diffusivity

DAB pXCXDAB
The Lewis number compares the relative thickness of thermal and concentration
boundary layers.
3. Sherwood number: It is the ratio of concentration gradient at the surface to overall
concentration gradient
Concentration gradient
Sh =
Overall concentration gradient
Molecular mass transport resistance
Convective mass transport resistance
k L
= c where k, = convective mass transfer coefficient
DAB
= function (Re, Sc)

31. Discuss the analogy of concentration boundary layer with the thermal and
hydrodynamic boundary layers.
(UPTU — 2003)

The concentration boundary layer has similarity to the thermal and hydrodynamic boundary
layers. The convective mass transfer mechanism is similar to convective heat transfer
y

Concentration boundary layer


Mass Transfer 465

mechanism. It is also classified as natural and forced convective mass transfer processes.
Consider a mass of species `A' is flowing over a solid surface having concentration profile
similar to velocity and temperature profile in the boundary layer.
Now concentration of species `A' at the surface (CAS) is greater than concentration of
species `A' at great distance from the surface (CA„,). The species 'A' diffuses from the
surface into the fluid through concentration boundary layer whose thickness is `Sc'. The
thickness of concentration boundary layer is defined in the same way as it is done for
hydrodynamic boundary layer or thermal boundary layer.
mA (dCA
= —D = k (CAs — CA_)
A dy )y=u
r‘ e
Here kc is called convective mass transfer coefficient.
dCA )
—Dx(
dy y=0
k=
c (CAS CA«)
Consider a boundary thickness dy which is less than `Sc' where concentration is CA.
(dCA (CA — CA0c )
Hence, we can have
dyj y=0 dy y=0
Therefore, we can write—
D(dCA
dyj y=0
kc =
(CAS — CA.)
CA—CAS]
—D
[ dy
CAS — CA«

kc _ dy / y=0
(CA—CAS
or
D (CAS — CA.)

(
kc xL _ dy )y=0
CA—CAS
D (CAS — CA«)
L

k xLi
The ratio of e s similar to I h L in convective mass transfer and it is called Sherwood
D
number similar to Nusselt number in heat transfer.
466 Heat and Mass Transfer

32. Give expression for equivalent diffusivity of non-diffusing, multicomponents


mixture having fixed components.
Equivalent diffusivity of non-diffusing, multicomponents mixture having fixed components
can be given in terms of mole fraction and diffusivity of each species as given under-
D=
1
nB nc nE,
DAB DAc DAD
where nB, nc and nD are mole fraction of the species in the mixture; and DAB, DAc and DAD
are diffusivity of species A through B, C and D.

33. Nitrogen gas diffuses through a 15 mm layer of non-diffusing gaseous mixture which
has: (i) C2H4 = 25%, (ii) C2H6 = 15%, and (iii) C4 H10 = 60% under steady condition
and at temperature and pressure of 20°C and 1 atmosphere. The partial pressures
of nitrogen at two planes of layer are 0.14 and 0.07 bar respectively. The diffusivity
of nitrogen through C2H4, C2H6 and C4Hio are 17 x 10-6, 15 x 10-6 and 10 x 10-6
respectively. Find the diffusion rate across the two planes.
The effective or equivalent diffusivity of gas with no diffusing species is-
D—
1
XB X c X D
DAB DAc DAD

1
0.25 + 0.15 ± 0.6
17 x 10-6 15 x 10-6 10x 10-6
= 11.8 x 10-6 m2/s

15 H

Consider suffixes N and M denote nitrogen and gaseous mixture respectively. Hence,
pressure is—
P = PN Pm
where PN = partial pressure of nitrogen
Pm = partial pressure of gaseous mixture
Given P N1 = 0.14 x 105 N/m2
PN2 = 0.07 x 105 N/m2
PAn = P — PN1 = 1.013 x 105 — 0.14 x 105
= 0.873 x 105 N/m2
Mass Transfer 467

PM2 = P - PN2 = 1.013 x 105 - 0.07 x 105


= 0.943 x 105 N/m2
Dx A x MN p
Now diffusion MN = x log PM2
RxT x2 -x1 Pm'

11.8x10-6 x1x28 1.013 x10 5


or MN = x x log 0.943
8314 x 293 15 x 10-- 0.873
= 7.06 x 10-5 kg/s.m2

34. Hydrogen gas is diffusing through air. Air has non-diffusing species: (1) 02, (ii) N2,
and (iii) H2. Mole fraction are 02 = 0.2, N2 = 0.7 and H2 = 0.1. Find diffusion coefficient
of H2 in air during steady state for temperature 300 K and pressure 4 bar. Diffusion
coefficients of hydrogen in oxygen DH_02 = 16 x 10-6 m2/s at 273 K and 1 bar while
in nitrogen DH_N2 = 18 x 10-6 m2/s at 300 K and 1 bar.

Guidance: Firstly, the diffusion coefficients DH_02 and DH_N2 have to be found out at
( 3/2 n
300 K and 4 bar using relation = x 1-2 . The equivalent diffusion coefficient of
D2 T2 P1
the gas mixture is to be found out from the relation D = 1
X02 + XN2

DH- 02 DH-N2
Diffusion coefficients at 300 K and at 4 bar are-
=( 3/2
x ( ) x (16 x10-6)
DH-02 273 ) 4
= 4.6 x 10-6 M2/S
( 300)3'2x 4
1 x (18 x 10 6)
DH-N2
300)
= 4.5 x 10-6 11112/S
The mole fraction of oxygen and nitrogen in mixture without hydrogen-
= 0 222
x02 = 1- 01 .
0.7
xN2= 1 01 = 0.777
Hence, effective or equivalent diffusion coefficient is-
D= 1
0.222 + 0.777
4.6 x 10-6 4.5 x 10-6
= 4.5 x 10-6 m2/s
468 Heat and Mass Transfer

35. Pressurised hydrogen gas is stored at 80°C in a 5 meter outer diameter spherical
vessel having wall thickness of 5 cm. Molar concentration of H2 at inner surface is
0.087 K mol/m3 and outer surface is zero. Find mass flow rate of H2 by diffusion
through spherical wall. Assume diffusion coefficient of H2 through spherical wall is
1.2 x 10-12 m2/s.

Molecular diffusion of H2 through spherical vessel is—


DAB X (CAA — CA2)
N=
r2 —
47r r2
Here r1 = 2.45 m, r2 = 2.5 m
DAB = 1.2 x 10-12 m2/s, CA1 = 0.087 kmol/m3
CA2 = 0 and MH2 = 2 kg/kmol
1.2 x 10-12 x (0.087)
N= 2.5 — 2.45
4irx2.45x2.5
= 1.607 x 10-10 K mol/s
Now m = N X MH2
= 1.607 x 10-1° x 2
= 3.214 x 10-10 kg

36. Estimate the diffusion rate of water from the bottom of the test tube 1.5 cm in
diameter and 15 cm long into dry atmosphere air at 25°C. Take diffusion coefficient
of 25.6 x 10-6 m2/s.
(JNTU — 2004)

Guidance: The diffusion of water through stagnant air in term of partial pressure can be
given as—
DAB xAxM, p
— x l og( P
R xT (x 2 —xi) P — Ps

Air

Water
vapour

Test tube
Mass Transfer 469
Now partial pressure of water vapour at free surface is Ps = 3.169 kPA (saturation
pressure at saturation temperature of 25°C). Also partial pressure of water vapour at the top
of tube is almost zero i.e. P = 0 due to presence of totally dry air.
ar 7r x (15 x10-3)12 x 18 x 1.013 x 105
6 101.32
25.6 x 10- x log
4 0.15 10132 — 3.169
8314 x 298
= 7.05 x 10-10 kg/s

37. A well is 30 m deep and 9 m in diameter is exposed to atmospheric pressure at 25°C.


The air at the top has 50% relative humidity. Find the diffusion rate of water vapour
through the well if DAB = 2.58 x 10-5 m2/s.

The well with partial pressure of water vapour as shown—


Air
T Waterpressure
i vapour partial
= PO

E
0

1Water vapour partial


pressure = Ps

-a— 9 m

Well

Ps at saturation temperature of 25°C from steam table = 3.169 kPa


11_ at top = Relative humidity x P,
= 50 x 3.169 = 1.585 kPa
100
DAB xAx (P — Poc)
Now = _ x x log
R xT (X2 X Xi) (P — Ps)

2.58 x10-5 x Lr x 92 x 18 1.013 x 105 (101.3 —1.585)


inn, = 4 X x log
8314 X 298 30 (101.3 — 3169)
= 3.63 x 10-7 kg/s

38. Salt is removed from the top of 2 m deep pond to ensure salt concentration at top as
zero. Salt concentration at bottom is 5 kg mol/m3. Find the rate of molar diffusion of
salt per unit area if DAB = 1.5 x m2/s. Total concentration of salt is 50 kg mol/m3

The molar diffusion is—


NA = DAB x C log C — CA2
A C — Cm
470 Heat and Mass Transfer

C = 50 kg mol/m3
CA = 0 -
ri2

7------------------
/
/
-----/
7T
/,
/- -,
_/ 2m
/- _.,
7-

,,,,„/„,,,,,,,,/ ,i.
c
Ai = 5 kg mol/m
3

Pond

NA = 1.5 x 10-9 (50 — 0)


x 50 x log
A 2 (50 — 5)
= 1.5 x 25 x l(r9 x 0.105
= 3.95 x 10-9 kg mol/m2s

39. Carbon dioxide is flowing through a rubber pipe having inside diameter 25 mm and
wall thickness of 2.5 mm at 25°C and 3 bar pressure. The solubility of CO2 in rubber
is 40.15 x K mol/m3 bar and diffusivity of CO2 through rubber is 0.11 x 10-9
m2/s. Find the loss of CO2 by diffusion per unit length of rubber pipe.
The rubber pipe carrying CO2 is as shown—

r2 = 15 mm

= 12.5 mm
CAI

CA2

Now CA2 = 0, CA1 = Solubility x Pressure


= 40.15 x x3
= 0.12 k mol/m3
Diffusion resistance for cylindrical surface is—
log r2
r1
Rdiffusion =
27rxLxDAB

log 15
or 12.5
Rdiffusion = 27r x 1 x 0.11 x 10-9
= 2.64 x 108 s/m2
NA CA1 — CA2
L Rdiffusion
0.12 — 0
2.64 x 108
= 4.57 x 10-10 k molls
Mass Transfer 47I

40. Calculate the effective diffusion coefficient of CO through air. The mole fractions of
species are-
02 = 0.18, N2 = 0.72 and CO = 0.10
The gas mixture has 3 atmospheric pressure and 37°C temperature. Diffusion
coefficients are-
Dco-o2 = 18.5 x 10-6 m2/s at 273 K and 1 atmosphere
Dco-N2 = 19.2 x 10-6 m2/s at 288 K and 1 atmosphere
The effective diffusion coefficient is—
D= 1
2 + XN,
DcXO°- 02 DCO N2
)312

Now = p2
D2 (1'2 )
103)3/2(13) X 18.5 X 10-6
Dco-02 = (327
= 7.46 x 10-6 m2/s
(310 )3/2(1) x 19.5 x 10-6
Dco-N2 = 288 3
= 7.25 x 10-6 m2/s
The mole fractions free of CO are-
018
= 02
1-0.1 — .
x — 0.72 — 0.8
N2 — 1 — 0.1 —
Hence, diffusion coefficient is—
D= 1
( 0.2 )+ ( 0.8
7.46 x10-6 ) 7.25 x10-6 )
= 7.3 x 10-6 m2/s

41. Using general mass differential equation for one dimension, without generation and
for steady state, show diffusion through a plain membrane is—

mA CA1 — CA2
A— L
472 Heat and Mass Transfer

General mass differential equation is-

d2
CA = 0
dx

L
y
//////// ////////// ///
Diffusion through membrane

On integration, we get-
d CA c
dx 1
On integrating again, we get-

CA = C X + C2
Now applying boundary conditions, we have-

CA Cit i at x=0
CA = CA2 at x =L
Hence, C2 = CA2
= CA2 - CA1
and C1

(CA2- CA1)
CA = X -I- CA2

Now mass transfer is-


m dCA -D(CA2 -CAD
a = D =
A dx

CA1 -CA2
L
D

42. A spherical container of fused silica (Si 02) has inside diameter of 0.25 m and wall
thickness of 3 mm, is storing helium gas at 20°C and 3 bar. Find the rate at which
this pressure decreases with time if diffusion coefficient is 0.4 x 10-13 m2/s and
absorptivity of helium is 0.45 x k mol/m3 bar.

The distribution of concentration of helium is as shown


Molar diffusion of helium gas is-

(CA1 -CA2)
NA = DAB
(x2 - x1)
Mass Transfer 473

CA
2

Now CA1 = Molecular weight x Absorptivity x Partial pressure of helium


= MA x S x PAl
CA2 = MA X S X PA2 but PA2 = 0
=0
DAB x MA X S X PAi
. NA —
(x2 — x1)
Whatever helium diffuses into fused silica is fully absorbed. Hence,
MA = mA, stored
d(pAV)
NA x A= MA, stored =
dt

= PA
But PA = MA X CA and CA
RxT

() A
—NA x A = d MxP
dt R x T

dPA —RxT x A
or x NA
dt MA XV
Now putting the value of NA, we have—
dPA —R x Tx A DAB x MA x S x PA
x
dt MA XV (x2 — xi)

ird2
Now A = ird2 and V —
6
dPA _ —6R xT xDAB xSx PA
dt d(x2 — x1)

—6 x 8314x293x 0Ax10-3 x0A5x10-3 x10-5 x5


0.25 x 0.003
= —1.75 x 10-6 Pa/s
474 Heat and Mass Transfer

43. An organic compound has molecular weight 78 and it is stored in an open tank having
diameter of 5 m and thickness of compound is 1 mm at its bottom. The compound
vapour pressure is 0.14 bar in the tank. Compound diffusion is taking place through
a stagnant air filter 3 mm thick. The operating pressure and temperature of the
system are 1 atmospheric pressure and 20°C. The diffusion coefficient is 8.5 x 104
m2/s. If density of the compound is 880 kg/m3, find time taken for the entire
compound to evaporate.
The arrangement is-
Air filter 3 mm thick
Organic compound
PA
,-,5 . 0.14 bar

,f /
H 5 m -H
Tank with compound

Diffusion rate is-


DABxAxMA P-I).
MA — — x P tog
RxT AX P-13,

8.5x10-6x4x52x78 x 1.013 x 105 (1.013 - 0)


= tog
8314 x 293 3 x 10-3 - (1.013 - 0.14)
= 2.68 x 10-2 kg/s
Mass of organic compound to be evaporated is-
= mass x density
= (7 x 52 x 0.001) x 880
4
= 17.2 kg
Now thA x t = 17.2
or t= 17.2
2.68 x 10-2
or t = 6445 sec.

44. Hydrogen gas is flowing through a pipe of inner diameter 2.5 cm and outer diameter
5 cm at 2 atmospheric pressure. Diffusion coefficient of hydrogen gas is 1.944 x 104
m2/s and solubility is 0.0532 cm3/cm3 of pipe at one atmospheric pressure. If
temperature is 25°C, find the loss of hydrogen by diffusion per unit length of pipe.
Solubility of hydrogen at 2 atmosphere is-
= 0.0532 x 2
Mass Transfer 475

= 0.1064 cm3/cm3 of pipe


PV = m RT
Px V x MHZ
or m = PV =
RT RxT

(2 x 1.03 x 105) x 0.1064


8314 x 298
= 0.0176 kg/m3 of pipe
Concentration of hydrogen at inner surface is Cm = 0.0176 kg/m3 while at outer surface
CA2 = 0
Diffusion of hydrogen is-
DAB X (CM C A2)
141A —
log
rl
2irL

1.944 x 10-8 x (0.0168 — 0)


log 5
2xxl
= 2.97 x 10-9 kg/s.
INDEX

A Condensation, 427
Absorptivity, 305 physical mechanism of, 425
Air-conditioning system, 5 types of, 426
Analogus electric circuit, 60 Condenser, 373
Artificial black body, 305 value of effectiveness of, 388
Average convective heat transfer coefficients, Conduction, 7
18 Continuity equation, 271
Convection, 8, 216
B law of, 17
Biot number, 178 Convective heat transfer,
Black body emissive power, 20 classification of, 216
Black body, 304 parameters of, 218
Boiler walls, 1 1 1 Convective mass transfer, 463
Boiling heat transfer, 432 Coordinate systems, 30
Boiling, 427 Correction factor, 209
Boundary conditions, 39 values of, 380
kinds of, 40 Critical radius of insulation, 86
Boundary layer, Critical thickness (of insulation), 85, 86
concept of, 220 Cylindrical coordinates system, 31
Cylindrical wall,
C thermal resistance of, 57
Cable with insulation, 118
Capacity ratio, 386 D
Cartesian coordinates system, 31 Diffuse grey body, 306
Circumferential rectangular, 167 Diffuse reflection, 308
Coefficient of friction, Diffusion flux, 447
variation of, 221 Dimensional analysis, 239
Coefficient of thermal expansion, 268 Direct contact heat exchanger, 371
Combined heat transfer mechanism, 20 Double pipe heat exchanger, 373
Combined mechanism of cooling, 20 Double tube heat exchanger, 375
Composite wall, 58 Dropwise condensation, 426, 427
Concentration boundary layer, 464
478 Index

E Friction drag coefficient, 224


Electric machines, 5 Furnace wall, 112, 113
Electrical analogy, 56
Emission from real body, 306 G
Emissive power, 304, 314 Gaussian error function, 202, 213
Emissivity from real body, 306 Geometric factor, 179
Energy equation for laminar flow, 271 Grashof number, 223, 270
Equivalent electrical network, 328 Greenhouse effect, 345
Evaporator, 373 Grey body factor, 329
value of effectiveness of, 389
H
F Hamilton chart, 326
Fick's law of diffusion, 450, 451 Heat conduction equation, 34, 39
Fick's law, 451 Heat conduction, 51
limitation of, 461 law of, 11
Film boiling with radiation, 434 Heat energy transfer,
Film boiling, 434 momentum and, 240
Film temperature, 227 Heat energy, 2
Filmwise condensation, 426, 427 magnitude of, 2
Fin effectiveness, Heat exchangers, 370
significance of, 143 classification of, 371
Fin of uniform cross-section, 137 effectiveness of, 383, 390
Finite difference form, 125 Heat flow,
Finite difference method, 124 with insulation, 110
Fins, 135 without insulation, 110
applications of, 136 Heat flux, 3
efficiency of, 144 Heat mass process,
heat transfer from a, 136 stoppage of, 6
types of, 136 Heat transfer by conduction, 8
Flow, Heat transfer by convection, 8
types of (in free convection), 277 Heat transfer coefficient, 59
Fluids, Heat transfer, 445
heat capacity of, 381 application of, 4
viscosity of, 215 classification of, 9
Forced convection boiling, 433 combined mechanism of, 21
Forced convection, 9, 217, 266, 272 modes of, 2, 7
Fouling, 371 steady state of, 4
Fourier number, 178 types of, 4
Fourier's law of heat conduction, 12, 451 unsteady state of, 4
Fourier's law, 11 Heisler charts, 191, 208, 210, 211
Frantz law, 17 Heisler correction factor, 212
Free convection boiling, 433 Hydraulic diameter, 229
Free convection, 266, 272 Hydrodynamic boundary layer, 219
Free electrons, Hyperbolic functions, 165
transport of, 8 derivatives of, 164
Index 479
I Molecular diffusion, 458
Indirect contact heat exchanger, 371 Momentum equation, 271
Infinite solids, 199 Monochromatic emissive power, 304
Internal energy, 3 Morgan chart, 326
Irradiation, 308
N
K Natural convection, 9, 216
Kirchhoff's law, 307 Newton's law of cooling, 17
Newton's law of viscosity, 451
L Nodal finite difference equations, 126
Laminar flow (viscous flow), 217 Nodal network difference form, 125
Laplace equation, 120, 124 Node difference form, 125
Lattice vibration, 8 Nuclear boiling, 433, 435
Local boiling, 433 Number of transfer units, 383
Local convective heat transfer coefficients, 18 Nusselt number, 222
Local heat transfer coefficient, 221 Nusselt's theory, 428
Log mean temperature difference, 375
Log-mean area for a hollow cylinder, 53 0
Lumped heat capacity system, 176 One-dimensional heat conduction equation, 32,
37
M Opaque body, 306
Mass average velocity, 446
Mass concentration, 445 P
Mass density, 445 Penetration depth, 203
Mass diffusion equation, 456 Penetration time, 203
Mass diffusion velocity, 447 Point,
Mass diffusion, definition of, 30
physical mechanism of, 450 Pool boiling, 433
Mass flux, 447 Prandtl number, 222
Mass fraction, 445 Provost theory, 303
Mass transfer process,
stoppage of, 6 Q
Mass transfer, 445 Quantum theory, 303
applications of, 444
by diffusion, 6, 444 R
by change of phase, 444 Radial heat flow, 52
by convection, 444 Radiant heat,
importance of, 6 properties of, 10
modes of, 7, 444 Radiation heat transfer,
Max-Planck's equation, 310, 311 mechanism of, 302
Max-Planck's law, 309 Radiation, 8, 10
Molar average velocity, 447 intensity of, 313
Molar concentration, 445 law of, 19
Molar diffusion velocity, 447 types of, 302
Molar flux, 447 Radiosity, 309
Mole fraction, 445 Rayleigh number, 224
480 Index

Reciprocity theorem, 316 Thermal conductivity, 12


Reflection of radiation, range of, 14
types of, 308 value of, 13
Reflectivity, 305 Thermal contact resistance, 61
Refrigeration system, 5 Thermal diffusivity, 33
Regenerator, Thermal equilibrium, 3
value of effectiveness of, 389 Thermocouple,
walls of, 117 response of, 180
Relaxation method, 127 sensitivity of, 180
Reynolds number, 218 Thermodynamics analysis, 4
Thermodynamics,
S heat transfer and, 3, 4
Saturated boiling, 433 Time constant, 179
Semi infinite solids, 199 Total emissive power, 20, 304
Shape factor, 121, 122, 315 Transient heat conduction, 173
Shell and tube heat exchanger, 374 analytical method of, 173
Simultaneous velocity, 221 Transient heat flow,
Skin drag, 224 characteristics of, 173
Skin friction coefficient, 224 Transit energy, 2
Solar radiation, 346 Transmissivity, 305
Solid angle, 312 Transparent body, 306
Specular reflection, 308 Turbulent flow, 217
Spherical coordinates system, 31 velocity profile for, 228
Spherical wall,
thermal resistance of, 57 U
Stable film boiling, 435 Unsteady heat conduction, 173
Stanton number, 223 characteristics of, 173
Steam pipe with insulation, 120
Stefan-Boltzman's law, 19 V
Stefan-Boltzmann equation, 303 Volume,
Stefan-Boltzmann law, 310 definition of, 30
Stored energy, 2 Volumetric coefficient of thermal expansion, 268
Sub-cooled boiling, 433 Von Korman integral technique, 273
Surface,
defmition of, 30
White body, 306
T Wiedermann law, 17
Temperature, 4 Wien's displacement law, 311
Thermal boundary layer, 220, 221, 228 Wien's displacement, 308

You might also like